Superpage
Soft tissue

Authors: Engy Abdellatif, M.B.B.Ch., M.D., Ph.D., Jamshid Abdul-Ghafar, M.D., Ph.D., Shahad Abdulameer, M.D., Asma K. Abu-salah, M.B.B.S., Rabia Ahmed, M.B.B.S., Borislav A. Alexiev, M.D., Rola H. Ali, M.D., Ohoud Aljarbou, M.D., Saba Anjum, M.B.B.S., Komal Arora, M.D., Tamanna Asghari, M.B.B.S., Zoonish Ashfaq, M.B.B.S., Amna Babar, M.B.B.S., Vindhya Bellamkonda, M.D., Cecilia Belzarena, M.D, M.P.H., Fireneh N. Beshah, M.D., Iva Brčić, M.D., Ph.D., Ali Chaudri, M.D., Qurratulain Chundriger, M.B.B.S., Michael R. Clay, M.D., Jiufa Cui, M.D., Ph.D., Carina Dehner, M.D., Ph.D., Louis P. Dehner, M.D., Josephine K. Dermawan, M.D., Ph.D., Deepti Dhall, M.D., Brendan C. Dickson, M.D., M.Sc., Gonçalo F. Esteves, M.D., Ana Félix, M.D., Ph.D., Saira Fatima, M.B.B.S., Karen Fritchie, M.D., Jerad M. Gardner, M.D., Raul S. Gonzalez, M.D., Mujtaba Haidari, M.D., Christopher S. Hale, M.D., Anjelica Hodgson, M.D., Kenneth A. Iczkowski, M.D., Christine E. Jabcuga, M.D., Lawrence J. Jennings, M.D., Ph.D., Ivy John, M.D., Dia Kamel, M.D., Ph.D., Erica Kao, M.D., Maria Tasneem Khattak, M.B.B.S., Kemal Kösemehmetoğlu, M.D., William B. Laskin, M.D., Bernadette Liegl-Atzwanger, M.D., S. Shawn Liu, M.D., Ph.D., Jose G. Mantilla, M.D., Jeanne Meis, M.D., Michael Michal, M.D., Ph.D., Diego M. Montoya-Cerrillo, M.D., Smitha Mruthyunjayappa, M.D., Alessandra F. Nascimento, M.D., Huy Q. Nguyen, M.D., Ayesha Nusrat, M.B.B.S., Farres Obeidin, M.D., Rajaguru Paramaguru, M.D., Nat Pernick, M.D., Raul Perret, M.D., M.Sc., Thuy L. Phung, M.D., Ph.D., Susan Potterveld, D.O., M.P.H., Jian-Hua Qiao, M.D., Madiha Bilal Qureshi, M.B.B.S., Alka Rani, M.B.B.S., Shima Rastegar, M.D., Muhammad Raza, M.B.B.S., Salvatore Lorenzo Renne, M.D., Andrea Saggini, M.D., Omar P. Sangueza, M.D., Matjaz Sebenik, M.D., Vijay Shankar, M.D., Sabeehuddin Siddique, M.B.B.S., Charanjeet Singh, M.D., Valeriya Skorobogatko, B.A., Jonathan C. Slack, M.D., Brian D. Stewart, M.D., Lauren N. Stuart, M.D., M.B.A., Norbert Sule, M.D., Ph.D., David Suster, M.D., Ummiya Tahir, M.B.B.S., Nasir Ud Din, M.B.B.S., Zeeshan Uddin, M.B.B.S., Brandon Umphress, M.D., Jaylou M. Velez-Torres, M.D., Meggen Walsh, D.O., M.S., P.A., Jigang Wang, M.D., Ph.D., Laura Warmke, M.D., Stacy D. Webb, M.D., Ghazi Zafar, M.B.B.S.
Resident / Fellow Advisory Boards: Josephine K. Dermawan, M.D., Ph.D., Erna Forgó, M.D., Farres Obeidin, M.D.
Deputy Editor-in-Chief: Borislav A. Alexiev, M.D.
Editor-in-Chief: Debra L. Zynger, M.D.

Copyright: 2002-2024, PathologyOutlines.com, Inc.

Soft Tissue & Bone related: Jobs, Fellowships, Conferences, Cases, CME, Board Review, What's New #1, What's New #2

Related chapters: Skin nonmelanocytic tumor, Bone & joints, Muscle & peripheral nerve nontumor

Editorial Board oversight: Borislav Alexiev, M.D. (last reviewed January 2023), Jose G. Mantilla, M.D. (last reviewed November 2023), Nasir Ud Din, M.B.B.S. (last reviewed January 2023), Farres Obeidin, M.D. (last reviewed November 2023)
Page views in 2024 to date: 26

Alveolar rhabdomyosarcoma
Definition / general
  • Highly aggressive type of rhabdomyosarcoma characterized by uniform, primitive, round cells showing skeletal muscle differentiation (Cancer 1969;24:18)
Essential features
  • High grade, cellular, round cell sarcoma with evidence of skeletal muscle differentiation
  • Nests and sheets of monomorphic cells may have pseudoalveolar pattern (Curr Top Pathol 1995;89:273)
  • Diffuse strong myogenin expression
  • Detection of PAX3::FOXO1 (60%) or PAX7::FOXO1 (20%) gene fusions by molecular analysis (Oncogene 2001;20:5736)
  • Predilection for deep soft tissue of lower extremities
  • High stage at presentation with overall worse prognosis
Terminology
  • Alveolar rhabdomyosarcoma
  • Preference towards switching to fusion positive and fusion negative rhabdomyosarcoma terminology among oncologists (Cancer Discov 2014;4:216)
ICD coding
  • ICD-O: 8920/3 - alveolar rhabdomyosarcoma
  • ICD-10: C49.9 - malignant neoplasm of connective and soft tissue, unspecified
  • ICD-11: 2B55.Z & XH7099 - rhabdomyosarcoma, unspecified primary site & alveolar rhabdomyosarcoma
Epidemiology
Sites
  • Commonly involves deep soft tissue of extremities, typically forearm
  • Other sites include head and neck, trunk, paraspinal, pelvic, genitourinary regions and retroperitoneum
  • Can present with widespread involvement, such as bone marrow infiltration (leukemia-like presentation) and lymphadenopathy (Diagn Pathol 2017;12:77)
  • 25 - 30% of patients present with metastatic disease
  • Most common metastatic sites include lung and lymph nodes (Int J Clin Oncol 2014;19:536)
  • Can metastasize to other sites such as the bone; however, rare visceral metastases to brain, breast, kidney and pancreas have also been reported (Arch Argent Pediatr 2021;119:e349)
Pathophysiology
  • Derived from cells with phenotypic and biologic features of myogenic lineage (undifferentiated mesodermal cells) (Cancer Lett 2009;279:126)
  • Genetic fusion events including translocations t(2;13)(q36;q14) and t(1;13)(p36;q14) lead to expression of fusion proteins PAX3::FOXO1 and PAX7::FOXO1
  • These proteins act as transcriptional activators, causing dysregulation of transcription affecting myogenesis
  • PAX::FOXO1 proteins behave as oncoproteins and contribute to tumor development by alteration of cell growth pathways and apoptosis, modulation of myogenic differentiation and stimulation of motility and other downstream pathways (Skelet Muscle 2012;2:25)
Etiology
  • Unknown
Diagrams / tables

Images hosted on other servers:
Role of PAX proteins in cell biology

Role of PAX proteins in cell biology

Clinical features
Diagnosis
  • Diagnosis is based on histologic features / immunomarkers expression in correlation with age, site and clinical findings (Mod Pathol 2001;14:506)
  • Small, round, blue cell tumor with diffuse positivity for desmin and myogenin
  • Molecular studies (PAX3::FOXO1 / PAX7::FOXO1 gene fusions) in morphologically challenging cases and cases with inconclusive immunohistochemistry (Skelet Muscle 2012;2:25)
  • WHO essential and desirable diagnostic criteria
    • Uniform round cells with or without alveolar growth pattern, diffuse homogenous expression of myogenin; heterogeneous expression for desmin or MyoD1; detection of PAX3::FOXO1 / PAX7::FOXO1 fusion by molecular analysis
Laboratory
  • Nonspecific
Radiology description
  • Radiological features are nonspecific in differentiating from other sarcomas (Clin Radiol 1978;29:53)
  • Location and demographics are most useful in truncating differentials
  • Plain radiograph: soft tissue density mass
  • Ultrasound: well defined heterogeneous nonuniform mass of low to medium echogenicity
  • CT: soft tissue density lesion with contrast enhancement
  • MRI:
    • T1: low to medium intensity lesion, isointense to adjacent muscle; prominent vascularity (Pediatr Radiol 2019;49:1516)
    • T2: hyperintense lesion; conspicuous vascularity (Radiographics 2020;40:791)
    • Postcontrast sequences: heterogeneous enhancement; ring-like enhancement when necrosis
Radiology images

Images hosted on other servers:
Left deltoid mass; exhibits restricted diffusion

Left deltoid mass; exhibits restricted diffusion

Hyperintense left thigh mass

Hyperintense left thigh mass

Prognostic factors
Case reports
Treatment
Clinical images

Images hosted on other servers:
Right facial swelling and nasal cavity mass

Right facial swelling and nasal cavity mass

Huge right submandibular mass

Huge right submandibular mass

Gross description
Gross images

Images hosted on other servers:
Fleshy, soft, gray-white tumor with necrosis

Fleshy, soft, gray-white tumor with necrosis

Large, pale, fleshy mass

Large, pale, fleshy mass

Frozen section description
  • Sensitive but not very specific (Turk Patoloji Derg 2015;31:16)
  • Clusters and single tumor cells with variable fibrous component
  • Dense cytoplasm with cytoplasmic vacuolation
  • Irregular, hyperchromatic nuclei with conspicuous nucleoli
Microscopic (histologic) description
  • Cellular round cell tumor
  • Large clusters, nests, cords and trabeculae of primitive round cells, separated by variably thick fibrovascular septa
  • Loss of cellular cohesion in the center forms alveolar-like, cystic and vague papillary appearance (Histopathology 2022;80:98)
  • Layer of cells adheres to the periphery of the spaces and fibrous septa
  • Small to intermediate sized monomorphic cells with scant cytoplasm
  • Hyperchromatic nuclei with variable conspicuous small nucleoli
  • Cells in the center have poor preservation and are necrotic; may appear floating
  • Multinucleated tumor giant cells with wreath-like lineup of nuclei are common (Acta Pathol Microbiol Immunol Scand A 1982;90:345)
  • Round to oval rhabdomyoblasts with abundant acidophilic cytoplasm may be present
  • Brisk mitosis and variable tumor necrosis
  • Occasional cases may show clear cell morphology with pale, glycogenated cytoplasm
  • Rare anaplasia
  • Some cases may show histologic features of embryonal rhabdomyosarcoma; molecular studies are essential to characterize such cases (Am J Clin Pathol 2013;140:82)
  • Solid variant:
    • Sheets of neoplastic cells
    • Lack fibrovascular septa, pseudoalveolar spaces and dyscohesion (Cancer Genet Cytogenet 2005;163:138)
    • May show rhabdomyoblastic differentiation
    • Abundant mitotic activity
Microscopic (histologic) images

Contributed by Nasir Ud Din, M.B.B.S.
Cellular round cell tumor

Cellular round cell tumor

Pseudoalveolar architecture

Pseudoalveolar architecture

Vague papillary appearance

Vague papillary appearance

Rhabdomyoblasts

Rhabdomyoblasts

Wreath-like multinucleated giant cells

Wreath-like multinucleated giant cells


Clear cell change

Clear cell change

Central floating cells

Central floating cells

Round blue cells

Round blue cells

Solid variant

Solid variant

Brisk mitosis

Brisk mitosis


Fibrovascular septa

Fibrovascular septa

Desmin

Desmin

Myogenin

Myogenin

CK AE1 / AE3

CK AE1 / AE3

Cytology description
  • Cellular smear with dyscohesive loose clusters of small to intermediate sized monotonous cells; may show pseudoalveolar pattern (Diagn Cytopathol 2014;42:1069)
  • Cells have scant cytoplasm, round to irregular hyperchromatic nuclei (Diagn Cytopathol 1992;8:465)
  • Variable fibrous tissue, wreath-like multinucleated cells and rhabdomyoblastic differentiation
Cytology images

Images hosted on other servers:
Alveolar structures

Alveolar structures

FNA biopsy of cervical lymph node

FNA biopsy of cervical lymph node

Positive stains
Negative stains
Electron microscopy description
Electron microscopy images

Images hosted on other servers:
Alveolar structures Alveolar structures

Copious intracytoplasmic actomyosin filaments and dark Z bands

Molecular / cytogenetics description
  • PAX3::FOXO1 / PAX7::FOXO1 fusion (85% of cases):
    • t(2;13)(q36;q14): PAX3::FOXO1 in 70 - 90% (Curr Mol Med 2007;7:47)
    • Amplification of MYCN and CDK4 in PAX3::FOXO1 positive cases
    • t(1;13)(p36;q14): PAX7::FOXO1 in 10 - 30%
    • Amplification of PAX7 and MIR17HG in PAX7::FOXO1 positive cases
  • Other fusions:
    • PAX3::FOXO4 fusion
    • PAX3::NCOA1 fusion
    • PAX3::INO80D fusion
    • FOXO1::FGFR1 fusion
  • Inactivating mutation of TP53 and CDKN2 (J Clin Oncol 2021;39:2859)
  • Activating mutation of FGFR4
  • Overexpression of ALK protein (Pediatr Dev Pathol 2009;12:275)
Molecular / cytogenetics images

Images hosted on other servers:
t(2;13) translocation and karyotype

t(2;13) translocation and karyotype

Disruption of <i>FOXO1</i> gene

Disruption of FOXO1 gene

Videos

Alveolar rhabdomyosarcoma

Sample pathology report
  • Forearm, wide margin excision:
    • Features are consistent with alveolar rhabdomyosarcoma (see comment)
    • Tumor size: 6 x 5 x 4.5 cm
    • Margins: superior margin: 1.8 cm
      • Inferior margin: 2.1 cm
      • Medial margin: 2 cm
      • Lateral margin: 2.2 cm
      • Anterior margin: 1.9 cm
      • Posterior margin: 2.2 cm
    • Comment: Histology shows an infiltrative cellular tumor composed of large clusters and nests of primitive round cells, separated by variably thick fibrovascular septa. Irregular pseudoalveolar spaces and cystic change are present within the nests. Scattered wreath-like multinucleated giant cells and rhabdomyoblasts are present along with brisk mitosis and necrosis. Tumor cells show strong and diffuse desmin and myogenin expression, focal MyoD1 and alpha smooth muscle actin positivity, whereas CD34, S100, CK AE1 / AE3, CD99, SALL4, LCA, TdT and synaptophysin are negative. Morphological features and immunoprofile strongly support the diagnosis of alveolar rhabdomyosarcoma. It is a malignant high grade soft tissue sarcoma with myogenic differentiation and an overall worse prognosis.
Differential diagnosis
Board review style question #1

An 18 year old boy presented with a rapidly growing mass in the right forearm, which was resected. Gross examination showed a skin covered fibromuscular tissue with an infiltrative, soft, fleshy gray-white tumor of 6 x 5 x 4.5 cm. Histology showed a small, round, blue cell tumor arranged in nests and aggregates separated by fibrous septa. Pseudoalveolar spaces with cystic change were present along with frequent mitoses and focal necrosis. Immunostains desmin and myogenin were diffuse strong positive. CD34, S100, CK AE1 / AE3, CD99, SALL4, LCA, TdT and synaptophysin were negative. Focal muscle specific actin and MyoD1 expression was present. What is the most likely diagnosis?

  1. Alveolar rhabdomyosarcoma
  2. Embryonal rhabdomyosarcoma
  3. Pleomorphic rhabdomyosarcoma
  4. Sclerosing rhabdomyosarcoma
  5. Spindle cell rhabdomyosarcoma
Board review style answer #1
A. Alveolar rhabdomyosarcoma. Alveolar rhabdomyosarcoma shows small, round, blue cell morphology with pseudoalveolar change, diffuse desmin and myogenin expression and focal MyoD1. Answer B is incorrect because embryonal rhabdomyosarcoma has round and spindle cell morphology with scattered differentiated rhabdomyoblasts as well as desmin positivity and heterogeneous nuclear staining for myogenin and MyoD1. Answer C is incorrect because pleomorphic rhabdomyosarcoma has sheets of large, atypical pleomorphic and frequently multinucleated polygonal, spindle or rhabdoid eosinophilic cells as well as diffuse strong desmin positivity with focal myogenin and MyoD1. Answer D is incorrect because sclerosing rhabdomyosarcoma has cords and trabeculae of round to ovoid cells in a sclerotic collagenous background as well as diffuse MyoD1 positivity, focal desmin and myogenin. Answer E is incorrect because spindle cell rhabdomyosarcoma has fascicles of spindle cells as well as diffuse MyoD1, focal desmin and myogenin.

Comment Here

Reference: Alveolar rhabdomyosarcoma
Board review style question #2

Which particular type of multinucleated giant cells is associated with alveolar rhabdomyosarcoma?

  1. Floret-like giant cells
  2. Langhans giant cells
  3. Osteoclast-like giant cells
  4. Touton type giant cells
  5. Wreath-like giant cells
Board review style answer #2
E. Wreath-like giant cells. Wreath-like giant cells are seen in alveolar rhabdomyosarcoma and clear cell sarcoma. Answer A is incorrect because floret-like giant cells are present in spindle cell / pleomorphic lipoma, neurofibroma and giant cell fibroblastoma. Answer B is incorrect because Langhans giant cells are seen in tuberculosis and sarcoidosis. Answer C is incorrect because osteoclast-like giant cells are seen in giant cell tumor and tenosynovial giant cell tumor. Answer D is incorrect because Touton type giant cells are seen in xanthoma, xanthelasma and myxofibrosarcoma.

Comment Here

Reference: Alveolar rhabdomyosarcoma

Alveolar soft part sarcoma
Definition / general
  • Alveolar soft part sarcoma (ASPS) is a rare sarcoma of uncertain histogenesis
  • First formally described by Christopherson et al. in 1952 (Cancer 1952;5:100)
  • Characterized by a specific translocation, der(17)t(X;17)(p11.2;q25), resulting in ASPSCR1-TFE3 gene fusion
Essential features
  • Rare malignant mesenchymal neoplasm frequently composed of large, polygonal cells with abundant eosinophilic cytoplasm, a nested or pseudoalveolar growth pattern and PASD+ intracytoplasmic rhomboid or rod shaped crystals
  • Predominantly affects the deep soft tissues of the extremities (thigh and buttock) in young adults and head and neck region (tongue and orbit) in children
  • Characterized by ASPSCR1-TFE3 gene fusion
ICD coding
  • ICD-O: 9581/3 - alveolar soft part sarcoma
  • ICD-11: 2B5F.2 & XH8V95 - sarcoma, not elsewhere classified of other specified sites & alveolar soft part sarcoma
Epidemiology
Sites
Pathophysiology
  • ASPSCR1-TFE3 fusion protein localizes to the nucleus, functioning as an aberrant transcription factor, resulting in c MET overexpression and activation of c MET signaling, making the tumor cells sensitive to c MET inhibition (Cancer Res 2007;67:919, J Pathol 2013;229:743, PLoS One 2017;12:e0185321)
  • Mouse model demonstrated the dependance of ASPS on lactate for growth, which may relate to its common occurrence in muscle with the expression of lactate transporter, monocarboxylate transporter (MCT1) protein and CD147 (Cancer Cell 2014;26:851, Am J Pathol 2002;160:1215)
  • Defined by a specific genetic alteration, der(17)t(X;17)(p11;q25) that results in the fusion of the TFE3 transcription factor gene (from Xp11) with ASPSCR1 at 17q25.3 (Oncogene 2001;20:48)
Etiology
  • ASPSCR1-TFE3 translocation is an instigating genetic event (Oncogene 2001;20:48)
  • No association with radiation or a cancer predisposition syndrome has been reported
Clinical features
  • Presents as a slow growing, painless mass (Clin Orthop Surg 2014;6:80)
  • Proptosis can be seen with orbital tumors
  • Vaginal bleeding can be seen with female genital tract tumors
  • At presentation, the tumor can be localized (38%), regional (11%) and metastatic (43%) (J Surg Oncol 2016;113:581)
Diagnosis
  • Eosinophilic polygonal cells with a nested growth pattern, rich capillary network and intracytoplasmic rod shaped crystals
  • TFE3 nuclear expression by immunohistochemistry
  • Confirmation of TFE3 gene rearrangement or ASPSCR1-TFE3 gene fusion (Arch Pathol Lab Med 2015;139:1459)
Radiology description
Radiology images

Contributed by Jeanne Meis, M.D. and Mark R. Wick, M.D.
Left leg mass

Left leg mass

Pulmonary metastases

Pulmonary metastases

CT scan, orbital



Images hosted on other servers:

MRI ankle mass T1

MRI ankle mass T2

Nuclear medicine bone scan

Chest CT

Thyroid gland ultrasound

MRI, tongue mass

Prognostic factors
Case reports
Treatment
  • Radical surgical resection is the treatment of choice
  • Excision of lung and brain metastasis may prolong survival (Cancer 1989;63:1)
  • Adjuvant chemotherapy does not seem to be effective
  • Radiation may reduce the risk of local recurrence (Cancer 2001;91:585)
  • Clinical trial of crizotinib (c MET inhibitor) showed disease stabilization without much tumor shrinkage (Ann Oncol 2018;29:758)
  • Multiple pulmonary metastases in select patients have responded to interferon alfa 2a (Br J Cancer 2003;89:243, Med Pediatr Oncol 2001;37:482)
  • Long term clinical follow up is mandatory given the risk of metastasis > 10 years after diagnosis
Clinical images

Contributed by Mark R. Wick, M.D.

Orbital



Images hosted on other servers:

Tongue

Orbital tumor

Nose and thigh

Gross description
  • Solid, partially circumscribed mass with fleshy nodules and fibrotic bands
  • Usually yellow to gray to white-tan
  • Tumor size varies from 1.2 to 24 cm (median: 6.5 cm) (Cancer 2001;91:585)
  • Frequently has large vessels at the periphery
Gross images

Contributed by Mark R. Wick, M.D.

Deep soft tissue mass

Frozen section description
  • Large polygonal cells with abundant eosinophilic cytoplasm
  • Eccentric nuclei with prominent nucleoli
  • Nests of tumor cells with surrounding vascular channels
  • Scrape preparations can demonstrate cytomorphology (Cancer 2009;117:500)
Frozen section images

Contributed by Laura Warmke, M.D.
Touch prep

Touch prep

Frozen section

Nest of tumor cells

Microscopic (histologic) description
Microscopic (histologic) images

Contributed by Laura Warmke, M.D., Jeanne Meis, M.D. and Mark R. Wick, M.D.
Polygonal cells

Polygonal cells

Prominent nucleoli

Prominent nucleoli

Eosinophilic cytoplasm

Eosinophilic cytoplasm

Pseudoalveolar-like structures

Pseudoalveolar-like structures

Central discohesion

Central discohesion

Fibrous septa

Fibrous septa


Clear cell change

Clear cell change

Capillary network

Capillary network

Vascular network

Vascular network

Lymphovascular invasion

Lymphovascular invasion

Fibrous septa

Fibrous septa

Pseudogland formation

Pseudogland formation


Pleomorphic foci

Pleomorphic foci

Large nests

Large nests

Uniform cells

Uniform cells

Hemangiopericytomatous vasculature

Hemangio-pericytomatous vasculature

Cytoplasmic vacuolization

Cytoplasmic vacuolization

Conventional and pleomorphic areas

Conventional and pleomorphic areas


Intracytoplasmic crystals

Intracytoplasmic crystals

TFE3 TFE3

TFE3

SMA

SMA

PAS

PAS

PASD

Virtual slides

Images hosted on other servers:

39 year old man with a distal extremity mass

Cytology description
Cytology images

Contributed by Laura Warmke, M.D. and Mark R. Wick, M.D.
Prominent nucleoli

Prominent nucleoli

Granular cytoplasm

Granular cytoplasm

Polygonal cells

Polygonal cells

FNA



Images hosted on other servers:

Fine needle aspiration

Clusters of tumor cells

Positive stains
Negative stains
Electron microscopy description
  • Tumor cells have numerous mitochondria and well developed Golgi apparatus
  • Characteristic rhomboid or rod shaped intracytoplasmic crystals with a regular lattice pattern (Arch Pathol Lab Med 2007;131:488)
  • Ultrastructural immunohistochemistry has shown that the crystals consist of aggregates of MCT1 and CD147 (Am J Pathol 2002;160:1215)
Electron microscopy images

Contributed by Mark R. Wick, M.D.

Rhomboid crystals



Images hosted on other servers:

Rod shaped crystals

Intracytoplasmic crystal

Molecular / cytogenetics description
Molecular / cytogenetics images

Images hosted on other servers:

FISH for TFE3

ASPL-TFE3 (ASPSCR1-TFE3) gene fusion

Videos

Alveolar soft part sarcoma versus alveolar rhabdomyosarcoma, by Dr. Gardner

Sample pathology report
  • Soft tissue, mass, anterior right thigh, core biopsy:
    • Alveolar soft part sarcoma (see comment)
    • Comment: This biopsy is composed of round to polygonal tumor cells with abundant eosinophilic cytoplasm and round nuclei with vesicular chromatin and prominent macronucleoli. The tumor cells grow in a nested or pseudoalveolar pattern with nests of cells separated by fibrous bands and delicate, thin walled blood vessels. Focal lymphovascular invasion is present. Immunohistochemical stains show that the tumor cells are positive for TFE3 (strong nuclear staining), while they are essentially negative for pancytokeratin, S100 protein, SMA, desmin, synaptophysin, MelanA and HMB45. Fluorescence in situ hybridization (FISH) confirms the presence of a TFE3 gene rearrangement. These results support the above diagnosis.
Differential diagnosis
Board review style question #1

Which of the following special stains is most helpful in diagnosing alveolar soft part sarcoma (ASPS)?

  1. Colloidal iron
  2. GMS
  3. PAS with diastase
  4. PAS without diastase
  5. Warthin-Starry
Board review style answer #1
C. PAS with diastase can highlight the characteristic rhomboid or rod shaped intracytoplasmic crystals within the tumor cells in approximately 80% of cases.

Comment Here

Reference: Alveolar soft part sarcoma
Board review style question #2

Metastases to which of the following locations are more common in alveolar soft part sarcoma (ASPS) than in any other soft tissue sarcoma?

  1. Bone
  2. Brain
  3. Liver
  4. Lung
  5. Lymph node
Board review style answer #2
B. Brain. While ASPS frequently metastasizes to the lungs and bone, as do many other sarcomas, metastases to the brain are much more common in ASPS than any other sarcoma. Metastatic involvement of the liver and lymph nodes is unusual in ASPS.

Comment Here

Reference: Alveolar soft part sarcoma

Anastomosing hemangioma
Definition / general
Essential features
  • Benign vascular tumor which displays overlapping features with well differentiated forms of angiosarcoma
  • Recently described in soft tissue, showing a predilection to the paraspinal areas (Am J Surg Pathol 2016;40:1084)
  • Composed of anastomosing sinusoidal capillary sized vessels with mild endothelial nuclear variability and scattered hobnailed endothelial cells
  • Cured by simple excision
Sites
Gross description
  • Well demarcated with a hemorrhagic mahogany spongy cut surface
Microscopic (histologic) description
  • Nonlobular architecture
  • Anastomosing proliferation of capillary sized vessels, reminiscent of splenic sinusoids (Am J Surg Pathol 2010;34:942), within a framework of nonendothelial supporting cells
  • Rare to absent mitotic activity
  • Mild endothelial nuclear variability and scattered hobnailed endothelial cells
  • Fibrin thrombi are typical
  • Extramedullary hematopoiesis and mature fat in roughly 50% of cases
Microscopic (histologic) images

Contributed by Ivy John, M.D.

Various images

Positive stains
Negative stains
Differential diagnosis
  • Angiosarcoma: diffusely infiltrative growth pattern; prominent cytological alterations, including high grade cytologic atypia, multilayering of endothelial cells and mitotic activity
  • Retiform hemangioendothelioma: predilection for the distal extremities; usually involves the skin; similar appearance to normal rete testis
  • Hobnail hemangioma: usually involves the skin; wedge shaped vascular proliferation with dilated vascular channels superficially and less conspicuous vessels in the deep aspect of the lesion
  • Splenic tissue:

Angiofibroma of soft tissue
Definition / general
  • Benign fibroblastic neoplasm of uniform spindle cells
  • Variable myxoid and collagenous stroma with a network of thin walled, branching blood vessels
Essential features
  • Clinically well defined mass located in the soft tissues of the extremities, often in association with joints or fibrotendinous structures
  • Histologically uniform spindle cells, myxoid / collagenous stroma, network of thin walled, branching blood vessels
  • Cytogenetically NCOA2 gene rearrangements
ICD coding
  • ICD-O: 9160/0 - angiofibroma, NOS
  • ICD-11: EE6Y & XH1JJ2 - other specified fibromatous disorders of skin and soft tissue & angiofibroma, NOS
Epidemiology
Sites
Pathophysiology
Etiology
  • Unknown
Clinical features
Diagnosis
  • Requires interpretation of clinical, radiological and histopathological findings
Radiology description
  • MRI
    • Limited descriptions of the appearance of angiofibroma of soft tissue on imaging
    • A few reports have described it as a well circumscribed mass
    • Signal characteristics and enhancement are presumed to be variable, subject to the cellular, myxoid, collageneous and vascular content of the lesion
    • T1: roughly isointense to skeletal muscle; T2: heterogeneous high signal intensity; T1C+ (Gd): variable from homogeneous to a peripheral enhancement (Mol Clin Oncol 2017;7:229)
Radiology images

Contributed by Nasir Ud Din, M.B.B.S.
Angiofibroma of ankle Angiofibroma of ankle

Angiofibroma of ankle

Angiofibroma of distal leg

Angiofibroma of distal leg



Images hosted on other servers:
MRI vastus lateralis muscle

MRI vastus lateralis muscle

Prognostic factors
  • Angiofibroma of soft tissue pursues a benign clinical course
  • Rare local recurrence and no evident metastatic potential
Case reports
Treatment
Clinical images

Images hosted on other servers:
Arthroscopic view angiofibroma knee

Arthroscopic view of knee

Gross description
  • Well demarcated, nodular or multinodular solid tumor
  • Infiltration of the surrounding tissues does occur
  • Range in size from 1.2 to 10.0 cm (mean 5.1 cm)
  • White to yellow, often glistening cut surface
  • Cystic or hemorrhagic areas may be present (Genes Chromosomes Cancer 2017;56:750)
Microscopic (histologic) description
  • Well circumscribed
  • Vaguely lobulated architecture
  • Alternating myxoid and collagenous areas
  • Regional variation in cellularity (Genes Chromosomes Cancer 2017;56:750)
  • Uniform bland spindle cells
  • Prominent vascular network
  • Spindle cells have inconspicuous palely eosinophilic cytoplasm
  • Short ovoid or tapering nuclei, irregular nuclear contours, fine chromatin and indistinct nucleoli
  • Cytological atypia and nuclear hyperchromasia are generally absent
  • Vascular network is composed of innumerable small, thin walled blood vessels evenly distributed throughout the lesion
  • Less prominent medium or large sized blood vessels with variably thick walls are usually present (Am J Surg Pathol 2012;36:500)
  • Perivascular collagen deposition and marked hyalinization or fibrinoid necrosis of medium sized vessel walls
  • Occasional degenerative atypia (Am J Surg Pathol 2012;36:500)
  • Degenerative changes may be focally present, including chronic inflammation, hemorrhage and aggregates of foamy histiocytes (Histopathology 2016;69:459)
  • Variably dense infiltrate of lymphocytes, sometimes perivascular in distribution
Microscopic (histologic) images

Contributed by Nasir Ud Din, M.B.B.S.
Diffuse spindle cells and vascular network

Diffuse spindle cells and vascular network

Variable cellular areas of spindle cells

Variable cellular areas of spindle cells

Hypo and hypercellular area of the lesion

Hypocellular and hypercellular area of the lesion

Hypocellular area

Hypocellular area

Spindle cells with thin vascular network

Spindle cells with thin vascular network


Thin walled blood vessels

Thin walled blood vessels

Spindle cell neoplasm with occasional ectatic vessels

Spindle cell neoplasm with occasional ectatic vessels

Extracellular collagenous matrix

Extracellular collagenous matrix

Collagenous stroma Collagenous stroma

Collagenous stroma


Network of thin walled blood vessels

Network of thin walled blood vessels

Variable cellularity and edema

Variable cellularity and edema

Edematous area Edematous area

Edematous area

CD34

CD34



Contributed by Komal Arora, M.D. (Case #311)
Circumscribed spindle cell tumor Circumscribed spindle cell tumor

Circumscribed spindle cell tumor

Spindle cells in collagenous stroma Spindle cells in collagenous stroma

Spindle cells in collagenous stroma


Spindle cells with thin vasculature Spindle cells with thin vasculature

Spindle cells with thin vasculature

Spindle cells in myxoid stroma

Spindle cells in myxoid stroma

Positive stains
Molecular / cytogenetics description
Molecular / cytogenetics images

Images hosted on other servers:
FISH revealing NCOA2 rearrangement

FISH revealing NCOA2 rearrangement

Videos

Angiofibroma of soft tissue

Sample pathology report
  • Soft tissue, thigh, excision:
    • Angiofibroma of soft tissue (see comment)
    • Comment: The sections reveal a fibroblastic neoplasm composed of uniform spindle cells in a variable myxoid and collagenous stroma with a network of innumerable small, thin walled, branching blood vessels. No necrosis, marked cytologic atypia or significant mitotic activity is identified. It is positive for CD34 and EMA on immunohistochemistry. These support the above diagnosis.
Differential diagnosis
  • Cellular angiofibroma:
    • Usually occurs in the pelviperineal region
    • More uniformly cellular
    • Vessels are more rounded having hyalinized walls
    • Usually ER+, PR+; loss of RB1
  • Solitary fibrous tumor:
    • Fibroblast-like cells with patternless appearance
    • Thick bands of collagen and prominent branching, hyalinized vessels
    • CD34+, CD99+, STAT6+
    • STAT6 rearrangement
  • Low grade fibromyxoid sarcoma:
    • Alternating myxoid and collagenous areas and whorled growth pattern
    • Arcades of thin wall blood vessels
    • 33% of cases contain giant collagen rosettes
    • EMA+, MUC4+
    • FUS::CREB3L2 fusion in most
  • Low grade myxofibrosarcoma:
    • Lobulated growth pattern, myxoid background, infiltrative borders
    • Long curvilinear vessels with perivascular hypercellularity
    • Cells have pleomorphic hyperchromatic nuclei and mitoses
  • Myxoid liposarcoma:
    • Prominent chicken wire vasculature
    • Numerous signet ring lipoblasts particularly at periphery of lobules
    • Mucoid matrix rich in hyaluronidase sensitive acid mucopolysaccharides; may have large mucoid pools
    • DDIT3 rearrangement
Board review style question #1
A 40 year old woman presented with a slowly growing mass on left thigh. Grossly, the tumor was well circumscribed with yellow-white cut surface. Microscopically, the tumor showed numerous small, branching, thin walled blood vessels in a collagenous and myxoid stroma. On immunohistochemistry, the tumor cells showed positivity for CD34 and EMA. What is the most likely diagnosis?

  1. Angiofibroma of soft tissue
  2. Cellular angiofibroma
  3. Low grade fibromyxoid sarcoma
  4. Myxofibrosarcoma
  5. Myxoma
Board review style answer #1
A. Angiofibroma of soft tissue. It is a fibroblastic neoplasm that commonly arises in lower extremities, often in relation to joints or fibrotendinous structures of middle aged females. It is composed of uniform spindle cells in a variable myxoid and collagenous stroma with a network of innumerable small, thin walled, branching blood vessels. It is positive for CD34 and EMA on immunohistochemistry.

Comment Here

Reference: Angiofibroma of soft tissue
Board review style question #2

A 55 year old woman presented with a mass on lateral aspect of left ankle for last 5 years. MRI revealed a well demarcated, enhancing, round mass that measured 4 cm. Excision biopsy was performed and microscopic examination revealed a lesion in given photomicrograph. Which cytogenetic aberration is associated with this tumor?

  1. AHHR::NCOA2
  2. FUS::CREBB3L2
  3. FUS::DDIT3
  4. NAB2::STAT6
  5. RB1 / FOXO1 deletion
Board review style answer #2
A. AHHR::NCOA2. The photomicrograph shows a tumor composed of uniform spindle cells in a variable myxoid and collagenous stroma with a network of innumerable small, thin walled, branching blood vessels, characteristic of angiofibroma of soft tissue. It is associated with NCOA2 rearrangement or AHRR::NCOA2 fusion gene in most cases.

Comment Here

Reference: Angiofibroma of soft tissue

Angioleiomyoma
Definition / general
  • Benign dermal / subcutaneous neoplasm arising in vascular smooth muscle; there is strong overlap with myopericytoma and these lesions are considered to be part of the same spectrum of disease (J Ultrasound Med 2016;35:1669)
Essential features
  • Angioleiomyoma is a benign smooth muscle tumor
  • Preoperative diagnosis is not certain, microscopic examination is needed for accurate diagnosis
  • Complete excision is curative
Terminology
ICD coding
Epidemiology
Sites
Pathophysiology
Etiology
Clinical features
  • Painful, solitary, slow growing nodule (J Ultrasound Med 2019;38:1201)
  • Well demarcated subcutaneous lesions classically found on the lower extremities but can also present on the head, trunk and upper extremities (J Cutan Pathol 2017;44:342)
  • Difficult to distinguish from other smooth muscle tumors and few are diagnosed pre-operatively (J Cutan Pathol 2017;44:342)
  • Presence of pain may be helpful in the diagnosis but is unreliable, as only 58% of tumors present with pain (J Cutan Pathol 2017;44:342)
  • Pain can be caused by local ischemia from contraction of the tumoral vessels and may be mediated by nerve fibers (J Ultrasound Med 2019;38:1201)
  • Multiple subcutaneous angioleiomyomas have been reported in a patient with AIDS (Br J Dermatol 2002;147:563)
  • Clinical differential diagnosis includes glomus tumors, spiradenomas, angiolipomas or neuromas
Diagnosis
Radiology description
  • On ultrasonography: subcutaneous masses with an oval shape, well defined margins, a homogeneous structure, hypervascularity and a small size (< 2 cm) (J Ultrasound Med 2016;35:1669)
  • On MRI: well circumscribed mass with slightly hyperintense signal on T1 weighted images, heterogeneous and hyperintense signal on T2 weighted and STIR images and heterogeneous enhancement on contrast enhanced T1 weighted images
Prognostic factors
  • No known prognostic factors
Case reports
Treatment
Clinical images

Images hosted on other servers:
Missing Image

Foot mass

Missing Image

Palate mass

Missing Image

Lip mass

Missing Image

Hand tumor

Gross description
  • Sharply demarcated, spherical, gray-white or brown tumor
  • Usually measuring less than 2 cm in diameter in approximately 80% of cases
  • Reference: Cancer 1984;54:126
Gross images

Images hosted on other servers:
Missing Image

Circumscribed lip mass

Missing Image

Homogenous cut surface

Microscopic (histologic) description
  • Encapsulated proliferation of eosinophilic smooth muscle cells with minimal nuclear pleomorphism
  • Rounded or slit-like veins with muscular walls present within the tumor (J Cutan Pathol 2017;44:342)
  • Tumors can further be classified into 3 subcategories, first described by Morimoto et al:
    • Solid type: smooth muscle bundles surround numerous small slit-like channels
    • Cavernous type: dilated vascular channels, the walls of which are difficult to distinguish from the intervascular smooth muscle
    • Venous type: thick walled vessels that are easily distinguished from the intervascular smooth muscle
  • Epithelioid and pleomorphic variants are reported in a few cases
  • Calcification and marked degeneration, including hyalinization and myxoid changes, may be present (J Cutan Pathol 2017;44:342, J Ultrasound Med 2019;38:1201)
  • Fat can be seen in a few cases; not to be mistaken for angiomyolipoma (Cancer 1984;54:126)
Microscopic (histologic) images

Contributed by Ohoud Aljarbou, M.D., Jijgee Munkhdelger, M.D., Ph.D. and Andrey Bychkov, M.D., Ph.D.

Smooth muscle fascicles and vascular channels

Lack atypia and mitosis

Well defined mass

Edema and hyalinization

Small fascicles of smooth muscles


Well defined mass with vascular component

Fascicles of smooth muscles

SMA

Desmin


Missing Image

Low power

Missing Image

Well defined nodule

Missing Image

Smooth muscle bundles, vessels

Missing Image

Organizing thrombus

Missing Image

Thin walled blood vessels

Missing Image

Thick walled blood vessels

Cytology description
  • Variable proportions of benign smooth muscle cells and uniform spindle cells
  • Dissociated or arranged in small fascicles
  • Small fragments of a collagenous matrix in the background (Monogr Clin Cytol 2017;22:68)
  • The limitations encountered are very small, hard or painful skin lesions that may be difficult to aspirate (Diagn Cytopathol 2002;27:161)
  • Abundant connective tissue and calcification can also cause problems in obtaining adequate material (Diagn Cytopathol 2002;27:161)
Positive stains
Negative stains
Molecular / cytogenetics description
Sample pathology report
  • Skin, lower leg mass, excision:
    • Angioleiomyoma, completely excised (see comment)
    • Comment: The grossly noted tumor is 1 cm. The resection margins are negative. There is no evidence of atypia or malignancy.
Differential diagnosis
Board review style question #1

    The image shown above is from a 45 year old woman with a right leg mass. SMA and desmin are positive. Which of the following is the most appropriate diagnosis?

  1. Angioleiomyoma
  2. Myopericytoma
  3. Glomus tumor
  4. Angiomyolipoma
Board review style answer #1
A. Angioleiomyoma. A mixture of smooth muscle bundles arranged in small fascicles and intervening vascular channels is noted. The concentric perivascular spindle cell proliferation which is characteristic for myopericytoma is absent. There are no glomus cells seen. Adipocytes are not seen in this image, which makes angiomyolipoma less likely.

Comment Here

Reference: Angioleiomyoma
Board review style question #2
    Which of the following modalities is the gold standard for the diagnosis of angioleiomyoma?

  1. Ultrasonography findings
  2. Dermoscopic findings
  3. MRI findings
  4. Microscopic examination
Board review style answer #2
D. Microscopic examination. Angioleiomyoma has no specific findings preoperatively. Microscopic findings are the gold standard for accurate diagnosis.

Comment Here

Reference: Angioleiomyoma

Angiolipoma
Definition / general
  • Benign, soft tissue tumor consisting of mature adipose tissue and clusters of thin walled vessels, often containing intraluminal fibrin thrombi
  • First described in 1912 by Bowen (Hum Pathol 1981;12:739)
  • Established as a distinct entity in 1960 by Howard and Helwig (Arch Dermatol 1960;82:924)
Essential features
  • Mature adipocytes
  • Clusters of capillary sized vessels
  • Fibrin thrombi in vessels
Terminology
  • Cellular angiolipoma (morphologic variant)
ICD coding
Epidemiology
  • Relatively common in young adults (Arch Dermatol 1960;82:924)
  • Usually appears in late second to early third decade of life
  • Rare in children or adults > 50 years
  • Male predominance
Sites
  • Forearm, trunk and upper arm common sites
  • Rare on scalp or face
  • Intramuscular hemangiomas represent different lesions (Arch Surg 1980;115:281)
  • So called angiolipomas of parenchymal organs and central nervous system represent different lesions containing larger vessels (World Neurosurg 2018;114:264)
Pathophysiology
  • Unknown
  • Association between sporadic angiolipoma and overlying cutaneous capillary malformation has been reported (Am J Clin Dermatol 2008;9:389)
Etiology
Clinical features
  • Frequently present as multiple subcutaneous small nodules
  • Nodules are usually tender or painful
  • There is no correlation between degree of vascularity and intensity of pain (Hum Pathol 1981;12:739)
Diagnosis
  • Tender, often multiple subcutaneous nodules (Arch Dermatol 1960;82:924)
  • Mature adipocytes
  • Clusters of capillary sized vessels
  • Intraluminal fibrin microthrombi
Radiology description
  • Ultrasound (Skeletal Radiol 2012;41:1055):
    • Oval shaped
    • Well defined margins
    • Hyperechoic lesions compared to muscle
    • Color Doppler flow present in < 25% of cases
  • CT (Arch Pathol Lab Med 2017;141:862):
    • Heterogeneous appearance
    • Mixed fat and soft tissue density
  • MRI (Skeletal Radiol 2018;47:859):
    • Fatty component shows high signal intensity on T1 weighted images and signal suppression on fat suppressed images
    • Vascular component shows areas of low signal intensity on T1 weighted images and high signal intensity on fat suppressed T2 weighted images
Radiology images

Images hosted on other servers:

Gastric angiolipoma

Prognostic factors
Case reports
Treatment
Clinical images

Images hosted on other servers:
Familial multiple angiomatosis

Familial multiple angiomatosis

Soft nodular mass

Multilobular

Gross description
Gross images

Images hosted on other servers:

Lesion and its microscopic appearance

Microscopic (histologic) description
  • Consists of mature adipose tissue and branching capillary sized vessels
  • Thin walled vessels often contain bright pink fibrin thrombi
  • Vascularity is more prominent in the periphery (Hum Pathol 1981;12:739)
  • Well formed capillaries often have a lobular arrangement
  • Occasionally, few thin fibrous septae present
  • Proportion of adipocytes and vessels varies
  • No nuclear atypia in the adipocytic or vascular component
  • Endothelial cells are bland and uniform
  • Morphologic variant: cellular angiolipoma (vessels predominate)
Microscopic (histologic) images

Contributed by Laura Warmke, M.D. and Jeanne Meis, M.D.
Angiolipoma

Angiolipoma

Subcutaneous angiolipoma

Subcutaneous angiolipoma

Cluster of vessels

Cluster of vessels

Mature adipose tissue

Mature adipose tissue

Peripheral vessels

Peripheral vessels


Mature adipose tissue with vessels

Mature adipose tissue with vessels

Well circumscribed nodule

Well circumscribed nodule

Vascular component

Vascular component

Fibrin thrombi

Fibrin thrombi

Intraluminal fibrin thrombi

Intraluminal fibrin thrombi



Cellular angiolipoma
Cellular angiolipoma

Cellular angiolipoma

Circumscription at lesion's periphery

Circumscription at lesion's periphery

Prominent spindle cell component

Prominent spindle cell component

Fibrin thrombi

Fibrin thrombi

Myxoid stromal change

Myxoid stromal change

Virtual slides

Images hosted on other servers:
Angiolipoma

Angiolipoma

Subcutaneous angiolipoma

Subcutaneous angiolipoma

Cytology description
  • Not clinically relevant
Immunofluorescence description
  • Fluorescein labeled antihuman fibrinogen demonstrates intense, irregularly homogeneous fluorescence of the fibrin thrombi (Hum Pathol 1981;12:739)
Positive stains
  • Immunohistochemical stains generally not needed for diagnosis (Arch Pathol Lab Med 2017;141:862)
  • S100 protein (focal to diffuse positivity in adipocytes)
  • ERG (vascular component)
  • CD31 (vascular component)
  • CD34 (vascular component)
  • CD61 (platelet marker highlights thrombi)
Negative stains
Electron microscopy description
  • Adipose tissue with central lipid droplet surrounded by thin rim of cytoplasm
  • Capillaries lined by one or more endothelial cells
  • Decreased number of Weibel-Palade bodies in endothelial cells (Hum Pathol 1981;12:739)
  • Endothelial cells frequently surrounded by pericytes
  • Variable numbers of fibrin thrombi within capillaries
Molecular / cytogenetics description
Molecular / cytogenetics images

Images hosted on other servers:

Karyotype of angiolipoma

Videos

Angiolipoma

Sample pathology report
  • Skin and subcutaneous tissue, left forearm, excision:
    • Angiolipoma (see comment)
    • Comment: Sections show an oval, well circumscribed lesion composed of mature adipose tissue with clusters of capillary sized vessels. Multiple fibrin microthrombi are identified with the lumen of the vessels, supporting the above diagnosis.
Differential diagnosis
  • Lipoma:
    • Mature adipose tissue
    • Lacks vascular component and fibrin thrombi
    • Various translocations of chromosome 12q or 6p
    • May mimic fat predominant angiolipoma
  • Intramuscular hemangioma:
    • Previously known as infiltrating angiolipoma
    • May have fatty overgrowth
    • Large, infiltrative lesion
    • Located deep in muscle (not subcutaneous tissue)
    • Large thick walled vessels present
  • Spindle cell hemangioma:
    • Multiple painful subcutaneous nodules in extremities
    • Cavernous vascular spaces
    • Cellular zones of spindled and epithelioid endothelial cells
    • Large calcified thrombi (phleboliths)
  • Angiomyolipoma:
    • Spindle cells that are HMB45+ smooth muscle cells
    • Large caliber and thick walled vessels
    • Rare in subcutaneous tissue
  • Kaposi sarcoma (distinguish from cellular angiofibroma):
    • Spindled endothelial proliferation
    • Extravasated red blood cells
    • HHV8+ endothelial cells
    • Lack of encapsulation
    • PAS+ hyaline globules instead of fibrin thrombi
  • Angiosarcoma (distinguish from cellular angiofibroma):
    • Nuclear atypia in endothelial cells
    • Aggressive, infiltrative growth
    • Mitotic activity and necrosis frequent
    • Loss of pericytic (perivascular smooth muscle) cells
    • Lack of fibrin thrombi
Board review style question #1

What is the most common anatomic location for sporadic angiolipoma?

  1. Face
  2. Forearm
  3. Lower extremity
  4. Retroperitoneum
Board review style answer #1
B. Forearm

Comment Here

Reference: Angiolipoma
Board review style question #2

Which of the following histologic features is associated with angiolipoma?

  1. Bundles of smooth muscle
  2. Cytologic atypia
  3. Fibrin thrombi
  4. Large, thick walled blood vessels
Board review style answer #2
C. Fibrin thrombi

Comment Here

Reference: Angiolipoma

Angiomatoid fibrous histiocytoma
Definition / general
  • Rare mesenchymal tumor of borderline malignant potential and uncertain histogenesis, characterized by solid nodules of epithelioid to spindled cells, blood filled pseudoangiomatous spaces, a fibrous pseudocapsule and a lymphoplasmacytic cuff toward the periphery
Essential features
  • Syncytial-like sheets of epithelioid to ovoid cells
  • Pseudoangiomatous blood filled spaces
  • Peripheral fibrous pseudocapsule and lymphoid tissue
Terminology
  • Originally described as angiomatoid malignant fibrous histiocytoma by Franz M. Enzinger in 1979 (Cancer 1979;44:2147)
ICD coding
  • ICD-O: 8836/1 - angiomatoid fibrous histiocytoma
  • ICD-11: 2F7C & XH9362 - neoplasms of uncertain behavior of connective or other soft tissue & angiomatoid fibrous histiocytoma
Epidemiology
  • Relatively rare entity, comprising 0.3% of all soft tissue tumors
  • More common in the first 2 decades of life without any sex predilection
  • Has been reported in a wide age range, extending from congenital presentation to the ninth decade (Arch Pathol Lab Med 2015;139:674)
  • Patients with extrasomatic soft tissue sites show higher mean age
Sites
  • Superficial soft tissues of extremities, followed by the trunk and the head and neck
Pathophysiology
  • Not known
Etiology
  • Originally described as a fibrohistiocytic tumor; however, WHO classification has kept it in the list of tumors of uncertain differentiation since no convincing ultrastructural or immunohistochemical evidence has been found to support this theory (Arch Pathol Lab Med 2015;139:674)
Clinical features
  • Predominantly presents as a slow growing, nontender, superficial, subcutaneous nodule in extremities, followed by the trunk and the head and neck
  • In the extremities, almost 66% of these tumors affect lymph node bearing regions, such as popliteal fossa, antecubital fossa, axilla, inguinal area and neck
  • Nonsomatic soft tissue sites of origin are increasingly being reported, including mediastinum, lung, bone, brain, gynecological tract, omentum and retroperitoneum (Mod Pathol 2011;24:1560, Br J Neurosurg 2021;35:233)
  • Often clinically mistaken with a vascular neoplasm or a hematoma
  • Constitutional symptoms are uncommon and are linked to cytokine production by tumor
Diagnosis
  • Diagnosis can be challenging due to nonspecific clinical, histological and immunohistochemical features compounded by rarity of this tumor
  • Key diagnostic histological features include:
    • Thick, fibrous pseudocapsule
    • Dense pericapsular cuff of lymphoplasmacytic cells
    • Presence of pseudoangiomatous / pseudovascular spaces
    • Syncytial proliferation of bland epithelioid to spindle shaped cells
    • Supportive immunohistochemical features: desmin+, CD99+, EMA+, CD68+
  • Reference: Arch Pathol Lab Med 2015;139:674
Radiology description
  • Circumscribed, lobulated mass
  • Cystic change with multiple areas of fluid - fluid levels
  • Signal characteristics and enhancement are presumed to be variable subject to the cellular, myxoid, collagenous and vascular content of the lesion
  • Heterogeneous intermediate to low signal intensity in T1 weighted image (T1WI) and heterogeneous high signal intensity in T2 weighted / short T1 inversion recovery (STIR) with the hemorrhagic component
  • After administration of IV contrast, enhancement of the soft tissue component within the lesion is seen with no enhancement of cystic components (Cureus 2021;13:e16465)
Radiology images

Images hosted on other servers:

MRI features

Prognostic factors
  • Morphological features of this tumor cannot predict the outcome
  • Most behave in an indolent fashion
  • Local recurrences have been reported in up to 15% of cases
  • Most recurrent cases show positive margins, deeper presentations and extrasomatic locations
  • < 5% metastasize, mostly to lymph nodes with exceptional cases of lung, liver and brain metastasis (BMC Musculoskelet Disord 2017;18:31)
Case reports
Treatment
  • Wide local excision with follow up is the treatment of choice
  • Irresectable and recurrent cases may be treated by adjuvant radiotherapy and or chemotherapy (Arch Pathol Lab Med 2015;139:674)
Clinical images

Images hosted on other servers:

Recurrent tumor with nodal metastasis

Gross description
  • Nodular and multicystic masses having grayish white to yellow cut surface with hemorrhagic areas / blood filled cysts
  • Median size is 2 cm (size range: 0.7 - 12 cm)
  • Reference: BMC Musculoskelet Disord 2017;18:31
Gross images

Contributed by Nasir Ud Din, M.B.B.S.

Lobulated lesion with dark brown cut surface

Microscopic (histologic) description
  • Thick, fibrous pseudocapsule with a dense pericapsular cuff of lymphoplasmacytic cells (with or without lymphoid follicles), seen in up to 80% of cases
  • Pseudocapsule imparts a low power resemblance to a lymph node involved by a metastatic tumor; absence of subcapsular sinus helps differentiate it from a lymph node
  • Circumscribed, often lobulated with syncytial sheets, short fascicles or whorls of epithelioid (histiocyte-like) to spindle shaped cells with moderate amount of eosinophilic cytoplasm, mildly pleomorphic vesicular nuclei and inconspicuous nucleoli
  • Presence of pseudoangiomatous / pseudovascular spaces or blood filled lakes lined by tumor cells is an important diagnostic feature but may be absent in up to 33% of cases showing a solid growth (Arch Pathol Lab Med 2015;139:674)
  • Mitotic activity is usually low
  • Interspersed inflammatory cells are commonly seen
  • Hemosiderin deposition may be prominent
  • May have areas of nuclear pleomorphism and atypical mitoses
  • Morphological variations:
    • Small round blue cell tumor-like morphology
    • Myxoid change particularly in intracranial ones (World Neurosurg 2019;126:113)
    • Pulmonary edema-like pattern
    • Microcystic pattern
    • Myxoid chondrosarcoma-like pattern (cord-like arrangement against myxoid stroma)
    • Marked sclerosis
    • Amianthoid fibers
    • Rhabdoid features (Brain Tumor Pathol 2021;38:138)
    • Schwannoma-like features (nuclear palisading and hyalinized vessels)
Microscopic (histologic) images

Contributed by Nasir Ud Din, M.B.B.S. and AFIP images

Lobulation with inflammatory cuff

Lobulation with capsule

Epithelioid to spindle cells

Spindle cells with hemosiderin

Histiocytoid morphology


Whorling pattern

Hemosiderin deposition

Cytological atypia

Chronic inflammatory cells


Resembles out of place lymph node

Round / oval nuclei

Moderate pleomorphism

Desmin

CD99

Cytology description
  • Cytological diagnosis of angiomatoid fibrous histiocytoma is challenging due to its rarity and nonspecific features
  • Smears are usually moderately cellular and show spindle shaped to ovoid (histiocyte-like to fibroblast-like) cells in cohesive clusters or dispersed cells (Diagn Cytopathol 2021;49:E36)
  • Nuclei show mild to moderate pleomorphism with vesicular chromatin
  • Nuclear grooves or intranuclear inclusions may be present
  • Background shows blood with lymphocytes and plasma cells
Positive stains
Molecular / cytogenetics description
  • Most cases show FET family gene rearrangement (mostly EWSR1 or less commonly FUS gene rearrangement)
  • FISH for EWSR1 rearrangement is useful diagnostically
  • Partner gene is mostly from the CREB family (CREB1, ATF1 or CREM)
  • Any of the following 3 characteristic translocations can be seen:
  • Some intracranial myxoid spindle cell tumors with EWSR1::CREB family gene fusion are described, which lack the features typical of angiomatoid fibrous histiocytoma; it is not clear whether they are subtypes of angiomatoid fibrous histiocytoma or belong to a novel entity (Brain Pathol 2018;28:183)
Molecular / cytogenetics images

Images hosted on other servers:

FISH and cytogenetic analysis

Videos

Brief overview of morphology

Sample pathology report
  • Right upper thigh, wide local excision and inguinal lymph node dissection:
    • Angiomatoid fibrous histiocytoma (see comment)
    • Tumor size: 5 x 4 x 2.5 cm
    • Lymphovascular invasion: absent
    • Margins of excision: all margins of excision are tumor free (give distances from each margin)
    • Skin: tumor free
    • Lymph nodes: all 06 lymph nodes are tumor free
    • Comment: Angiomatoid fibrous histiocytoma is considered as a borderline malignancy of uncertain histogenesis. It may recur in up to 15% cases and rarely can metastasize. Clinical follow up at adequate interval is advised.
    • FISH studies to detect EWSR1 rearrangement may be helpful for confirmation of diagnosis (optional comment for difficult cases)
Differential diagnosis
  • Organized hematoma:
    • No atypical spindle cells
    • No pseudocapsule or lymphoplasmacytic cuff
    • Negative for desmin
  • Nodal metastasis:
    • Metastatic melanoma in particular can mimic angiomatoid fibrous histiocytoma
    • Arises in regions populated by lymph nodes
    • True nodal architecture present
    • Subcapsular and medullary sinuses
    • History of malignancy may or may not be present
    • Relevant IHC may help
  • Granulomatous inflammation in a lymph node:
    • Lacks solid growth pattern of angiomatoid fibrous histiocytoma
    • Granulomas are usually dispersed and discrete or irregular and confluent
    • Large granulomas often show central necrosis and lack cystic hemorrhage
    • May contain foreign body type giant cells
    • Predisposing factors may be noted clinically (exposure to infectious agents, etc.)
    • CD68+, CD168+
    • Negative for desmin
  • Aneurysmal benign fibrous histiocytoma:
    • Morphologic variant of benign fibrous histiocytoma (dermatofibroma)
    • Dermal localization, often with overlying epidermal changes
    • No pseudocapsule or lymphoplasmacytic cuff
    • Peripheral collagen trapping often present
    • Mixed cell population, including chronic inflammatory cells, siderophages and giant cells
    • Negative for desmin
  • Follicular dendritic cell sarcoma:
    • Most arise in lymph nodes
    • Cervical lymph nodes are more commonly affected
    • Infiltrating lymphocytes are common
    • Binucleation and multinucleation often seen
    • CD21+, CD23+, CD35+, EMA+
    • Negative for desmin
  • Kaposi sarcoma (nodular):
    • May involve lymph nodes
    • Predisposing factors often present (AIDS history)
    • Moderately cellular neoplasm of spindled endothelial cells
    • CD34+, CD31+, ERG+, D2-40+, HHV8+
    • Negative for desmin
  • Spindle cell hemangioma:
    • Distal extremities, particularly acral
    • No pseudocapsule or lymphoplasmacytic cuff
    • Resembles cavernous hemangioma with cellular septa composed of spindle cells
    • CD31+, CD34+, ERG+, SMA+
    • Negative for desmin
  • Inflammatory myofibroblastic tumor:
    • Mostly in abdominal soft tissue
    • No pseudocapsule or lymphoplasmacytic cuff
    • Lacks pseudovascular spaces
    • More commonly myxoid
    • Eosinophils and neutrophils are common
    • SMA+, ALK+ in 50 - 60%
  • Intranodal palisaded myofibroblastoma:
    • Occurs in lymph nodes, predominantly in inguinal region
    • Common in males
    • Fascicles of spindle cells with nuclear palisading
    • Amianthoid fibers are characteristic
    • Lacks pseudovascular spaces
    • SMA+; negative for desmin
Board review style question #1


A 13 year old boy presented with a left axillary swelling, gradually increasing in size over the period of 6 months. No other symptoms were reported. On examination, a 3 cm subcutaneous, cystic, freely mobile, nontender lump was identified in the medial wall of axilla. The lump was excised and sent for histopathology. The H&E and immunohistochemistry slides of this lump are shown above. What is the most likely diagnosis?

  1. Angiomatoid fibrous histiocytoma
  2. Follicular dendritic cell sarcoma of lymph node
  3. Inflammatory myofibroblastic tumor
  4. Kaposi sarcoma
  5. Spindle cell rhabdomyosarcoma
Board review style answer #1
A. Angiomatoid fibrous histiocytoma

Comment Here

Reference: Angiomatoid fibrous histiocytoma
Board review style question #2
Which of the following is true regarding angiomatoid fibrous histiocytoma?

  1. Associated with immunodeficiency
  2. Considered as a borderline malignancy
  3. Does not carry a risk of metastases
  4. Has a characteristic immunohistochemical profile
  5. More common in abdominal cavity
Board review style answer #2
B. Considered as a borderline malignancy

Comment Here

Reference: Angiomatoid fibrous histiocytoma

Angiomyofibroblastoma
Definition / general
Essential features
Terminology
ICD coding
  • ICD-10: D21.9 - benign neoplasm of connective and other soft tissue, unspecified
Epidemiology
Sites
Pathophysiology
Etiology
Clinical features
Diagnosis
Radiology description
Radiology images

Images hosted on other servers:

Ultrasound scan

Transvaginal / Doppler ultrasound

Sagittal section of MRI

MRI with gadolinium contrast

Case reports
Treatment
Clinical images

Images hosted on other servers:

Left vulvar mass

Gross description
Gross images

Images hosted on other servers:

Pendunculated,
well circumscribed,
nonencapsulated
tumor

Well demarcated, yellow-white mass

Frozen section description
  • Spindle cells or cords of epithelioid cells around abundant blood vessels with thin to thick vascular walls embedded in loose stroma (Heliyon 2020;6:e04123)
Frozen section images

Images hosted on other servers:

Cords of epithelioid to spindle cells

Microscopic (histologic) description
Microscopic (histologic) images

Contributed by Nasir Ud Din, M.B.B.S.

Numerous blood vessels

Variable cellularity

Myxoedematous stroma

Stromal cells around blood vessels


Oval to spindled cells

Bland cytology

Collagenous stroma

Desmin+

ER+

Negative stains
Electron microscopy description
Molecular / cytogenetics description
Videos

Histopathology

Sample pathology report
  • Vulva, wide local excision:
    • Angiomyofibroblastoma (see comment)
    • Comment: Histological examination reveals a well circumscribed, unencapsulated tumor with alternating hypercellular and hypocellular areas set in edematous to collagenous stroma. Numerous thin walled blood vessels and bland looking spindled to epithelioid cells, arranged singly or in small nests or cords concentrating around blood vessels are seen. No mitotic activity, nuclear atypia or necrosis is noted. It is a benign condition and has been completely excised.
Differential diagnosis
Board review style question #1

A 38 year old woman presented with a gradually growing nodule in left labium majus. On physical examination, there is a well demarcated, firm, subcutaneous mass protruding from the left labium majus. Radiology shows well demarcated subcutaneous mass. Histology is shown in the image above. Which of the following is the most likely diagnosis in this case?

  1. Aggressive angiomyxoma
  2. Angiomyofibroblastoma
  3. Cellular angiofibroma
  4. Epithelioid leiomyoma
  5. Superficial myofibroblastoma
Board review style answer #1
B. Angiomyofibroblastoma. The history of slow growing subcutaneous nodule in vulvovaginal areas in this middle aged woman suggests a soft tissue lesion. In this area, the most common mesenchymal tumors include aggressive angiomyxoma, angiomyofibroblastoma and cellular angiofibroma. The alternating hypocellular and hypercellular areas with prominent blood vessels and tumor cells concentrating around these vessels seen on the photomicrograph are characteristic features of angiomyofibroblastoma. In contrast, the aggressive angiomyxoma have low cellularity with abundant hyalinized blood vessels, extravasated red blood cells and plump stromal cell scattered in mucin rich stroma. Cellular angiofibroma and superficial myofibroblastoma also lack this characteristic, alternating cellularity pattern.

Comment Here

Reference: Angiomyofibroblastoma
Board review style question #2
A 27 year old woman noticed a small gradually increasing nodule in her vulvar region. Physical examination revealed a firm, well demarcated nodule in right labium majus. Reactivity of which of the following markers would help you to reach final diagnosis?

  1. Desmin
  2. Keratin
  3. NSE
  4. SMA
  5. S100
Board review style answer #2
A. Desmin. The differential diagnosis of gradually increasing subcutaneous nodules in vulvovaginal areas in young women includes site specific mesenchymal tumors (i.e., aggressive angiomyxoma, angiomyofibroblastoma and cellular angiofibroma). The alternating areas of cellularity with prominent blood vessels and tumor cells concentrating around these vessels as seen on photomicrographs are characteristic features of angiomyofibroblastoma. This tumor commonly shows reactivity for desmin and ER / PR receptors. They are negative for keratin, NSE and S100, while SMA can be rarely positive.

Comment Here

Reference: Angiomyofibroblastoma

Angiosarcoma
Definition / general
  • Angiosarcoma is a malignant neoplasm showing morphological or immunophenotypic evidence of endothelial differentiation
Essential features
  • Irregularly shaped anastomosing vascular channels or sheet-like growth
  • Highly infiltrative architecture and poor demarcation
  • Multilayering of endothelial cells, nuclear atypia, increased mitoses, necrosis
  • Positive for vascular markers, such as CD31, CD34, ERG, VEGF and factor VIII, by immunohistochemistry
ICD coding
  • ICD-O: 9120/3 - Angiosarcoma
  • ICD-11: 2B56.Y & XH6264 - Angiosarcoma, other specified primary site and hemangiosarcoma
Epidemiology
Sites
  • Most common sites of angiosarcoma are skin of the head and neck (about 60% of cases); can also present within the soft tissues, visceral organs, bone and retroperitoneum
  • Skin:
    • Cutaneous angiosarcoma is the most common form, typically primary and occurring in sun exposed areas of older adults, with secondary cases less common (Ann Diagn Pathol 2011;15:93, Curr Probl Cancer 2015;39:258)
    • Head and neck are the most common locations for cutaneous angiosarcoma, with rare occurrence on the trunk and extremities
  • Breast:
    • Primary angiosarcoma developing de novo
    • Secondary angiosarcoma developing as a consequence of previous breast cancer treatment (e.g. postoperative radiotherapy or chronic lymphedema after treatment for breast cancer, associated with Stewart-Treves syndrome)
    • They display varying degrees of nuclear atypia, hyperchromatic nuclei, enlarged nucleoli and frequent mitoses (BMC Cancer 2018;18:463)
  • Extremities:
    • Tumor can arise 5 - 15 years after radical mastectomy and axillary lymph node dissection associated from chronic lymphedema (Stewart-Treves syndrome)
  • Kidney:
    • Primary renal angiosarcoma, also referred to as renal hemangiosarcoma, is an extremely rare neoplasm
    • To date, about 60 primary renal angiosarcomas have been described in the literature
    • Most cases occur in the sixth and seventh decades (Curr Urol Rep 2018;19:4)
  • Liver:
    • Hepatic angiosarcoma is a rare malignancy accounting for up to 2% of primary hepatic neoplasms (Am J Surg Pathol 2019;43:581)
    • Associated with vinyl chloride, arsenic and thorium dioxide (Thorotrast)
  • Lung:
    • Primary pulmonary angiosarcoma is rare and can present with respiratory symptoms such as hemoptysis, cough and dyspnea
    • Less than 30 cases have been reported in the literature
    • Has a very poor prognosis (Thorac Cancer 2016;7:607)
  • Spleen:
    • Rare and aggressive neoplasm with a high metastatic rate and poor prognosis
    • Presents with left upper quadrant pain and systemic symptoms such as GI bleeding, splenomegaly and hemoperitoneum due to splenic rupture (J Surg Oncol 2005;92:312)
  • Small intestine:
    • Primary angiosarcoma of the small intestine is extremely rare with a very poor prognosis
  • Ovary:
    • Patients with ovarian angiosarcomas most commonly present with abdominal pain; however, some patients present with distant metastases, often to the lungs
    • Tumor can spread beyond the ovary at the time of initial diagnosis in most reported cases with disease progression within less than a year after diagnosis (J Med Case Rep 2014;8:47)
  • Salivary glands:
    • Angiosarcomas of the salivary glands and oral cavity are extremely rare and comprise only 2% of all angiosarcomas
    • Prognosis of salivary gland angiosarcoma is not known due to the paucity of cases in the literature (Head Neck Pathol 2020;14:837)
Pathophysiology
  • Pathogenesis of angiosarcoma is characterized by a rapid and extensive infiltrating overgrowth of vascular endothelial cells
  • Angiosarcoma is a locally aggressive tumor with a high rate of lymph node infiltration and metastases
  • Angiosarcoma has been shown to have upregulation of vascular specific receptor tyrosine kinases, including TIE1, KDR, TEK and FLT
  • Upregulation of these genes and overexpression of VEGFR can cause endothelial cell expansion, angiogenesis and also vascular leaks
  • KDR mutations are seen in primary breast angiosarcoma regardless of exposure to radiation
  • c-MYC amplification is seen in radiation induced and lymphedema associated angiosarcoma
  • FLT4 amplification has been detected in 25% of secondary angiosarcomas
Etiology
  • Etiology is unknown in most cases
  • A minority arise after radiation exposure or longstanding lymphedema (Stewart-Treves syndrome)
  • Angiosarcoma is associated with exposure to vinyl chloride, arsenic and thorium dioxide (Thorotrast)
  • Small numbers occur in association with implanted foreign material, in pre-existing hemangioma / vascular malformation and in regions of prior trauma or surgery
  • Angiosarcomas occur in patients with certain syndromes (neurofibromatosis and Maffucci sundrome) and rarely as heterologous components of other neoplasms (benign or malignant nerve sheath tumors) (Cancer 1988;62:2436, Histopathology 1999;35:114, Am J Surg Pathol 2002;26:1319, Am J Nephrol 2011;34:42)
Clinical features
  • Patients typically present with a blue or purple lesion on the scalp or face that has been present for several months and can be rapidly growing
  • These lesions may appear macular, nodular or plaque-like
  • Diffuse, clinically undetectable intradermal spread leads to indistinct borders and a high incidence of multicentricity
  • Advanced lesions can show hemorrhage or ulceration
  • Cervical lymphadenopathy is found in approximately 10% of patients at the time of presentation
Diagnosis
  • Clinical presentation:
    • Angiosarcomas often look like a bruised, purpuric area on the skin which can bleed easily when traumatized
    • They grow larger over time and the skin around the bruised area can swell, accompanied by pain
  • Imaging: MRI, CT or PET scan to assess tumor size and anatomic localization
  • Biopsy of the lesion for microscopic examination
Laboratory
  • Laboratory studies are usually unremarkable and not unique to angiosarcoma, unless mass effect causes compression of critical organs, leading to laboratory abnormalities (such as compression of the ureter, which can lead to renal failure) or the disease is fairly advanced to cause subtle lab abnormalities (such as anemia of chronic disease and elevated sedimentation rate)
  • There are no specific lab abnormalities or biomarkers that could point towards a diagnosis of angiosarcoma
Radiology description
  • Head and neck:
    • MRI shows intermediate T1 signal intensity with possible areas of hyperintensity, indicating the presence of hemorrhage (Br J Radiol 2017;90:20170039)
  • Soft tissue:
  • Breast:
  • Liver:
    • Consistent with an aggressive vascular tumor
    • Predominantly hypoattenuating mass with or without hyperattenuating foci on unenhanced CT with heterogenous enhancement, indicating necrosis and fibrosis on contrast enhanced CT (AJR Am J Roentgenol 2000;175:165)
  • Bone:
Prognostic factors
Case reports
Treatment
  • Surgical resection is the main treatment modality, in addition to chemotherapy and radiotherapy
  • Targeted therapy and immunotherapy have been studied as a promising new treatment of angiosarcoma
Clinical images

Contributed by Huong Giang Tran, M.D., Ph.D. and Yu-Hung Wu, M.D.
Angiosarcoma on the forehead Angiosarcoma on the forehead

Angiosarcoma on the forehead

Angiosarcoma on the foot

Angiosarcoma on the foot

Angiosarcoma on the scalp

Angiosarcoma on the scalp

Gross description
  • Skin:
    • Purple or maroon nodules or plaques with hemorrhagic or necrotic cut surface
  • Breast:
    • Lesions can be cutaneous or intraparenchymal
    • Cutaneous lesion is grossly similar to other cutaneous forms in setting of postsurgical lymphedema associated with history of irradiation to breast or chest wall
    • Intraparenchymal lesion is either solitary or multiple masses in breast
    • Overlying skin may present with purple discoloration
  • Deep soft tissue:
    • Lesion most likely presents as a slow growing, painful and large mass associated with coagulopathy (in 33% of cases)
Gross images

Contributed by Zan Ahmed, M.D. and Thuy L. Phung, M.D., Ph.D.
Angiosarcoma on the foot

Angiosarcoma on the foot

Liver angiosarcoma

Liver angiosarcoma

Cut surface of liver angiosarcoma

Cut surface of liver angiosarcoma

Microscopic (histologic) description
  • Angiosarcoma has a wide morphologic appearance, ranging from lesions that are cytologically bland and vasoformative, to solid sheets of highly pleomorphic cells without definitive vasoformation
  • Numerous irregularly shaped anastomosing vascular channels lined by atypical endothelial cells with a highly infiltrative architecture and poor demarcation
  • Tumor cells are typically plump, pleomorphic and mitotically active
    • They can be spindle shaped, polygonal, epithelioid and primitive round cells, forming papillae or solid nests within vascular lumina
  • Tumor vessels ramify the dermis and intercalate through dermal collagen and subcutaneous soft tissues
  • Because of the heterogeneous histologic features in poorly differentiated tumors, the histological identification of an angiosarcoma can be challenging
  • Solid growth pattern often presents in poorly differentiated angiosarcoma
  • Intratumoral hemorrhage is common
  • Stromal lymphoid aggregates may also be present
Microscopic (histologic) images

Contributed by Zan Ahmed, M.D., S. Shawn Liu, M.D., Ph.D. and Thuy L. Phung, M.D., Ph.D.
Cutaneous angiosarcoma

Cutaneous angiosarcoma

Anastomosing vascular channels Anastomosing vascular channels

Anastomosing vascular channels

Atypical endothelial cells

Atypical endothelial cells

Solid pattern of angiosarcoma

Solid pattern of angiosarcoma

Multiple mitotic figures

Multiple mitotic figures


CD31 immunostain CD31 immunostain

CD31 immunostain

D2-40 immunostain D2-40 immunostain

D2-40 immunostain

Solid pattern of angiosarcoma Solid pattern of angiosarcoma

Solid pattern of angiosarcoma


Solid pattern of angiosarcoma

Solid pattern of angiosarcoma

CD31 immunostain

CD31 immunostain

CD34 immunostain CD34 immunostain

CD34 immunostain

Ki67 immunostain Ki67 immunostain

Ki67 immunostain


Epithelioid angiosarcoma Epithelioid angiosarcoma Epithelioid angiosarcoma

Epithelioid angiosarcoma

CD31 immunostain CD31 immunostain

CD31 immunostain




Contributed by Kyle Haggerty (M.D. candidate) and David Loeffler, D.O. (Case #509)

Epithelioid angiosarcoma, involving the adrenal gland


CD31

CK7

CKCKT

ERG

SF1

Positive stains
  • Angiosarcoma expresses endothelial cell markers, including CD31, CD34, ERG, FLI1, VEGF and factor VIII
  • Cytokeratin, EMA and CD30 may be expressed in a subset of epithelioid angiosarcoma
  • Lymphatic marker podoplanin as determined by D2-40 immunostain is variably expressed and suggests focal lymphatic differentiation in the tumor
Negative stains
  • Tumor is negative for HHV8 (human herpesvirus 8, a Kaposi sarcoma biomarker)
Electron microscopy description
  • Weibel-Palade bodies, which are long, cylindrical, rod shaped bodies with a single membrane and containing microtubules in matrix, can be identified in the cytoplasm of endothelial cells
Electron microscopy images

Contributed by Guillermo A. Herrera, M.D.
Weibel-Palade body Weibel-Palade body

Weibel-Palade body

Molecular / cytogenetics description
  • Angiosarcoma is characterized by upregulation of vascular specific receptor tyrosine kinases, such as TIE1, KDR, TEK and FLT (Curr Urol Rep 2018;19:4)
  • Has overexpression of HIF1 alpha and HIF2 beta, which are upstream regulators of VEGF (Curr Urol Rep 2018;19:4)
  • High level of c-MYC gene amplification by FISH is seen in radiation induced and chronic lymphedema associated angiosarcoma (Curr Urol Rep 2018;19:4)
  • Some tumors harbor recurrent PTPRB and PLCG1 mutations, which are involved in angiogenesis (Curr Urol Rep 2018;19:4)
Sample pathology report
  • Skin, superior medial right forehead, biopsy:
    • Superficial portion of an angiosarcoma, transected at the base (see comment)
    • Comment: Sections show a dense and hypercellular dermal proliferation of spindle shaped and epithelioid malignant cells with enlarged pleomorphic and hyperchromatic nuclei with speckled chromatin pattern. The tumor is highly vascularized with numerous irregularly shaped blood vessels and hemorrhagic areas. Foci of tissue necrosis are present. The tumor stroma is desmoplastic and edematous with abundant lymphocytic infiltrates. There are numerous mitotic figures in the tumor. The tumor cells are strongly positive for the vascular markers CD31 and CD34 and for the lymphatic marker D2-40. Ki67 immunostain shows high cell proliferative activity throughout the tumor. The tumor cells are negative for S100, MART1, SMA (smooth muscle actin), pancytokeratin AE1 / AE3 and desmin. Immunostains and H&E findings support the diagnosis of an angiosarcoma.
Differential diagnosis
  • Atypical fibroxanthoma:
    • Admixture of spindle cells, histocyte-like cells, xanthomatous cells and multinucleated giant cells
    • Marked cytological atypia but without vascular differentiation
    • A study shows over 33% of cases express CD31, which may lead to erroneous diagnosis of angiosarcoma
    • Negative for CD34 and ERG
  • Hemangioma:
    • Well demarcated lobular architecture with well established vascular channels and single luminal lining of benign endothelial cells
  • Kaposi sarcoma:
    • Patch stage shows a mild dermal vascular proliferation with minimal endothelial atypia
    • Associated with lymphocytes or plasma cell infiltration and hemosiderin deposits
    • At the plaque stage, dermal vascular proliferation varies in size and is associated with eosinophilic spindle cells, instead of lymphocytes or plasma cells
    • Nodular stage presents predominantly as a well circumscribed dermal mass of eosinophilic spindle cells that often lack endothelial lining
    • Nodular stage is associated with red blood cell extravasation
    • Positive for HHV8 (human herpesvirus 8)
  • Retiform hemangioendothelioma:
    • Resembles well differentiated angiosarcoma
    • Presents in the extremities of young patients with slit-like space lined by hobnail neoplastic endothelial cells
    • No significant cytological atypia
  • Kaposiform hemangioendothelioma:
    • Consists of fascicles of spindle shaped endothelial cells, congested capillaries, slit-like vascular spaces in lobular and infiltrative patterns
    • Areas of spindle shaped cells resemble Kaposi sarcoma
    • Negative for HHV8
  • Epithelioid hemangioendothelioma:
    • Round, polygonal cells with eosinophilic cytoplasm and vesicular nuclei arranged in short cords or nests
    • Surrounded by hyaline and myxoid stroma
  • Spindle cell hemangioma:
    • Biphasic tumor
    • Admixture of thin walled, congested cavernous vascular spaces and solid areas of spindled to epithelioid endothelial cells
    • Large, calcified thrombi may be present
Board review style question #1

A 78 year old man presents with a rapidly growing purpuric and nodular cutaneous lesion on the left forehead. The H&E histological images of the biopsy of the lesion are shown above. What is the most likely diagnosis of this lesion?

  1. Angiosarcoma
  2. Epithelioid hemangioendothelioma
  3. Hemangioma
  4. Kaposi sarcoma
  5. Targetoid hemangioma
Board review style answer #1
A. Angiosarcoma

Comment Here

Reference: Angiosarcoma
Board review style question #2
Which of the following immunohistochemical stains is most likely negative for angiosarcoma?

  1. CD31
  2. CD34
  3. D2-40
  4. ERG
  5. HHV8
Board review style answer #2
E. HHV8

Comment Here

Reference: Angiosarcoma
Board review style question #3
Which stain will help differentiate epithelioid angiosarcoma from a poorly differentiated adrenal cortical carcinoma?

  1. ERG
  2. Pancytokeratin
  3. SF1
  4. VEGFR3
  5. Vimentin
Board review style answer #3
A. ERG. ERG is sensitive and specific for vascular differentiation. Although not on the list of potential answers, CD31 and Factor VIII are also sensitive and specific markers for vascular differentiation and can additionally be useful in this case. Steroidogenic factor 1 (SF1) is a transcription factor that has a role in steroidogenesis and is expressed in the adrenal cortex and other gonadotrophic cells in the pituitary and gonads. While usually a very specific marker, its expression may be lost in up to 12% of adrenal cortical carcinomas (Hum Pathol 2013;44:822). Both epithelioid angiosarcoma and carcinoma will stain positive for pancytokeratin. While VEGFR3 is specific for vascular differentiation, it is less sensitive and will have variable staining in angiosarcoma. While vimentin is quite sensitive for angiosarcoma, it may not be helpful because carcinomas may also stain positive for vimentin.

Comment Here

Reference: Angiosarcoma

Arteriovenous malformation
Definition / general
  • Arteriovenous malformation (AVM) is a fast flow vascular anomaly characterized by abnormal arteriovenous communications and malformed arteries, veins and capillaries
  • These are usually congenital but may be acquired later in life
Essential features
  • Angiography essential in diagnosis
  • Arteriovenous shunting may induce localized hypertrophy, coagulopathy or heart failure
  • Clinically, the lesions are classically red, warm and pulsatile
  • AVMs are usually sporadic but can be seen in several syndromic settings
  • Histology demonstrates an admixture of malformed arteries, thickened veins and capillaries
  • Arteriovenous (AV) shunts are rarely demonstrated histologically as they generally require many serial sections
Terminology
  • Arteriovenous hemangioma (no longer recommended by International Society for the Study of Vascular Anomalies)
  • Intramuscular hemangioma, Enzinger type (intramuscular fast flow vascular anomaly is now considered a subtype of AVM)
ICD coding
  • ICD-11: LA90.3Z - peripheral arteriovenous malformations, unspecified
Epidemiology
  • Most AVMs are congenital lesions and are recognized at birth (Angiogenesis 2019;22:547, Circulation 2017;136:1037)
  • Some do not become apparent until later in life
  • Syndrome associated AVMs, such as those in hereditary hemorrhagic telangiectasia (HHT), may not become apparent for several decades (Circulation 2017;136:1037)
  • Syndromes associated with AVMs include Parkes Weber syndrome (PWS), capillary malformation / arteriovenous malformation 1 and 2 (CM-AVM1 / 2) and hereditary hemorrhagic telangiectasia (HHT)
Sites
  • Sporadic AVM can involve any site in the body, although seem to be most common in the head and neck
  • Those occurring in the setting of PWS almost invariably involve a lower extremity
  • Multiple small AVMs are often seen in the setting of HHT
Pathophysiology
  • Abnormal RAS / MAPK and or PI3K / mTOR / AKT pathway signaling due to a mutation in 1 or more associated genes leads to abnormal vascular morphogenesis
Etiology
Diagrams / tables
Not relevant to this topic
Clinical features
Repetitive with above
Diagnosis
  • Per WHO, it is essential to demonstrate malformed large vascular channels with an increased small vessel component in correlation with clinical and imaging features
  • Demonstration of somatic MAP2K1 (or other) mutations is desirable
Laboratory
Not relevant to this topic
Radiology description
  • Arteriovenous malformations are fast flow vascular malformations characterized by abnormal connections between feeding arteries and draining veins (Semin Pediatr Surg 2014;23:203)
Radiology images
Not applicable
Prognostic factors
  • Clinical significance largely predicated by degree of arteriovenous shunting
  • Syndrome associated AVMs have unique clinical associations dictated by the specific syndrome (HHT, PWS, etc.)
Case reports
Treatment
  • Embolization
  • Resection
  • Targeted inhibition of RAS / MAPK / ERK pathway
Clinical images
None applicable
Gross description
No information provided
Gross images

Contributed by Harry P. Kozakewich, M.D. and Jonathan C. Slack, M.D.
Cut surface of resected AVM Cut surface of resected AVM

Cut surface of resected AVM

Frozen section description
Not relevant to this topic
Frozen section images
Not relevant to this topic
Microscopic (histologic) description
  • Per WHO, it is essential to demonstrate malformed large vascular channels with an increased small vessel component in correlation with clinical and imaging features
  • There are no applicable grading / staging criteria
Microscopic (histologic) images

Contributed by Jonathan C. Slack, M.D. and @Andrew_Fltv on Twitter
Arteriovenous shunt

Arteriovenous shunt

Intramuscular fast flow vascular anomaly Intramuscular fast flow vascular anomaly

Intramuscular fast flow vascular anomaly

Untreated

Untreated

Treatment effect

Treatment effect


Elastin stain (VVG) Elastin stain (VVG)

Elastin stain (VVG)

Smooth muscle actin

Smooth muscle actin

Virtual slides
None applicable
Cytology description
None applicable
Cytology images
None applicable
Immunofluorescence description
None applicable
Immunofluorescence images
None applicable
Positive stains
Not applicable
Negative stains
Not applicable
Electron microscopy description
Not applicable
Electron microscopy images
Not applicable
Molecular / cytogenetics description
Not applicable
Molecular / cytogenetics images
Not applicable
Videos
Not applicable
Sample pathology report
  • Soft tissue, left scalp, excision:
    • Vascular anomaly, consistent with arteriovenous malformation in the appropriate clinical setting (see comment)
    • Comment: The examined soft tissues are involved by an infiltrative vascular anomaly consisting of a haphazard proliferation of malformed arteries, veins, capillaries and indeterminate channels. In light of the radiologic findings of a fast flow vascular anomaly, the findings are consistent with the clinical impression of arteriovenous malformation. If clinically indicated, ancillary molecular studies could be pursued on remaining formalin fixed paraffin embedded tissue.
Differential diagnosis
Board review style question #1

This red, warm, pulsatile forehead soft tissue mass has been present since birth and recently enlarging in this 17 year old boy. What is the diagnosis?

  1. Arteriovenous malformation
  2. Congenital hemangioma
  3. Pyogenic granuloma
  4. Venous malformation
Board review style answer #1
A. Arteriovenous malformation. The image shows an arteriovenous malformation (AVM) composed of an admixture of abnormal arteries, veins and small vessels. The clinical history of a red, warm, pulsatile congenital lesion is consistent with AVM. Answer C is incorrect because pyogenic granulomas lack abnormal arteriovenous connections, are not typically congenital and are nonpulsatile. Answer D is incorrect because venous malformations are nonpulsatile, cool to touch, white-blue and lack an arterial component. Answer B is incorrect because congenital hemangiomas are typically well circumscribed, may involute rapidly after birth, are not warm or pulsatile and lack abnormal arteriovenous connections.

Comment Here

Reference: Arteriovenous malformation

Arteriovenous malformation (pending)
[Pending]
Microscopic (histologic) images

Contributed by @Andrew_Fltv on Twitter

Atypical lipomatous tumor / well differentiated liposarcoma
Definition / general
  • Locally aggressive mesenchymal tumor composed of mature adipocytes and stromal cells with at least focal cytologic atypia
  • Adipocytic component and background cellular constituents vary in concentration and lead to several recognizable morphologic subtypes
Essential features
  • Low grade lipogenic tumor with multiple morphologic subtypes and significant histologic variability
  • Molecularly characterized by ring or giant marker / rod chromosomes composed of material from 12q13-15
    • Results in localized amplification of several neighboring genes, including MDM2
  • Behavior is dependent upon location, with deep seated lesions having the ability to dedifferentiate and subsequently metastasize
  • Terminology is based on location (see Terminology)
  • Wide local excision with negative margins is often curative
Terminology
  • Use of the terminology atypical lipomatous tumor (ALT) and well differentiated liposarcoma (WDL) is based on a tumor's location in the body and primarily relates to resectability
    • Tumors are morphologically and genetically identical, with the variation in terminology intended to avoid both undertreatment and overtreatment
  • Tumors located in the periphery have no risk of metastasis
    • For such lesions, complete resection is generally curative
    • For these tumors, the designation as sarcoma is inappropriate and the term ALT is preferred
  • If the tumor is deep seated (retroperitoneum, mediastinum, spermatic cord), the chance of achieving negative margins is significantly diminished and the risk of local recurrence, dedifferentiation (DDL) and death are increased
    • Lesions are best regarded as true sarcomas and the terminology of well differentiated liposarcoma is more appropriate
  • Suggestion: use of atypical lipoma is not recommended
ICD coding
  • ICD-O:
    • 8850/1 - atypical lipomatous tumor
    • 8851/3 - liposarcoma, well differentiated
  • ICD-11:
    • 2F7C & XH0RW4 - neoplasms of uncertain behavior of connective or other soft tissue & atypical lipomatous tumor
    • 2B5H & XH7Y61 - well differentiated lipomatous tumor, primary site & liposarcoma, well differentiated
Epidemiology
Sites
  • Most common site is the deep thigh of the lower extremity, followed by retroperitoneum, trunk, head and neck region and spermatic cord
  • Numerous anatomic sites have been reported, including the subcutis and skin
Diagnosis
  • Diagnosis typically requires surgical resection to ensure adequate sampling, as diagnostic accuracy is limited by sampling bias on core needle biopsy (AJR Am J Roentgenol 2021;216:997)
Radiology description
  • Fat tissue density mass with thick or nodular enhancing septa on CT imaging
  • Mass of fat isointense signal in all sequences, thick septa or nodular nonlipomatous areas with contrast enhancement on MRI imaging
  • Retroperitoneal WDL typically shows fat with an abnormal appearance on MRI (stranding) that represents the thick fibrous bands
  • Reference: Radiographics 2005;25:1371
Prognostic factors
  • Most important factor is anatomic location
  • Tumors do not metastasize unless they dedifferentiate, which is associated with significantly shorter overall survival (Am J Surg Pathol 2007;31:1)
  • Subcutaneous or intramuscular tumors may recur but typically do not dedifferentiate or metastasize
  • Risk of dedifferentiation is directly related to location and duration of growth
Case reports
Treatment
  • Surgical resection with widely negative margins is generally curative
  • Retroperitoneal or central body site tumors are difficult to resect with a clear margin, frequently recur, can dedifferentiate and cause death (J Surg Res 2012;175:12)
  • Positive margin status and sclerosing subtype are associated with reduced local recurrence free survival (Ann Surg Oncol 2004;11:78)
Clinical images

Images hosted on other servers:

Pelvic tumor during surgery

Intraoperative image of axillary tumor

Gross description
  • Typically multilobulated and well circumscribed
  • Rarely are grossly infiltrative
  • Marbled yellow cut surface in lipoma-like lesions, with more firm / fibrotic white areas grossly in lesions with less adipocytic differentiation (Semin Diagn Pathol 2019;36:112)
  • Fat necrosis in large lesions, especially at the periphery
  • Sample carefully to look for dedifferentiated components
    • Dedifferentiated areas are nonlipogenic and can stand out as firm nodules or be more diffusely admixed with low grade areas
Gross images

Images hosted on other servers:

Mesenteric tumor

Pelvic tumor

Retroperitoneal tumor

Dedifferentiated tumor in small bowel mesentery

Microscopic (histologic) description
  • Depends on subtype, generally composed of mature fat with variably sized adipocytes and bands of fibrotic stroma containing spindle cells with enlarged, hyperchromatic nuclei
  • Can be markedly atypical
  • Cellularity is low and mitotic figures are uncommon
  • Atypical cells are more commonly found in fibrous septa and in a perivascular distribution
  • Rarely display heterologous differentiation (Hinyokika Kiyo 2010;56:697)
  • There are 3 histologic subtypes that have limited clinical significance (Semin Diagn Pathol 2001;18:258, Ann Surg Oncol 2004;11:78)
    • Lipoma-like subtype
      • Most common subtype
      • Scattered atypical cells may be diffuse or exceedingly rare
      • Frequently contains lipoblasts
      • Grossly, can be indistinguishable from lipoma
    • Sclerosing subtype
      • Second most common subtype
      • Predilection for retroperitoneal or paratesticular location
      • Collagenous fibrous tissue with scattered adipocytes and atypical multinucleated stromal cells
      • Scant lipogenic component may be missed in small samples
    • Inflammatory subtype
      • Rare
      • Almost always in the retroperitoneum where it is confused with nonlipogenic tumors
      • Chronic inflammatory cells (B > T cells) with occasional lymphoid follicles scattered in a cellular fibrocollagenous stroma with sparse multinucleated atypical cells
      • May obscure adipocytes
  • Mixed subtype
    • Not uncommon to see an admixture of these subtypes with sufficient sampling
  • Rarest variants
    • Lipoleiomyosarcoma
      • Liposarcoma with leiomyosarcomatous differentiation
      • Both components are low grade
      • Variable amount of the smooth muscle component, which can be seen in association with large vascular walls
      • Similar biology and can dedifferentiate
    • ALT / WDL with low grade osteosarcoma-like areas
      • In contrast to osseous metaplasia, these lesions have foci reminiscent of parosteal osteosarcoma or low grade central osteosarcoma (Am J Surg Pathol 2010;34:1361)
  • Pitfalls and tips
    • Nuclei with sharply outlined vacuoles (Lochkern cells) are enlarged and can appear hyperchromatic but are normal adipocytes cut in cross section
      • These do not qualify for the diagnosis; the same goes for atrophic skeletal muscle fibers, which can simulate atypia on an otherwise typical lipoma
    • Histiocytes in areas of fat necrosis can simulate atypical cells
    • Lipoblasts are neither necessary nor sufficient for diagnosis
      • When present they are generally multivacuolated versus the signet ring lipoblasts commonly seen in myxoid liposarcoma
    • Fatty differentiation may be very focal and difficult to differentiate from background adipocytes; when in doubt test molecularly (see Molecular / cytogenetics description)
    • Lipomas do occur in the retroperitoneum and the distinction with WDL may require molecular testing (BMC Res Notes 2015;8:75)
      • Retroperitoneal WDL typically shows fat with an abnormal appearance on MRI (stranding) that represents the thick fibrous bands
    • Avoid using subtypes as diagnoses (i.e., inflammatory liposarcoma); the preferred terminology would be well differentiated liposarcoma, with the addition of inflammatory subtype (or that information conveyed in the comment)
Microscopic (histologic) images

Contributed by Michael Clay, M.D. and AFIP

Inflammatory ALT

Lipoma-like ALT

Recurrent ALT with ropey collagen


Recurrent ALT with ropey collagen

Sclerotic ALT

Sclerotic ALT



Morphologic variability:

Enlarged, dense irregular nuclei

Bizarre shaped cells

Lipoblasts are multivacuolated with atypical nuclei


Atypical cells within fibrous tissue

Lochkern cells with sharply outlined nuclear vacuoles

Floret cells


Thick collagen fibers

Thin collagen fibers

Moderately cellular (but not dedifferentiated)

Atypical cells within blood vessel walls



Metaplastic change:

Metaplastic bone

Metaplastic smooth muscle

Inflammatory subtype

Cytology description
  • Large cells with multilobulated nuclei and mature appearing adipocytes
  • May have bizarre tumor cells (Acta Cytol 2000;44:459)
Positive stains
Negative stains
Molecular / cytogenetics description
Molecular / cytogenetics images

Images hosted on other servers:

Giant ring or rod marker chromosome

Videos

Well differentiated liposarcoma / atypical lipomatous tumor mimicking lipoma

Well differentiated liposarcoma / atypical lipomatous tumor

Sample pathology report
  • Retroperitoneal mass, resection:
    • Well differentiated liposarcoma (7.5 cm) (see comment)
    • No evidence of dedifferentiation
    • Peripheral resection margins are positive for tumor
    • Comment: Histologic sections of this retroperitoneal mass show a well differentiated fatty neoplasm with fibrotic bands containing rare enlarged, hyperchromatic cells. The malignant cells show strong nuclear expression of MDM2 by immunohistochemistry, supporting the diagnosis.
Differential diagnosis
Board review style question #1

A retroperitoneal tumor is surgically resected and the microscopic image provided is representative of the entire lesion. Ancillary FISH testing is positive for MDM2 amplification. What is the diagnosis?

  1. Lipoma
  2. Myxofibrosarcoma
  3. Myxoid liposarcoma
  4. Pleomorphic lipoma
  5. Well differentiated liposarcoma
Board review style answer #1
E. Well differentiated liposarcoma. Given the findings of a retroperitoneal mass composed of a proliferation of variably sized adipocytes intermixed with rare atypical cells characterized by enlarged, hyperchromatic nuclei, the best diagnosis is well differentiated liposarcoma. Evidence of MDM2 amplification by FISH testing supports the diagnosis. A is incorrect, since although deep seated and large lipomas can occur, these lesions lack atypical nuclei and do not have 12q13-15 amplification. B is incorrect because myxofibrosarcomas are extremely rare in the retroperitoneum and although low grade lesions have scattered cells with enlarged, hyperchromatic nuclei, they do not have 12q13-15 amplification. C is incorrect because myxoid liposarcoma tumor cells lack significant atypia, are typically evenly distributed in a prominent myxoid stroma with delicate branching vasculature and do not have 12q13-15 amplification. D is incorrect because pleomorphic lipomas are most frequently seen in the head and neck region and do not have 12q13-15 amplification.

Comment Here

Reference: Atypical lipomatous tumor / well differentiated liposarcoma
Board review style question #2
Which of the following is true about atypical lipomatous tumor / well differentiated liposarcomas (ALT / WDL)?

  1. Frequently present in the upper trunk / back of the neck in adults
  2. Have genomic alterations at 8q11-13
  3. Lack the ability to metastasize
  4. Must contain lipoblasts
  5. Show a predilection to metastasize to other soft tissue sites
Board review style answer #2
C. Lack the ability to metastasize. ALT / WDL cannot metastasize unless it progresses to a dedifferentiated liposarcoma. A is incorrect because spindle cell lipomas show this predilection. B is incorrect because lipoblastomas have PLAG1 gene fusions, which is located at 8q11-13. D is incorrect since ALT / WDL neither need nor always have lipoblasts. E is incorrect because myxoid liposarcoma shows the unusual predilection among fatty tumors to metastasize to other soft tissue sites.

Comment Here

Reference: Atypical lipomatous tumor / well differentiated liposarcoma

Atypical spindle cell / pleomorphic lipomatous tumor
Definition / general
  • Atypical spindle cell / pleomorphic lipomatous tumor is a benign adipocytic neoplasm characterized by ill defined margins and the presence of variable proportions of mild to moderate atypical spindle cells, adipocytes, lipoblasts, pleomorphic cells, multinucleated giant cells and a myxoid or collagenous extracellular matrix
Essential features
Terminology
  • Atypical spindle cell lipoma
  • Not recommended: spindle cell liposarcoma, fibrosarcoma-like lipomatous tumor
ICD coding
  • ICD-O: 8857/0 - spindle cell lipoma
  • ICD-11: 2E80 & XH4E98 - benign lipomatous neoplasm & spindle cell lipoma
Epidemiology
Sites
Pathophysiology
Etiology
  • Unknown
Clinical features
  • Presents as a persistent or enlarging soft tissue mass
Diagnosis
  • Tissue sampling is the gold standard for a definitive diagnosis; molecular testing for MDM2 / CDK4 and RB1 may be useful on biopsy
Radiology description
  • Description of radiographic features of this entity is very scarce in the literature
  • MRI is likely the most suitable imaging modality for the characterization of these tumors
Radiology images

Contributed by Borislav A. Alexiev, M.D.

MRI

Prognostic factors
Case reports
Treatment
  • Complete surgical excision (i.e., lesion free margin) is the appropriate treatment (Cureus 2021;13:e19410)
Clinical images

Images hosted on other servers:

Right groin region mass

Gross description
  • Nonencapsulated mass with nodular or multinodular growth pattern
  • Gray-yellow cut surface, without necrotic areas (Cureus 2021;13:e19410)
Gross images

Contributed by Borislav A. Alexiev, M.D.

Soft tissue mass

Frozen section description
Microscopic (histologic) description
  • Ill defined tumor margins (Pathol Int 2018;68:550, Am J Surg Pathol 2017;41:1443, Am J Surg Pathol 2017;41:234, Int J Surg Pathol 2019;27:521, Virchows Arch 2014;465:97, J Clin Pathol 2018;71:483)
  • Variable proportions of mild to moderately atypical spindle cells, adipocytes, lipoblasts, floret-like multinucleated cells
  • Adipocytic component has a predominantly mature morphology, with variation of adipocyte size and shape
  • Focal, mild to moderate adipocytic atypia with nuclear enlargement and chromatin coarsening can be observed
  • Lipoblasts can vary from small and univacuolated or bivacuolated to larger and multivacuolated (pleomorphic)
  • Bizarre, hyperchromatic, pleomorphic multinucleated cells are often scattered within the spindle cells or adipocytic components
  • Myxoid or collagenous extracellular matrix with ropy collagen bundles
  • Mitotic figures are often present but are mostly scarce
  • Tumor necrosis is absent
  • Heterologous differentiation, including the presence of smooth muscle, cartilage or bone, is rarely observed
Microscopic (histologic) images

Contributed by Borislav A. Alexiev, M.D. and Farres Obeidin, M.D.

Adipocytic neoplasm

Pleomorphic lipoma-like morphology

Atypical lipoblasts

Pleomorphic multinucleated cells

Atypical mitosis


Variation in adipocyte size

Mild to moderate cytologic atypia

Cellular areas

CD34

RB1

Cytology description
  • Description of cytologic features of this entity is very scarce in the literature
Negative stains
Molecular / cytogenetics description
  • Deletions or losses in 13q14, including RB1, have been identified in a significant subset of cases (Histol Histopathol 2020;35:769)
  • Consistent absence of MDM2 or CDK4 amplification (Histol Histopathol 2020;35:769)
  • Chromosome 12 polysomy (MDM2/CEP12 ratio of < 2.0 with > 2 signals of both probes) (unpublished author observation)
Molecular / cytogenetics images

Contributed by Xinyan Lu, M.D.

MDM2 FISH

Sample pathology report
  • Abdominal mass, excision:
    • Atypical spindle cell / pleomorphic lipomatous tumor, 3.2 cm in greatest dimension (see comment)
    • Tumor extends to the inked soft tissue margin
    • Comment: Tumor consists of poorly marginated proliferation of mild to moderately atypical spindle cells, adipocytes, lipoblasts and floret-like giant cells set in a fibromyxoid stroma with ropy collagen. The adipocytic component shows variation in adipocytic size and scattered nuclear atypia. Frequent univacuolated and multivacuolated (pleomorphic) lipoblasts are seen. Bizarre, hyperchromatic and pleomorphic multinucleated cells are scattered within the spindle cell and adipocytic components. Mitotic figures are present (2 mitoses/10 high power fields) but tumor necrosis is absent. By immunohistochemistry, tumor cells stain positive for CD34 and S100 (focal) and show loss of RB1 expression. The findings support the above diagnosis. Atypical spindle cell / pleomorphic lipomatous tumor has a low rate of local recurrence for incompletely excised lesions. There is no documented risk for metastasis.
Differential diagnosis
  • Spindle cell / pleomorphic lipoma:
    • Atypical multivacuolated lipoblasts and atypical mitoses are not compatible with the diagnosis of spindle cell / pleomorphic lipoma (Hum Pathol 2018;74:188)
    • Chromosome 12 polysomy (MDM2/CEP12 ratio of < 2.0 with > 2 signals of both probes) in 89% of cases (Mod Pathol 2008;21:943)
  • Atypical lipomatous tumor / well differentiated liposarcoma:
    • Variation in adipocytic size associated with nuclear atypia
    • Scattered hyperchromatic atypical stromal cells and hyperchromatic bizarre stromal cells
    • Varying number of lipoblasts (from many to none)
    • Fibrillary collagenous stroma
    • Detection of MDM2 or CDK4 amplification serves to distinguish atypical lipomatous tumors / well differentiated liposarcoma from benign adipose tumors and malignant mimickers (Am J Surg Pathol 2007;31:1476)
    • RB1 expression is retained
  • Dedifferentiated liposarcoma:
    • Abrupt or gradual transition from atypical lipomatous tumor / well differentiated liposarcoma to spindle cell or pleomorphic nonlipogenic (rarely lipogenic) tumor of low or high grade (Virchows Arch 2020;476:455)
    • Detection of MDM2 amplification is helpful to distinguish dedifferentiated liposarcoma from benign and malignant mimickers in the appropriate clinical context
    • RB1 expression is retained
  • Pleomorphic liposarcoma:
    • Despite some overlapping morphologic, immunohistochemical and genetic features between atypical pleomorphic lipomatous tumor (APLT) and pleomorphic liposarcoma (PLS) (for example: infiltration, pleomorphism including pleomorphic lipoblasts, and loss of RB1), PLS can be differentiated from APLT by a higher degree of pleomorphism, high mitotic activity and tumor necrosis (Am J Surg Pathol 2017;41:1443)
    • Presence of a pleomorphic lipoma-like component demonstrating floret-like multinucleated cells and ropy collagen is a defining feature of APLT, not present in PLS (Am J Surg Pathol 2017;41:1443)
    • Pleomorphic lipoblasts in the background of a high grade, usually pleomorphic, undifferentiated sarcoma (Histopathology 2014;64:38)
    • Almost half of the cases contain focal areas similar to intermediate or high grade myxofibrosarcoma associated with pleomorphic lipoblasts (Am J Surg Pathol 2002;26:601, Am J Surg Pathol 2004;28:1257)
    • Epithelioid morphology is seen in about 25% of cases
    • Necrosis is present in the majority of cases
    • Amplification of MDM2 or CDK4 is absent
    • Strong and diffuse CD34 expression is absent
Board review style question #1
Which of the following is true about atypical spindle cell / pleomorphic lipomatous tumor?

  1. High recurrence rate
  2. Loss of RB1 expression is observed in 50 - 70% of cases
  3. MDM2 amplification is almost always present
  4. Most common locations are head and neck and genital area
  5. Risk for dedifferentiation
Board review style answer #1
B. Loss of RB1 expression is observed in 50 - 70% of cases

Comment Here

Reference: Atypical spindle cell / pleomorphic lipomatous tumor
Board review style question #2

A 56 year old man presented with a left thigh mass. Histologically, the tumor was characterized by ill defined margins and the presence of mild to moderately atypical spindle cells, mature adipocytes, univacuolated and multivacuolated (pleomorphic) lipoblasts and multinucleated floret-like cells set in a myxoid stroma with ropy collagen. Bizarre, hyperchromatic and pleomorphic multinucleated cells were scattered within the spindle cell and adipocytic components. Occasional mitotic figures, including 1 atypical mitosis, were identified. No tumor necrosis was seen. On immunohistochemistry, the tumor was positive for CD34 and S100 and showed loss of RB1.

Which of the following is most likely the correct diagnosis?

  1. Atypical lipomatous tumor / well differentiated liposarcoma
  2. Atypical spindle cell / pleomorphic lipomatous tumor
  3. Dedifferentiated liposarcoma
  4. Pleomorphic liposarcoma
  5. Spindle cell / pleomorphic lipoma
Board review style answer #2
B. Atypical spindle cell / pleomorphic lipomatous tumor

Comment Here

Reference: Atypical spindle cell / pleomorphic lipomatous tumor

Atypical vascular lesion post radiation
Definition / general
  • Due to obstruction / destruction of lymphatic drainage, often postradiation therapy (Histopathology 1999;35:319) or idiopathic in elderly
Terminology
  • In breast, often called lymphangioma circumscriptum, although the classic lesion of lymphangioma circumscriptum appears at birth or in the early years, and most breast cases are actually postsurgery or radiation therapy within the field of radiation (Am J Clin Pathol 1994;102:757)
Epidemiology
  • In women, age 33 - 72 years
  • Arises 3 - 20 years after radiation therapy for breast (93%) or ovarian (7%) carcinoma
  • Associated with pain, chronic drainage, cellulitis (South Med J 1999;92:69)
Clinical features
Case reports
Treatment
  • Cryosurgery, electrocautery or laser therapy to vaporize surface lymphatics (Dermatol Surg 1998;24:893); frequently recurs and causes substantial morbidity so must monitor for recurrence
Clinical images

Images hosted on other servers:

Breast: multiple small vesicles

Breast: multiple vesicles of lymphangioma circumscriptum

Chest: asymptomatic vesicular eruption

Gross description
  • One or more circumscribed papules, small vesicles or erythematous plaques, usually in irradiated field, median 0.5 cm
  • Frequently multiple synchronous lesions with discoloration
Microscopic (histologic) description
  • Irregular dilated vascular spaces with branching and anastomosing pattern in superficial and deep dermis
  • Thin walls, lymphatic appearance
  • Vascular channels lined by single discontinuous layer of endothelial cells with numerous small stromal papillary formations and flattened nuclei; also lined by endothelial cells projecting into lumina (lymphatic counterpart of intravascular papillary endothelial hyperplasia / Masson’s tumor)
  • May have poorly circumscribed and focally infiltrating irregular jagged vascular spaces involving the entire dermis, lined by inconspicuous endothelial cells, dissecting collagen bundles of the dermis and mimicking Kaposi's sarcoma (Am J Surg Pathol 2002;26:328)
  • No necrosis, no mitotic figures, no blood lakes

  • Lymphatic type: predominantly thin walled, variably anastomosing lymphatics primarily in superficial dermis
  • Vascular type: predominantly small, irregularly dispersed, capillary type vessels, invested by pericytes, often blood filled, in superficial or deep dermis; associated with extravasated red blood cells or hemosiderin, minor lymphatic type component
Microscopic (histologic) images

Images hosted on other servers:

Benign lymphangio-endothelioma-like lesions

Superficial lymphangioma-like patterns

Mixed histologic patterns

Nuclear hyperchromasia

D2-40+ lymphangioma-like lesion


Breast: ectatic lymphatic spaces in papillary dermis

Chest: asymptomatic vesicular eruption

Positive stains
Negative stains
Electron microscopy description
  • Endothelial cells have cytoplasmic microfilaments and pinocytotic vesicles; intermediate junctions are present between adjacent cells (Jpn J Clin Oncol 1991;21:129)
Differential diagnosis

Benign triton tumor / neuromuscular choristoma
Definition / general
  • Rare developmental lesion of mature skeletal muscle and nerve
Terminology
  • Also called neuromuscular hamartoma
  • Malignant triton tumor refers to rhabdomyosarcoma plus MPNST)
Clinical features
  • Usually < 2 years old, affects brachial plexus or sciatic nerve
  • May involve large cranial nerve trunks and present as intracranial mass
  • Very rarely involves peripheral nerve and presents as small nodules
  • Aggressive behavior
Case reports
Treatment
  • Biopsy for diagnosis plus observation, may develop fibromatosis after biopsy or complete excision
  • May recur, mandating regular followup after excision
Gross description
  • Circumscribed, firm, gray-brown-white, multinodular, attached to nerve
Microscopic (histologic) description
  • Multiple nodules, each 3 - 5 mm, separated by narrow bands of connective tissue
  • Nodules are composed of fascicles of striated muscle of varying size with nerve fibers (myelinated or not) within same perimysial fibrous sheath
  • Stroma may be more cellular with bland spindle cells and resemble fibromatosis
Microscopic (histologic) images

AFIP images

Nodules of skeletal muscle and neural elements

Fascicles and nerve fibers

Haphazardly distributed skeletal muscle and nerve fibers

Nerve fibers are S100+

Positive stains
Differential diagnosis

Calcifying aponeurotic fibroma
Definition / general
  • Rare tumor of children and adolescents, that involves the distal extremities, in association with aponeuroses, tendons and fascia
  • Characterized by bland spindle cells and less cellular zones of calcifications that have epithelioid to plump fibroblasts
Essential features
  • Infiltrative lesion composed of bland spindle cells within a collagenous matrix
  • Calcified areas that contain epithelioid fibroblasts or scattered giant cells
Terminology
  • Juvenile aponeurotic fibroma
ICD coding
  • ICD-O: 8816/0 - calcifying aponeurotic fibroma
Epidemiology
Sites
  • Most commonly occurs on the palmar aspect of hands and fingers, followed by plantar aspect of feet and toes (Cancer 1970;26:857)
  • Wrists and ankles are less commonly involved
  • Unusual locations include the proximal extremities and trunk (Hum Pathol 1998;29:1504)
  • Rare examples are documented in head and trunk regions
Pathophysiology
  • Unknown
Etiology
  • Unknown
Clinical features
  • Presents as painless, poorly circumscribed soft tissue swelling of prolonged duration
  • Propensity to recur
Diagnosis
  • Appropriate clinical, radiological and histological examination
Radiology description
  • Xray and ultrasound may show nonspecific soft tissue mass with variable extent of fine stippled calcifications (Radiographics 2009;29:2143)
  • CT scan: optimal for evaluation of calcified areas
  • MRI: superficial, ill defined, subcutaneous soft tissue mass with a tendency to infiltrate or adhere to surrounding tissues
Radiology images

Contributed by Nazmul Baqui, M.B.B.S., M.D.

Xray of left foot



Images hosted on other servers:

Plain Xray of foot and axial CT

CT of lesion on medial aspect of foot

46 year old woman

36 year old woman with distal phalangeal bone involvement

Prognostic factors
  • Benign but locally aggressive
  • Due to infiltrative nature, local recurrences may occur
Case reports
Treatment
  • Complete surgical excision is warranted
Clinical images

Images hosted on other servers:

Lesion in foot, intraoperative

Lesion at tip of index finger

Gross description
  • Ill defined firm mass with variable grittiness
  • Usually ≤ 3 cm
Gross images

Contributed by Nazmul Baqui, M.B.B.S., M.D.

Vaguely nodular mass

Firm gray cut surface



Images hosted on other servers:

Soft tissue mass with calcification

Gritty lesion resected from thigh

Microscopic (histologic) description
  • Fibromatosis-like, infiltrative and nodular calcified components
  • Infiltrative cellular component is composed of uniform plump spindle cells
  • No significant nuclear atypia or mitoses are seen
  • Calcified hypocellular component is either hyalinized or shows chondrocytes
  • Osteoclast-like giant cells are usually present
  • Lesion infiltrates the surrounding soft tissue
Microscopic (histologic) images

Contributed by Nasir Ud Din, M.B.B.S., Nazmul Baqui, M.B.B.S., M.D. and AFIP

Circumscribed calcified lesion

Giant cells

Spindle and giant cells

Prominent calcified area

Chondroid nodules


Poorly circumscribed fibroproliferative process

Amorphous
calcification
surrounded by
palisading cells

Typical zonation pattern

Rounded cells adjacent to hyalinized layer

Chondroid area adjacent to calcification

Cytology description
  • Cytologic examination reveals benign appearing spindled cells, chondroid cells, multinucleated giant cells and calcific debris (Diagn Cytopathol 2001;24:336)
Positive stains
Negative stains
Molecular / cytogenetics description
Molecular / cytogenetics images

Images hosted on other servers:

Schematic diagrams of genes and RT PCR images

Videos

Calcifying aponeurotic fibroma

Sample pathology report
  • Left hand, swelling, excision:
    • Calcifying aponeurotic fibroma (see comment)
    • Comment: Histology showed a lesion composed of bland spindle cells and less cellular zones of calcifications that have epithelioid to plump fibroblasts. These tumors are prone to recur if incompletely excised.
Differential diagnosis
  • Inclusion body fibromatosis:
    • Myofibroblastic tumor that usually occurs on the digits of children
    • Composed of bland spindle cells with characteristic intracytoplasmic inclusions
  • Ganglion cyst:
  • Tenosynovial giant cell tumor, localized type:
    • Mostly occurs on the volar aspect of the first 3 fingers
    • Usually occurs in patients aged 30 - 50 years
    • Moderately cellular with abundant mononuclear cells
    • Scattered multinucleated osteoclast-like giant cells, hemosiderin pigment, foamy histiocytes and collagenized stroma (Medicine (Baltimore) 2021;100:e26445)
  • Schwannomas and neurofibromas:
    • Both tumors occur less frequently in the hands and are composed of spindle shaped cells with serpentine nuclei
    • Scwannomas typically show Verocay bodies
    • Neurofibromas have a uniform spindle cell population within a collagenous to myxoid background and may show nerves at the periphery
    • Both usually lack calcifications and usually are not circumscribed (J Orthop Surg (Hong Kong) 2019;27:2309499019840736)
  • Rheumatoid arthritis:
    • Usually affects older individuals and affects the synovium of the wrist
    • Proliferative synovitis with villous hypertrophy and fibrinoid necrosis
    • Extensive lymphoplasmacytic and histiocytic infiltration and lymphoid follicle / germinal center formation (Medicine (Baltimore) 2021;100:e26445)
Board review style question #1
A 2 year old boy presented with mass lesion of the hand. Radiology shows a soft tissue mass near finger tendons with specks of calcifications. What is the most appropriate diagnosis with respect to age and findings?

  1. Calcifying aponeurotic fibroma
  2. Epidermal inclusion cyst
  3. Ganglion cyst
  4. Inclusion body fibromatosis
  5. Rheumatoid arthritis
Board review style answer #1
A. Calcifying aponeurotic fibroma

Comment Here

Reference: Calcifying aponeurotic fibroma
Board review style question #2

This lesion, as shown in the photomicrograph above, was resected from the hand of a 2 year old boy. What is the most likely diagnosis?

  1. Calcifying aponeurotic fibroma
  2. Epidermal inclusion cyst
  3. Fibroma of tendon sheath
  4. Fibromatosis
  5. Nuchal type fibroma
Board review style answer #2
A. Calcifying aponeurotic fibroma

Comment Here

Reference: Calcifying aponeurotic fibroma

Calcifying fibrous tumor
Definition / general
  • Benign fibrous lesion with abundant hyalinized collagen, psammomatous or dystrophic calcifications and lymphoplasmacytic infiltration
Essential features
  • Paucicellular
  • Bland spindle cells in a collagenous tissue
  • Calcifications and inflammatory infiltrate
Terminology
ICD coding
  • ICD-10: D21.9 - benign neoplasm of connective and other soft tissue, unspecified
Epidemiology
  • Adolescents / young adults
  • M:F = 1:1.27
Sites
  • Most common sites: tubular gastrointestinal tract, solid organs, peritoneal and pleural surfaces but can occur anywhere
Pathophysiology
  • Calcifying fibrous tumor may represent different stages of IgG4 related disease and fits with the unifying concept of IgG4 related pseudotumor
  • Recently, gastrointestinal calcifying fibrous tumor has been thought to be a gastrointestinal lesion of immunoglobulin 4 (IgG4) related disease (Surg Case Rep 2019;5:150)
Etiology
  • Previous infection, history of trauma and surgical intervention
  • No definitive mechanisms or causes have been confirmed
Diagrams / tables

Images hosted on other servers:

Sites

Clinical features
  • Patients can present with various symptoms depending on the location of the tumor
Diagnosis
  • Histologic examination of tissue
Radiology images

Images hosted on other servers:

CT scan

Gastrohepatic ligament tumor

Esophageal tumor

Prognostic factors
  • Local recurrence rate is ~10%
Case reports
Treatment
  • Excision
Clinical images

Images hosted on other servers:

Endoscopy of gastric tumor

Endoscopy of ileum tumor

Gross description
  • Single or multiple
  • Well circumscribed but unencapsulated
  • Spherical to lobulated mass with a solid, white to gray, gritty cut surface
  • Variable size, may infiltrate into surrounding tissue (Biomed Res Int 2019;2019:5026860)
Gross images

Contributed by Mary Wong, M.D.

Ileum tumor



Images hosted on other servers:

Gastrohepatic ligament tumor

Ileum tumor

Microscopic (histologic) description
  • Paucicellular fibroblastic proliferation with bland spindle cells embedded in dense collagenous tissue
  • Varying degrees of lymphocytes (possibly lymphoid follicles), plasma cells
  • Scattered dystrophic or psammomatous calcification (Biomed Res Int 2019;2019:5026860)
Microscopic (histologic) images

Contributed by Deepti Dhall, M.D.

Bland spindle cells and inflammation

Bland spindle cells and calcification


Case #249

Omental tumor

Electron microscopy description
  • Immature fibroblastic cells, collagen fibrils, dystrophic and psammomatous calcifications
Sample pathology report
  • Ileum, resection:
    • Calcifying fibrous tumor, 3.5 cm in greatest dimension
    • Surgical margins are negative
Differential diagnosis
Board review style question #1
A 30 year old man presents with abdominal pain and a CT reveals a mass within the gastrointestinal system, which is resected. Histopathologic examination reveals paucicellular spindle cells with dystrophic calcifications and lymphoid aggregates. Immunohistochemical stains for CD34 and vimentin are positive; CD117, DOG1, SMA and ALK1 are negative. Which of the following is the diagnosis for this patient?



  1. Calcifying fibrous tumor
  2. Desmoplastic fibroblastoma
  3. Gastrointestinal stromal tumor
  4. Inflammatory myofibroblastic tumor
Board review style answer #1
A. Calcifying fibrous tumor

Comment Here

Reference: Calcifying fibrous tumor

Cellular angiofibroma
Definition / general
  • Cellular angiofibroma (CAF) is a benign, cellular and richly vascularized fibroblastic neoplasm of the genital tract
Essential features
  • Benign, well circumscribed lesion arising in the superficial soft tissue of the genital region
  • Characterized by bland spindle shaped cells arranged without any pattern in a stroma with wispy collagen, numerous medium sized thick walled vessels and a variable adipocytic component
  • Complete excision is curative with extremely rare recurrence and even rarer sarcomatoid transformation
ICD coding
  • ICD-O: 9160/0 - cellular angiofibroma
  • ICD-11: EE6Y & XH4E06 - other specified fibromatous disorders of skin and soft tissue & cellular angiofibroma
Epidemiology
  • Peak incidence in the fifth decade of life in women and the seventh decade in men
  • M = F
Sites
Etiology
Clinical features
Diagnosis
  • Suggested by characteristic clinical presentation and supporting radiology and diagnosed by histopathological findings
Radiology description
  • Ultrasound: hypoechoic mass (BMJ Case Rep 2020;13:e235241)
  • MRI: CAF demonstrates heterogeneous increased signal intensity on T2 weighted imaging and heterogeneous contrast enhanced pattern due to hypervascularity (IJU Case Rep 2020;3:69)
  • CT: hyperintense on post contrast images (IJU Case Rep 2020;3:69)
  • Cases with intratumoral fat component show heterogeneous appearance on both CT and MRI
Radiology images

Images hosted on other servers:
Nodular mass inguinal region

Nodular mass inguinal region

Lower rectal exophytic lesion

Lower rectal exophytic lesion

Right pharynx mass

Right pharynx mass

Hypopharyngeal mass

Hypopharyngeal mass

Cervicovaginal mass Cervicovaginal mass

Cervicovaginal mass

Prognostic factors
  • Clinically benign with a limited capacity for local recurrence
  • Only rare cases show nuclear atypia and sarcomatous transformation (Mod Pathol 2011;24:82)
  • Complete local excision is effective in preventing recurrence and damage to surrounding tissues (BMJ Case Rep 2020;13:e235241)
Case reports
Treatment
  • Complete local excision by shelling out is the treatment of choice
  • Presence of atypical / sarcomatous features do not predispose to a malignant or aggressive biological behavior or recurrence (BMJ Case Rep 2020;13:e235241)
Clinical images

Images hosted on other servers:

Circumscribed, nodular mass

Gross description
  • Circumscribed dermal or subcutaneous round, oval or lobulated masses, measuring a few centimeters in diameter (larger in men)
  • Size ranges from 0.6 cm to 25 cm
  • Consistency varies from soft to rubbery, with a solid grayish pink to yellowish brown cut surface (Asian J Androl 2018;20:95, Medicine (Baltimore) 2019;98:e18385)
  • Foci of hemorrhage or necrosis are exceptional
Gross images

Images hosted on other servers:

Mass attached to spermatic cord

Polypoid mass

Well circumscribed mass

Frozen section description
  • Frozen section examination of the lesion or margins is not required for diagnosis or to declare margin clearance
Microscopic (histologic) description
  • Well circumscribed, unencapsulated dermal or subcutaneous lesions that may be more infiltrative in men
  • The 2 components present are spindle cells and medium sized thick to variably hyalinized blood vessels (Diagn Pathol 2015;10:114)
  • Spindle / fusiform cells have bland cytologic features and may be arranged in fascicles, short whorls at random (Am J Surg Pathol 2004;28:1426)
  • Stroma can be fibrous to myxoid and contain delicate pale short to thick collagen fibers
  • Mature fat can be seen in ~25% of cases (Weiss: Enzinger and Weiss's Soft Tissue Tumors: Expert Consult, 7th Edition, 2019)
  • Variable stromal edema, hyalinization or myxoid change is often seen, especially in men
  • Stroma may show scattered lymphocytes and perivascular lymphoid aggregates, mast cells are frequent
  • Necrosis or hemorrhage are not usually observed and mitotic activity is low (< 1/10 high power fields)
  • Rarely, sarcomatous features may be seen as well as scattered cells with marked nuclear atypia or areas that mimic liposarcoma (both well differentiated and pleomorphic types) (Lindberg: Diagnostic Pathology - Soft Tissue Tumors, 3rd Edition, 2019)
  • Occasional reports show abrupt transition to a discrete nodule with sarcomatous features or multivacuolated lipoblasts, pleomorphic and hyperchromatic spindle cells that morphologically resemble pleomorphic liposarcoma, atypical lipomatous tumor or pleomorphic spindle cell sarcoma
Microscopic (histologic) images

Contributed by Nasir Ud Din, M.B.B.S.
Stroma and vessels

Stroma and vessels

Stromal cells

Stromal cells

Edematous stroma

Edematous stroma

Collagen fibers

Collagen fibers

Bland cells

Bland cells


Inflammatory cells

Inflammatory cells

Desmin

Desmin

CD34

CD34

Estrogen receptor

Estrogen receptor

PR

PR

Virtual slides

Images hosted on other servers:
Soft tissue seminar Rosai collection

Soft tissue seminar Rosai collection

Positive stains
Molecular / cytogenetics description
Molecular / cytogenetics images

Images hosted on other servers
G banding and FISH analysis

G banding and FISH analysis

RB1 loss on FISH analysis

RB1 loss on FISH analysis

Videos

Cellular angiofibroma of the genitals

Sample pathology report
  • Vaginal mass, excision specimen:
    • Cellular angiofibroma, completely resected with clear margins (see comment)
    • Comment: This tumor shows classic combination of benign spindle cells with admixed population of thick hyalinized medium sized blood vessels, which given with the site is compatible with a diagnosis of cellular angiofibroma. Any atypical or sarcomatous component is not identified. These are benign lesions and complete excision is generally considered curative with very low chances of local recurrence.
Differential diagnosis
  • Aggressive angiomyxoma:
    • Large, infiltrative, gelatinous masses with poor demarcation
    • Extends deeper into the soft tissues
    • Paucicellular, myxoid lesions with smooth muscle and lesional cells that appear to be radiating from thick medium to large blood vessels
    • Variable CD34, desmin, smooth muscle actin, ER and PR
  • Angiomyofibroblastoma:
    • Vulvar, well marginated lesions of reproductive aged and early postmenopausal women
    • Alternating areas of high and low cellularity
    • Epithelioid cells with abundant eosinophilic cytoplasm that are commonly seen aggregated around blood vessels
    • Spindle cells morphology may also be seen, particularly in postmenopausal cases
    • Variable CD34, desmin, smooth muscle actin, ER and PR
  • Solitary fibrous tumor:
    • Cellular lesion with short spindle cells in a collagenous background
    • Hemangiopericytoma-like vasculature is the characteristic feature
    • STAT6 positivity is highly specific for this entity
Board review style question #1

A 40 year old woman presented with vaginal swelling. Intraoperatively, a circumscribed nodular mass was seen and the cut surface showed a gray-white to yellow multinodular lesion. Microscopically, the tumor was well circumscribed, composed of bland spindle cells and had the striking presence of hyalinized blood vessels as shown in the photomicrograph above. What is the correct diagnosis?

  1. Angiomyofibroblastoma
  2. Cellular angiofibroma
  3. Leiomyoma
  4. Solitary fibrous tumor
Board review style answer #1
B. Cellular angiofibroma. The tumor shows bland spindle cells and a prominent component of hyalinized blood vessels, which is characteristic of cellular angiofibroma. There is no perivascular arrangement as seen in angiomyofibroblastoma, no interlacing fascicles of cells as seen in leiomyoma and the blood vessels do not show hemangiopericytoma-like pattern to call it solitary fibrous tumor. Answer A is incorrect because angiomyofibroblastoma shows presence of epithelioid to spindle cells, which are prominently arranged around blood vessels. Answer C is incorrect because leiomyoma is composed of interlacing fascicles of smooth muscle without hyalinized vessels or another spindle cell component. Answer D is incorrect because solitary fibrous tumor shows characteristic staghorn-like blood vessels and not hyalinized ones.

Comment Here

Reference: Cellular angiofibroma
Board review style question #2
A 65 year old man presented with inguinal pain and swelling that was slow growing in nature. MRI showed a hyperintense lesion in the inguinal region that was attached to the spermatic cord. Excision was done and a diagnosis of cellular angiofibroma was made. Which of the following statements is correct regarding this entity?

  1. Excision followed by radiation is curative
  2. It is a locally aggressive neoplasm
  3. RB1 gene deletion is characteristic
  4. STAT6 is positive by immunohistochemistry
Board review style answer #2
C. RB1 gene deletion is characteristic. RB1 gene deletion, which is located at chromosome 13q14, is characteristic of cellular angiofibroma. Answer B is incorrect because cellular angiofibroma is a benign lesion that does not require radiation or any other aggressive therapy in most cases. Answer A is incorrect because complete surgical excision is curative and local recurrence is extremely rare. Answer D is incorrect because STAT6 positivity by immunohistochemistry is a feature of solitary fibrous tumor, which surrogates the genetic abnormality (NAB2::STAT6 fusion) in this tumor.

Comment Here

Reference: Cellular angiofibroma

Chondroid lipoma
Definition / general
  • Benign adipose tissue tumor composed of lipoblasts intermixed with mature adipocytes in a myxohyaline chondroid matrix (Ann Diagn Pathol 2012;16:230)
Essential features
  • Rare variant of lipoma with features of immature fat and immature cartilage
  • Rare, slow going, painless mass in adult women
  • Lobulated, circumscribed, deep seated tumor
  • Mostly affects the proximal extremities and limb girdles of adult women
  • Nests and cords of uni and multivacuolated lipoblasts with variable adipose tissue in a myxochondroid matrix
  • Characterized by t(11;16)(q13;p13) translocation (Histopathology 2013;62:925)
  • Surgical excision is curative with rare recurrence
Terminology
  • Extraskeletal chondroma with lipoblast-like cells (first described terminology, subsequently disapproved) (Hum Pathol 1986;17:1285)
ICD coding
Epidemiology
Sites
  • Most common sites: proximal extremities and limb girdles
  • Less common sites: trunk, distal extremities, head and neck (J Clin Diagn Res 2017;11:ED17)
  • Deep seated tumor involves skeletal muscles, deep fibrous connective tissue or deep subcutaneous fat
  • Superficial in 20% of cases
Pathophysiology
  • Shows recurrent t(11;16)(q13;p13) translocation with resultant C11orf95 and MRTFB fusion (Genes Chromosomes Cancer 2010;49:810)
  • Megakaryoblastic leukemia 2 (MKL2) / myocardin related transcription factor B (MRTFB) acts as coactivator of transcription factor serum response factor (SRF)
  • SRF controls cellular processes including cytoskeleton organization, cell migration, growth and differentiation
  • C11orf95 encodes protein of unknown function
Etiology
  • Not known
Clinical features
  • Benign, painless, slow growing tumor of variable duration
  • History of recent increase in size in 50% of cases
  • Reference: Exp Ther Med 2021;22:1087
Diagnosis
  • Requires presence of classic histologic features (i.e., presence of lipoblasts, chondroid cells and adipocytes in a myxohyaline matrix in a circumscribed lobulated tumor)
  • Diagnosis can also be made on cytology (Am J Surg Pathol 1999;23:1300)
Radiology description
  • CT typically shows a well defined, heterogeneously enhancing mass with variable areas of fat attenuation
  • Can exhibit calcification and ossification on radiographs (Australas Med J 2012;5:355, Skeletal Radiol 2004;33:670)
  • MRI usually shows a well defined mass with lobulated areas of T2 hyperintense or fluid-like signal intensity (Balkan Med J 2015;32:107, Skeletal Radiol 1996;25:592)
  • Appears isointense or hypointense to muscle, with focal hyperintense areas (fat areas) on T1 weighted MR images
  • May show hypermetabolic activity at PET / CT
Radiology images

Contributed by Nasir Ud Din, M.B.B.S.
Right upper limb, coronal view

Right upper limb, coronal view

Right upper limb, T1

Right upper limb, T1

Prognostic factors
  • Does not recur or metastasize if completely excised
  • Rare local recurrence
Case reports
Treatment
Gross description
  • Well delineated, frequently encapsulated
  • Multilobulated (33%)
  • Cut surface: yellowish tan, gelatinous
  • Hemorrhagic foci can be seen
  • 1.5 - 11 cm, hemorrhage is associated with size increase
  • Reference: Medicine (Baltimore) 2019;98:e15587
Gross images

Contributed by Nasir Ud Din, M.B.B.S.
Cut surface

Cut surface

Gelatinous appearance

Gelatinous appearance

Microscopic (histologic) description
  • Encapsulated with occasional lobulations
  • Lobulations are formed by fibrous septa
  • Composed of 3 components arranged in nests, cords and sheets in variable proportion:
    • Mature adipose tissue
    • Cells displaying lipoblastic differentiation
    • Myxohyaline chondroid matrix
  • Mature adipocytes contain eccentrically placed nuclei and vacuolated cytoplasm
  • Cells with lipoblastic differentiation may show various patterns:
    • Undifferentiated bland cells with minimal cytoplasm
    • Small univacuolated to multivacuolated lipoblasts with fat droplets scalloping bland nuclei
  • Cells with granular eosinophilic cytoplasm may be seen
  • Rich vascular network: thick walled blood vessels alternate with large, gaping thin walled vascular spaces
  • No significant nuclear atypia or mitotic activity
  • Hemorrhage and hemosiderin deposition can be present (Hum Pathol 1995;26:706)
  • Fibrosis, calcification and metaplastic bone formation may be seen (Skeletal Radiol 2008;37:475)
Microscopic (histologic) images

Contributed by Nasir Ud Din, M.B.B.S.
Lobulated appearance

Lobulated appearance

Cellular morphology

Cellular morphology

Prominent matrix

Prominent matrix

Multivacuolated lipoblasts

Multivacuolated lipoblasts

Cytology description
Cytology images

Images hosted on other servers:
Cell block section

Cell block sections

Positive stains
  • Immunostains are not very helpful in differential diagnosis
  • S100:
    • Positive in mature adipocytes (strong)
    • Lipoblasts (weak)
    • Negative in cells that do not show lipoblastic differentiation
  • Keratins: rarely positive
  • Cyclin D1: positive (Sarcoma 2011;2011:638403)
  • PAS special stain highlights intracytoplasmic glycogen
  • Toluidine and Alcian blue special stains highlight chondroitin sulfate in matrix at low pH
Negative stains
Electron microscopy description
  • Mature adipose tissue (Am J Surg Pathol 1995;19:1272)
  • Small embryonal cells with characteristics of lipoblasts, chondroblasts or both
  • Matrix shows thin filament network, thin collagen fibers and copious proteoglycan particles
Molecular / cytogenetics description
  • Chromosomal translocation t(11;16)(q13;p12-13) (C11orf95-MKL2) (Histopathology 2013;62:925)
  • Rare 3 way translocation involving chromosomes 1, 2 and 5
Molecular / cytogenetics images

Images hosted on other servers:
11;16 translocation

t(11;16) translocation

Videos

Chondroid lipoma pathology

Sample pathology report
  • Brachialis muscle lump, excision:
    • Chondroid lipoma (see comment)
    • Comment: Chondroid lipoma is a rare variant of lipoma characterized by admixture of chondroid cells, lipoblasts and adipocytes in a myxohyaline matrix. The tumor is benign and excision is curative.
Differential diagnosis
  • Soft tissue chondroma:
    • Occurs in hands and feet
    • True hyaline cartilage is present
    • Lacks adipocytic component
  • Atypical lipomatous tumor:
    • Adipocytes in sheets with variation in size and shape
    • Thick fibrous septa that have atypical stromal cells
    • May exhibit myxoid stroma
    • May show metaplastic cartilage
    • MDM2 amplification present
  • Myxoid liposarcoma:
    • Spindle to stellate cells with myxoid stroma
    • Plexiform vasculature
    • Lipoblasts of signet type are typically present
    • Round cell component shows uniform round cells with moderate pleomorphism and scant cytoplasm
    • t(12;16), t(12;22) translocation
  • Extraskeletal myxoid chondrosarcoma:
    • Larger infiltrative mass
    • More lobulated architecture
    • Anastomosing and reticular pattern of uniform spindle to epithelioid cells
    • Prominent myxoid stroma
    • Lacks adipocytes and lipoblasts
    • Characteristic translocation t(9;17), t(9;22)
  • Myoepithelioma of soft tissue:
    • Cells can be round, spindle or plasmacytoid
    • May show myxoid background and chondroid metaplasia
    • Lipoblasts are absent
    • Immunohistochemistry required for diagnosis: cytokeratin, EMA and S100 positive
    • Variable expression of SMA, calponin, p63, GFAP, desmin
  • Chondrolipoma:
    • Typical lipoma with mature hyaline cartilage
Additional references
Board review style question #1

A 30 year old woman presented with a painless lump in right thigh. Excision specimen showed an encapsulated lesion of 4 x 4 cm. Histology showed a circumscribed adipocytic lesion composed of admixture of chondroid cells with vacuolated to eosinophilic cytoplasm, multivacuolated lipoblasts and mature adipocytes in a chondromyxoid background. Mature hyaline cartilage, necrosis and atypia were absent. Occasional mitotic figures were present. What is the most likely diagnosis?

  1. Chondroid lipoma
  2. Chondrolipoma
  3. Extraskeletal myxoid chondrosarcoma
  4. Myxoid liposarcoma
  5. Well differentiated liposarcoma
Board review style answer #1
A. Chondroid lipoma

Comment Here

Reference: Chondroid lipoma
Board review style question #2
Which of the following lipomas shows the presence of lipoblasts?

  1. Angiolipoma
  2. Chondroid lipoma
  3. Chondrolipoma
  4. Myelolipoma
  5. Osteolipoma
Board review style answer #2
B. Chondroid lipoma

Comment Here

Reference: Chondroid lipoma

CIC rearranged sarcoma
Definition / general
Essential features
Terminology
  • Undifferentiated round cell sarcoma with CIC-DUX4 fusion
ICD coding
  • ICD-O: 8803/3 - Small cell sarcoma
Epidemiology
Sites
Etiology
  • CIC-DUX4 fusion oncoprotein potentiates the transcriptional activity of CIC and activates the expression ETV1/4/5, which is a member of the E26 transformation specific (ETS) family of transcription factors (Sci Rep 2020;10:684)
  • MYC amplification in majority of cases (Mod Pathol 2015;28:57)
Clinical features
Diagnosis
Radiology description
  • Large heterogeneous appearing hypermetabolic mass on PET / CT
Radiology images

Contributed by Borislav Alexiev, M.D.

PET / CT

Prognostic factors
Case reports
Treatment
Gross description
Gross images

Contributed by Borislav Alexiev, M.D.

Soft tissue mass

Frozen section description
  • Diagnosis of CIC-DUX4 sarcoma on a small tissue fragment without molecular studies would be challenging
Microscopic (histologic) description
Microscopic (histologic) images

Contributed by Borislav Alexiev, M.D.

Chest wall mass

Small round cell morphology

Solid growth pattern

Spindle cell morphology

Mitoses

Myxoid stroma


CD99 expression

WT1 expression

MYC expression

Cytology description
  • Hypercellular smears, with tumor cells arranged in large groups and singly dispersed
  • Individual cells with high nuclear to cytoplasmic ratio, eccentric round to ovoid nuclei, irregular nuclear contours and small nucleoli (Diagn Cytopathol 2018;46:958)
  • Cytoplasmic vacuoles (Cancer Cytopathol 2016;124:350)
  • Mitotic figures
  • Necrosis
  • Myxoid stromal component
Electron microscopy description
  • Heterogeneity: in cell density, from tightly packed to loosely unconnected areas (Ultrastruct Pathol 2020;44:237)
  • Polygonal to pleomorphic cells with small processes
  • Round, oval, polygonal or elongated nuclei
  • Abundant glycogen in the cytoplasm and rare cell adhesions
  • No neuroendocrine granules present
Molecular / cytogenetics description
Molecular / cytogenetics images

Contributed by Lawrence J. Jennings, M.D., Ph.D.

Real time RT-PCR

Sample pathology report
  • Chest wall mass, excision:
    • CIC-DUX4 associated undifferentiated small round cell sarcoma (see comment)
    • Comment: There is a subcutaneous solid nodular proliferation of small to medium sized round / ovoid and spindle cells with scant amount of amphophilic or lightly eosinophilic cytoplasm. The cells contain medium sized round to oval vesicular nuclei with small nucleoli. High mitotic activity (21 mitoses/10 high power fields) and areas of necrosis are present. Patchy myxoid or edematous stromal changes are seen.
    • Immunohistochemically the cells have strong expression of CD99, WT1 (N terminal) and MYC while are negative for pankeratin AE1 / AE3, EMA, myogenin, S100 and SOX10. INI1 is preserved. The tumor is positive for CIC-DUX4 fusion transcript.
    • This constellation of morphological, immunohistochemical and molecular features strongly supports the diagnosis of CIC-DUX4 associated undifferentiated small round cell sarcoma. It is a sarcoma associated with an aggressive clinical course, with an inferior overall survival compared to Ewing sarcoma.
Differential diagnosis
Board review style question #1
Which of the following is true about CIC-DUX4 rearranged sarcoma?

  1. High metastatic rate, brain most common
  2. Diagnosis always requires clinicopathological and radiological correlation
  3. Prognosis is poor
  4. Tumor is always negative for ERG
  5. Tumor is characterized by cords of epithelioid cells distributed in a desmoplastic stroma
Board review style answer #1
C. Prognosis is poor

Comment Here

Reference: CIC-DUX4 fusion tumor
Board review style question #2

A 65 year old man presented with a left thigh mass. Hematoxylin eosin stains demonstrated proliferation of small to medium sized round / ovoid cells with medium sized round to oval vesicular nuclei and clear or pale eosinophilic cytoplasm. Increased mitotic activity, geographic necrosis and patchy myxoid stromal change were seen. Immunohistochemical stains for CD99, WT1 and DUX4 were positive in tumor cells while all of the following were negative: pankeratin AE1 / AE3, S100, SOX10, myogenin, NY-ESO-1, NKX2.2 and CCNB3. INI1 was retained. Which of the following is most likely the correct diagnosis?

  1. Synovial sarcoma, poorly differentiated
  2. Extraskeletal Ewing sarcoma
  3. Epithelioid sarcoma
  4. BCOR-CCNB3 (Ewing-like) sarcoma
  5. CIC-DUX4 associated undifferentiated round cell sarcoma
Board review style answer #2
E. CIC-DUX4 associated undifferentiated round cell sarcoma

Comment Here

Reference: CIC-DUX4 fusion tumor

Clear cell sarcoma
Definition / general
  • Clear cell sarcoma (CCS) is a malignant soft tissue sarcoma composed of monotonous epithelioid and spindle cells with clear to eosinophilic cytoplasm characterized by melanocytic differentiation and EWSR1-ATF1 / CREB1 rearrangement
Essential features
  • Young adult, M:F = 1, distal extremity, slow growing
  • Not related to epidermis (hence, tendon and aponeuroses)
  • Do not confuse with clear cell sarcoma of kidney
  • Sheets and nests of monotonous epithelioid spindle cells with clear to eosinophilic cytoplasm separated by a delicate framework of fibrocollageneous stroma
  • Scattered Touton type or wreath-like multinucleated giant cells are relatively common
  • S100 / SOX10+, Melan A+, HMB45+
  • EWSR1-ATF1 or EWSR1-CREB1
Terminology
  • Melanoma of soft parts (not recommended)
  • Clear cell sarcoma of tendon and aponeuroses
  • Clear cell sarcoma of soft parts
ICD coding
  • ICD-O: 9044/3 - clear cell sarcoma, NOS (except of kidney M-8964/3)
  • ICD-10: C49.9 - malignant neoplasm of connective and soft tissue, unspecified
  • ICD-11: 2B5K & XH77N6 - unspecified malignant soft tissue tumors or sarcomas of bone or articular cartilage of other or unspecified sites & clear cell sarcoma of soft tissue
Epidemiology
  • < 1% of all soft tissue tumors
  • Young adults (third decade)
  • No gender predilection
Sites
Pathophysiology
  • Microphthalmia associated transcription factor (MITF) seems to be critical oncogenic target of EWSR1-ATF1 in CCS leading to melanocytic phenotype but not in angiomatoid fibrous histiocytoma with the same EWSR1-ATF1 rearrangement (Cancer Cell 2006;9:473, Am J Surg Pathol 2012;36:e1)
  • Regarding gene expression profile, CCSs cluster with melanomas, not soft tissue sarcomas (J Clin Oncol 2003;21:1775)
  • Classified as tumor of uncertain differentiation in WHO 2020
Etiology
  • Unknown
Clinical features
  • Not uncommon to get a long history (prebiopsy duration)
  • Pain and mass are common presenting symptoms
Diagnosis
  • Magnetic resonance imaging (MRI)
  • Biopsy and molecular pathology
Radiology description
  • Well defined, homogeneous and benign looking mass
  • Magnetic resonance imaging
    • Homogeneous iso hyperintense on T1 and heterogeneous on T2 with variable contrast enhancement
  • Reference: Skeletal Radiol 2000;29:187
Radiology images

Contributed by Ustun Aydingoz, M.D.
Magnetic resonance imaging

Magnetic resonance imaging



Images hosted on other servers:

MRI of CCS

Prognostic factors
Case reports
Treatment
  • Complete surgical excision with or without radiotherapy
  • Chemotherapy / immunotherapy has no effect
Clinical images

Images hosted on other servers:

Clear cell sarcoma of large toe

Gross description
Gross images

Contributed by Kemal Kösemehmetoğlu, M.D. and Mark R. Wick, M.D.
Magnetic resonance imaging

Clear cell sarcoma of soft tissue

Frozen section description
  • Sentinel lymph node can be encountered
  • Melanoma is the main differential diagnosis and may be impossible to separate in this setting
Microscopic (histologic) description
  • Essential: nests of epithelioid spindle cells with clear eosinophilic cytoplasm and prominent nucleoli plus melanocytic differentiation
  • Morphological patterns (Am J Surg Pathol 2008;32:452, J Clin Pathol 2010;63:416):
    • Vaguely organoid (neuroendocrine-like) pattern: nests and short fascicles of epithelioid or spindle cells, surrounded by a delicate framework of fibrocollagenous tissue contiguous with the adjacent tendons and aponeurosis
    • Diffuse pattern: solid sheets of epithelioid to spindle cells
    • Pseudoalveolar pattern: reminiscent of alveolar soft part sarcoma
    • Myxoid / microcystic pattern
    • Inflammatory pattern: reminiscent of seminoma
  • Cells:
    • Epithelioid to spindled
    • Monotonous round to oval nuclei with prominent nucleoli
    • Lightly eosinophilic to amphophilic cytoplasm
    • Clear cells typically comprise only a subset of the tumor
    • Scattered rhabdoid and pleomorphic cells
    • Scattered Touton type or wreath-like multinucleated giant cells are relatively common
    • Rare mitosis
  • Necrosis usually focally in 33%
  • Melanin pigment in scattered cells in half of the cases
  • No involvement of epidermis or no epidermal melanocytic proliferation but longstanding lesions (especially at fingers) may enlarge and therefore tumors may ulcerate
Microscopic (histologic) images

Contributed by Kemal Kösemehmetoğlu, M.D.
CCS attached to aponeurosis

CCS attached to aponeurosis

Tumor adjacent to tendon and aponeurosis

Tumor adjacent to tendon and aponeurosis

Classical (organoid / neuroendocrine-like) morphology

Classical (organoid / neuroendocrine-like) morphology

Short fascicles of spindle cells

Short fascicles of spindle cells

Mixed spindle and epithelioid cells

Mixed spindle and epithelioid cells


Intermixed spindle and epithelioid cells

Intermixed spindle and epithelioid cells

Solid sheets of epithelioid cells

Solid sheets of epithelioid cells

High power appearance of classical CCS

Classical CCS

Rare pleomorphic cells

Rare pleomorphic cells

Pseudoalveolar pattern in CCS

Pseudoalveolar pattern


Melanin pigment in CCS

Melanin pigment

Myxoid / microcystic pattern

Myxoid / microcystic pattern

Lymph node metastasis

Lymph node metastasis

Fontana stain

Fontana stain


S100

S100

SOX10

SOX10

HMB45

HMB45

MelanA

MelanA

Cytology description
  • Hypercellular smears composed of monomorphic epithelioid / polygonal cells, including singly scattered or loosely clustered spindle cells, with prominent nucleoli
  • Granular to vacuolated, well defined cytoplasm
  • Binucleation / multinucleation (50%), abrupt anisonucleosis, racquet shaped cells (20%)
  • Tigroid background (10%)
  • Focal intracytoplasmic pigment (10%)
  • If available, immunohistochemical stains and molecular testing are crucial in reinforcing a diagnosis (Cytopathology 2020;31:280)
Cytology images

Contributed by Bharat Rekhi, M.D.
Cytology of CCS - Giemsa

Giemsa

Pap

Positive stains
Electron microscopy description
  • Resembles synovial sarcoma given the presence of basement membrane material, a biphasic pattern of cells and the presence of pseudoglandular structures with filopodia
  • Presence of melanosomes and interdigitating cellular processes wrapping around cells and extracellular structures are similar to melanoma and nerve sheath tumors
  • May contain aggregates of glycogen and inclusions composed of multilayered membrane structures forming myelin-like figures
  • References: Am J Pathol 1984;114:264, Virchows Arch A Pathol Anat Histopathol 1983;401:109
Molecular / cytogenetics description
  • t(12;22)(q13;q12) for EWSR1-ATF1 [4 splice variants including the most common EWSR1(ex8)-ATF1(ex4)] in 90 - 95%
  • t(2;22)(q32.3;q12) for EWSR1(ex7)-CREB1(ex7) in 5 - 10% (Mod Pathol 2009;22:1201)
  • Aberrations in chr 2, 3, 7, 8, 12, 13, 14, 15, 17, 21 and 22
  • Rare BRAF V599E and NRAS codon 61 mutations (Dermatol Res Pract 2012;2012:984096)
Molecular / cytogenetics images

Contributed by Kemal Kösemehmetoğlu, M.D.
EWSR1 break apart FISH

EWSR1 break apart FISH



Images hosted on other servers:

PCR showing EWSR1-ATF1 fusion product (81 bp)

Sequence of PCR product showing EWSR1-ATF1

Videos

Woman in 20s with gluteal mass

Sample pathology report
  • Right toe, incisional biopsy:
    • Clear cell sarcoma of soft tissue (see comment)
    • Comment: Tumor was composed of nests of clear cells with prominent nucleoli and melanocytic differentiation. FISH break apart study revealed the presence of an EWSR1 rearrangement, which excluded the diagnosis of melanoma. These tumors are definitionally regarded as high grade - grade 3 sarcomas. Wide excision is recommended.
Differential diagnosis
Board review style question #1

An 18 year old woman presented with a 1 cm mass in the right great toe. Morphological features are shown above. Which one of the following immunohistochemical panels is the most helpful for reaching the correct diagnosis?

  1. EMA, SMA, INI1
  2. HMB45, MelanA, desmin
  3. PanCK, INI1, S100
  4. PanCK, synaptophysin, CD31
  5. SMA, desmin, SOX10
Board review style answer #1
C. PanCK, INI1, S100. If you order just HMB45, MelanA, and desmin and all of them turn out to be positive you may end up with the diagnosis of PEComa. PEComa is in the differential, even desmin is negative. So IHC panel should include an S100 or Sox10 and not necessarily HMB45 and MelanA at first. HMB45 and MelanA are better requested after S100/Sox10 positivity. panCK and INI1 are to exclude epithelioid sarcoma. INI1 could be replaced by HMB45 if requested.

Comment Here

Reference: Clear cell sarcoma
Board review style question #2

An 18 year old woman presented with a 1 cm toe mass. There was diffuse S100 expression plus SOX10, HMB45 and MelanA positivity. What is the next step?

  1. Diagnose as melanoma
  2. Order PAX8 and TFE3
  3. Order whole genome sequencing
  4. Perform an EWSR1 break apart FISH
  5. Request PCR for EWSR1-CREB1
Board review style answer #2
D. Perform an EWSR1 break apart FISH. The differential diagnosis is between melanoma and CCS and EWSR1 FISH is less time consuming and cheaper than NGS to reach the correct diagnosis.

Comment Here

Reference: Clear cell sarcoma

Composite hemangioendothelioma
Definition / general
Essential features
  • Vascular neoplasm which is considered to be of intermediate (rarely metastasizing) malignant potential
  • Striking morphologic variation at low power, both within individual tumors and between tumors of different patients
  • Distinguishing composite hemangioendothelioma from other vascular neoplasms largely lies in finding different histologic vascular patterns within 1 lesion
ICD coding
  • ICD-O:
    • 9130/1 - hemangioendothelioma, NOS
    • 9136/1 - composite hemangioendothelioma
  • ICD-11: 2F72.Y & XH8D24 - other specified neoplasms of uncertain behavior of skin & composite hemangioendothelioma
Epidemiology
Sites
Etiology
Clinical features
Diagnosis
  • Tissue sampling is the gold standard for a definitive diagnosis
  • Because of the wide morphologic spectrum, can be difficult to recognize in small tissue samples (Adv Anat Pathol 2015;22:254)
Prognostic factors
Case reports
Treatment
  • Because of the rarity of these tumors, the best therapeutic approach has not been established
  • Surgical excision is often curative (Adv Anat Pathol 2015;22:254)
  • Local radiotherapy and chemotherapy for recurrent and metastatic disease have been used with some success
Clinical images

Images hosted on other servers:

Eyelid swelling

Gross description
Microscopic (histologic) description
  • Poorly circumscribed lesion with infiltrative borders, centered in the deep dermis and subcutis
  • The most striking feature at low power is the variability in appearance from area to area
  • Admixture of different vascular components, including but not limited to:
    • Epithelioid hemangioendothelioma
    • Retiform hemangioendothelioma
    • Low grade angiosarcoma
    • Lymphangioma
    • Hemangioma (spindle cell, capillary, cavernous, hobnail, epithelioid)
  • Different components merge imperceptibly, making it difficult to identify the pure components
  • Vacuolated epithelioid endothelial cells with pseudo lipoblastic appearance
  • Few mitotic figures
Microscopic (histologic) images

Contributed by Borislav Alexiev, M.D.
Vascular neoplasm

Vascular neoplasm

Retiform hemangioendothelioma pattern

Retiform hemangio-endothelioma pattern

Epithelioid hermangioma pattern

Epithelioid hermangioma pattern

Angiosarcoma pattern

Angiosarcoma pattern

Vacuolated endothelial cells

Vacuolated endothelial cells


ERG

ERG

CD31

CD31

Positive stains
Negative stains
Sample pathology report
  • Scalp, excision:
    • Composite hemangioendothelioma (see comment)
    • Tumor is closest at less than 0.1 cm from the specimen margin
      • Comment: There is a poorly circumscribed, infiltrative lesion, which is centered in the dermis and subcutis. It comprises a complex admixture of vascular components, which include arborizing channels simulating the rete testis, chains and cord of eosinophilic epithelioid cells intermixed with large adipocyte-like cells, capillary sized vessels lined by epithelioid endothelial cells and superficial dilated vessels lined by small endothelial cells protruding into the lumina. Irregular anastomosing channels lined by mildly atypical endothelial cells are also noted. No coagulative tumor necrosis is seen. Immunohistochemically the lesional cells have strong expression of ERG and CD31 while are negative for HHV8. This constellation of morphological and immunohistochemical features strongly supports the diagnosis of composite hemangioendothelioma. It is vascular neoplasm prone to local recurrence unless completely excised, with intermediate (rarely metastasizing) malignant potential.
Differential diagnosis
  • Distinguishing composite hemangioendothelioma from other vascular neoplasms largely lies in finding different histologic vascular patterns within 1 lesion
  • Retiform hemangioendothelioma (Am J Surg Pathol 1994;18:115):
    • Long arborizing blood vessels arranged in a retiform pattern (reminiscent of normal rete testis)
    • Blood vessels lined by monomorphic hobnail endothelial cells with scant cytoplasm
    • Very prominent lymphocytic infiltrate in most cases
    • Focal presence of papillae with hyaline collagenous cores, similar to those seen in papillary intralymphatic angioendothelioma (Dabska tumor)
    • Absence of endothelial atypia or mitotic activity
    • Lymphatic marker PROX1 is positive (Am J Surg Pathol 2012;36:351)
    • Other lymphatic markers, including podoplanin (D2-40) are usually but not always negative (J Cutan Pathol. 2009 Sep;36(9):987)
    • CD31+, CD34+, ERG+
  • Dabska type hemangioendothelioma (Cancer 1969 Sep;24:503):
    • More common in infants and children
    • Poorly delineated dermal or subcutaneous proliferation of lymphatic channels
    • Distinctive intravascular growth of well differentiated columnar / matchstick-like / hobnail endothelial cells with eosinophilic cytoplasm and indistinct nucleoli (Am J Surg Pathol 1999;23:1004)
    • Intraluminal papillary tufts with hyaline cores
    • Varying degrees of stromal or intraluminal lymphocytes
    • Minimal cytologic atypia and rare to absent mitotic activity
    • D2-40+, VEGFR3+, CD31+, CD34 variable
  • Kaposiform hemangioendothelioma:
    • Tumor occurs nearly exclusively in children
    • Often associated with Kasabach-Merritt phenomenon
    • Typically seen in the retroperitoneum or deep soft tissues, although can occur in the skin and superficial soft tissues
    • Features reminiscent of both capillary hemangioma and Kaposi sarcoma (Am J Surg Pathol 2004;28:559)
    • Tumor nodules composed of fascicles of spindle shaped endothelial cells and slit-like or crescentic vascular channels (Int J Clin Exp Pathol 2015;8:13711)
    • Tumor infiltrates soft tissue in a cannonball fashion
    • Signature feature of the lesion is glomeruloid structures, which comprise tightly coiled capillary vessels invested with pericytes
    • Glomeruloid structures are ERG+, CD31+ and CD34+
    • Lymphatic markers (PROX1, D2-40, VEGFR3) are highly expressed in the slit-like vessels, whereas the glomeruloid structures lack these antigens (Am J Surg Pathol 2010;34:1563)
    • HHV8-
    • GLUT1-
    • No tendency to regress
  • Epithelioid hemangioendothelioma:
    • Histologic hallmark of these tumors is the presence of chains and cords of round or slightly spindled eosinophilic endothelial cells with primitive lumen formation by single cells (Cancer 1982;50:970, Adv Anat Pathol 2014;21:254)
    • Presence of focal vascular channels
    • Distinct lack of mitotic activity or significant pleomorphism in most cases
    • Myxohyaline stroma
    • Recurrent translocations involving chromosomal regions 1p36.3 and 3q25, resulting in the formation of a WWTR1-CAMTA1 fusion gene in approximately 90% of cases
    • A small subset (< 5%) have a YAP1-TFE3 fusion gene
    • CAMTA1+, ERG+, CD31+, CD34+
    • Most of CAMTA1 - tumor express TFE3 (Am J Surg Pathol 2016;40:94)
  • Kaposi sarcoma:
    • The disease is uniformly associated with human herpesvirus (HHV8) infection (Science 1994;266:1865, N Engl J Med 2019;380:1181)
    • Patch stage:
      • Abnormal vessels lined by thin endothelial cells dissecting the dermis
      • Ramifying proliferating vessels often surround larger ectatic vessels, producing the so called promontory sign (not pathognomonic for Kaposi sarcoma)
    • Plaque stage:
      • All characteristic of the patch stage are exaggerated
      • Vascular spaces show jagged outlines
    • Nodular stage:
      • Well circumscribed, cellular nodules of intersecting fascicles of spindle cells and numerous slit-like spaces containing erythrocytes
    • Variable chronic inflammatory infiltrate composed of lymphocytes, plasma cells, and dendritic cells
    • Several histologic variants of Kaposi sarcoma have been described, including anaplastic Kaposi sarcoma (Arch Pathol Lab Med 2013;137:289)
    • ERG+, CD31+, D2-40+, HHV8+
  • Angiosarcoma (J Clin Pathol 2017;70:917):
    • Wide range of morphological appearances ranging from areas of well formed, anastomosing vessels to solid sheets of epithelioid or spindled cells without clear vasoformation
    • Multiple patterns may be present in the same tumor
    • Vast majority are high grade neoplasms with brisk mitotic activity, significant nuclear atypia and coagulative necrosis
    • Amplification of MYC in 8q24 is a consistent hallmark of radiation induced and chronic lymphedema associated angiosarcoma (Genes Chromosomes Cancer 2011;50:25, Am J Pathol 2010;176:34)
    • ERG+, CD31+, CD34+, FLI1+, AE1 / AE3 variable, CAM5.2 variable
Additional references
Board review style question #1

Which of the following is true about composite hemangioendothelioma?

  1. Harbors WWTR1-CAMTA1 fusions
  2. Harbors ZFP36-FOSB fusions
  3. HHV8+
  4. Typically does not recur after excision
  5. Distinguishing from other vascular neoplasms largely lies in finding different histologic vascular patterns within 1 lesion
Board review style answer #1
E. Distinguishing composite hemangioendothelioma from other vascular neoplasms largely lies in finding different histologic vascular patterns within 1 lesion

Comment here

Reference: Composite hemangioendothelioma
Board review style question #2
A 65 year old man presents with a poorly circumscribed, infiltrative lesion of the scalp, which is centered in the dermis and subcutis. H&E stained tissue sections demonstrate a complex admixture of vascular components, which include arborizing channels simulating the rete testis, chains and cord of eosinophilic epithelioid cells intermixed with large adipocyte-like cells, capillary sized vessels lined by epithelioid endothelial cells and superficial dilated vessels lined by small endothelial cells protruding into the lumina. Irregular anastomosing channels lined by mildly atypical endothelial cells are also noted. Rare mitoses are seen (up to 2 mitoses/10 high power fields). Immunohistochemically the lesional cells have strong expression of ERG and CD31 while are negative for HHV8. Which of the following is most likely the correct diagnosis?

  1. Angiosarcoma
  2. Composite hemangioendothelioma
  3. Epithelioid hemangioendothelioma
  4. Kaposi sarcoma
  5. Pseudomyogenic hemangioendothelioma
Board review style answer #2
B. Composite hemangioendothelioma

Comment here

Reference: Composite hemangioendothelioma

Cystic / cavernous lymphangioma
Definition / general
  • Benign vascular lesion composed of a collection of dilated lymphatic channels that may be superficial, deep or diffusely involve organ systems
  • First described by Rodender in 1828 (Arch Pathol Lab Med 2015;139:278)
Essential features
  • Benign proliferation of lymphatic vessels
  • Immunohistochemical expression of CD31 and D2-40
Terminology
  • Lymphatic malformation
  • Lymphangioma circumscriptum (superficial cutaneous lymphangioma)
  • Cavernous lymphangioma (deep lymphangioma)
  • Cystic hygroma (cystic lymphangioma)
  • Intra-abdominal cystic lymphangioma
  • Hemangiolymphangioma
  • Lymphangiomatosis (generalized lymphangioma, systemic angiomatosis)
ICD coding
  • ICD-O:
  • ICD-11:
    • LA90.12 & XH9MR8 - lymphatic malformations of certain specified sites & lymphangioma, NOS
    • 2E81.10 - disseminated lymphangiomatosis
Epidemiology
Sites
Pathophysiology
Etiology
  • Early or congenital lesions are favored to be developmental malformations
  • Sequestered lymphatics fail to communicate with normal lymphovascular system
  • Most are considered to be malformations / hamartomas (not true neoplasms)
  • Genetic abnormalities play a role (Virchows Arch 2008;453:1)
Clinical features
  • Superficial lesions may present as multiple small, grouped vesicular lesions involving the skin
  • Tongue lesions present as a mass surfaced by pebbly, vesicle-like nodules said to resemble frog eggs (Head Neck Pathol 2020;14:512)
  • Cystic hygroma usually presents as a unilateral, diffuse, nonpulsatile, painless swelling of the posterior cervical triangle
  • Deeper lesions may present as a large, slow growing painless mass
  • Intra-abdominal lesions can displace organs and cause intestinal obstruction
Diagnosis
  • Swollen soft tissue mass with positive transillumination test
  • Ultrasound can confirm cystic or multicystic lesion
  • Diagnosing these lesions may be very challenging in small biopsies given their cystic nature and bland cytologic features
Radiology description
  • Xray:
  • Ultrasound:
    • Unilocular or multilocular anechoic mass
    • Sharply defined cystic of multicystic mass with internal septations
    • May be diagnosed on prenatal ultrasound (Prenat Diagn 1988;8:405)
  • CT findings:
    • Nonenhancing cystic lesions
    • Septated cystic mass of variable size (Pediatr Radiol 2002;32:88)
    • Contains fluid of homogeneous density
    • May displace adjacent organs
  • MRI findings:
    • T1 weighted image may be hypointense or hyperintense when filled with hemorrhage or proteinaceous material (Magn Reson Imaging 2003;21:81)
    • T2 weighted image shows a multiloculated mass with hyperintense areas
Radiology images

Contributed by Jeanne Meis, M.D.

Radiograph of left lower extremity

Angiogram of
lymphangiomatosis

MRI of left extremity

MRI of lower abdomen

Radiograph of lower extremity


MRI of mediastinal lymphangioma

Radiograph of mediastinal lymphangioma

Arteriogram of lymphangioma



Images hosted on other servers:

CT of splenic lymphangioma

CT of orbital lymphangioma

Ultrasound of mesenteric lymphangioma

Prognostic factors
  • Benign lesions with excellent prognosis
  • Recurrence is high with incomplete removal
  • When involving deeper tissue planes, lesions can recur in up to 20% of patients (J Pediatr Surg 1992;27:220)
  • Complications include infection, hemorrhage, rupture and intestinal obstruction
  • Diffuse lymphangiomatosis with mediastinal or visceral organ involvement can be fatal
  • No malignant transformation has been reported
Case reports
Treatment
  • Surgical resection may be indicated for large, symptomatic lesions
  • Intralesional injection of sclerosing agents, including bleomycin and OK-432
  • Radiofrequency ablation (Int J Pediatr Otorhinolaryngol 2008;72:953)
Clinical images

Contributed by Jeanne Meis, M.D.

Lower extremity



Images hosted on other servers:

Cystic hygroma of neck

Lymphangioma of dorsal tongue

Omentum: 30 cm tumor in 4 year old girl


Scrotal tumor with hemorrhage in 3 year old boy

Abdominal cyst: 25 cm

Broad ligament: 19 kg tumor in 70 year old woman

Mesentery: 1 year old boy


Thigh tumor

Mesenteric tumor: 12 year old girl

Mesenteric tumors


Mesenteric tumors

Gross description
  • Multicystic and spongy lesions
  • Reddish / brown translucent cystic mass
  • Cystic spaces often contain watery, thick or milky fluid (Hum Pathol 2005;36:426)
Gross images

Contributed by Jeanne Meis, M.D.

Resection of lymphangiomatosis

Mediastinal lymphangioma



Images hosted on other servers:

Splenic lymphangioma

Microscopic (histologic) description
  • Variably sized, thin walled, dilated lymphatic vessels lined by flattened endothelium
  • Frequently surrounded by lymphoid aggregates, sometimes with reactive germinal centers
  • Lumina may contain eosinophilic and amorphous proteinaceous fluid with occasional lipid laden macrophages and lymphocytes
  • Longstanding lesions may show interstitial fibrosis
  • Walls of larger vessels may contain smooth muscle
  • Stromal mast cells and hemosiderin deposits are frequently seen
  • Lining of the cysts may rarely form papillary projections
  • Lymphangiomatosis frequently shows an anastomosing growth pattern, dissecting around normal structures
  • Extensive granulation tissue and inflammation may obscure lymphatic nature (Hum Pathol 2005;36:426)
Microscopic (histologic) images

Contributed by Laura Warmke, M.D. and Jeanne Meis, M.D.
Dilated lymphatic spaces

Dilated lymphatic spaces

Focal areas of fibrosis

Focal areas of fibrosis

Proteinaceous fluid Proteinaceous fluid

Proteinaceous fluid

Lymphatic channels

Lymphatic channels

Retroperitoneal lymphanagioma

Retroperitoneal lymphanagioma


Retroperitoneal lymphanagioma Retroperitoneal lymphanagioma

Retroperitoneal lymphanagioma

Lymphangioma circumscriptum

Lymphangioma circumscriptum

Lymphangioma of tongue Lymphangioma of tongue Lymphangioma of tongue

Lymphangioma of tongue


Superficial lymphangioma of vulva Superficial lymphangioma of vulva

Superficial lymphangioma of vulva

Superficial lymphangioma

Superficial lymphangioma

Scattered macrophages

Scattered macrophages

Lymphoid aggregate Lymphoid aggregate

Lymphoid aggregate


Lymphoid aggregates Lymphoid aggregates

Lymphoid aggregates

Calcification

Calcification

Thick wall

Thick wall

Endothelial lining

Endothelial lining

Irregular vascular channels

Irregular vascular channels


Irregular vascular channels

Irregular vascular channels

Trichrome stain

Trichrome stain

D2-40

D2-40

CD31

CD31

Cytokeratin

Cytokeratin

Virtual slides

Images hosted on other servers:

Lymphangioma involving adrenal gland

Buccal lymphangioma

Positive stains
Negative stains
Electron microscopy description
  • May assist in determining endothelial origin of cells lining the cysts
  • Presence of Weibel-Palade bodies, storage granules of endothelial cells
Molecular / cytogenetics description
Videos

Lymphangioma (cystic hygroma)

Sample pathology report
  • Soft tissue, retroperitoneal mass, resection:
    • Cavernous lymphangioma (see comment)
    • Comment: Sections show a lesion composed of dilated lymphatic spaces lined by flattened endothelium without cytologic atypia. Immunohistochemical stains show that the endothelial cells are positive for D2-40 and CD31, supporting a lymphatic nature, while they are negative for cytokeratin. These results, along with the morphologic features, support the above diagnosis.
Differential diagnosis
Board review style question #1

Cystic hygroma (cystic lymphangioma), which frequently involves the posterior triangle of the neck, is associated with which of the following syndromes?

  1. Down syndrome
  2. Klippel-Trenaunay syndrome
  3. Maffucci syndrome
  4. Turner syndrome
Board review style answer #1
D. Turner syndrome

Comment Here

Reference: Cystic / cavernous lymphangioma
Board review style question #2

Which of the following immunohistochemical stains, when positive, is the most helpful in making the diagnosis of lymphangioma?

  1. Calretinin
  2. Cytokeratin
  3. D2-40
  4. GLUT1
Board review style answer #2

Dedifferentiated liposarcoma
Definition / general
  • Well differentiated liposarcoma (WDL) with transition, either in the primary tumor or as a recurrence to a sarcoma that is typically nonlipogenic
  • Most cases are intermediate to high grade; the existence of low grade dedifferentiated liposarcoma, although now officially recognized, has been controversial
Essential features
  • Nonlipogenic sarcoma (typically) that arises from well differentiated liposarcoma
    • Precursor well differentiated liposarcoma may or may not be identifiable in the background
  • Both low and high grade classifications are now recognized (see Terminology)
  • Molecularly characterized by ring or giant marker / rod chromosomes composed of material from 12q13-15
    • Results in localized amplification of several neighboring genes, including MDM2
  • Rarely, the high grade component can be lipogenic and resemble pleomorphic liposarcoma (homologous lipoblastic differentiation) (Am J Surg Pathol 2010;34:1122)
  • First tumor type to consider in cases of high grade sarcoma of the retroperitoneum in an adult
    • Retroperitoneal tumors historically regarded as inflammatory subtype of malignant fibrous histiocytoma (MFH) are now considered to represent a common morphologic pattern of dedifferentiation in dedifferentiated liposarcoma
Terminology
  • Although originally these tumors were all considered high grade, low grade dedifferentiation is now a recognized phenomenon
    • Low grade dedifferentiation can be histologically indistinguishable from cellular well differentiated liposarcoma; the terminology that is used varies among practicing pathologists
    • Although some report a comparable prognosis for well differentiated and low grade dedifferentiated cases, others report an unfavorable outcome with any percentage or degree of dedifferentiation (Am J Surg Pathol 2007;31:1, Am J Surg Pathol 1997;21:271)
  • Minimal dedifferentiation
    • Although controversial, some require macroscopic evidence of dedifferentiation (> 1.0 cm) to label a tumor as truly dedifferentiated
    • Even cases with so called minimal dedifferentiation (< 1.0 cm) can still carry an inferior prognosis when compared with well differentiated liposarcoma; prolonged clinical follow up is recommended regardless of size
ICD coding
  • ICD-O: 8858/3 - dedifferentiated liposarcoma
  • ICD-11: 2B59 & XH1C03 - liposarcoma, primary site & dedifferentiated liposarcoma
Epidemiology
  • Typically occurs in older adults, with a slight predilection for men
  • Occurs in up to 10% of well differentiated liposarcomas, with more frequent dedifferentiation noted in retroperitoneal primaries
Sites
  • Most common site is the retroperitoneum, followed by the extremities
  • Other frequent sites include the spermatic cord
  • Rare in the head and neck
  • Extremely rare in the subcutis; as a general rule, subcutaneous atypical lipomatous tumors do not dedifferentiate
Diagnosis
  • Imaging studies are recommended for initial diagnostic workup and may show both fatty and nonfatty components
  • Percutaneous biopsy has low sensitivity for detecting dedifferentiated liposarcoma (Ann Surg Oncol 2015;22:1068)
  • Surgical resection is recommended for accurate tumor classification
Radiology description
  • Characterized by a nonlipomatous mass contiguous with a well demarcated fatty component
  • On MRI, nonfatty regions typically show low signal intensity on T1 weighted images, high signal intensity on T2 weighted images and marked enhancement after IV gadolinium (Eur J Radiol 2004;50:257)
  • Other imaging features suggestive of dedifferentiation include central necrosis, contrast enhancement and increased FDG uptake on PET / CT (J Comput Assist Tomogr 2016;40:872, Sarcoma 2020;2020:8363986)
  • Calcification on imaging may suggest heterologous osteosarcomatous dedifferentiation (AJR Am J Roentgenol 2011;197:W37)
Radiology images

Images hosted on other servers:
Large retroperitoneal mass with central necrosis

Large retroperitoneal mass with central necrosis

Fatty mass with enhancing nodule on CT scan

Fatty mass with enhancing nodule on CT scan

Heterologous osteosarcomatous differentiation

Heterologous
osteosarcomatous
differentiation

Prognostic factors
  • Better prognosis than other high grade pleomorphic sarcomas but still recurs in 40 - 75% of cases, metastasizes in 10 - 15% of cases and is associated with a 28% mortality rate (Am J Surg Pathol 1994;18:1213, Am J Surg Pathol 1997;21:271)
  • Higher Fédération Nationale des Centres de Lutte Contre le Cancer (FNCLCC) grade and myogenic differentiation are associated with a worse clinical outcome (Am J Surg Pathol 2015;39:383)
    • A modified FNCLCC grade, which assigns a tumor differentiation score (1 - 3) according to the dedifferentiated component of the tumor (in contrast to the standard FNCLCC grading system, which assigns a tumor differentiation score of 3 to all dedifferentiated liposarcomas) has been shown to predict local recurrence, supporting the hypothesis that tumor behavior is influenced by the degree of differentiation within the dedifferentiated component (Mod Pathol 2015;28:37)
  • Metastasis can occur in cases with low and high grade dedifferentiation and there is no minimal amount of dedifferentiation that mitigates this risk (Am J Surg Pathol 1997;21:271)
  • Cases with myxofibrosarcoma-like features are particularly aggressive (Mod Pathol 2005;18:976)
  • Retroperitoneal dedifferentiated liposarcoma has higher rates of local recurrence and disease specific death (Curr Opin Oncol 2011;23:373)
    • If followed long enough, nearly all retroperitoneal tumors will recur
  • Must rule out dedifferentiated liposarcoma in any retroperitoneal sarcoma; this can be aided by
    • Adequate sampling, particularly of peripheral areas
    • Cytogenetics
  • Immunostaining for MDM2 and CDK4 or molecular testing for 12q13-15 amplification
Case reports
Treatment
  • Resectable disease is treated with surgery to obtain negative margins (NCCN Guidelines: Neuroendocrine and Adrenal Tumors [Accessed 28 March 2023])
  • Neoadjuvant radiation therapy can be considered in selected patients at high risk for local recurrence but adjuvant radiation therapy is discouraged for retroperitoneal / intra-abdominal sarcoma
  • Palbociclib (CDK4 inhibitor) may be useful as single agent therapy for the treatment of unresectable tumors
Clinical images

Images hosted on other servers:
Intraoperative image of axillary tumor

Intraoperative image of axillary tumor

Gross description
  • Large firm mass (may resemble fish flesh) with coarse lobulation that can be surrounded by the more grossly fatty appearing well differentiated component
  • Dedifferentiation can be discrete and nodular or more gradual (Semin Diagn Pathol 2019;36:112)
  • Look for foci of necrosis in the high grade component
Gross images

Images hosted on other servers:
Whitish, elastic, hard nodule

Whitish, elastic, hard nodule

Microscopic (histologic) description
  • High grade dedifferentiated liposarcoma
    • Well differentiated and dedifferentiated components are often both present and can have abrupt or gradual transitions
    • Dedifferentiated component is a cellular and typically a nonlipogenic sarcoma with significant pleomorphism
    • Although some propose a mitotic rate of > 5 mitoses/10 high power fields, this is not uniformly adopted (Am J Surg Pathol 2023;47:649)
    • Often resembles malignant fibrous histiocytoma (MFH), now referred to as undifferentiated pleomorphic sarcoma (UPS), with short fascicles of pleomorphic spindle cells associated with mixed inflammatory infiltrate
    • Can show a peculiar whirling pattern reminiscent of meningothelial structures (Histopathology 1998;33:414, Am J Surg Pathol 1998;22:945)
    • Heterologous elements in 5 - 10% of cases
      • Heterologous elements can easily mislead pathologists in poorly sampled cases, especially in metastatic sites
      • Can manifest as neural differentiation, leiomyosarcoma, osteosarcoma / chondrosarcoma, rhabdomyosarcoma or pleomorphic liposarcoma (homologous lipoblastic dedifferentiation)
      • Rhabdomyoblastic differentiation has been associated with worse outcomes
      • Angiosarcomatous differentiation has been reported (Virchows Arch 2005;446:456)
      • Myxoid liposarcoma-like morphologic features may exist in a subset of cases (often with coexisting amplification of DDIT3) and are important to recognize as a unique histologic pattern of dedifferentiated liposarcoma to avoid misclassification as myxoid liposarcoma (Mod Pathol 2019;32:585)
  • Low grade dedifferentiated liposarcoma
    • Less common low grade tumor resembling fibromatosis or well differentiated fibrosarcoma
    • Nonlipogenic (in contrast to well differentiated spindle cell liposarcoma, which contains atypical fat / lipoblasts)
    • Considered by some to be the same as cellular atypical lipomatous tumor, although there is evidence that even low grade dedifferentiation is associated with a poorer prognosis when compared with conventional atypical lipomatous tumor
    • Now a recognized WHO classification
  • Pitfalls and tips
    • Look at the edges of dedifferentiated liposarcoma to identify a rim of background well differentiated liposarcoma that may mimic compressed background fat with reactive change
    • Dedifferentiated liposarcoma often shows a significant amount of heterogeneity (multiple patterns of differentiation and growth) and the confusing nature of the tumor can be a clue to the correct diagnosis
    • Imaging may show both a fatty and solid nonfatty component
    • Metastases often contain only the dedifferentiated component
    • When in doubt use ancillary testing to confirm either protein overexpression or gene amplification
Microscopic (histologic) images

Contributed by Michael R. Clay, M.D. and AFIP
Low grade component

Low grade component

Delicate branching vasculature

Delicate branching vasculature

High grade component with entrapped lipoblasts

High grade component with entrapped lipoblasts

Meningothelial-like whirls Meningothelial-like whirls

Meningothelial-like whirls

Dedifferentiated liposarcoma

Dedifferentiated liposarcoma


Fibrosarcoma-like areas

Fibrosarcoma-like areas

Hemangiopericytoma type pattern

Hemangiopericytoma
type pattern

MFH-like areas

MFH-like areas

Neural appearance

Neural appearance

Rhabdomyosarcomatous elements

Rhabdomyo-
sarcomatous
elements

Storiform pattern

Storiform pattern

Virtual slides

Images hosted on other servers:
Dedifferentiated liposarcoma

Dedifferentiated liposarcoma

Cytology description
  • Hypercellular with multinucleated, pleomorphic giant cells with abundant cytoplasm, small clusters of cells with high N:C ratio, spindled cells with elongated nuclei (Acta Cytol 2001;45:641)
  • Occasional osteoclast type giant cells (Cytojournal 2010;7:5)
  • Most samples are suitable for molecular confirmation
Cytology images

Images hosted on other servers:
FNAB of dedifferentiated liposarcoma

FNAB of dedifferentiated liposarcoma

Positive stains
Molecular / cytogenetics description
Molecular / cytogenetics images

Images hosted on other servers:
FISH studies performed on FNA cell block sections

FISH studies performed on FNA cell block sections

Videos

Dedifferentiated liposarcoma with heterologous osteosarcoma component

Dedifferentiated liposarcoma

Sample pathology report
  • Retroperitoneal mass, resection:
    • Dedifferentiated liposarcoma (10.0 cm) (see comment)
    • Resection margins are negative for tumor
    • Comment: Histologic sections of this retroperitoneal mass show a malignant spindle cell lesion with prominent scattered pleomorphic cells and well differentiated adipose tissue at the periphery. The malignant cells show strong nuclear expression of MDM2 by immunohistochemistry, supporting the diagnosis.
Differential diagnosis
  • Leiomyosarcoma:
    • Even in pleomorphic cases there are usually areas with morphology distinctive of well differentiated leiomyosarcoma
    • There is no background well differentiated liposarcomatous component and no 12q13-15 amplification
    • Immunostaining can be supportive but beware of desmin / actin expression in dedifferentiated liposarcoma
  • Malignant peripheral nerve sheath tumor (MPNST):
    • No well differentiated liposarcomatous component
    • History of neurofibromatosis can be helpful
    • Be wary of molecular / immunohistochemical testing as 20% of MPNST can have MDM2 gene amplification and protein overexpression
    • More commonly located in the deep soft tissues of the extremities than in the retroperitoneum
    • Typically more uniform in growth pattern than dedifferentiated liposarcoma, although the distinction may be very difficult
  • Melanoma:
    • Look for a clinical history of melanoma
    • Immunohistochemical staining can be informative with S100 being consistently positive, although other melanocytic markers are frequently negative (HMB45 and MelanA)
  • Pleomorphic liposarcoma (PLS):
    • Has lipoblasts set in the background of a pleomorphic sarcoma
    • Only way to distinguish PLS from dedifferentiated liposarcoma with secondary homologous differentiation is to either identify background well differentiated liposarcoma or verify MDM2 overexpression or amplification by ancillary studies (Cancer Cytopathol 2014;122:128)
      • This is an important distinction as pleomorphic liposarcoma has a significantly worse prognosis
  • Rhabdomyosarcoma:
    • No well differentiated liposarcomatous component, no 12q13-15 amplification or MDM2 protein overexpression
    • Rhabdomyoblastic stains can be unreliable given heterologous differentiation in dedifferentiated liposarcoma
  • Other sarcoma subtype infiltrating background adipose tissue:
    • Try to look in areas outside of the main tumor bulk to identify a well differentiated liposarcoma component; imaging can be particularly useful in showing a well differentiated fatty component associated with the higher grade sarcoma
    • Molecular and immunohistochemical studies are helpful
      • This may be particularly necessary when the background entrapped fat takes on a reactive appearance with fat necrosis, as this can closely mimic an underlying well differentiated liposarcoma
  • Sarcomatoid carcinoma:
    • Look for epithelial markers (EMA / cytokeratin) and clinical history indicative of a primary epithelial malignancy
    • No well differentiated liposarcomatous component, no 12q13-15 amplification
  • Sarcomatoid mesothelioma:
  • Undifferentiated pleomorphic sarcoma (UPS) / malignant fibrous histiocytoma (MFH):
    • Retroperitoneal tumors are dedifferentiated liposarcoma until proven otherwise
      • Be very wary of this diagnosis in that location
    • Although UPS may have a complex karyotype, MDM2 gene amplification would indicate the diagnosis of dedifferentiated liposarcoma (Mod Pathol 2003;16:256)
Board review style question #1

A 10 cm retroperitoneal mass was surgically resected with the histologic appearance shown in the image above. Which one of the following ancillary tests would best help support the diagnostic impression?

  1. FISH positive for DDIT3 gene rearrangement
  2. FISH positive for EWSR1 gene rearrangement
  3. FISH positive for FOXO1 gene rearrangement
  4. FISH positive for MDM2 gene amplification
  5. FISH positive for MDM2 gene rearrangement
Board review style answer #1
D. FISH positive for MDM2 gene amplification. The microscopic image demonstrates a proliferation of variably sized adipocytes admixed with scattered enlarged, hyperchromatic tumor cells that abruptly transitions to a cellular, spindle cell component with significant pleomorphism. These are the key histologic features of dedifferentiated liposarcoma. Detection of MDM2 amplification by FISH is a useful ancillary tool to confirm the diagnosis of both well differentiated and dedifferentiated liposarcoma. It is important to recognize, however, that MDM2 gene amplification is not entirely specific for well differentiated and dedifferentiated liposarcoma as it can also be found in other tumors, including intimal sarcoma, parosteal osteosarcoma, low grade central osteosarcoma, malignant peripheral nerve sheath tumor, endometrial stromal sarcoma and sclerosing rhabdomyosarcoma.

Comment Here

Reference: Dedifferentiated liposarcoma
Board review style question #2
Which of the following is associated with an adverse prognosis in patients with dedifferentiated liposarcoma?

  1. Age > 65 years old
  2. Amount of the dedifferentiated component
  3. Heterologous differentiation into rhabdomyosarcoma
  4. Located in the extremity
  5. Lower FNCLCC grade
Board review style answer #2
C. Heterologous differentiation into rhabdomyosarcoma. Rhabdomyoblastic differentiation and a higher FNCLCC grade have been associated with worse clinical outcomes, whereas the amount and grade of the dedifferentiated component have not. Overall, anatomic location is the most important prognostic factor in dedifferentiated liposarcomas, with retroperitoneal tumors behaving the most aggressively.

Comment Here

Reference: Dedifferentiated liposarcoma

Deep fibrous histiocytoma
Definition / general
  • Fibrous histiocytoma of subcutaneous tissue, deep soft tissue or parenchymal organs, with no dermal involvement (Am J Surg Pathol 1994;18:677, Am J Surg Pathol 1990;14:801, Stanford Medicine)
  • Rare painless lesions, < 1% of fibrohistiocytic tumors
  • Usually adult males > 25 years (range 6 - 84 years)
  • Usually extremities, head and neck, trunk; rarely in deep soft tissue of retroperitoneum, mediastinum or pelvis
Case reports
Treatment
Clinical images

Images hosted on other servers:

Nodule in arch of foot

Gross description
  • Well circumscribed with pseudocapsule, typically 4 cm (range 0.5 to 25 cm), variable hemorrhage
Microscopic (histologic) description
  • Prominent storiform pattern of uniform spindle cells with ill defined eosinophilic cytoplasm and bland, elongated or plump vesicular nuclei with no atypia
  • Often hemangiopericytoma-like vasculature
  • Scattered lymphocytes, either multinucleated giant cells, osteoclastic giant cells or foam cells in 59%
  • Usually less than 5 mitotic figures / 10 HPF
  • Stroma is myxoid or hyaline
  • Borders are non-infiltrative, with no trapping of fat cells
  • Necrosis or angiolymphatic invasion are rare
Microscopic (histologic) images

AFIP images

Circumscribed tumor with no trapped fat

Cellular tumor

Tumor with more fibrous stroma

Foam cells help distinguish from DFSP



Focal storiform pattern and cellular uniformity

Hemangiopericytoma-like vascular pattern



Case #360



Images hosted on other servers:

Various images

Positive stains
Negative stains
Differential diagnosis

Dermal nerve sheath myxoma
Definition / general
  • Benign tumor, usually cutaneous, derived from Schwann cells (peripheral nerve sheath) but without the characteristic features of conventional schwannoma
  • Probably a myxoid variant of schwannoma
Essential features
  • Benign tumor with Schwannian differentiation
  • Multilobulated and myxoid without the characteristic features of conventional schwannoma
  • Diffuse S100 positivity is key to distinguishing it from neurothekeoma
Terminology
  • Historically it has been equated with the classic or myxoid variant of neurothekeoma but given their altogether different clinical, immunohistochemical, ultrastructural and molecular features, that is no longer considered to be true by most authors
ICD coding
  • ICD-O: 9562/0 - Neurothekeoma, nerve sheath myxoma
Epidemiology
  • All ages but mainly young and middle aged adults are affected
  • M = F
  • More than 5 times rarer than neurothekeoma
Sites
  • Extremities (85%), most often the fingers
  • Only rarely does it involve the head and neck region
Etiology
  • Unknown
Clinical features
  • A solitary, small, mobile and flesh colored to translucent nodule, undergoing slow, usually painless growth
  • Frequently presents as a longstanding mass (up to more than 20 years) (Am J Surg Pathol 2005;29:1615)
  • Nonpolarized contact dermoscopy showed diffuse gray to yellow background, cobblestone-like areas, and gray structureless areas with hairpin, linear and twisted vessels around them (Clin Exp Dermatol 2019;44:206)
Diagnosis
  • No specific clinical diagnostic features have been described associated with this entity
  • Diagnosis is made by pathology examination in excised lesions
Clinical images

Images hosted on other servers:

Eyelid nodule

Tongue nodule

Scalp swelling

Case reports
Treatment
  • Complete local excision with free margins and subsequent follow up is advisable, as a high recurrence rate (47%) was found to be associated with simple excision (Am J Surg Pathol 2005;29:1615)
  • It is an otherwise totally benign tumor
Gross description
  • Majority < 2.5 cm in greatest axis (0.4 - 4.5 cm)
  • Rubbery to firm consistency
  • Well demarcated, translucent to white, mucoid and glistening upon sectioning
Gross images

Images hosted on other servers:

Perianal tumor

Cervical tumor

Microscopic (histologic) description
  • Unencapsulated multilobulated tumor occupying dermis or subcutis
  • Well circumscribed myxoid lobules with fibrous rims
  • Neoplastic cells are small and may be spindled, stellate, epithelioid and ring shaped (often with cytoplasmic nuclear invaginations)
  • Mild nuclear atypia at the most, with rare or no mitotic figures
  • Scant eosinophilic cytoplasm envelops small round nuclei, with thin bipolar or multipolar (spider-like) processes
  • Cellular interconnections are common so cord-like, nested and syncytial arrangements may develop
  • A hyperplastic and hyperpigmented epidermal reaction, similar to benign fibrous histiocytomas, was reported (Clin Exp Dermatol 2019;44:206)
Microscopic (histologic) images

Contributed by Instituto Português de Oncologia de Lisboa (IPOLFG)

Vulvar skin nodule

Nuclear cytoplasmic inclusion

Cytoplasmic processes

Diffuse S100

Focal AE1 / AE3

Negative SMA

Cytology description
  • Fine needle aspiration yields a greyish jelly-like material
  • Isolated round to stellate cells with bland nuclei
  • Metachromatic fibrillary myxoid stroma (pink on Diff-Quik stain and pale blue on Papanicolaou stain)
  • Some cells may appear in cords and loose clusters (Diagn Cytopathol 2002;27:173)
  • Multiple elongated cytoplasmic processes may be apparent (J Clin Diagn Res 2015;9:ED07)
  • Binucleated and multinucleated cells may be present
  • Consider also the differential diagnoses of myxoid neurofibroma, myxoid schwannoma, myxoid neurothekeoma and cutaneous mucinosis, from which it is usually impossible to distinguish on cytology grounds
Positive stains
Negative stains
Molecular / cytogenetics description
  • Very similar molecular genetic signatures to dermal schwannoma, in contrast to that of neurothekeoma, which more closely resembles cellular fibrous histiocytomas (Mod Pathol 2011;24:343)
Sample pathology report
  • Finger, left middle, excision:
    • Dermal nerve sheath myxoma, 0.8 cm
    • Surgical margins, negative for tumor
Differential diagnosis
  • Neurothekeoma (especially myxoid and mixed variants)
    • More often affects females and the head and neck region
    • Multilobulated or micronodular tumor but with spindled and epithelioid cells
    • Nonspecific immunophenotype (ie, uncertain histogenesis): positive for CD63, NSE (89%) and sometimes at least focally for SMA (34 - 60%)
    • Negative for S100 and GFAP
    • Consistently positive for CD10 (Mod Pathol 2014;27:701)
  • Plexiform neurofibroma
    • Frequently associated with neurofibromatosis type 1
    • Tumor nodules are less compact
    • Stroma contains collagen fibers with shredded carrot appearance
    • Some cells will stain with CD34 (fibroblasts), others with S100 (Schwann cells)
  • Myxoid schwannoma
    • Usually encapsulated as a whole, not in individual lobules
    • Distinctive morphologic features include alternating Antoni A and B areas, nuclear palisading (Verocay bodies) and hyalinized vessels
  • Cutaneous myoepithelioma
    • May show a lobular, reticular or syncytial growth pattern, with myxoid stroma (although not as extensively so as the soft tissues)
    • Poorly circumscribed, often with an overlying hyperplastic epidermis
    • Expresses both S100 and GFAP, yet also AE1 / AE3 or EMA
  • Cutaneous myxoma (superficial angiomyxoma)
    • Reported in half of patients with Carney complex
    • Most common in the head, neck and trunk
    • Well circumscribed, vaguely lobulated tumor with prominent mucinous matrix and vascularity
    • Contains variably shaped fibroblasts but is less cellular
    • Accompanying inflammatory infiltrate containing neutrophils
    • Up to 25% of cases have entrapped epithelial structures
    • Expresses CD34 but not S100 (Am J Surg Pathol 1999;23:910)
  • Dermal mucinosis
    • Group of essentially disparate nonneoplastic diseases (except for cutaneous myxoma), having in common only dermal mucin deposition
    • May be cell poor, with few stellate-like forms (e.g. pretibial myxedema, scleredema), or show some degree of fibroblastic proliferation (e.g. scleromyxedema)
    • A perivascular lymphocytic infiltrate is usually present
    • No sharply demarcated lobules
Board review style question #1
    Which of the following tumors usually does not have an S100 staining like the one depicted?

  1. Schwannoma
  2. Malignant melanoma
  3. Dermal nerve sheath myxoma
  4. Paraganglioma
  5. Epithelioid malignant peripheral nerve sheath tumor
Board review style answer #1
D. Paraganglioma

Comment Here

Reference: Dermal nerve sheath myxoma
Board review style question #2
    Which of the following is a purely neural tumor?

  1. Neurofibroma
  2. Dermal nerve sheath myxoma
  3. Neurothekeoma
  4. Chondroid syringoma
  5. Benign Triton tumor
Board review style answer #2
B. Dermal nerve sheath myxoma

Comment Here

Reference: Dermal nerve sheath myxoma

Dermatofibrosarcoma protuberans (DFSP)
Definition / general
  • Locally aggressive, superficial mesenchymal neoplasm with fibroblastic differentiation
Essential features
  • Locally aggressive, superficial mesenchymal neoplasm with fibroblastic differentiation
  • Virtually all cases contain fusion genes; COL1A1::PDGFB is the most common fusion product, although others have recently been reported
  • Fibrosarcomatous transformation imparts an increased risk of recurrence and metastasis
ICD coding
  • ICD-O:
    • 8832/3 - dermatofibrosarcoma protuberans, NOS
    • 8833/3 - pigmented dermatofibrosarcoma protuberans
    • 8834/1 - giant cell fibroblastoma
  • Fibrosarcomatous dermatofibrosarcoma protuberans: no distinct coding identified
Epidemiology
Sites
  • Can ostensibly involve any area of skin but the trunk and extremities are the most common locations
Pathophysiology
  • Tumors are generally presumed to occur sporadically
  • Virtually all cases contain fusion genes; COL1A1::PDGFB is the most common fusion product, although others have been reported
  • Possible association with adenosine deaminase deficient severe combined immunodeficiency (J Allergy Clin Immunol 2012;129:762)
Clinical features
  • Classically an exophytic, nodular cutaneous mass; however, often initially presents as a flat plaque (JAMA Netw Open 2019;2:e1910413)
  • Initially may show persistent slow growth, often for many years, then sudden progression (Cancer 1962;15:717)
  • Fibrosarcomatous transformation is associated with metastatic potential (Cancer 2000;88:2711)
Diagnosis
  • Tumors are morphologically distinctive and frequently amenable to classification based on H&E
  • Immunohistochemistry for CD34 is a useful adjunct since most cases are diffusely positive
  • Molecular testing is helpful, particularly in the context of limited sampling or unusual morphology
Radiology description
Radiology images

Images hosted on other servers:
Multiple examples from radiopaedia.org

Nodular soft tissue mass

Prognostic factors
  • Incomplete resection is a risk factor for recurrence
  • Fibrosarcomatous transformation (fibrosarcoma ex DFSP) imparts an increased risk of recurrence and metastasis (Am J Surg Pathol 2006;30:436)
  • Metastases typically occur following multiple local recurrences
  • Increased age, male sex and tumor size are associated with worse overall survival (JAMA Dermatol 2016;152:1365)
Case reports
Treatment
Clinical images

Images hosted on other servers:
Papulonodular thigh lesions

Papulonodular thigh lesions

Before and after imatinib treatment

Before and after imatinib treatment

Gross description
Gross images

Images hosted on other servers:
Scalp tumor

Scalp tumor

Microscopic (histologic) description
Microscopic (histologic) images

Contributed by Brendan C. Dickson, M.D., M.Sc.
Infiltration along fibrous septa

Infiltration along fibrous septa

Honeycomb pattern

Honeycomb pattern

Nuclear monomorphism

Nuclear monomorphism

Storiform pattern

Storiform pattern

Herringbone pattern in fibrosarcoma ex DFSP

Herringbone pattern in fibrosarcoma ex DFSP

Brisk mitotic activity in fibrosarcoma ex DFSP

Brisk mitotic activity in fibrosarcoma ex DFSP


Pigmented DFSP

Pigmented DFSP

Myxoid DFSP

Myxoid DFSP

DFSP post imatinib therapy

DFSP post imatinib therapy

Diffuse CD34 staining

Diffuse CD34 staining

Diminished CD34 in fibrosarcoma ex DFSP

Diminished CD34 in fibrosarcoma ex DFSP

Virtual slides

Images hosted on other servers:
35 year old man with lesion on right upper arm

35 year old man with lesion on right upper arm

81 year old woman with breast lesion

81 year old woman with breast lesion

40 year old man with lesion on abdomen

40 year old man with lesion on abdomen

27 year old man with lesion on abdomen

27 year old man with lesion on abdomen

Positive stains
Negative stains
Electron microscopy description
Molecular / cytogenetics description
Molecular / cytogenetics images

Images hosted on other servers:
Fusion product of <i>COL1A1</i> to <i>PDGFB</i> Fusion product of <i>COL1A1</i> to <i>PDGFB</i>

Fusion product of COL1A1 to PDGFB

<i>PDGFB</i> break apart FISH

PDGFB break apart FISH

Sample pathology report
  • Skin, back, biopsy:
    • Dermatofibrosarcoma protuberans (see comment)
    • Comment: Within the dermis and subcutis there is a spindle cell neoplasm with a storiform pattern. The cytoplasm is eosinophilic. The nuclei are ovoid and monomorphic with rare mitotic activity. There is sparing of adnexal structures and infiltration of the subcutaneous fat with a honeycomb pattern. The cells are diffusely positive for CD34; they are negative for desmin, smooth muscle actin, S100 and keratin (AE1 / AE3).
Differential diagnosis
  • Cellular fibrous histiocytoma:
    • Plump spindle cells with peripheral collagen entrapment
    • Inflammation is often present, including foamy macrophages, lymphocytes and plasma cells; occasionally multinucleated giant cells (J Cutan Pathol 2012;39:747)
    • Immunohistochemistry for CD34 may highlight peripheral dermal fibroblasts; rarely, tumors may be positive (J Cutan Pathol 2012;39:747)
  • Cutaneous leiomyosarcoma:
    • Plump spindle cells with a fascicular architecture
    • Ovoid / cigar shaped nuclei, atypical mitoses may be identified
    • Immunohistochemistry typically positive for desmin and h-caldesmon and negative for CD34
  • Solitary fibrous tumor:
    • Spindle cells with a patternless distribution; prominent branching vasculature
    • Keloid-like collagen bundles
    • Immunohistochemistry will also be positive for STAT6
Board review style question #1

Which immunohistochemical stain would be diffusely positive in typical dermatofibrosarcoma protuberans tumor (such as the one shown in the image above)?

  1. CD34
  2. Desmin
  3. HMB45
  4. Pankeratin
  5. S100
Board review style answer #1
A. CD34 is typically diffusely positive in cases of dermatofibrosarcoma protuberans while desmin, HMB45, pankeratin and S100 are not expressed.

Comment Here

Reference:Dermatofibrosarcoma protuberans (DFSP)
Board review style question #2
The presence of which of the following fusion genes can be used to support a diagnosis of dermatofibrosarcoma protuberans in a primary dermal spindle cell neoplasm?

  1. COL1A1::PDGFB
  2. EWSR1::FLI1
  3. FUS::DDIT3
  4. JAZF1::SUZ12
  5. MYB::NFIB
Board review style answer #2
A. COL1A1::PDGFB fusion product is present in most cases of dermatofibrosarcoma protuberans. It is important to note that additional fusion genes are possible but are missed by FISH or reverse transcription PCR assays that are restricted to PDGFB. It is also important to note that other tumors may harbor this fusion product (e.g., uterus, cervix). The other fusion gene options do not occur in dermatofibrosarcoma protuberans and are characteristic of other neoplasms:
  • EWSR1::FLI1: Ewing sarcoma
  • FUS::DDIT3: myxoid liposarcoma
  • JAZF1::SUZ12: low grade endometrial stromal sarcoma
  • MYB::NFIB: adenoid cystic carcinoma

Comment Here

Reference: Dermatofibrosarcoma protuberans (DFSP)
Board review style question #3

Which of the following is true regarding the entity shown above?

  1. Positive for Fontana-Masson and CD34
  2. Positive for Fontana-Masson and factor XIIIa
  3. Positive for Prussian blue and CD34
  4. Positive for Prussian blue and factor XIIIa
Board review style answer #3
A. Positive for Fontana-Masson and CD34. This is an example of pigmented dermatofibrosarcoma protuberans (Bednar tumor). The pigment is melanin, which is positive with the Fontana-Masson stain but not Prussian blue. The tumor cells are typically diffusely positive for CD34 but negative for factor XIIIa.

Comment Here

Reference: Dermatofibrosarcoma protuberans (DFSP)
Board review style question #4
The molecular pathogenesis of most cases of dermatofibrosarcoma protuberans is characterized by which of the following fusion gene products?

  1. COL6A3::CSF1
  2. COL1A1::PDGFB
  3. COL6A3::PDGFD
  4. COL1A1::USP6
Board review style answer #4
B. COL1A1::PDGFB. COL6A3::CSF1 is found in a subset of tenosynovial giant cell tumors. COL6A3::PDGFD is only rarely encountered in dermatofibrosarcoma protuberans. COL1A1::USP6 may be present in myositis ossificans, a fibro-osseous pseudotumor of the digits and aneurysmal bone cyst.

Comment Here

Reference: Dermatofibrosarcoma protuberans (DFSP)

Desmoplastic fibroblastoma
Definition / general
  • Fibroblastic lesion centered in subcutaneous tissue with reactive fibroblasts, low cellularity and abundant collagen (Hum Pathol 1998;29:676)
  • Also called collagenous fibroma
Epidemiology
  • Rare benign lesion of adult men (70% between ages 40 and 69 years)
Sites
  • Usually upper extremities, back, feet
Case reports
Treatment
  • Conservative excision, does not recur or metastasize
Gross description
  • Usually 1 - 4 cm, well circumscribed, may be lobulated, has firm and homogeneous gray cut surface resembling cartilage
Microscopic (histologic) description
  • Paucicellular, bland spindled (stellate) and reactive appearing fibroblasts and myofibroblasts separated by abundant collagen with variable myxoid stroma
  • Fibroblasts have amphophilic cytoplasm, vesicular nuclei and distinct nucleoli
  • 70% of cases involve subcutis, 25% extend into skeletal muscle
Microscopic (histologic) images

AFIP images

Large, reactive
appearing spindled
fibroblasts in
fibrous matrix

Focus of low cellularity and dense collagen

More cellular area with reactive type fibroblasts

Positive stains
Negative stains
Electron microscopy description
Molecular / cytogenetics description
Differential diagnosis

Desmoplastic small round cell tumor
Definition / general
  • Desmoplastic small round cell tumor (DSRCT) is a malignant mesenchymal neoplasm composed of small round tumor cells associated with prominent stromal desmoplasia, polyphenotypic differentiation and EWSR1-WT1 gene fusion
Essential features
  • Nests of small round cells in a desmoplastic stroma
  • Expression of cytokeratin, desmin and WT1 (antibody to carboxy terminus [C terminus])
  • EWSR1-WT1 fusion transcripts
Terminology
  • Intra-abdominal desmoplastic round cell tumor
ICD coding
  • ICD-0: 8806/3 - desmoplastic small round cell tumor
  • ICD-11: 2B5F.2 & XH5SN6 - sarcoma, not elsewhere classified of other specified sites & desmoplastic small round cell tumor
Epidemiology
Sites
Pathophysiology
  • Recurrent t(11;22)(p13;q12) translocation leading to formation of the EWSR1-WT1 fusion gene, which generates a chimeric protein with transcriptional regulatory activity (Int J Surg Pathol 2016;24:672)
Etiology
  • Unknown
Clinical features
  • Clinical symptoms are usually related to the primary site of presentation, such as pain, palpable mass, abdominal distention, obstruction and ascites (J Gastrointest Cancer 2019;50:560)
  • At initial presentation, 90% of patients have multifocal disease, nodular disease, diffuse peritoneal and omental disease or a combination of these conditions
  • Substantial number of patients have diaphragmatic involvement
Diagnosis
  • Tissue sampling is the gold standard for a definitive diagnosis
  • Fluorescence in situ hybridization (FISH) to detect EWSR1 rearrangement and reverse transcription polymerase chain reaction (RT-PCR) to assess for EWSR1-WT1 fusion transcripts are routine diagnostic ancillary tools (Virchows Arch 2017;471:631)
  • Both methods (FISH and RT-PCR) complement each other by confirming cases that the other method may not (Virchows Arch 2017;471:631)
Radiology description
Radiology images

Contributed by Borislav A. Alexiev, M.D.
CT Abdomen / pelvis

CT of abdomen / pelvis

Prognostic factors
Case reports
Treatment
  • Current management of DSRCT includes a combination of chemotherapy, radiation and aggressive cytoreductive surgery, plus intraperitoneal hyperthermic chemotherapy (Cancers (Basel) 2021;13:498)
Gross description
  • Typical gross appearance consists of multiple tumor nodules
  • Cut surface is firm and grayish white, with foci of necrosis and hemorrhage
Gross images

Contributed by Borislav A. Alexiev, M.D.
Multiple tumor nodules

Multiple tumor nodules

Frozen section description
  • Sharply outlined nests of small round, epithelioid or spindled cells surrounded by prominent desmoplastic stroma
Microscopic (histologic) description
Microscopic (histologic) images

Contributed by Borislav A. Alexiev, M.D.
Well defined nests

Well defined nests

Desmoplastic stroma

Desmoplastic stroma

Uniform cells

Uniform cells

Mild anisonucleosis

Mild anisonucleosis

Large cell pattern

Large cell pattern


Desmin

Desmin

Keratin AE1 / AE3

Keratin AE1 / AE3

WT1 (C terminus)

WT1 (C terminus)

WT1 (N terminus)

WT1 (N terminus)

Cytology description
  • Hypercellular smears of small round cells (Cancer Cytopathol 2014;122:386)
  • Round or kidney shaped nuclei with inconspicuous nucleoli
  • Nuclear molding usually present
  • Scant cytoplasm
  • Paranuclear cytoplasmic densities
  • Numerous mitotic figures, crushed nuclei and apoptosis
  • Connective tissue fragments (Acta Cytol 2012;56:576)
Cytology images

Contributed by Borislav A. Alexiev, M.D.
Touch smear cytology

Touch smear cytology

Immunofluorescence description
  • EWSR1-WT1 break apart FISH demonstrates separation of signals indicative of translocation involving the EWS locus (Oncol Lett 2012;4:423)
  • Paired locus specific EWS and WT1 probes show EWSR1-WT1 fusion signals in nuclei of DSRCT (Oncol Lett 2012;4:423)
Immunofluorescence images

Contributed by Jose G. Mantilla, M.D.
EWSR1 gene break apart

EWSR1 gene break apart



Images hosted on other servers:

Fluorescence in situ hybridization

Positive stains
Electron microscopy description
Molecular / cytogenetics description
Sample pathology report
  • Peritoneum, biopsy:
    • Desmoplastic small round cell tumor (see comment)
    • Comment: The tumor is characterized by sharply outlined nests of small round cells surrounded by a prominent desmoplastic stroma. The tumor cells are uniform, with small hyperchromatic nuclei, inconspicuous nucleoli, scant cytoplasm and indistinct cytoplasmic borders. Mitoses are frequent and focal tumor cell necrosis is present. Immunohistochemically, the tumor cells are immunoreactive for keratin AE1 / AE3 and desmin and show strong nuclear expression for WT1 (C terminus), while all of the following are negative: myogenin, MYOD1, S100 and WT1 (N terminus). The findings support the above diagnosis. Despite multimodal treatment, DSRCT is associated with dismal outcomes.
Differential diagnosis
  • Blastemal predominant Wilms tumor:
    • Overlapping histologic features with DSRCT, including poorly differentiated morphology, desmoplastic stroma and immunoreactivity for desmin and cytokeratin (Am J Surg Pathol 2014;38:1220)
    • Focal triphasic elements and glomeruloid bodies
    • WT1 immunoreactivity with antibodies to both the amino terminus and carboxy terminus
    • Negative for EWSR1-WT1 rearrangements
  • Ewing sarcoma:
    • Classic Ewing sarcoma:
      • Composed of uniform small round cells with round nuclei containing finely stippled chromatin and inconspicuous nucleoli, and scant clear or eosinophilic cytoplasm
    • Atypical Ewing sarcoma:
      • Tumor cells are larger, with prominent nucleoli and irregular nuclear contours
  • Rhabdomyosarcoma, embryonal and alveolar variants:
    • Embryonal rhabdomyosarcoma
      • Composed of primitive mesenchymal cells that show variable degrees of skeletal muscle differentiation
    • Alveolar rhabdomyosarcoma
      • Classic subtype: nests of neoplastic cells arranged in alveolar spaces
      • Solid subtype: sheets of neoplastic cells; nests separated by thin fibrovascular septae but alveoli are not seen
    • Mogenin and MyoD1 positive
    • WT1 (C terminus) negative
    • PAX3-FOXO1 and PAX7-FOXO1 fusion transcripts (alveolar rhabdomyosarcoma) (Mod Pathol 2021;34:748)
    • Negative for EWSR1-WT1 rearrangements
  • Lymphoma:
    • Diffuse or nodular growth pattern, large atypical cells with abundant cytoplasm, nuclei with clumped chromatin and prominent nucleoli
    • Positive for CD45, B cell and T cell markers (Appl Immunohistochem Mol Morphol 2013;21:116)
    • WT1 (C terminus) negative
    • Negative for EWSR1-WT1 rearrangements
  • Small cell carcinoma:
Board review style question #1

A 31 year old man presented with abdominal distention and multiple tumor nodules studding the peritoneal surface. Hematoxylin eosin stains demonstrate well defined nests of small round cells separated by desmoplastic stroma. The tumor cells are uniform, with small hyperchromatic nuclei, inconspicuous nucleoli and scant cytoplasm. Mitoses are frequent and focal tumor cell necrosis is present. Perineural invasion is identified. Immunohistochemically, the tumor cells are immunoreactive for keratin AE1 / AE3, desmin, NSE and WT1 (C terminus), while all of the following are negative: myogenin, MYOD1, S100 and WT1 (N terminus). Which of the following is most likely the correct diagnosis?

  1. Desmoplastic small round cell tumor
  2. Ewing sarcoma
  3. Lymphoma
  4. Small cell carcinoma
  5. Wilms tumor
Board review style answer #1
A. Desmoplastic small round cell tumor

Comment Here

Reference: Desmoplastic small round cell
Board review style question #2
Which of the following is true about desmoplastic small round cell tumor?

  1. Desmoplastic stroma is always present
  2. Prognosis of DSRCT has improved considerably with current multimodal therapy
  3. Striking female predominance
  4. Tumor is positive for WT1 (N terminus)
  5. Widespread abdominal / peritoneal involvement is common
Board review style answer #2
E. Widespread abdominal / peritoneal involvement is common

Comment Here

Reference: Desmoplastic small round cell

Ectomesenchymoma
Definition / general
  • Ectomesenchymoma is a rare, fast growing tumor of the nervous system or soft tissue that occurs mainly in children
Essential features
Sites
  • Sites described most often include head / neck, abdomen, perineum, scrotum and limbs
Clinical features
  • Fast growing tumors that eventually cause pain
Radiology description
  • CT scan may show anterior displacement of bladder and rectal compression
Prognostic factors
  • Stage and size < 10 cm
Case reports
  • 5 month old boy with a 7.5 cm mass emanating from the prostate (Pediatr Surg Int 1999;15:68)
    • Tumor was resected and after a course of chemotherapy, recurred 8 months later, causing fatal intestinal obstruction
    • This is the only case report of ectomesenchymoma of the prostate
Treatment
  • Surgery and chemotherapy have been described; prognosis is guarded
Gross description
  • Rubbery, white
Microscopic (histologic) description
  • 2 histologic components:
    • Skeletal muscle component: round to oval cells with abundant glycogenated cytoplasm and cross striations
    • Component analogous to ganglioneuroma: hypocellular area of spindle cells with wavy nuclei and long eosinophilic cytoplasmic processes; admixed ganglion cells
Microscopic (histologic) images

Contributed by Brendan C. Dickson, M.D., M.Sc.

Spindle cells and neuronal elements

Rhabdomyoblastic differentiation and mitoses

Myogenin

Synaptophysin



Images hosted on other servers:

Male infants with pelvic / urogenital tumors

34 year old man with paratesticular tumor

Positive stains
Negative stains
Molecular / cytogenetics description
Differential diagnosis
Board review style question #1
Which of the following is true of ectomesenchymoma of the prostate?

  1. It has 3 histologic components
  2. It is S100 positive in the neural component
  3. It is a slow growing tumor
  4. It should be cytokeratin positive
  5. It should be negative for desmin
Board review style answer #1
B. It is S100 positive in the neural component

Comment Here

Reference: Ectomesenchymoma

Ectopic meningioma (pending)
Table of Contents
Definition / general
Definition / general
[Pending]

Elastofibroma
Definition / general
  • Benign, ill defined proliferation of fibroelastic tissue with excessive abnormal degenerated elastic fibers
Essential features
  • Mostly asymptomatic mass at infrascapular region, incidentally found on radiological examination for other causes
  • Well circumscribed lesion, comprised of mature fat, fibrous tissue and characteristic abnormal eosinophilic looking elastic fibers
  • Surgical excision is curative, required mostly in symptomatic cases, with very rare local recurrence
Terminology
  • Acceptable: elastofibroma dorsi
ICD coding
  • ICD-O: 8820/0 - elastofibroma
  • ICD-11: FB51.Y & XH3BQ8 - other specified fibroblastic disorders & elastofibroma
Epidemiology
  • Exact prevalence is unknown as most are detected incidentally
  • Arises almost exclusively in the elderly, with peak incidence between the seventh and eighth decades of life
  • F > M (Ann R Coll Surg Engl 2020;102:84)
Sites
Pathophysiology
  • Elastotic degeneration of collagen or abnormal elastotic fibrogenesis may underlie the pathogenesis of elastofibroma and active neovascularization or endothelial mesenchymal transition plays a potential pathogenetic role (Eur J Histochem 2015;59:2459)
  • Most recently, chromosomal abnormalities have been described in a few cases, including those for chromosome 1, suggesting a neoplastic nature for this tumor (Cancer Genet Cytogenet 2001;126:68)
  • Traditionally, it was thought to be associated with microtrauma, being reported more commonly in laborers
  • Case reports show positive relationship with a patient's dominant hand side (Turk Gogus Kalp Damar Cerrahisi Derg 2022;30:250)
Etiology
  • Unknown at this time
Diagrams / tables

Images hosted on other servers:
Proposed pathogenesis of elastofibroma

Proposed pathogenesis

Clinical features
  • Lesion usually presents as a slow growing, asymptomatic mass or rarely causes stiffness, pain, scapular snapping and impingement
Diagnosis
  • Radiology is characteristic, particularly if the tumor arises at the typical subscapular location
  • Most asymptomatic cases do not require a biopsy unless there is a need to rule out malignancy
  • Symptomatic ones are excised and histopathology confirms the diagnosis suggested on imaging
  • Reference: Saudi Med J 2022;43:156
Radiology description
  • Ultrasound: complex hypoechoic lesion
  • CT scan: attenuation appears similar to skeletal muscle
  • MRI: heterogeneous masses with internal fatty areas
  • Diffusion MRI (Sisli Etfal Hastan Tip Bul 2020;54:103)
    • Newly proposed technique
    • Lesions appear heterogeneous with hyperintensity on T2 weighted images
    • More sensitive than conventional MRI in differentiating malignant lesions
  • PET CT: moderate increased uptake, symmetrical in bilateral lesions (Indian J Nucl Med 2019;34:258)
Radiology images

Images hosted on other servers:
Coronal CT images

Coronal CT

Axial CT images

Axial CT

MRI of subscapular mass

MRI of subscapular mass

CT of subscapular and gluteal masses

CT of subscapular and gluteal masses

CT scan

CT scan

Ultrasonographic appearance

Ultrasonographic appearance


MRI findings MRI findings

MRI findings

PET CT findings PET CT findings

PET CT findings

MRI with diffusion sequences

MRI with diffusion sequences

Prognostic factors
  • Benign lesion cured by simple excision
  • Local recurrence is extremely rare with only occasional case reports (J Orthop 2014;12:S133)
Case reports
Treatment
  • Most asymptomatic cases do not require any treatment
  • Complete excision is curative
Clinical images

Images hosted on other servers:
Bilateral subscapular masses

Bilateral subscapular masses

Bilateral painful masses

Bilateral painful masses

Left parotid region mass

Left parotid region mass

Intraoperative view of subscapular mass

Intraoperative view of subscapular mass

Surgical excision of the elastofibroma dorsi

Surgical excision of the elastofibroma dorsi

Typical location (arrow)

Typical location (arrow)

Gross description
  • May or may not be well circumscribed lesion, without a definite capsule
  • Cut surface is firm to rubbery and shows fibrous and fatty areas
Gross images

Contributed by Nasir Ud Din, M.B.B.S
Fibrous and fatty areas

Fibrous and fatty areas



Images hosted on other servers:
excised masses

Excised masses

Poorly circumscribed lesion Poorly circumscribed lesion

Poorly circumscribed lesion

Bilateral excised masses

Bilateral excised masses

Whole and cut up specimen

Whole and cut up specimen

Pancreatic lesion

Pancreatic lesion

Microscopic (histologic) description
  • Essential diagnostic criteria: bland appearing fibrofatty tumor with abnormal elastic fibers
    • Infiltrative, ill defined, unencapsulated hypocellular lesion, which may entrap skeletal muscle fibers at the periphery or invade periosteum
    • Collagenized stroma with variable hypocellular myxoid to edematous areas
    • Admixed mature adipocytes present
    • Scattered abnormal elastic fibers appear as densely eosinophilic, round to irregular, variably sized globules, elongated structures with irregular fuzzy outlines or beaded cords
    • Scattered spindle to stellate, bland looking fibroblasts are seen
    • Lacks significant nuclear atypia or foci of necrosis
  • Reference: Anticancer Res 2021;41:2211
Microscopic (histologic) images

Contributed by Nasir Ud Din, M.B.B.S.
Stroma with fat and elastic fibers

Stroma with fat and elastic fibers

Bland looking fibroblasts

Bland looking fibroblasts

Eosinophilic elastic fibers

Eosinophilic elastic fibers

Globular appearance of elastic fibers

Globular appearance of elastic fibers


Clustering of elastic fibers

Clustering of elastic fibers

Abnormal elastic fibers

Abnormal elastic fibers

Elastic stain

Elastic stain

Virtual slides

Images hosted on other servers:
Elastofibroma WSI

Elastofibroma whole slide image (WSI)

Infrascapular lesion

Infrascapular lesion

Chest wall lesion

Chest wall lesion

Cytology description
  • Cytological smears show degenerated abnormal elastic fibers in the form of globules, braids and elongated fibers
  • Altered elastic fibers have green-yellow autofluorescence with ultraviolet light (Diagn Cytopathol 2002;26:310)
Positive stains
Electron microscopy description
  • Abnormal elastic fibers appear as electron dense masses of 3 - 5 micron diameter, removed by elastase digestion
  • Some areas may show homogeneous, extremely dense core surrounded by dense, irregular fibers of diameter 350 A to 400 A with an indistinct outline (J Clin Pathol 1968;21:463)
Electron microscopy images

Images hosted on other servers:
Electron dense fibers Electron dense fibers

Electron dense fibers

Electron dense fibers Electron dense fibers

Electron dense fibers

Molecular / cytogenetics description
  • Pathognomonic cytogenetic abnormalities have not been described to date
  • Recent studies have described DNA copy number changes in a few cases, particularly involving chromosome 1 and X chromosome (Cancer Genet Cytogenet 2001;126:68)
Videos

Elastofibroma pathology

Elastofibroma radiology

Sample pathology report
  • Right subscapular mass, excision:
    • Benign soft tissue lesion, consistent with elastofibroma (see comment)
    • Comment: An ill defined, hypocellular lesion showing spindle cells and collagenized stroma in the background, with entrapment of fat and skeletal muscle at the periphery. Scattered abnormal elastic fibers appear as densely eosinophilic, round to irregular, variably sized globules to beaded structures. These tumors follow a benign course with complete excision being curative. Recurrence is extremely rare.
Differential diagnosis
Board review style question #1

A 60 year old man undergoes MRI for staging of biopsy proven lung cancer. Imaging shows bilateral subscapular masses with attenuation similar to adjacent skeletal muscle. Surgical excision is performed and light microscopic appearance is shown in the photomicrograph. What is the most likely diagnosis in this case?

  1. Elastofibroma
  2. Metastatic carcinoma
  3. Neurofibroma
  4. Nodular fasciitis
  5. Spindle cell lipoma
Board review style answer #1
A. Elastofibroma. The photomicrograph shows a fibrous tumor with loose background, few thin walled vessels and scattered abnormally shaped eosinophilic fibers. Admixed elements of adipose tissue are also seen. These are characteristic features of elastofibroma. There is no abnormal epithelial population. Answer C is incorrect because neurofibroma shows spindle cells with a background of characteristic small collagen fibers (shredded carrot-like). Answer D is incorrect because in nodular fasciitis, a tissue culture-like growth pattern, is seen with a myxoid background. Answer E is incorrect because spindle cell lipoma contains both fibrous and adipocytic components but the spindle cells are more elongated and there is an absence of abnormal elastic fibers in any of these lesions. Answer B is incorrect as elastofibromas are benign tumors.

Comment Here

Reference: Elastofibroma
Board review style question #2

A 60 year old man undergoes MRI for staging of biopsy proven lung cancer. Imaging shows bilateral subscapular masses with attenuation similar to adjacent skeletal muscle. Surgical excision is performed and light microscopic appearance is shown in the photomicrograph. Which of the following special stains will help the most in reaching the correct diagnosis?

  1. Alcian blue
  2. Elastic stain
  3. Reticulin stain
  4. Schiff
  5. Trichrome
Board review style answer #2
B. Elastic stain. The abnormal elastic fibers stain black with Verhoeff van Gieson elastic stain. Answer A is incorrect because Alcian blue may be positive in the background stroma but it is not specific for this entity and may be seen in a variety of soft tissue lesions. Answers C and D are incorrect because similarly, reticulin fibers are not a feature of this entity nor is Schiff positivity. Answer E is incorrect because trichrome stains the background collagen but not the elastic fibers.

Comment Here

Reference: Elastofibroma

Embryonal rhabdomyosarcoma
Definition / general
  • Subtype of the rhabdomyosarcoma (RMS) soft tissue cancer family whose lineage derives from the undifferentiated mesoderm
  • Embryonal rhabdomyosarcoma (ERMS) is the most common RMS subtype
  • Several distinct and prognostic patterns of growth have been noted, including the botryoid and anaplastic variants
Essential features
  • Most common subtype of rhabdomyosarcoma in the pediatric and adolescent setting
  • Displays a wide spectrum of morphologic features, including cases with spindled morphology
  • Displays some degree of myogenic differentiation, including either MyoD1 or myogenin positivity
  • Not associated with recurrent gene fusions but displays some recurrent copy number changes
  • Diagnostic criteria are: (a) round and spindle cell morphology with scattered differentiated rhabdomyoblasts; (b) desmin positivity and heterogenous nuclear staining for myogenin or MYOD1; (c) lack of FOXO1 gene rearrangements distinguishes poorly differentiated ERMS from solid alveolar RMS
Terminology
  • Botryoid embryonal rhabdomyosarcoma (sarcoma botryoides) only occurs in certain locations, specifically beneath mucosal epithelial lined viscera, such as the bladder, biliary tract, vagina or upper respiratory tract, extrahepatic bile ducts or near a space; rarely in eyelid or anal region
    • Often has a grape-like (botryoid) growth pattern
    • 25% of rhabdomyosarcomas, 10% of embryonal subtype
    • Considered a subtype of embryonal rhabdomyosarcoma (ERMS) (Am J Surg Pathol 2001;25:856)
  • Not recommended to use: sarcoma botryoides; mixed embryonal and alveolar rhabdomyosarcoma
ICD coding
  • ICD-O: 8910/3 - embryonal rhabdomyosarcoma, NOS
  • ICD-11: 2B55.Z & XH83G1 - rhabdomyosarcoma, unspecified primary site & embryonal rhabdomyosarcoma NOS
Epidemiology
  • Most common in children 0 - 4 years old (with a maximum reported incidence of 4 cases per 1 million children) but can be identified at any age (Cancer 2009;115:4218)
  • Represents approximately 60 - 70% of childhood cases and 20% of adult cases
  • Rare congenital cases have also been reported (Int J Surg Case Rep 2017;31:47)
Sites
  • Most common in the head and neck region, including the nasal and oral cavities, as well as the orbit and middle ear (J Clin Oncol 1995;13:610)
  • Also common in the paratesticular soft tissues and the genitourinary tract
  • Extremity involvement of the somatic soft tissues is less common (< 9%)
  • Metastasizes to soft tissue, serosa, lung, lymph nodes and bone marrow
Pathophysiology
  • Derived from the undifferentiated mesoderm and shows phenotypic and biologic features of primitive skeletal muscle
Etiology
Diagnosis
  • Clinical and morphologic features: primitive hypercellular tumor with spindled to round shaped cells; mitotic activity and necrosis are common
  • Positivity for desmin, myogenin and MyoD1
  • Confirmation of no evidence of FOXO1 gene fusion in challenging cases (to exclude alveolar rhabdomyosarcoma)
Prognostic factors
  • Intergroup Rhabdomyosarcoma Study Group (IRSG) has subdivided rhabdomyosarcoma into low, intermediate and high risk groups for the purpose of protocol placement in pediatric patients (Pediatr Blood Cancer 2012;59:5)
    • Based on clinicopathologic variables including histologic type, extent of surgical control of local disease and primary tumor site
  • Botryoid variant has a particularly good prognosis, although may have late relapse (Zhonghua Bing Li Xue Za Zhi 2004;33:225, Pediatr Blood Cancer 2008;51:140)
  • Other favorable prognostic factors are age (between 1 and 9 years), orbital and paratesticular location and absence of metastatic disease at the time of resection
  • Anaplasia is defined similar to anaplastic Wilms tumor as a significant nuclear variation (cells that are 3x larger than background tumor cells) and the presence of atypical multipolar mitotic figures
  • Extremity involvement is associated with more relapses and lower survival
Case reports
Treatment
  • Surgery is the preferred local therapy, with radiotherapy typically used only in the event of residual disease, nodal disease or poor response to combined chemotherapy
  • Intergroup Rhabdomyosarcoma Study Postsurgical Clinical Grouping System is recommended to plan treatment in pediatric cases (Pediatr Blood Cancer 2012;59:5)
  • American Joint Committee on Cancer (AJCC) TNM staging system remains appropriate for planning treatment in adult patients (Amin: AJCC Cancer Staging Manual, 8th Edition, 2018)
  • Anaplastic tumors often require more intensive treatment (Cancer 2008;113:3242)
  • Botryoid variant: conservative surgery plus radiation and chemotherapy (Int J Gynecol Cancer 2008;18:190)
Gross description
  • Poorly circumscribed mass, white, soft or firm, infiltrative
  • Botryoid variant: resembles cluster of grapes or allergic nasal polyp, fleshy nodular polypoid projections of variable size into lumen
Gross images

Contributed by Mark R. Wick, M.D. and AFIP

Orbital

Botryoid



Images hosted on other servers:

White tumor mass with focal necroses

Incision of the oral mucosa

Pitfalls and tips
  • Anaplasia specifically requires multipolar mitotic figures
    • Make sure you are not overinterpreting degenerating or apoptotic cells
    • Look for organization of the division and exclude cells that appear to be exploding
    • Also try to avoid interpreting overlapping cells as 1 larger cell
  • Absence of a translocation does not result in the default diagnosis of embryonal rhabdomyosarcoma; tumor must adhere to the described histologic features
  • Some rhabdomyosarcomas defy classification (rhabdomyosarcoma, not otherwise specified); this designation is preferable to misclassification, especially in small or otherwise limited samples
  • Beware nonspecific staining (aberrant positivity for cytokeratin or S100 protein) versus true staining (S100 protein as a component of malignant peripheral nerve sheath tumor)
  • Damaged skeletal muscle can express MyoD1 and myogenin; a different sarcoma that otherwise infiltrates skeletal muscle will very much mimic a rhabdomyosarcoma on immunohistochemistry staining
    • Look for the MyoD1 / myogenin to be positive in tumor cells in a linear pattern, which suggests the staining is of background muscle and not the tumor
Microscopic (histologic) description
  • Composed of primitive mesenchymal cells that show variable degrees of skeletal muscle differentiation
  • They are moderately cellular but in the typical pattern often contain both hypocellularity and hypercellular areas with a loose, myxoid stroma
  • Perivascular condensations of tumor cells in the less cellular regions are common
  • Sheets of small, stellate, spindled or round cells with scant or deeply eosinophilic cytoplasm and eccentric, small oval nuclei with a light chromatin pattern and inconspicuous nucleoli
  • Can occasionally identify tumor cells that contain generous amounts of eosinophilic cytoplasm, a feature of rhabdomyoblastic differentiation (so called strap cells)
    • These may become more prominent with chemotherapy (chemotherapeutic induced cytodifferentiation)
  • May have cells with elongated tails of cytoplasm (tadpole cells)
  • If densely cellular, may resemble solid alveolar rhabdomyosarcoma (Am J Clin Pathol 2013;140:82)
  • Botryoid variant frequently shows a cambium layer: a hypercellular zone immediately beneath the epithelial surface
    • Cells are undifferentiated, round or spindled with minimal cytoplasm, frequent mitotic figures
    • Deeper layers of the tumor are typically less cellular but overall conform to the histology of embryonal rhabdomyosarcoma (ERMS) with variation by region
  • Rare morphological patterns:
Microscopic (histologic) images

Contributed by Erdener Özer, M.D., Ph.D. and Mark R. Wick, M.D.
Spindled and round cells

Spindled and round cells

Desmin

Desmin

Myogenin

Myogenin



Contributed by Carolina Martinez Ciarpaglini, M.D., Ph.D. (Case #276) - tonsillar mass
Nests and sheets

Nests and sheets

Crush artifact

Crush artifact

Nuclear molding

Nuclear molding

Focally epithelioid

Focally epithelioid


Actin

Desmin

Myogenin

MyoD1



AFIP images - anaplastic rhabdomyosarcoma

Hyperchromatic nuclei 3x larger than adjacent tumor cells

Bizarre mitotic figure

Rare muscle differentiation



AFIP images - botryoid variant
Polypoid growth

Polypoid growth

Overlying mucosa

Overlying mucosa

Spindle cells

Spindle cells

Deep foci of hypercellularity Deep foci of hypercellularity

Deep foci of hypercellularity

Focal cambium layer

Focal cambium layer

Cytology description
Cytology images

Contributed by Erdener Özer, M.D., Ph.D.

Small round cell tumor

Positive stains
Electron microscopy description
  • Developing striated muscle, thick and thin filaments
Molecular / cytogenetics description
  • 11p15.5 loss of heterozygosity or loss of imprinting is a frequent finding
  • Chromosomal aneuploidies are very common, especially with gains of chromosome 8 (seen in 90% of patients)
    • Additional gains frequently seen include chromosomes 2, 11, 12, 13 and 20
  • No diagnostic translocation found to date (see Terminology regarding spindled rhabdomyosarcoma)
  • Inactivating mutations of TP53 and CDKN2A and activating mutations of RAS family genes (Genes Chromosomes Cancer 2011;50:397)
  • Frequency of ALK dysregulation is debatable, as are any significant associated clinical features (Mod Pathol 2013;26:772)
Sample pathology report
  • Soft tissue mass, left chest wall, excision:
    • Embryonal rhabdomyosarcoma, negative for evidence of anaplasia (see comment)
    • Comment: Histologic sections of this left chest wall excision demonstrate a spindle cell sarcoma comprised of hyperchromatic cells arranged into disorganized intersecting fascicles. No evidence of anaplastic features is identified. Immunohistochemical staining shows diffuse positivity for desmin, with scattered cells positive for myogenin and MyoD1. These findings are compatible with embryonal rhabdomyosarcoma.
Differential diagnosis
  • Alveolar rhabdomyosarcoma (ARMS):
    • Diffuse and strong nuclear staining for myogenin and MyoD1; molecular studies (including FISH) show PAX-FOXO1 fusion gene product in approximately 85% of cases (Am J Clin Pathol 2013;140:82)
    • Histologically, ARMS is more uniform and shows more predictable patterns of growth (a finding which correlates with its consistent driver fusion event; it looks like a translocation sarcoma)
  • Desmoplastic small round cell tumor:
    • Tumor nodules on serosal surfaces, strongly keratin+ and EMA+, may be desmin+ but muscle specific actin-
    • Associated with desmoplasia and histologically more closely mimic alveolar rather than embryonal rhabdomyosarcoma (ERMS)
    • Will not display myogenin or MyoD1 positivity; beware false positivity if they infiltrate background skeletal muscle
    • See Pitfalls and tips above
  • Ewing / PNET:
    • Another round cell tumor, often displays Homer Wright rosettes
    • Nuclei are far more uniform and pale, not dense and hyperchromatic; CD99+, desmin-, MyoD1-, myogenin-
    • Look for characteristic rearrangements including t(11;22) or t(12;22)
    • Although there can be histologic overlap, if you sample the tumor enough you will find more atypia and cell to cell variability in rhabdomyosarcoma
  • Large cell lymphoma:
  • Monophasic synovial sarcoma:
    • Negative for muscle markers, usually far more spindled and organized (long fascicles) when compared to ERMS
    • Cytokeratin and EMA expression will show scattered and focal positivity in the spindled cells, the characteristic pattern seen in nearly all of these lesions
    • Molecular or FISH testing for the t(X;18) fusion, if needed
  • Myxoid liposarcoma (MLS):
    • Although MLS typically is very easy to distinguish from ERMS, it can have a very similar background matrix that is loose and lightly basophilic
    • Cells can focally resemble one another as well
    • Look for signet ring lipoblasts in MLS, more bland histology and more uniformity when compared to ERMS
    • Immunostaining and molecular features are also easily utilized to differentiate the 2 entities
  • Neuroblastoma:
    • Undifferentiated neuroblastoma can be very difficult to distinguish histologically; you may need to rely on the clinical presentation and the immunoprofile
    • Elevated urinary catecholamines, rosettes, granular chromatin, neuropil
    • S100+ (often), chromogranin+, synaptophysin+, GFAP+
    • Absence of myogenic markers
  • Pleomorphic rhabdomyosarcoma:
    • Exclusively adults, usually in their 60s - 70s
    • Usually deep soft tissue of the extremity and remarkable for its universal diffuse cytologic atypia
    • Uniformly pleomorphic and does not contain elements of embryonal rhabdomyosarcoma
    • Anaplastic, as opposed to ERMS, which occasionally displays anaplasia in a background of classic EMRS
  • Spindle cell rhabdomyosarcoma:
    • Historically, tumors with spindled morphology were regarded as a variant of ERMS but the 2020 WHO now has a separate spindle cell / sclerosing subtype category (i.e. this is not a variant of ERMS)
    • ERMS can display spindled growth but there are several molecularly defined subtypes of spindle cell / sclerosing RMS, which are considered separate and will likely be better defined in the future by WHO (Am J Surg Pathol 2016;40:224)
  • Wilms tumor:
    • Can have rhabdomyoblastic differentiation, especially in the setting of chemotherapy
    • Beware any tumor in the kidney or any metastatic site in a patient with a previous Wilms tumor
    • Epithelial and blastemal components can be very focal following chemotherapy
    • If a Wilms tumor displays this pattern, it usually shows abundant cytodifferentiation, as opposed to classic rhabdomyosarcoma, which is usually focal
Board review style question #1
Which FISH testing may assist in making distinction between the dense pattern of embryonal rhabdomyosarcoma and the alveolar subtype of rhabdomyosarcoma?

  1. ETV6::NTRK3
  2. EWS::FLI1
  3. PAX3::FOXO1
  4. SYT::SSX1
  5. TPM3::ALK
Board review style answer #1
C. PAX3::FOXO1. This fusion gene product occurs in approximately 80% of alveolar rhabdomyosarcoma cases. Additionally, PAX7::FOXO1 rearrangements are present in a minority of cases.

Comment Here

Reference: Embryonal rhabdomyosarcoma
Board review style question #2

Which histologic features in embryonal rhabdomyosarcoma are needed to diagnose anaplasia?

  1. 3x nuclear size variability and tripolar mitotic figures
  2. Alternating hypercellular and hypocellular areas
  3. Coagulative necrosis
  4. More than 20 mitoses per 10 high power fields
  5. Solid growth pattern
Board review style answer #2
A. 3x nuclear size variability and tripolar mitotic figures. Multipolar mitotic figures (including tripolar mitoses) and nuclear anaplasia (3x variation in nuclear size) are required to diagnose anaplastic embryonal rhabdomyosarcoma (ERMS). Solid growth pattern is irrelevant and necrosis can be seen in any subtype of rhabdomyosarcoma. ERMS typically displays the alternating pattern of hypocellularity and hypercellularity (also a buzzword for high grade malignant peripheral nerve sheath tumors). There is no requirement for a specific mitotic rate.

Comment Here

Reference: Embryonal rhabdomyosarcoma

Epithelioid hemangioendothelioma
Definition / general
  • Malignant endothelial neoplasm that most commonly involves soft tissue, bone, lung, skin and liver; can be locally aggressive and has metastatic potential
  • 2 subtypes defined by WWTR1-CAMTA1 or YAP1-TFE3 rearrangement; former expressing CAMTA1 and latter TFE3 (less specific)
Essential features
  • Endothelial neoplasm that most commonly involves soft tissue, bone, lung, skin and liver
  • Locally aggressive tumor with metastatic potential
  • WWTR1-CAMTA1 rearranged tumors: composed of cords or small nests of large endothelial cells with abundant eosinophilic cytoplasm embedded in a myxohyaline stroma; CAMTA1 positive staining
  • YAP1-TFE3 rearranged tumors: composed of solid nests or pseudo alveolar formations of epithelioid cells enmeshed in a fibrous stroma; TFE3 positive staining
ICD coding
  • ICD-O: 9133/3 - epithelioid hemangioendothelioma, malignant
  • ICD-10: D18.0 - hemangioma
  • ICD-11: 2B5Y & XH9GF8 - epithelioid hemangioendothelioma, NOS
Epidemiology
Sites
Pathophysiology
Etiology
  • Unclear
Clinical features
  • Clinical presentation depends on the tumor location, most frequently pain
  • If arising from a vein, occurring symptoms caused by vascular occlusion, such as edema or thrombophlebitis
  • Recurs locally, may metastasize (usually to lymph nodes and lungs) (Diagn Pathol 2014;9:131, Am J Surg Pathol 1997;21:363)
Diagnosis
  • Diagnostic workup includes radiology, histology (biopsy, resection) with immunohistochemistry and molecular analysis
Radiology description
  • CT scan: poorly circumscribed lesion with ground glass appearance
Radiology images

Images hosted on other servers:

Multifocal liver lesions

Prognostic factors
  • 13% recur, 20 - 30% metastasize (lung, lymph nodes), 13% die of disease; for lung, mortality is 65% (Am J Surg Pathol 1997;21:363)
  • High risk (> 3 mitotic figures per 50 high power fields and size > 3 cm) have 5 year disease specific survival of 59% versus 100% for low risk (Am J Surg Pathol 2008;32:924)
Case reports
Treatment
  • Wide local excision
Gross description
Frozen section description
  • Hypercellular tumor composed of groups of epithelioid cells embedded in a myxohyaline stroma
Microscopic (histologic) description
  • WWTR1-CAMTA1 subtype (classic EHE):
    • Cords, strands or small nests of large endothelial cells with abundant eosinophilic cytoplasm embedded in a myxohyaline stroma
    • Tumor cells have vesicular, round to oval, sometimes indented nuclei
    • Some tumor cells have intracytoplasmic, round, clear vacuoles representing small vascular lumina, which may contain erythrocytes
  • YAP-TFE3 subtype:
    • Solid nests or pseudo alveolar arrangement of epithelioid cells enmeshed in a fibrous stroma
    • Tumor cells have abundant, densely eosinophilic cytoplasm and can form vascular spaces
    • Intracytoplasmic vacuoles are rare
  • Usually minimal mitotic activity, atypia or necrosis
  • Up to 10% of cases exhibit frank malignant features of prominent nuclear pleomorphism, increased mitotic activity, solid growth or necrosis; these tumors resemble epithelioid angiosarcoma and have a more aggressive behavior (Am J Surg Pathol 2008;32:924)
Microscopic (histologic) images

Contributed by Iva Brčić, M.D., Ph.D. and Bernadette Liegl-Atzwanger, M.D.
Poorly circumscribed

Poorly circumscribed

Cords and nests

Cords and nests

EHE in the bone

Tumor within bone

Erythrocytes in lumen

Erythrocytes in lumen

Cords in myxohyaline stroma

Cords in myxohyaline stroma

Cellular atypia

Cellular atypia


Mitosis and atypical cells

Mitosis and atypical cells

Nuclear CAMTA1 staining

Nuclear CAMTA1 staining

Diffuse CD31

Diffuse CD31

Diffuse ERG

Diffuse ERG

CAM5.2 focally positive

CAM5.2 focally positive

Cytology description
Positive stains
Negative stains
Molecular / cytogenetics description
Molecular / cytogenetics images

Images hosted on other servers:

YAP1-TFE3 fusion

Sample pathology report
  • Right thigh, excision:
    • WWTR1-CAMTA1 rearranged epithelioid hemangioendothelioma (see comment)
    • Comment: Tumor is composed of cords and small nests of large endothelial cells with abundant eosinophilic cytoplasm embedded in a myxohyaline stroma. Immunohistochemically, the tumor cells are positive for ERG, CD31, CAMTA1, focally positive for keratin and are negative for CD34 and TFE3. The morphology and immunoprofile strongly support the diagnosis of WWTR1-CAMTA1 rearranged epithelioid hemangioendothelioma.
Differential diagnosis
Additional references
Board review style question #1

Which of the following is true regarding epithelioid hemangioendothelioma?

  1. Cut surface is white with myxoid areas
  2. It is a benign neoplasm with vascular differentiation
  3. Most of the cases are negative for CAMTA1
  4. Tumor cells stain positive for ERG
  5. Tumor consists of spindle cells embedded in hyalinized stroma
Board review style answer #1
D. Tumor cells stain positive for ERG

Comment Here

Reference: Epithelioid hemangioendothelioma
Board review style question #2

A 30 year old woman presented with a mass of the trunk. Immunohistochemistry shows positive staining with CD31, CD34, ERG and CAMTA1 and focal positivity with keratin. Which of the following is most likely the correct diagnosis?

  1. Epithelioid angiosarcoma
  2. Epithelioid hemangioendothelioma
  3. Epithelioid hemangioma
  4. Malignant melanoma
  5. Myoepithelioma of soft tissue
Board review style answer #2
B. Epithelioid hemangioendothelioma

Comment Here

Reference: Epithelioid hemangioendothelioma

Epithelioid hemangioma
Definition / general
Essential features
  • Numerous stromal eosinophils and lymphoid reaction with follicles in most cases
  • Most frequent sites are head and distal extremities
  • All cases show diffuse strong expression of vascular markers
  • FOS rearrangement in a third of epithelioid hemangiomas across different locations and histologic variants
Terminology
  • Angiolymphoid hyperplasia with eosinophilia
  • Nodular angioblastic hyperplasia with eosinophilia and lymphofolliculosis
  • Histiocytoid hemangioma
  • Inflammatory angiomatous nodule
  • Atypical or pseudopyogenic granuloma
ICD coding
  • ICD-O: 9125/0 - epithelioid hemangioma
  • ICD-11: 2E81.0Y & XH10T4 - neoplastic haemangioma of other specified site & epithelioid haemangioma
Epidemiology
Sites
Etiology
  • The etiology of most epithelioid hemangiomas is unknown (Am J Surg Pathol 2004;28:523)
  • There has been considerable debate as to whether this is a reactive vascular proliferation or a benign vascular neoplasm, although most available data favor the latter (Am J Surg Pathol 2015;39:1313)
Clinical features
  • Most patients present with a slow growing, solitary subcutaneous or dermal nodule (J Cutan Pathol 2018;45:395)
  • Multifocal disease in the same anatomical region is encountered with some frequency
Diagnosis
  • Tissue sampling is the gold standard for a definitive diagnosis
Radiology images

Images hosted on other servers:

Right scapula mass

Prognostic factors
  • Local recurrence occurs in up to one third of patients (J Craniofac Surg 2018;29:e736)
  • The vast majority of recurrences are indolent
  • There are no reports of distant metastases
Case reports
Treatment
Clinical images

Images hosted on other servers:
Missing Image

Erythematous nodule

Multiple papules

Gross description
  • Most have a nonspecific nodular appearance with pale pink to red-brown color (Am J Surg Pathol 2004;28:523)
  • Subcutaneous lesions may be confused with lymph nodes because of circumscription and a prominent lymphoid reaction
Frozen section description
  • Diagnosis may be difficult due to extensive inflammation
Microscopic (histologic) description
  • Lobular proliferation of small, capillary sized vessels lined by plump, epithelioid endothelial cells, often surrounding central vessel (Br J Dermatol 1969;81:1)
  • Endothelial cells with abundant eosinophilic or amphophilic cytoplasm that is sometimes vacuolated (Am J Surg Pathol 2004;28:523)
  • Cytoplasm may appear to project into lumen (hobnail or tombstone appearance)
  • Single, large nucleus with fine chromatin and central nucleolus
  • Rare spindle cell morphology
  • Mitotic figures are uncommon
  • No necrosis
  • No nuclear pleomorphism
  • Abundant stromal chronic inflammation (numerous eosinophils, lymphocytes, mast cells and plasma cells)
  • Lymphoid tissue may form reactive germinal centers, particularly at periphery of lesion
  • Dermal examples lack association with an artery, vessels have a more mature appearance and endothelial cells are less plump and more cobblestone-like in appearance
  • Epithelioid hemangioma with atypical features (Genes Chromosomes Cancer 2014;53:951)
    • Solid growth
    • Increased cellularity
    • Mild to moderate nuclear pleomorphism
    • Necrosis may be seen
    • No overt sarcomatous areas
Microscopic (histologic) images

Contributed by Borislav A. Alexiev, M.D. and @JMGardnerMD on Twitter
Missing Image

Vascular neoplasm with inflammation

Missing Image

Capillary sized vessels with inflammation

Missing Image

Epithelioid endothelial cells

Missing Image

ERG

Epithelioid hemangioma

Epithelioid hemangioma

Cytology description
Positive stains
Negative stains
Molecular / cytogenetics description
  • FOS rearrangement in a third of epithelioid hemangioma across different locations and histologic variants (Am J Surg Pathol 2015;39:1313)
  • A novel FOS-LMNA fusion transcript in a typical variant of epithelioid hemangioma (Am J Surg Pathol 2015;39:1313)
  • Epithelioid hemangiomas with atypical features harbor ZFP36-FOSB fusions (recommended diagnostic terminology for a pathology report for these cases is "epithelioid hemangiomas with atypical features") (Genes Chromosomes Cancer 2014;53:951)
Sample pathology report
  • Left upper lip mass, excision:
    • Epithelioid hemangioma (see comment)
    • Comment: There is a dermal and subcutaneous proliferation of small, capillary sized vessels lined by epithelioid endothelial cells. The process is well demarcated and intimately associated with a small artery. A brisk stromal eosinophilic and lymphocytic inflammatory infiltrate is present. Immunohistochemically the epithelioid endothelial cells have strong expression of ERG and CD31 while are negative for HHV8. This constellation of morphological and immunohistochemical features strongly supports the diagnosis of epithelioid hemangioma. It is a benign vascular neoplasm prone to local recurrence unless completely excised. No distant metastatic spread has been reported.
Differential diagnosis
Board review style question #1
    Which of the following is true about epithelioid hemangioma?

  1. ~30% of cases metastasize
  2. Diagnosis always requires clinicopathological and radiological correlation
  3. Epithelioid hemangiomas harbor SERPINE1-FOSB fusions
  4. Epithelioid hemangiomas with atypical features harbor ZFP36-FOSB fusions
  5. Tumor is characterized by cords of epithelioid endothelial cells distributed in a myxohyaline stroma
Board review style answer #1
D. Epithelioid hemangiomas with atypical features harbor ZFP36-FOSB fusions

Comment Here

Reference: Epithelioid hemangioma
Board review style question #2
    A 65 year old man presented with a left upper lip mass. Hematoxylin-eosin stains demonstrated proliferation of small, capillary sized vessels lined by plump, epithelioid endothelial cells. Numerous stromal eosinophils, plasma cells and lymphocytes were also seen. Immunohistochemical stains for CD31 and ERG were positive in epithelioid endothelial cells. SMA stain highlighted the preserved myopericytic layer around vessels. Which of the following is most likely the correct diagnosis?

    Missing Image

  1. Angiomatosis
  2. Epithelioid angiosarcoma
  3. Epithelioid hemangioendothelioma
  4. Epithelioid hemangioma
  5. Pseudomyogenic hemangioendothelioma
Board review style answer #2
D. Epithelioid hemangioma

Comment Here

Reference: Epithelioid hemangioma

Epithelioid sarcoma
Definition / general
  • A malignant mesenchymal neoplasm that exhibits epithelioid cytomorphology and a predominantly epithelial phenotype
Essential features
  • Malignant epithelioid tumor occurring in pediatric and adult populations
  • Unpredictable clinical course; better prognosis in pediatric patients
  • Two typical morphologies, including classic type with epithelioid to spindled cells with central pseudogranulomatous architecture; and proximal type with predominant epithelioid and rhabdoid cells
  • IHC: Pancytokeratin+, INI1 lost
  • Molecular: Mutations in INI1/SMARCB1
Terminology
  • Classic or conventional type is also known as distal type
  • Proximal type is also known as large cell type
  • Defunct name: sarcoma aponeuroticum
ICD coding
  • ICD-10: C49.0 - malignant neoplasm of other connective and soft tissue
  • ICD-O: 8804/3 - epithelioid sarcoma
Epidemiology
  • Rare, < 1% of all adult soft tissue sarcomas, 4 - 8% of pediatric non rhabdomyoblastic sarcomas
  • Classic type epithelioid sarcoma is more common than the proximal type epithelioid sarcoma
  • Classic type: Most common in adolescents and young adults, between ages 10 and 40 years; M:F = 1.9:1 (Adv Anat Pathol 2016;23:41)
  • Proximal type: Middle aged to older adults, > 80% presenting between ages of 20 and 65 years; M:F = 1.6:1
Sites
  • Either type may arise anywhere
  • Classic type: usually distal upper extremity, > 60% arising in the fingers and hand
  • Proximal type: more common in deep soft tissue, truncal tissue (including pelvic peritoneal, genital and inguinal) and buttock / hip
Pathophysiology
Etiology
Clinical features
  • Solitary or multiple, slowly growing, usually painless, firm nodules (J Clin Aesthet Dermatol 2009;2:49)
  • Lesion often results in non healing skin ulcers that have a tendency to clinically mimic other ulcerative dermal processes
Diagnosis
  • Core needle biopsy for deep tumors and punch biopsy for dermal tumors
  • Followed by wide resection
Radiology description
  • No distinct MR imaging characteristic of this tumor that would enable early specific diagnosis
  • Tumors may show variable amounts of fibrosis, granulomatous pattern, necrosis and cellularity, corresponding to variable T2-weighted signal; T1-weighted images usually isointense to muscle but may have variability due to hemorrhage or necrosis
  • Suspicion should arise when presenting with multiple soft tissue nodules or persistent punched out ulcers involving the skin and subcutaneous tissues (Skeletal Radiol 2002;31:400)
Radiology images

Contributed by Borislav Alexiev, M.D.
Missing Image

T2-weighted MRI, hand

Prognostic factors
  • Course is often unpredictable; common to present with extensive disease, lymph node metastases or distant metastases (J Clin Aesthet Dermatol 2009;2:49)
  • Adverse prognostic factors include proximal site, large size, male sex, older age, necrosis, vascular invasion, rhabdoid cytomorphology and inadequate excision (Adv Anat Pathol 2016;23:41, Adv Anat Pathol 2006;13:114)
  • In adults: 5 year overall survival of 50 - 85%
  • In pediatric population: better prognosis; 5 year overall survival of 92.4%, more likely to have localized, classic type morphology, fewer nodal or distant metastases at presentation (Adv Anat Pathol 2016;23:41)
  • Proximal epithelioid sarcoma is an aggressive tumor; 65% of patients die of disease (Mod Pathol 2001;14:655)
Case reports
Treatment
Clinical images

Contributed by Mark R. Wick, M.D.
Missing Image

Hand

Missing Image

Heel

Gross description
  • Classic subtype usually presents as one or more indurated, ill defined, dermal or subcutaneous nodules (Cancer 1970;26:1029)
  • Deep seated tumors are multinodular masses and involve tendons or fascia
  • Cut surface is glistening with gray-white or gray-tan color and multiple areas of hemorrhage and necrosis (Arch Pathol Lab Med 2009;133:814)
Gross images

Contributed by Borislav Alexiev, M.D.
Missing Image

Mass in subcutis

Microscopic (histologic) description
  • Classic and proximal epithelioid sarcoma can each occur in either proximal or distal sites and may show features of both types (Histopathology 2001;39:641)
  • Classic or conventional, distal type (Adv Anat Pathol 2006;13:114)
    • Dermal / subcutaneous nodule(s)
    • Can simulate granulomatous process, with or without necrosis (pseudogranulomatous morphology)
    • Fairly uniform plump small to medium sized cells with eosinophilic cytoplasm (Adv Anat Pathol 2016;23:41)
    • Spindle cells can also be identified and often appear more conspicuous at the periphery of a nodule
    • Mildly atypical nuclei with vesicular chromatin and small nucleoli
    • Variable mitotic activity
    • Mixed chronic inflammatory infiltrate common
    • Dystrophic calcification and metaplastic bone formation
    • Other variant morphologies (Adv Anat Pathol 2016;23:41)
      • Angiomatoid variant
      • Myxoid variant
      • Other features include intracytoplasmic vacuoles, multinucleated giant cells and storiform pattern
  • Proximal, large cell type (Am J Surg Pathol 1997;21:130, Adv Anat Pathol 2016;23:41)
    • Infiltrative growth pattern with less defined peripheral borders
    • Multiple large nodules
    • Large polygonal cells with abundant eosinophilic cytoplasm
    • Rhabdoid morphology common
    • Vesicular nuclei with prominent macronucleoli
    • Mitoses, necrosis and hemorrhage common
Microscopic (histologic) images

Contributed by Borislav Alexiev, M.D.
Missing Image

Soft tissue neoplasm

Missing Image

Pseudo-granulomatous morphology

Missing Image

Epithelioid morphology

Missing Image

Mild nuclear atypia

Missing Image

Pleomorphic epithelioid cells

Missing Image

Rhabdoid morphology


Missing Image

Pancytokeratin AE1/AE3 expression

Missing Image

CD34 expression

Missing Image

INI1 expression

Virtual slides

Images hosted on other servers:
Missing Image

Epithelioid sarcoma, distal type

Missing Image

Epithelioid sarcoma, proximal type

Cytology description
  • Smears in most cases were moderate to hypercellular, composed of polygonal cells and spindle cells
  • Cells are arranged in loosely cohesive groups, non overlapping clusters and scattered singly, containing moderate to abundant cytoplasm, defined cell borders, vesicular nuclei and discernible nucleoli
  • Variable cytopathologic features include rhabdoid-like intracytoplasmic inclusions, giant cells, marked nuclear atypia and interspersed scanty, metachromatic stroma (Diagn Cytopathol 2016;44:636)
  • Necrotic and inflammatory background (Cancer Cytopathol 2018;126:934)
Positive stains
Negative stains
Electron microscopy description
  • Shows a spectrum of cellular differentiation, from undifferentiated cells or primitive fibrohistiocytic cells to epithelial type cells with junctions, tonofilaments and microvilli (Hum Pathol 1988;19:265)
  • Other findings have included desmosomes, surface microprocesses and interdigitating cell membranes, indicating epithelial (sometimes interpreted as synovial sarcoma-like) differentiation (Adv Anat Pathol 2006;13:114)
  • Proximal epithelioid sarcoma often shows prominent intracytoplasmic intermediate filament aggregates that often take the shape of paranuclear whorls in keeping with the rhabdoid phenotype (Adv Anat Pathol 2016;23:41)
Molecular / cytogenetics description
Sample pathology report
  • Right hand mass, excision:
    • Epithelioid sarcoma, classic type (see comment)
    • Comment: There is a dermal and subcutaneous multinodular lesion composed of small to medium sized oval or polygonal epithelioid cells and plump spindle shaped cells with eosinophilic cytoplasm and mildly atypical nuclei with vesicular chromatic and small nucleoli. The nodules contain centralized necrosis and hemorrhage imparting a granulomatous appearance to the process on low power magnification. The epithelioid cells are concentrated towards the center of the nodules with peripheral spindling. Aggregates of chronic inflammatory cells are present at the periphery of the nodules. The mitotic activity is low: 4 mitoses / 10 high power fields. The lesion involves the underlying facia. Immunohistochemically the epithelioid and spindled cells show strong expression of CAM5.2, EMA and CD34 and are negative for CD31 and CD68. There is loss of INI1 expression in lesional cells. This constellation of morphological and immunohistochemical features strongly supports the diagnosis of epithelioid sarcoma, classic type. The course of epithelioid sarcoma is often unpredictable and it is common for patients to present with extensive disease, lymph node metastases, or distant metastases.
Differential diagnosis
Board review style question #1
    Which of the following is true about epithelioid sarcoma?

  1. ~30% of cases metastasize
  2. Diagnosis always requires clinicopathological and radiological correlation
  3. Epithelioid sarcoma is positive for MDM2 amplification by FISH
  4. Pediatric patients appear to have a better prognosis
  5. Tumor is characterized by marked nuclear pleomorphism in most cases
Board review style answer #1
D. Pediatric patients appear to have a better prognosis

Comment Here

Reference: Epithelioid sarcoma
Board review style question #2

    A 53 year old man presents with a left hip mass. Hematoxylin-eosin stains demonstrate multinodular and sheet-like growth of large epithelioid cells with enlarged vesicular nuclei and prominent nucleoli. Aggregates of rhabdoid cells with glassy intracytoplasmic hyaline inclusions and foci of tumor necrosis are frequently encountered. Mitotic figures are uncommon (5 mitoses / 10 high power fields). The lesion involves deep soft tissue and underlying bone. Immunohistochemical stains for EMA, CAM5.2 and CD34 are positive in epithelioid cells and there is loss of INI1 expression. Stains for SMA, myogenin, MyoD1, CD31, FLI1 and SALL4 are negative. Which of the following is most likely the correct diagnosis?

  1. Epithelioid angiosarcoma
  2. Epithelioid hemangioendothelioma
  3. Epithelioid sarcoma, proximal type
  4. Extrarenal rhabdoid tumor
  5. Pleomorphic rhabdomyosarcoma
Board review style answer #2
C. Epithelioid sarcoma, proximal type

Comment Here

Reference: Epithelioid sarcoma

Epstein-Barr virus associated smooth muscle tumor
Definition / general
Essential features
  • Neoplasm with smooth muscle differentiation
  • Clinical history of immunosuppression
  • Positivity for Epstein-Barr virus encoded small RNA (EBER) transcripts by in situ hybridization
  • Fascicles of spindled cells with eosinophilic cytoplasm and elongated nuclei; a second population of round, more primitive appearing smooth muscle cells in a subset of cases
  • Tumor infiltrating lymphocytes; the lymphocytic infiltrate is composed primarily of T cells
Terminology
  • Not recommended: AIDS associated EBV positive smooth muscle tumor, EBV associated posttransplant smooth muscle tumor
ICD coding
  • ICD-O: 8897/1 - smooth muscle tumor of uncertain malignant potential
  • ICD-11: 2F7Y & XH00B4 - neoplasms of uncertain behavior of other specified site & smooth muscle tumor, NOS
Epidemiology
  • Wide age range (1 - 66 years), with a slight female predominance
  • Most cases occur in 1 of 3 main settings (Am J Surg Pathol 2006;30:75, Head Neck Pathol 2021;15:1162, Front Immunol 2018:9:368)
    • Immunodeficiency due to HIV / AIDS
    • Immunodeficiency after transplantation of a solid organ or hematopoietic stem cells
    • Congenital or primary immunodeficiency (least commonly)
  • Most primary immunodeficiency patients with EBV associated smooth muscle tumors are children (88%)
  • 68% of posttransplant and 72% of HIV / AIDS patients are adults
  • In the posttransplant EBV SMT group, they present as late complications (median: 48 months; range: 5 - 348 months) (Front Immunol 2018:9:368)
Sites
  • EBV SMT can occur anywhere in the body, including sites unusual for sporadic leiomyomas and leiomyosarcomas
  • Posttransplant EBV SMTs most commonly involve the liver (56%), followed by the lungs (31%) and gastrointestinal tract (15%) (Pathobiology 2013;80:297)
  • In patients with HIV, EBV SMTs have a particular predilection for the intra-axial or extra-axial CNS (41% of cases)
  • EBV SMTs can occur at single or multiple sites synchronously or metachronously and may grow continuously (Front Immunol 2018:9:368)
Pathophysiology
  • Pathogenesis appears to be related to infection and transformation of smooth muscle cells by EBV (Patholog Res Int 2011;2011:561548)
  • Having entered a smooth muscle cell, it is unclear how EBV infection causes neoplastic transformation with clonal proliferation
  • Majority of HIV EBV SMTs are positive for complement receptor 2 (CR2 or CD21) that is bound by EBV during B cell infection, while a substantial number of EBV SMTs resulting from organ transplantation associated immunosuppressive treatment and all analyzed cases of EBV SMTs arising from primary immunodeficiency disorders are CD21 negative (Front Immunol 2018:9:368)
  • EBV generally achieves a latency type III-like pattern in EBV SMTs (i.e., cells are positive for EBV nuclear antigen 2 [EBNA2], EBNA3 and late membrane protein 1) (Front Immunol 2018:9:368)
  • MYC overexpression and AKT-mTOR pathway activation appear as the main mediators of EBV SMT proliferation (Clin Cancer Res 2009;15:5350, Clin Transplant 2013;27:E462)
  • Epigenetic alterations may play an important role in tumorigenesis as recurrent copy number gains were found in histone deacetylases (Mod Pathol 2023;36:100127)
Etiology
  • EBV infection in the setting of T lymphocyte immunosuppression
  • EBV SMTs usually develop in the context of secondary immunodeficiency caused by human immunodeficiency virus infection or immunosuppressive treatment after solid organ transplantation (Front Immunol 2018:9:368, Am J Surg Pathol 2006;30:75, Head Neck Pathol 2021;15:1162)
  • In a small fraction of patients (mostly pediatric), EBV associated smooth muscle tumors may occur with primary immunodeficiency disorders, such as GATA2 and CARMIL2 deficiency (Front Immunol 2018:9:368)
Clinical features
  • Clinical manifestation of EBV SMTs is unspecific and mainly depends on the tumor localization, the tumor size and the particular organ displacement or disruption (Front Immunol 2018:9:368)
Diagnosis
  • EBV SMTs are suspected in the context of secondary immunodeficiency and primary immunodeficiency
  • No pathognomonic radiological picture
  • Biopsy with histopathology, immunohistochemistry and EBER in situ hybridization are mandatory to formally establish the diagnosis (Eur J Cancer 2014;50:2417)
Radiology description
Radiology images

Contributed by Borislav A. Alexiev, M.D.
Cervical spine MRI

Cervical spine MRI



Images hosted on other servers:
CT and PET scans

CT and PET scans

Prognostic factors
  • Prognosis is mainly dependent on the condition of the individual patient's immune system
  • Most EBV SMTs do not metastasize
  • Adverse prognosis is mainly related to comorbidities associated with immunosuppression but not with the extent of histological atypia or tumor size (Pathobiology 2013;80:297)
  • EBV SMTs do not correlate well with their histologic features and apparent grade of lesion, unlike somatic smooth muscle tumors (Arch Pathol Lab Med 2016;140:718)
    • It has been proposed that these tumors be designated as smooth muscle tumors of uncertain potential
  • Poor prognosis is mainly associated with intracranial manifestation and nonresectable tumors
Case reports
Treatment
  • Surgery or reduced immunosuppression are the main therapeutic approaches and both provide comparable overall survival rates (Am J Transplant 2012;12:1908)
  • Other therapies are chemotherapy and palliative radiation
Clinical images

Images hosted on other servers:
Endobronchial mass

Endobronchial mass

Right arytenoid lesion

Right arytenoid lesion

Gross description
Gross images

Images hosted on other servers:
Liver nodules

Liver nodules

Frozen section description
  • Intersecting fascicles of spindled cells with ample eosinophilic cytoplasm and elongated nuclei (Arch Pathol Lab Med 2016;140:718, Am J Surg Pathol 2006;30:75, Head Neck Pathol 2021;15:1162)
  • In ~50% of cases, a second population of round appearing cells may be seen
  • Cytological atypia is mild to moderate but can be marked in HIV positive patients
  • Tumor infiltrating lymphocytes
  • Low mitotic activity
  • Tumors in HIV positive patients may show higher mitotic activity or necrosis
Microscopic (histologic) description
  • Fascicular arrangement of relatively well differentiated smooth muscle cells with brightly eosinophilic cytoplasm and elongated, blunt ended nuclei exhibiting variable atypia (Arch Pathol Lab Med 2016;140:718, Am J Surg Pathol 2006;30:75, Head Neck Pathol 2021;15:1162)
  • There are 2 important defining and unique features
    • Presence of variable numbers of intratumoral lymphocytes; the lymphocytic infiltrate is composed primarily of T cells
    • Presence of so called primitive round cell areas (round, oval and epithelioid) arising gradually or abruptly from the well differentiated smooth muscle cells
    • These features may vary considerably in different cases
  • Subset of these tumors exhibits a hemangiopericytoma-like pattern
  • Vesicular nuclei with small nucleoli
  • Moderate increased cellularity, lack of nuclear pleomorphism and low mitotic rate
  • Focal areas of necrosis may be present
  • Histopathologic features may vary considerably in terms of cellular atypia, mitotic activity and necrosis, with no correlation to the clinical behavior (Arch Pathol Lab Med 2016;140:718)
  • HIV / AIDS EBV associated smooth muscle tumor subtype shows the most histologic variation, ranging from standard leiomyoma-like to leiomyosarcoma-like and even angioleiomyoma or myopericytoma-like features
    • In such cases, the detection of EBV in the tumor cells remains the mainstay for distinguishing them from conventional leiomyosarcoma
Microscopic (histologic) images

Contributed by Borislav A. Alexiev, M.D.
Moderate increased cellularity

Moderate increased cellularity

Spindle cell proliferation

Spindle cell proliferation

Blunt ended nuclei

Blunt ended nuclei

Low degree of atypia

Low degree of atypia


Intratumoral lymphocytes

Intratumoral lymphocytes

Smooth muscle actin

Smooth muscle actin

EBER ISH

EBER ISH

CD3

CD3

Positive stains
Electron microscopy description
  • Spindle cells with abundant cytoplasmic aggregates of actin microfilaments, including fusiform dense bodies and attachment plaques (Sarcoma 2008:2008:859407)
  • Cytoplasmic organelles are mainly represented by mitochondria, while rough endoplasmic reticulum cisternae are sparse
  • In studies in which electron microscopy was performed, no nuclear viral particles were detected (Patholog Res Int 2011;2011:561548)
Electron microscopy images

Images hosted on other servers:
Spindle cells with microfilaments

Spindle cells with microfilaments

Molecular / cytogenetics description
  • Overall copy number alteration profile is closer to leiomyoma
  • Recurrent copy number gain of oncogenes, such as RUNX1, CCND2 and ETS2 (Mod Pathol 2023;36:100127)
  • Epigenetic alterations may play an important role in tumorigenesis as recurrent copy number gains were found in histone deacetylases
Molecular / cytogenetics images

Images hosted on other servers:
Ideograms

Ideograms

Unsupervised hierarchical clustering

Unsupervised hierarchical clustering

Copy number alteration score

Copy number alteration score

Enrichment plots

Enrichment plots

Sample pathology report
  • C4 / C5 intradural tumor, resection:
    • EBV associated smooth muscle tumor (see comment)
    • Comment: The patient is a 35 year old woman, status post-kidney pancreas transplant. She presented with back pain and right upper extremity weakness and was found to have an intradural extramedullary lesion at C4 - C5 measuring 2.5 x 1.5 x 1.0 cm. She underwent C3 - C6 posterior cervical fusion with intradural tumor resection.
    • Histologic sections of the intradural mass show a moderately cellular lesion composed of spindle cells with eosinophilic cytoplasm and elongated, blunt ended vesicular nuclei with small nucleoli. Mitotic activity is low (2 mitoses/10 high power fields). There is no tumor necrosis. An intratumoral lymphocytic infiltrate is present.
    • The spindle cells are positive for smooth muscle actin and h-caldesmon and negative for all other markers analyzed (keratin AE1 / AE3, SOX10, desmin, ALK1, CD30, MyoD1). In situ hybridization for EBER shows strong diffuse positivity in the spindle cells.
    • The findings are consistent with an EBV associated smooth muscle tumor in a posttransplant setting.
Differential diagnosis
  • Mycobacterial spindle cell pseudotumor (Head Neck Pathol 2023;17:782):
    • Monotonous, spindle cell population in a vaguely storiform arrangement mixed with a heavy infiltrate of neutrophils and sparse lymphocytes
    • Spindle cell population stains strongly and diffusely for CD68 and is negative for muscle markers
    • Ziehl-Neelsen stain discloses numerous intracytoplasmic acid fast bacilli
    • EBER negative
  • Inflammatory myofibroblastic tumor (Diagn Pathol 2023;18:105):
    • Spindle cells are arranged in fascicular or haphazard patterns, accompanied by admixtures of inflammatory infiltrate with an abundance of plasma cells and lymphocytes in a hyalinized or myxoid background
    • Variable expression of smooth muscle markers, CD34 and MDM2
    • > 50% of inflammatory myofibroblastic tumors contain rearrangements of the anaplastic lymphoma kinase (ALK) gene on chromosome 2p23, resulting in the aberrant expression of ALK chimeric protein by IHC
    • ROS1 and RET rearrangements in rare cases of ALK negative inflammatory myofibroblastic tumor (Am J Surg Pathol 2015;39:957)
    • EBER negative
  • Inflammatory leiomyosarcoma / rhabdomyoblastic tumor (Genes Chromosomes Cancer 2022;61:653):
    • Morphologic and immunohistochemical features show overlap of smooth and skeletal muscle phenotypes
    • Sheets and fascicles of variably pleomorphic tumor cells showing spindled and epithelioid to rhabdoid morphology and a prominent histiocyte rich inflammatory infiltrate
    • Calcospherites (psammomatous calcifications) are a striking feature in the majority of cases
    • May show immunopositivity for some combination of smooth muscle actin, desmin and h-caldesmon, although h-caldesmon was absent in 1 cohort of 13 patients (Mod Pathol 2021;34:758)
    • Expression of MyoD1
    • Characteristic genetic finding is a near haploid loss of heterozygosity
    • EBER negative
  • Leiomyoma (Mod Pathol 2014;27:S17):
    • Histologically indistinguishable from EBV associated smooth muscle tumor in most cases
    • Rare in visceral organs outside uterus
    • Prominent intratumoral lymphocytic infiltrate, a characteristic feature of EBV associated smooth muscle tumor, is rarely seen
    • EBER negative
  • Angioleiomyoma (Mod Pathol 2014;27:S17):
    • Bundles of bland, well differentiated smooth muscle cells and intervening variably sized blood vessels
    • Most commonly in the subcutis or dermis of extremities; rare in visceral organs
    • Histologic variations include adipocytic metaplasia, prominent hyalinization / calcification and degenerative atypia
    • Prominent intratumoral lymphocytic infiltrate, a characteristic feature of EBV associated smooth muscle tumor, is rarely seen
    • EBER negative
  • Myopericytoma, including myofibroma (J Clin Pathol 2006;59:67):
    • Myopericytomas are nodular or lobular neoplasms composed of cytologically bland, oval to spindle shaped, myoid cells with characteristic multilayered, concentric growth around numerous small blood vessels
    • Myofibromas are nodular, well circumscribed neoplasms characterized by a distinctive biphasic growth pattern; the center of the lesion is composed of immature appearing, plump, spindled cells associated with hemangiopericytoma-like vasculature, whereas the periphery of the lesion contains nodules and fascicles of variable hyalinized myoid cells
    • Prominent intratumoral lymphocytic infiltrate, a characteristic feature of EBV associated smooth muscle tumor, is rarely seen
    • EBER negative
  • Leiomyosarcoma (Mod Pathol 2014;27:S17):
    • Focal pleomorphism is common even in low grade tumors with low mitotic activity
    • TP53 / p53 germline mutations
    • EBER negative
Board review style question #1

Which of the following is true for EBV associated smooth muscle tumors?

  1. Most occur in the setting of congenital or primary immunodeficiency
  2. Posttransplant tumors most commonly involve the gastrointestinal tract
  3. In patients with HIV, EBV associated smooth muscle tumors have a particular predilection for the intra-axial or extra-axial CNS
  4. Radiological imaging can establish the diagnosis of EBV associated smooth muscle tumors
  5. Most HIV / AIDS patients with EBV associated smooth muscle tumor are children
Board review style answer #1
C. In patients with HIV, EBV associated smooth muscle tumors have a particular predilection for the intra-axial or extra-axial CNS. Answer A is incorrect because EBV associated smooth muscle tumors rarely occur in the setting of congenital or primary immunodeficiency. Answer B is incorrect because posttransplant tumors most commonly involve the liver. Answer D is incorrect because radiological imaging cannot establish the diagnosis of EBV associated smooth muscle tumors as there are no pathognomonic findings. Answer E is incorrect because most HIV / AIDS patients with EBV associated smooth muscle tumors are adults (72%).

Comment Here

Reference: Epstein-Barr virus associated smooth muscle tumor
Board review style question #2
Which of the following is true for EBV associated smooth muscle tumors?

  1. Poor prognosis is mainly associated with intracranial manifestation and nonresectable tumor
  2. Most EBV associated smooth muscle tumors do metastasize
  3. Adverse prognosis is mainly related to histological atypia
  4. Malignant behavior and clinical outcome are predicted by the histologic features
  5. Chemotherapy and palliative radiation are the main therapeutic approaches
Board review style answer #2
A. Poor prognosis is mainly associated with intracranial manifestation and nonresectable tumor. Answer B is incorrect because most EBV associated smooth muscle tumors do not metastasize. Answer C is incorrect because adverse prognosis is mainly related to comorbidities associated with immunosuppression but not with the extent of histological atypia or tumor size. Answer D is incorrect because unlike in somatic smooth muscle tumors, where the malignant behavior and clinical outcome are predicted by the histologic features, the behavior of EBV associated smooth muscle tumors does not correlate well with their histologic features and apparent grade of lesion. Answer E is incorrect because surgery or reduced immunosuppression are the main therapeutic approaches and both provide comparable overall survival rates.

Comment Here

Reference: Epstein-Barr virus associated smooth muscle tumor

EWSR1::SMAD3 positive fibroblastic tumor
Definition / general
Essential features
  • Extremely rare, fusion defined, locally aggressive soft tissue tumor
  • So far described only on the extremities, with a strong predilection for acral parts
  • Wide age range (1 - 68 years); M:F = 1:4
  • 2 main components: hypercellular spindled fibroblasts merging with acellular hyalinized areas
  • All cases show diffuse strong nuclear expression of ERG, which seems to be very specific in the pertinent differential diagnosis
Terminology
  • EWSR1::SMAD3 positive fibroblastic tumor
ICD coding
  • No official coding established
Epidemiology
  • Extremely rare tumor
  • Wide age range (1 - 68 years)
  • M:F = 1:4
Sites
  • Extremities, particularly acral parts
Pathophysiology
  • Gene fusion driven
Etiology
  • Unknown
Clinical features
  • Small, painless, infiltratively growing tumor located in the dermis / subcutis
  • Frequently recurs due to infiltrative growth (4 of 6 cases with follow up recurred)
  • May recur many years after initial excision (Am J Surg Pathol 2018;42:1325)
  • Malignant variant not described
Diagnosis
  • Clinical and morphological features: hypercellular tumor, usually located on the distal extremities, composed of monomorphic bland spindled fibroblasts with minimal mitotic activity merging with acellular hyalinized areas
  • Diffuse strong nuclear expression of ERG
  • Confirmation of EWSR1 rearrangements in challenging cases
Radiology description
Prognostic factors
  • Negative margin status seems to affect the likelihood of recurrence
Case reports
Treatment
  • Complete excision seems to be curative
Gross description
Microscopic (histologic) description
  • Usually located in the subcutis but dermal examples directly abutting the epidermis have been described (J Cutan Pathol 2021;48:255)
  • Nodular or less frequently vaguely lobulated / plexiform growth pattern
  • Usually infiltrative growth, sometimes leading to engulfment of the surrounding subcutaneous adipose tissue
  • 2 main components: hypercellular spindled fibroblasts gradually merging with acellular hyalinized areas
  • Spindled cells form hypercellular, well organized fascicles that frequently intersect with each other
  • Spindled cells have relatively uniform, elongated, focally wavy nuclei that are round when observed on a cross section
  • Moderate amount of eosinophilic cytoplasm
  • Lack of pleomorphism, atypia or mitoses
  • Some cases show distinct zonation pattern with a hyalinized center and cellular areas at the periphery; others do not
  • Only 1 case showed focal stippled calcifications in the hyalinized area
Microscopic (histologic) images

Contributed by Michael Michal, M.D., Ph.D.
Nodular growth in most cases

Nodular growth in most cases

Engulfment of surrounding adipose

Engulfment of surrounding adipose

No pleomorphism, atypia or mitoses

No pleomorphism, atypia or mitoses

Merging of spindled and hyalinized areas

Merging of spindled and hyalinized areas

Elongated nuclei, round on cross section

Elongated nuclei, round on cross section


Intersecting hypercellular fascicles

Intersecting hypercellular fascicles

Hyalinized areas

Hyalinized areas

Diffuse ERG

Diffuse ERG

Focal SATB2

Focal SATB2

Cytology description
  • Unknown
Positive stains
Negative stains
Electron microscopy description
  • Unknown
Molecular / cytogenetics description
Sample pathology report
  • Left foot, excision:
    • EWSR1::SMAD3 rearranged fibroblastic tumor (see comment)
    • Comment: There is a hypercellular proliferation of spindled cells with a large hyalinized area in the center of the lesion. Immunohistochemically the tumor cells have strong nuclear expression of ERG and are negative for SMA, desmin, CD34, S100, OSCAR and EMA. This constellation of morphological and immunohistochemical features strongly supports the diagnosis of EWSR1::SMAD3 rearranged fibroblastic tumor. It is a recently characterized, locally aggressive mesenchymal neoplasm prone to recur unless completely excised. Metastatic spread has not been reported.
Differential diagnosis
  • No entity in the pertinent differential diagnosis shows strong and diffuse ERG expression (Am J Surg Pathol 2018;42:1325)
  • Cellular fibrous histiocytoma:
    • Contains slightly shorter fascicles composed of usually less uniform spindled cells
    • Exhibits more prominent storiform pattern and many cases show a characteristic collagen entrapment at the periphery, which is not typically seen in EWSR1::SMAD3 positive fibroblastic tumor
    • Most cases express factor XIIIa
    • Rearrangements of EWSR1 gene are absent
  • Myofibroma:
    • Shows plumper spindled cells
    • Hemangiopericytoma-like vessels with a pericytic arrangement of the surrounding cells
    • Mostly positive with SMA
    • Does not have strong and diffuse ERG expression
    • Most cases show PDGFRB gene mutation (Am J Surg Pathol 2017;41:195)
  • Lipofibromatosis:
    • Lacks the prominent hyalinized areas
    • Has more abundant fat component that is present throughout the tumor and not only at the periphery
    • Tumor cells are oval rather than spindled
    • No prominent intersecting of tumor fascicles
    • Does not have strong and diffuse ERG expression
    • Molecular background characterized by various gene fusions (Mod Pathol 2019;32:423)
  • Calcifying aponeurotic fibroma:
    • Contains calcifications / chondroid foci rather than hyalinized areas
    • Variable and overall lower cellularity
    • More haphazard arrangement of the tumor fascicles
    • Focal weak ERG expression (Am J Surg Pathol 2018;42:1325)
    • Defined by FN1-EGF gene fusion (J Pathol 2016;238:502)
  • Lipofibromatosis-like neural tumor:
    • Lacks prominent hyalinized areas
    • More atypical and hyperchromatic cells
    • No prominent intersection of tumor fascicles
    • Usually positive for S100 and CD34
    • Does not have strong and diffuse ERG expression
    • Most cases have NTRK1 gene fusions (those also express NTRK1 by immunohistochemistry), subset shows ALK or ROS1 rearrangement
  • Fibromatosis:
    • Lacks the prominent hyalinized areas
    • Much less cellular
    • Nuclei are less plump, wavier and more curved
    • Positive with SMA
    • Does not have strong and diffuse ERG expression
  • Monophasic synovial sarcoma:
    • Much higher mitotic and proliferative indices
    • Usually has vaguely epithelioid areas which can be confirmed by immunohistochemistry
    • Hemangiopericytomatous vasculature
    • Prominent mast cell infiltrate
    • Stains with EMA, keratins, TLE1
    • Does not have strong and diffuse ERG expression
    • Characterized by SYT-SSX gene fusion
  • Cutaneous forms of myoepithelioma:
    • Significantly different morphologically and immunohistochemically
    • Does not have strong and diffuse ERG expression
    • May potentially enter the differential due to the EWSR1 gene break as detected by FISH
Board review style question #1
Which of the following is true about EWSR1::SMAD3 rearranged fibroblastic tumor?

  1. ~50% of cases metastasize
  2. All cases show diffuse and strong nuclear staining with ERG antibody
  3. It typically occurs in the head and neck area
  4. Tumor consists of neoplastic adipose tissue and epithelioid cells
  5. Tumor has a prominent hemangiopericytoma-like vasculature and chondroid areas
Board review style answer #1
B. All cases show diffuse and strong nuclear staining with ERG antibody

Comment Here

Reference: EWSR1::SMAD3 positive fibroblastic tumor
Board review style question #2

A 1 year old infant presented with a tumor on her palm. It is negative with the standard soft tissue immunohistochemical panel consisting of SMA, desmin, CD34, S100, OSCAR and EMA. It was additionally negative for NTRK1 immunohistochemistry. Which of the following is most likely the correct diagnosis?

  1. EWSR1::SMAD3 rearranged fibroblastic tumor
  2. Fibromatosis
  3. Lipofibromatosis-like neural tumor
  4. Monophasic synovial sarcoma
  5. Myofibroma
Board review style answer #2
A. EWSR1::SMAD3 rearranged fibroblastic tumor

Comment Here

Reference: EWSR1::SMAD3 positive fibroblastic tumor

Extrarenal rhabdoid tumor
Definition / general
  • Heterogeneous group of neoplasms unified by the presence of globular cytoplasmic inclusions, vesicular nuclei with prominent nucleoli and aggressive behavior
  • Response to therapy is poor
  • Metastasizes to lung, liver and lymph nodes early in the course of disease
Essential features
  • Abundant eosinophilic cytoplasm (due to hyaline globular inclusions of intermediate filaments) with eccentric vesicular nuclei and prominent nucleoli
  • Skeletal muscle markers and S100 negative
  • Karyotypic rearrangements of 22q11.2 resulting in homozygous inactivation of SMARCB1 (hSNF5 / INI1), with subsequent loss of INI1 nuclear expression by immunohistochemistry
Terminology
  • Malignant rhabdoid tumor (synonym)
ICD coding
  • ICD-O: 8963/3 - rhabdoid tumor, NOS
  • ICD-11: 2B5F.2 & XH3RF3 - sarcoma, not elsewhere classified of other specified sites & malignant rhabdoid tumor
Epidemiology
  • Genuine rhabdoid tumors exclusively belong to the pediatric age group and are very rare
  • Adult cases represent expression of a particular phenotype, which can occur in any type of sarcoma or even carcinoma but it is virtually universally associated with an aggressive course
  • Mean age at diagnosis < 1 year for tumors with germ line SMARCB1 alterations and 1.5 years for sporadic tumors (Crit Rev Oncog 2015;20:199)
  • M = F
Sites
Pathophysiology
Clinical features
  • Clinical presentation is related to the site of involvement
  • Intraocular tumors present with proptosis (J Pediatr Hematol Oncol 2020;42:228)
  • Deep / visceral involvement causes local pain and pressure symptoms and may result in loss of function of an anatomic site if local neural bundle / plexus is compromised (Brain Dev 2017;39:717)
Diagnosis
  • Rhabdoid cell morphology in a pediatric tumor along with loss of INI1
  • Requires thorough sampling and immunohistochemical studies to rule out rhabdoid phenotype in another underlying tumor, particularly in adult cases (Mod Pathol 2016;29:1232)
  • Demonstration of 22q11.2 mutations in a tumor with well developed rhabdoid morphology (Genes Chromosomes Cancer 2011;50:379)
Radiology description
Radiology images

Images hosted on other servers:

Intratumoral hemorrhage, bone erosion

Lobulated mediastinal mass

Multilobulated hyperintense mass

Prognostic factors
Case reports
Treatment
  • No definite chemotherapy regimen is available; some cases report successful therapy using alkaloids, platinum agents and combinations of chemotherapy, surgery and radiation (Crit Rev Oncog 2015;20:199)
  • Targeted therapy is under investigation, utilizing various epigenetic pathways including DNA and histone methylation, histone deacetylation, cell cycle arrest and antimitotic mechanisms (Crit Rev Oncog 2015;20:199)
Clinical images

Images hosted on other servers:

Ulcerating tumor at heel

Gross description
  • Tumor is infiltrative with a tan-white solid appearance
  • Calcifications and hemorrhage may be seen (Urology 2020;137:164)
Gross images

Contributed by Mark R. Wick, M.D.

Superficial, subcuticular



Images hosted on other servers:

Polypoid mass with hemorrhage

Frozen section description
  • Frozen sections are rarely performed and show sheets of cells with typical rhabdoid appearance and vesicular nuclei with nucleoli
Microscopic (histologic) description
  • Tumor cells have classic rhabdoid or skeletal-like profile, comprised of abundant cytoplasm with eosinophilic hyaline globules and vesicular nuclei with prominent nucleoli (Anticancer Res 2005;25:4573, Ann Diagn Pathol 2012;16:504)
  • Some cases may show areas of spindling of tumor cells
  • Nuclear pleomorphism can be prominent
  • Background stroma may be variably myxoid to fibromyxoid
  • Tumor necrosis and mitotic activity can be variable
Microscopic (histologic) images

Contributed by Nasir Ud Din, M.B.B.S.
Rhabdoid cells

Rhabdoid cells

Prominent nucleoli

Prominent nucleoli

Sheet-like growth

Sheet-like growth

Nuclear pleomorphism

Nuclear pleomorphism

Spindling of tumor cells

Spindling of tumor cells

Myxoid stroma

Myxoid stroma


Fibromyxoid stroma

Fibromyxoid stroma

Tumor necrosis

Tumor necrosis

EMA

EMA

CD99

CD99

CD56

CD56

SALL4

SALL4


Loss of nuclear INI1 Loss of INI1

Loss of INI1

Cytology description
  • Variably cellular smears with individual cells and structureless clusters of rhabdoid cells, spindle cells or round cells
  • Cytoplasmic inclusions are eosinophilic on Giemsa and pale gray on Papanicolaou stain (Indian J Pathol Microbiol 2011;54:819)
  • Differentials on cytology include extrarenal Wilms tumor, rhabdomyosarcoma, spindle cell sarcoma and round blue cell tumors including lymphoma (Indian J Pathol Microbiol 2011;54:819)
Cytology images

Contributed by Mark R. Wick, M.D.
Rhabdoid cells in smears

Rhabdoid cells in smears



Images hosted on other servers:

Giemsa and Papanicolaou stains

Positive stains
Negative stains
Electron microscopy description
Electron microscopy images

Images hosted on other servers:

Cytoplasmic whorls

Molecular / cytogenetics description
Molecular / cytogenetics images

Images hosted on other servers:

Homozygous deletion on MLPA

Sample pathology report
  • Right arm, excision:
    • Extrarenal rhabdoid tumor (see comment)
    • Comment: Tumor cells are rhabdoid in appearance with eccentric nuclei. Positive cytokeratin, negative skeletal markers and loss of INI1 are seen. 22q11.2 mutation confirms the diagnosis.
Differential diagnosis
In children:
  • Rhabdomyosarcoma:
    • Embryonal rhabdomyosarcoma:
      • Common in children < 10 years old
      • M > F
      • Most common sites: head and neck region, genitourinary region
      • May have rhabdomyoblasts / strap cells with cross striations
      • May show eccentric cytoplasm similar to extrarenal rhabdoid tumor
      • Shows positive staining for markers of skeletal differentiation (desmin, myogenin and MyoD1)
      • Retained nuclear INI1
    • Alveolar rhabdomyosarcoma:
      • Occurs in adolescents and young adults
      • Grows as alveoli-like spaces formed by loss of cohesion centrally
      • Monomorphic round cells with hyperchromatic nuclei and scant cytoplasm
      • Strong and diffuse desmin and myogenin positivity
      • May express focal / weak cytokeratins (5 - 50% cases)
      • Intact nuclear INI1
  • Ewing sarcoma:
    • Shows sheet-like growth of round blue cells with scant cytoplasm and hyperchromatic nuclei
    • Diffuse CD99 and NKX2.2 expression
    • Retained nuclear INI1
    • May show cytokeratin positivity in up to 20% cases
    • EWSR1 mutations are diagnostic
  • Desmoplastic round blue cell tumor:
    • Mean age 25 years
    • M > F
    • Abdominal cavity is the most common site
    • Contains prominent desmoplastic stroma
    • Round cells grow in nodules separated by stroma
    • May show polyphenotypic differentiation with positive keratins, vimentin, desmin, CD56, NSE and nuclear WT1
    • Retained nuclear INI1
    • EWSR1-WT1 gene fusion is consistent

In adults:
  • Extrarenal rhabdoid tumor should be considered only after other tumors with a rhabdoid appearance have been ruled out
  • Carcinoma:
    • Age over 50, a history of carcinoma and areas of cohesive epithelial differentiation help in reaching correct diagnosis
  • Malignant melanoma:
    • Nonpigmented melanoma mimics rhabdoid tumor
    • Occurs in older age group
    • Positive melanocytic markers HMB45, MelanA and negative cytokeratins
  • Anaplastic large cell lymphoma:
    • May occasionally exhibit rhabdoid features
    • Positive for CD30, weak / focal positive for CD45 / LCA and CD3 and negative for keratins
  • Proximal type epithelioid sarcoma:
    • Positive for keratins, CD34 (50% cases) and may show loss of nuclear INI1
    • Harbors mutations of 22q11 with resultant abnormalities of SMARCB1 but the mechanism of these mutations is different than that underlying extrarenal rhabdoid tumors
Board review style question #1

A 2 month old baby presented with a progressively enlarging soft tissue mass in the right axillary region. Microscopic examination of the resected tumor showed morphology depicted in the picture. Immunohistochemical stains to help reach a diagnosis are

  1. Negative desmin and nuclear INI1
  2. Negative vimentin and positive S100
  3. Positive cytokeratin and CD34
  4. Positive cytokeratin and desmin
  5. Positive cytokeratin and negative vimentin
Board review style answer #1
A. Negative desmin and nuclear INI1. This is an extrarenal rhabdoid tumor.

Comment Here

Reference: Extrarenal rhabdoid tumor
Board review style question #2
Homozygous inactivation of SMARCB1 seen in extrarenal rhabdoid tumors are due to mutations in

  1. 9q22
  2. 11p13
  3. 11q24
  4. 21q12
  5. 22q11
Board review style answer #2

Extraskeletal myxoid chondrosarcoma
Definition / general
  • Extraskeletal myxoid chondrosarcoma (EMC) is a malignant mesenchymal neoplasm of uncertain differentiation with abundant myxoid matrix, multilobular architecture, uniform cells arranged in cords, clusters and reticular networks and NR4A3 rearrangement
Essential features
  • Bland cells with eosinophilic cytoplasm and round to oval nuclei with evenly distributed chromatin and inconspicuous nucleolus
  • Tumor cells are characteristically interconnected with one another to form cords and small clusters and complex trabecular or cribriform arrays
  • Despite the name, there is no evidence of cartilaginous differentiation
  • NR4A3 rearrangement
Terminology
  • NR4A3 rearranged myxoid sarcoma (provisional)
ICD coding
  • ICD-O: 9231/3 - Extraskeletal myxoid chondrosarcoma
  • ICD-11: 2B5F.2 & XH9344 - Sarcoma, not elsewhere classified of other specific sites and myxoid chondrosarcoma
Epidemiology
Sites
Pathophysiology
Etiology
  • Unknown
Clinical features
Diagnosis
Radiology description
Radiology images

Contributed by Borislav A. Alexiev, M.D.
MRI of soft tissue mass

MRI of soft tissue mass

Prognostic factors
Case reports
Treatment
  • Standard treatment for localized disease is surgery, plus or minus radiation therapy with an expected prolonged survival, even though the risk of relapse is about 50% (Cancers (Basel) 2020;12:2703)
  • In advanced cases, besides the standard chemotherapy currently used for soft tissue sarcoma, antiangiogenic agents have recently shown promising activity (Cancers (Basel) 2020;12:2703)
Gross description
  • Well demarcated, lobulated (Cancer 1998;83:1504)
  • Myxoid nodules separated by fibrous septa
  • Cystic areas with hemorrhages
Gross images

Contributed by Borislav A. Alexiev, M.D.
Soft tissue mass

Soft tissue mass

Frozen section description
  • Uniform cells with round or spindled shaped nuclei and eosinophilic cytoplasm arranged in cords or clusters within abundant myxoid matrix
Microscopic (histologic) description
  • Multinodular architecture (Cancer 1998;83:1504)
  • Fibrous septa divide the tumor in pools of abundant myxoid or chondromyxoid matrix containing tumor cells (Cancer 1998;83:1504, Pathol Int 2005;55:453)
  • Uniform cells with eosinophilic to vacuolated cytoplasm, often with long delicate cytoplasmic processes and round to oval nuclei with evenly distributed chromatin and inconspicuous nucleolus (Am J Surg Pathol 1999;23:636)
  • Tumor cells are characteristically interconnected with one another to form cords, small clusters and complex trabecular or cribriform arrays
  • Spindle cell differentiation is common
  • Mitotic activity is low
  • Stroma is hypovascular
  • Foci of intralesional hemorrhages are frequently seen in various proportions (Cancer 1998;83:1504)
  • Usually lacks discernible cartilaginous histology (Pathol Int 2005;55:453)
  • 80% of EMCs with variant (non-EWSR1) NR4A3 gene fusions (TAF15, TCF12) demonstrate high grade morphology with increased cellularity, proliferation and cytologic atypia, showing epithelioid / rhabdoid features in half of the cases (Hum Pathol 2014;45:1084)
Microscopic (histologic) images

Contributed by Borislav A. Alexiev, M.D.
Lobular architecture

Lobular architecture

Intralesional hemorrhage

Intralesional hemorrhage

Spindle cell differentiation

Spindle cell differentiation

Cohesive clusters

Cohesive clusters

Low grade nuclei

Low grade nuclei


NSE

NSE

AE1 / AE3

AE1 / AE3

Cytology description
Cytology images

Images hosted on other servers:

Thigh tumor

Negative stains
Electron microscopy description
Molecular / cytogenetics description
Molecular / cytogenetics images

Images hosted on other servers:
EWSR1 and NR4A3 FISH

EWSR1 and NR4A3 FISH

Sample pathology report
  • Left thigh, excision:
    • Extraskeletal myxoid chondrosarcoma (see comment and synoptic report)
    • Comment: The tumor has a multinodular architecture. Thick fibrous bands separate pools of myxoid stroma containing uniform tumor cells with eosinophilic or vacuolated cytoplasm and round to oval nuclei with evenly distributed chromatin and inconspicuous nucleolus. The cells are interconnected with one another to form cords and small clusters. Foci of intralesional hemorrhages are frequently seen. Mitotic activity is low (1 mitosis/10 high power fields). Immunohistochemically, the tumor cells are positive for NSE and negative for SMA, desmin, GFAP, CD34, S100, AE1 / AE3 and EMA. FISH studies demonstrate rearrangements of NR4AE at 9q22.33 and EWSR1 at 22q12 gene regions. The findings strongly support the diagnosis of extraskeletal myxoid chondrosarcoma. Although often associated with prolonged survival, extraskeletal myxoid chondrosarcoma has high rates of distant recurrence.
Differential diagnosis
Board review style question #1

The most common gene rearranged in extraskeletal myxoid chondrosarcoma is

  1. FOS
  2. NR4A3
  3. PHF1
  4. SS18
  5. ZNF444
Board review style answer #1
Board review style question #2

A 50 year old man presented with a left thigh mass. Hematoxylin eosin stains demonstrated hypocellular lobules with abundant pale blue myxoid matrix containing interconnecting cords and clusters of uniform cells with round to ovoid nuclei and eosinophilic or vacuolated cytoplasm. Foci of intralesional hemorrhages were seen. Occasional mitotic figures were identified (2 mitoses/10 high power fields). Immunohistochemical stain for NSE was positive in tumor cells while all of the following were negative: S100, GFAP, AE1 / AE3, desmin, CD34, ERG and h-caldesmon. FISH studies demonstrated rearrangement of the NR4AE gene at 9q22.33.

Which of the following is most likely the correct diagnosis?

  1. Epithelioid sarcoma
  2. Extraskeletal myxoid chondrosarcoma
  3. Metastatic carcinoma
  4. Myoepithelioma
  5. Ossifying fibromyxoid tumor
Board review style answer #2
B. Extraskeletal myxoid chondrosarcoma

Comment Here

Reference: Extraskeletal myxoid chondrosarcoma

Extraskeletal osteosarcoma
Definition / general
  • Adults, extremities
  • May occur after Xray exposure
  • 60% mortality, worse than chondrosarcoma
  • Subtypes: osteoblastic, chondroblastic, fibroblastic, MFH-like, telangiectactic, well-differentiated (parosteal)
Case reports
Clinical images

Contributed by Mark R. Wick, M.D.
Missing Image

Upper arm x-ray



Images hosted on other servers:
Missing Image

Skin of lower abdomen

Gross images

Case #370

Case #1: 75 year old man

Case #2: 55 year old woman



Images hosted on other servers:
Missing Image

Retroperitoneal mass

Missing Image

Upper arm mass

Microscopic (histologic) description
  • Osteoid and bone formation produced by tumor cells, without interposition of cartilage
Microscopic (histologic) images

Case #370 - Case #1

CK negative

Vimentin positive



Case #370 - Case #2:

40x

100x

400x



Contributed by Mark R. Wick, M.D.
Missing Image Missing Image

Upper arm



Images hosted on other servers:
Missing Image Missing Image Missing Image

Cutaneous tumor on scar of previous bone graft

Missing Image

Upper arm



Missing Image Missing Image

Chest wall

Missing Image Missing Image

Retroperitoneal mass

Differential diagnosis

Fibrohistiocytic tumors-general
Table of Contents
Definition / general
Definition / general

Fibroma of tendon sheath
Definition / general
  • Benign fibroblastic / myofibroblastic nodular proliferation usually attached to a tendon / tendon sheath
Essential features
  • Benign nodular, paucicellular spindle cell lesion with slit-like spaces mostly on finger tendon sheath
  • Cellularity may be higher at the periphery
  • Has the propensity to recur in 5 - 10% cases
ICD coding
  • ICD-O: 8810/0 - fibroma, NOS
  • ICD-11: EE6Y & XH0WB3 - other specified fibromatous disorders of skin & soft tissue and fibroma of tendon sheath
Epidemiology
Sites
  • Mostly on finger tendons
  • Intra-articular, rarely
Etiology
  • Not known at this time
Clinical features
Diagnosis
  • Diagnosis requires correlation of site with typical histological features
Radiology description
  • Plain Xrays show a soft tissue shadow without calcification or bone involvement
  • Ultrasound: well circumscribed hypoechoic mass
  • MRI: iso signal intensity to muscle on T1 weighted images, low signal intensity to muscle on T2 weighted images (BMC Musculoskelet Disord 2020;21:732)
Radiology images

Images hosted on other servers:

MRI

Prognostic factors
  • Benign lesion
  • Can recur in 5 - 10% cases
Case reports
Treatment
  • Surgical excision (marginal excision) is warranted in all cases
Clinical images

Images hosted on other servers:

Palmar surface

Multiple nonmovable deep seated nodules on palm

Gross description
Gross images

Images hosted on other servers:

Excised mass

Microscopic (histologic) description
  • Well circumscribed tumor of variable cellularity
  • Cellularity mostly higher at tumor edges
  • Bland spindle cells in a collagenous background
  • Tumor has characteristic thin walled slit-like vessels
  • Degenerative changes like myxoid / cystic change, osseous / chondroid metaplasia can be seen
  • Bizarre pleomorphic cells can also be present
  • Mitotically inactive
  • Necrosis not present
  • Cellular variant of fibroma of tendon sheath also exists; it overlaps morphologically with nodular fasciitis and fibrous histiocytoma (Cancer 1979;44:1945)
  • Reference: Geschickter: Tumors of Bone, 1949
Microscopic (histologic) images

Contributed by Nasir Ud Din, M.B.B.S.
Circumscribed nodular growth

Circumscribed nodular growth

Circumscribed spindle cell nodule

Circumscribed spindle cell nodule

Slit-like vessels

Slit-like vessels

Slit-like vascular spaces

Slit-like vascular spaces


Paucicellular lesion with collagenous stroma

Paucicellular lesion with collagenous stroma

Markedly collagenized stroma

Markedly collagenized stroma

Spindle cell proliferation

Spindle cell proliferation

Spindle cell lesion

Spindle cell lesion



AFIP images

Multinodular proliferation

Transition from collagenous to cellular area

Fibroblasts are bland and separated by collagen

Cellular area
resembles
leiomyosarcoma
or fibrosarcoma

Extensive
collagenization
of nodules

Most cases are
paucicellular
with scattered
spindled fibroblasts

Cytology description
  • H&E stained slides (J Cytol 2015;32:207):
    • Low cellularity
    • Few loose clusters and singly dispersed bland appearing fibrotic spindle cells and stellate cells admixed with hyalinized fibrocollagenous matrix
    • Necrosis and atypical mitoses not seen
Cytology images

Images hosted on other servers:

FNA

Positive stains
Electron microscopy images

Images hosted on other servers:

Organelles within a spindle cell

Densities of myofilament bundles

Cytoplasmic organelles

Spindled myofibrofibroblasts

Molecular / cytogenetics description
Sample pathology report
  • Finger nodule, excision:
    • Fibroma of tendon sheath (see comment)
    • Comment: Histology showed a well circumscribed, variably cellular lesion composed of bland spindle cells having regular nuclei arranged in sheets and fascicles. Thin walled vessels are present. The background is collagenous.
    • It is a benign condition with recurrence in 5 - 10% cases.
Differential diagnosis
  • Deep benign fibrous histiocytoma:
    • Involves extremities and head and neck region
    • Affects wide age range (i.e. 6 - 84 years)
    • Slight male predominance
    • Histologically well circumscribed and cellular lesion, prominent histiocyte-like cells, foam cells, giant cells and hemosiderin
    • IHC: CD34 positive
  • Tenosynovial cell tumor, localized type:
    • Present in hands; located in close proximity to synovium of tendon sheath or interphalangeal joint
    • Age range of 30 - 35 years
    • Female predominance
    • Histologically cellular tumor exhibiting variable composition of mononuclear cells, multinucleated giant cells, foamy macrophages, inflammatory cells and hemosiderin
    • IHC: positive for CD68, CD163 and CD45
  • Inclusion body fibromatosis:
    • Affects toes of children < 5 years of age
    • Ill defined, paucicellular, plump spindle cells exhibiting intracytoplasmic inclusion and bland nucleus with low mitotic activity
    • IHC: expresses SMA
    • Recurrence rate is high
Board review style question #1
What is the most common location for fibroma of tendon sheath?

  1. Face
  2. Fingers
  3. Oral cavity
  4. Pelvis
  5. Toes
Board review style answer #1
B. Fingers. The most common location for fibroma of tendon sheath is finger tendons (i.e. thumb, index finger and middle finger).

Comment Here

Reference: Fibroma of tendon sheath
Board review style question #2

A 32 year old man has had painless swelling in the palm of his hand for 6 months. It was excised and the histology is shown in the above image. What is the most likely diagnosis?

  1. Benign fibrous histiocytoma
  2. Fibroma of tendon sheath
  3. Nodular fasciitis
  4. Palmar fibromatosis
  5. Tenosynovial giant cell tumor, localized type
Board review style answer #2
B. Fibroma of tendon sheath

Comment Here

Reference: Fibroma of tendon sheath

Fibromatosis colli
Definition / general
  • Fibromatosis that appears at birth, often bilateral, affecting lower 1/3 of sternocleidomastoid muscle, causing thickened muscle
  • Also called congenital torticollis (torticollis: twisting of neck causing unnatural position of head, usually caused by spasm of neck muscles, Am Fam Physician 1996;54:1965)
Epidemiology
  • Associated with congenital anomalies (14% have congenital dislocations of hip, also breech deliveries)
  • May be due to birth injury (breech presentation, forceps)
  • Uncommon (0.4% of live births), usually diagnosed by age 6 months
  • Recommended to diagnose by FNA since excision usually is not required
Treatment
  • Early - stretching and physiotherapy, resolves in 70%
  • Some cases require resection of affected muscle
  • Does not recur
Gross description
  • Tan gritty mass of muscle up to 3 cm, no hemorrhage or necrosis
Microscopic (histologic) description
  • Diffuse proliferation of uniform plump fibroblasts and myofibroblasts and scar like collagen in muscle, with entrapped reactive and degenerating skeletal muscle fibers (loss of cross striations, nuclear enlargement and hypercellularity, multinucleation, atrophy)
  • Surgical specimens are usually less cellular than FNA specimens because they are obtained later in time course of disease
Microscopic (histologic) images

AFIP images

Multinodular
proliferation

Skeletal muscle
fibers are trapped
at advancing
edge of lesion

Scattered, bland
fibrocytes are widely
separated by
dense collagen

Cytology description
  • Early - cellular specimen with clusters or parallel arrays of bland appearing spindle cells in fibromyxoid matrix
  • Also atrophic skeletal muscle in clean background, frequent muscle giant cells, bland bare nuclei and collagen (Acta Cytol 2003;47:359)
  • Usually no significant inflammation (Diagn Cytopathol 2000;23:338)
Positive stains
Differential diagnosis
  • Fibromatosis: no muscle fibers which are replaced by fibrous tissue except at periphery, does not typically affect sternocleidomastoid muscle
  • Proliferative myositis: doesn’t affect this site, stroma resembles granulation tissue and is not collagenous
  • Fibrodysplasia ossificans progressiva: doesn’t affect this site, hand malformations are present, bone is present

Fibromatosis-desmoid
Definition / general
  • Locally aggressive fibroblastic / myofibroblastic tumor arising in deep soft tissues with no metastatic potential
Essential features
  • Benign tumor with infiltrative borders and a propensity for local recurrence
  • May be associated with familial adenomatous polyposis (Gardner syndrome)
  • Histology: long, sweeping fascicles with thin walled vessels and microhemorrhages; bland cells with mild to moderate cellularity and minimal atypia
  • Immunohistochemistry: SMA+ and nuclear beta catenin+
  • Molecular: CTNNB1 and APC mutations may be seen
Terminology
  • Synonyms: aggressive fibromatosis, musculoaponeurotic fibromatosis, desmoid tumor, deep fibromatosis
ICD coding
  • ICD-O: 8821/1 - desmoid type fibromatosis
  • ICD-10: D48.1 - neoplasm of uncertain behavior of connective and other soft tissue
  • ICD-11:
    • 2F7C & XH13Z3 - neoplasm of uncertain behavior of connective or other soft tissue & desmoid type (aggressive) fibromatosis
    • 2F7C & XH6116 - neoplasm of uncertain behavior of connective or other soft tissue & abdominal (mesenteric) fibromatosis
Epidemiology
Sites
  • 37 - 50% occur in the abdominal region; shoulder girdle, chest wall and inguinal regions are the most prevalent extraabdominal sites
    • In FAP: majority (51 - 67%) are intraabdominal or in the abdominal wall
    • Sporadic: extraabdominal are more common
  • Tendency of premenopausal and pregnant women to develop fibromatosis in the abdominal wall musculoaponeurosis (Fam Cancer 2006;5:191)
Pathophysiology / etiology
  • CTNNB1 and APC gene mutations can account for up to 89% of cases (Genes Chromosomes Cancer 2010;49:560)
  • May be driven by a combination of genetic mutations, high estrogen states and antecedent trauma which leads to activation of the canonical Wnt / β catenin pathway
Diagrams / tables

Images hosted on other servers:

Canonical Wnt pathway

Site distribution

Clinical features
  • Extraabdominal tumors are deep seated, poorly circumscribed and painless
  • Abdominal wall tumors typically arise in gravid or postpartum women
  • Intraabdominal tumors arise as slowly growing, painless masses that may mimic ovarian tumors
Radiology description
  • Ultrasound:
    • Can be used with palpable masses of the extremities, abdominal, chest wall, breast, etc.
    • Oval, well to poorly marginated solid soft tissue mass with variable echogenicity
    • May have thin, linear extension along fascial planes (fascial tail sign) or intramuscular finger-like extensions (staghorn sign)
  • CT:
    • Most commonly used for intraabdominal tumors and associated complications (like small bowl obstruction)
    • Soft tissue mass with well to poorly defined margins, variable attenuation and mild to moderate enhancement
  • MRI:
    • Preferred modality for abdominal wall and extraabdominal tumors
    • Heterogeneous iso to hyperintense to skeletal muscle on T2, isointense to muscle on T1
    • Nonenhancing linear bands (Band sign) can be seen in 60 - 90% of cases
    • Moderate to marked enhancement after gadolinium based contrast (Radiographics 2016;36:767)
Radiology images

Contributed by Farres Obeidin, M.D.

Abdominal wall mass

Abdominal wall mass


Medial thigh mass

Breast mass

Paraspinal mass

Mesenteric mass

Prognostic factors
  • Local recurrence in 20 - 30%
  • Margin status shown to predict local recurrence in primary tumors but was not significant in recurrent presentations
  • Other factors such as age, gender, presentation status, tumor size, tumor focality and histologic type have shown inconsistent predictive value
  • Complete excision may cause significant morbidity and mortality in certain locations (Cancer 1999;86:2045, J Surg Oncol 2009;100:563, Ann Surg Oncol 2012;19:4028)
  • CTNNB1 S45F mutation associated with significantly increased risk of recurrence (Ann Surg Oncol 2015;22:1464)
Case reports
Treatment
  • Highly variable natural course, with up to 25% undergoing spontaneous regression
  • Increasing tendency to follow with observation only (Ann Surg Oncol 2013;20:4096)
  • Mainstay of treatment is surgical resection, which can result in significant morbidity
  • Radiotherapy alone and surgery plus adjuvant radiotherapy have shown improved local control over surgery alone; however, long term effects of radiation therapy prevent standard use, especially in young patients
  • Systemic therapy for advanced disease (Oncologist 2011;16:682):
    • Antihormonal therapy (tamoxifen) can be used as first line in asymptomatic patients
    • Nonsteroidal anti-inflammatory drugs (NSAIDs) have shown good response in some studies but no randomized trials yet
    • Several conventional chemotherapy have been assessed, doxorubicin and dacarbazine combination or pegylated liposomal doxorubicin preferred
    • Response evaluation criteria in solid tumors (RECIST) trial shows some response with receptor tyrosine kinase inhibitors; sorafenib and sunitinib may be more efficacious than imatinib but studies are ongoing
Clinical images

Contributed by Mark R. Wick, M.D.

Calf



Images hosted on other servers:

Paraspinous subcutaneous mass

Tongue mass

Gross description
  • Typically 5 - 10 cm; may be poorly defined or well circumscribed
  • Firm, glistening white, gritty and coarsely trabeculated cut surface resembling scar tissue
Gross images

Contributed by Farres Obeidin, M.D. and AFIP images

Whirling, fibrous cut surface

Gritty cut surface

Mass in the mesentery

Extension into bowel wall

Infiltrative borders


Arising in the abdominal wall muscles

Can be well circumscribed

Extraabdominal fibromatosis

Intraabdominal fibromatosis: gray glossy cut surface

Microscopic (histologic) description
  • Typically poorly defined borders
  • Conventional pattern (Am J Clin Pathol 2016;145:332):
    • Most common pattern; present at least focally in almost all tumors
    • Long sweeping fascicles with elongated, slender, spindled cells of uniform appearance and pale cytoplasm set in a collagenous stroma
    • No nuclear hyperchromasia, minimal cytologic atypia and variable mitotic rate (typically low but may be focally increased)
    • Thin walled and prominent blood vessels with perivascular edema
    • Vascular microhemorrhages may be seen
  • Hypocellular / hyalinized pattern:
    • Second most common pattern
    • More hypocellular, prominent stromal hyalinization, compressed and thin walled vessels
  • Staghorn vessel pattern:
    • ~20% of cases
    • Prominent, dilated, branching blood vessels
  • Myxoid pattern:
    • ~15% of cases
    • Myxoid stroma with less discrete fascicles; lacks the typical thin walled blood vessels
  • Keloidal pattern:
    • ~15% of cases
    • Variably sized bands of keloidal collagen fibers throughout the tumor; fascicles are disrupted and typical vascular pattern is lacking
  • Nodular fasciitis-like pattern:
    • ~10% of cases
    • Spindle to stellate cells in a loose tissue culture arrangement with variable amounts of stromal hemorrhage
  • Hypercellular pattern:
    • ~5% of cases
    • Resembles conventional pattern but with increased cellular density and frequently overlapping nuclei
    • Without cytologic atypia or nuclear hyperchromasia
Microscopic (histologic) images

Contributed by Farres Obeidin, M.D., Raul S. Gonzalez, M.D. and AFIP images

Spindling mimics dense breast fibrous tissue

Infiltrative margins

Fascicular to herringbone architecture

Microhemorrhages

Bowel masses may mimic GIST

Fascicular architecture


Infiltration into muscle

Storiform or whirling architecture

Wavy, bland, spindled nuclei

Extension into GI submucosa

Bland with usually minimal mitoses

Occasional staghorn vessels


Keloid-like areas

Spindle cell proliferation with thin vessels

Mast cells

Rarely may be hypercellular

Inconspicuous nucleoli with no cytologic atypia

Extraabdominal: myxoid tumor


Extraabdominal:
cellular, scattered
mitoses, none
atypical

Colonic fibromatosis

Colonic fibromatosis

Intraabdominal
(mesenteric):
extrinsic to bowel
muscularis propria


Intraabdominal
(mesenteric):
collagen seen
on trichrome

SMA+

Nuclear beta catenin

Ki67 low

Cytology description
  • Bland spindle cells with long, fusiform nuclei and metachromatic matrix material
  • Tumor cells are individual or as fragments within matrix (Cancer 2007;111:166)
  • Fine needle aspiration is fairly reliable for diagnosis but core needle biopsy is better (Acta Orthop 2006;77:926)
  • May have long fascicular arrangement of spindle cells (Diagn Cytopathol 2012;40:45)
Positive stains
Electron microscopy description
  • Fibroblastic and myofibroblastic features, including intrareticular collagen fibers, thin filament bundles, cytoplasmic dense bodies
Molecular / cytogenetics description
Molecular / cytogenetics images

Images hosted on other servers:

Trisomy 20 and 8

Sample pathology report
  • Abdominal wall, mass, resection:
    • Desmoid type fibromatosis (see comment)
    • Margins of resection are negative, with tumor closest at 1 cm from the inked margin.
    • Comment: Microscopic sections show a proliferation of bland spindle cells with thin blood vessels and focal areas of hemorrhage infiltrating into surrounding skeletal muscle and adipose tissue. A beta catenin immunostain shows nuclear positivity in the majority of cells, supporting the diagnosis of desmoid type fibromatosis.
Differential diagnosis
Board review style question #1

A mass was excised from the abdominal wall of a 28 year old woman. The histology is shown above. Stains were performed and the lesion shows nuclear positivity for beta catenin. Which of the following is true about this lesion?

  1. It harbors an ALK1 gene rearrangement
  2. It has high potential for recurrence if incompletely excised
  3. It has metastatic potential
  4. The lesional cells arose from the interstitial cells of Cajal
Board review style answer #1
B. It has high potential for recurrence if incompletely excised. This is a desmoid type fibromatosis. They don't have metastatic potential but have a high change for recurrence, particularly in the abdomen. GIST cells arise from the interstitial cells of Cajal in the muscularis propria of the bowel and inflammatory myofibroblastic tumors may have ALK1 gene rearrangements.

Comment Here

Reference: Fibromatosis-desmoid
Board review style question #2
Which of the following genetic syndromes is associated with an increased chance of developing desmoid type fibromatosis?

  1. Gardner syndrome
  2. Li-Fraumeni syndrome
  3. Maffucci syndrome
  4. Neurofibromatosis
Board review style answer #2
A. Gardner syndrome is an expansion of familial adenomatous polyposis (FAP) that includes additional features such as desmoid type fibromatosis, Gardner fibromas and rhabdomyosarcomas.

Comment Here

Reference: Fibromatosis-desmoid

Fibromatosis-palmar / plantar
Definition / general
  • Palmar and plantar fibromatosis are benign nodular fibroblastic / myofibroblastic proliferations typically arising in the volar aspect of the hands and fingers or involving plantar aponeuroses, respectively
  • Both processes are histologically similar, composed of a bland cellular proliferation of spindle cells with a bluish appearance and with a variable amount of background collagen, depending on the age of the lesion (J Pathol Transl Med 2021;55:265)
Essential features
  • Bland, variably cellular proliferation of spindled fibroblasts / myofibroblasts
  • Collagenous stroma
  • Involvement of aponeurosis and variably subcutis and dermis
  • A subset shows nuclear beta catenin expression, despite the absence of CTNNB1 or APC gene mutations (Histopathology 2007;51:509, Mod Pathol 2001;14:695)
Terminology
  • Palmar fibromatosis: Dupuytren disease or contracture
  • Plantar fibromatosis: Ledderhose disease, morbus Ledderhose
ICD coding
  • ICD-10
    • M72.0 - palmar fascial fibromatosis (Dupuytren)
    • M72.2 - plantar fascial fibromatosis
  • ICD-11
    • FB51.0 - palmar fascial fibromatosis
    • FB51.1 - knuckle pads
    • FB51.Y - other specified fibroblastic disorders (for plantar)
    • XH75J5 - palmar / plantar type fibromatosis
Epidemiology
  • Palmar fibromatosis
    • Most common type of superficial fibromatosis (1 - 2% of population)
      • Prevalence increases with age (~20% of population at age 65)
    • M:F = 3 - 4:1
    • 50% bilateral
    • Most common in northern Europeans
      • Rare in Black population
  • Plantar fibromatosis
Sites
  • Palmar fibromatosis
  • Plantar fibromatosis
    • Most often the medial and central bands of plantar aponeurosis
Pathophysiology
Etiology
  • Palmar fibromatosis
    • Unknown (WHO 5th edition)
    • May be caused by fibrogenic cytokines (J Hand Surg Br 2005;30:557)
    • Associations with diabetes, smoking and repetitive vibrational trauma reported but not validated (WHO 5th edition)
  • Plantar fibromatosis
Clinical features
  • Palmar fibromatosis
    • Often presents with subcutaneous nodules on distal palmar crease
    • Puckers overlying skin as it ages
    • Causes flexion contracture, most commonly of digits 4 and 5, due to cord-like expansion of digital aponeurotic slips
    • Does not involve deep structures (i.e., tendons or skeletal muscle)
    • 10% also have plantar disease; 1 - 4% have penile fibromatosis
  • Plantar fibromatosis
    • 0.5 - 3.0 cm in diameter subcutaneous nodule(s), slow growing and located in the medial or central plantar aponeurosis
    • Initially painless but later associated with pain after standing or walking, typically on the medial aspect of the sole (arch)
    • Associated with concomitant palmar and penile fibromatoses and keloids
    • Usually not associated with contractures (Foot Ankle Int 2018;39:751)
Diagnosis
  • Usually clinical; however, occasionally may be a histologic confirmation
Radiology description
Radiology images

Contributed by Mark R. Wick, M.D.

Plantar fibromatosis
CT of heel

CT of heel



Images hosted on other servers:

Palmar fibromatosis
Ultrasound

Ultrasound

MRI

MRI

Palmar and plantar fibromatosis
Axial T1 weighted MRI

Axial T1 weighted MRI

Prognostic factors
  • Palmar fibromatosis
    • Worse in White men with a strong family history, bilateral involvement, severe disease and ectopic manifestations (BMJ 2006;332:397)
  • Plantar fibromatosis
Case reports
Treatment
Clinical images

Contributed by Brian D. Stewart, M.D.

Palmar fibromatosis

Nodules and contractures

Excision of contraction band

Postoperative improvement

Gross description
  • Palmar fibromatosis
    • Small nodules or nodular masses associated with aponeurosis and subcutaneous fat with gray-yellow-white cut surface (color depends on collagen content)
  • Plantar fibromatosis
    • 2 - 3 cm nodules associated with aponeurosis and subcutis with gray-yellow-white cut surface (color depends on collagen content)
  • Reference: Weiss: Enzinger and Weiss's Soft Tissue Tumors, 7th Edition, 2019
Gross images

Contributed by Brian D. Stewart, M.D.

Palmar fibromatosis

Excised contraction band

Frozen section description
Frozen section images

Contributed by Brian D. Stewart, M.D.

Plantar fibromatosis

Spindle cell proliferation

Microscopic (histologic) description
  • Both processes are histologically similar
  • Typically, tumors involve a thickened palmar / plantar aponeurosis and form single or multiple discontinuous, moderately cellular spindle cell nodules in a collagenous stroma (Am J Surg Pathol 2005;29:1095)
  • Morphologically, there are 3 phases of growth (proliferative, involutional and late stage)
    • Proliferative phase
      • Cellular, parallel fascicles of bland, plump, relatively uniform spindled fibroblasts with tapering nuclei, vesicular chromatic and small or inconspicuous nucleoli, with minimal stromal collagen
      • Some plantar lesions are hypercellular and can mimic spindle cell sarcomas but they lack atypia
      • Scattered chronic inflammation and occasional typical mitotic figures, especially in early lesions
      • Mitotic activity may be prominent in pediatric patients
      • Occasional, particular plantar lesions show interspersed multinucleated cells (Am J Surg Pathol 2002;26:244)
    • Late stage
      • Increase collagenous matrix and decreased cellularity
  • Reference: WHO 5th edition
Microscopic (histologic) images

Contributed by Brian D. Stewart, M.D. and AFIP

Palmar fibromatosis

Fibroblastic proliferation

Fibroblastic nodules

Fibroblastic nodule

Tendinous infiltration

Infiltrates an aponeurosis

Mitotic figures



Plantar fibromatosis

Nodular proliferation

Plantar aponeurotic infiltration

Demarcation from plantar fascia

Virtual slides

Images hosted on other servers:
Missing Image

Male with 6 month history of palm nodule

Cytology description
  • Usually limited to touch preps during rare frozen sections (Acta Cytol 1993;37:323)
  • Low cellularity, clusters of bland spindle cells with oval to elongated nuclei, no atypical features, rare or absent mitotic activity
Cytology images

Contributed by Brian D. Stewart, M.D.

Plantar fibromatosis

Touch prep

Positive stains
Negative stains
Electron microscopy description
  • Fibroblasts and myofibroblasts
Electron microscopy images

Images hosted on other servers:
Missing Image

Fibroblast

Molecular / cytogenetics description
Videos

Plantar fibromatosis

Sample pathology report
  • Soft tissue, hand nodule, excision:
    • Palmar fibromatosis (Dupuytren contracture) (see comment)
    • Comment: Sections show a variably cellular proliferation of spindled fibroblasts and myofibroblasts without atypia set within a collagenous stroma, morphologically consistent with palmar fibromatosis (Dupuytren contracture).

  • Soft tissue, foot nodule, excision:
    • Plantar fibromatosis (see comment)
    • Comment: Sections show a variably cellular proliferation of spindled fibroblasts and myofibroblasts without atypia set within a collagenous stroma, morphologically consistent with plantar fibromatosis.
Differential diagnosis
Board review style question #1

Which of the following is true about the disease pictured in this photomicrograph of a painful nodule on the sole of the foot?

  1. Associated with similar processes in other body sites
  2. Beta catenin will show diffuse nuclear reactivity, confirming the diagnosis
  3. Most common in adolescents
  4. Most common in the central and lateral portions of the foot
  5. t(x;18) is pathognomonic
Board review style answer #1
A. Associated with similar processes in other body sites. Plantar fibromatosis is associated with similar processes in other body sites. Answer B is incorrect because beta catenin usually shows nuclear reactivity in palmar fibromatosis but not in plantar fibromatosis. Answers C and D are incorrect because plantar fibromatosis is most common in the medial and central bands of the plantar aponeurosis and in middle aged patients. Answer E is incorrect because synovial sarcoma has a pathognomonic t(x;18).

Comment Here

Reference: Palmar / plantar fibromatosis
Board review style question #2
Which statement about plantar fibromatosis is correct?

  1. Associated with diabetes
  2. Bilateral in 5% of cases
  3. Often cured without recurrence after excision
  4. Synonymous with plantar fasciitis
  5. Usually rapidly growing
Board review style answer #2
A. Associated with diabetes. Plantar fibromatosis has been shown to have an association with diabetes. Answer B is incorrect because plantar fibromatosis is bilateral in about 25% of cases. Answer C is incorrect because recurrence is common after excision. Answer D is incorrect as plantar fasciitis is inflammation of the ligament itself and does not form nodules; it is part of the differential diagnosis of plantar fibromatosis. Answer E is incorrect because plantar fibromatosis is usually slow growing.

Comment Here

Reference: Palmar / plantar fibromatosis
Board review style question #3
Which of the following stains may be positive in palmar or plantar fibromatosis although not specific?

  1. CD34
  2. CD56
  3. DOG1
  4. Keratin
  5. Smooth muscle actin
Board review style answer #3
E. Smooth muscle actin is the correct answer as it is reactive in many spindle cell proliferations and is not specific. Answers A - D are incorrect as all of these markers are negative in palmar and plantar fibromatosis.

Comment Here

Reference: Palmar / plantar fibromatosis

Fibrosarcoma-adult
Definition / general
  • Malignant tumor of fibroblasts with herringbone architecture and variable collagen
  • Rare (up to 3% of adult sarcomas)
  • Some limit diagnosis to those age 10+ years, most patients are ages 40 - 55 years
  • Many cases formerly called fibrosarcoma are actually dedifferentiated liposarcoma, fibromatosis, fibrosarcomatous DFSP, low-grade fibromyxoid sarcoma, MPNST, synovial sarcoma or MFH-pleomorphic
  • Usually deep soft tissue of lower extremities or trunk, only rarely in retroperitoneum or mediastinum
  • 50% recur, 25% metastasize (lung, bone)
  • More metastases if more cellular and higher mitotic activity
  • Survival:
    • 5 year - 41%, 10 year - 29%
    • Better if tumor is superficial and better differentiated, low mitotic rate, no necrosis
  • See also peripheral ameloblastic fibrosarcoma, sclerosing epithelioid fibrosarcoma
Treatment
  • Radical excision, radiation if residual tumor or positive margins
  • Possibly chemotherapy if high grade
Gross description
  • May appear well circumscribed but nonencapsulated
  • Fleshy, hemorrhagic, necrotic, white-tan
Gross images

Contributed by Mark R. Wick, M.D. and AFIP

Various images

High grade fibrosarcoma (right)

Microscopic (histologic) description
  • Highly cellular fibroblastic proliferation in herringbone pattern (cells in columns of short parallel lines with all the lines in one column sloping one way and lines in adjacent columns sloping the other way)
  • Cells have scant cytoplasm, tapering elongated dark nuclei with increased granular chromatin, variable nucleoli
  • Mitotic activity present, often with abnormal forms
  • Variable collagen
  • Usually no giant cells
  • No pleomorphism (or call pleomorphic MFH), no other distinct cell types
  • Patterns:
    • Keloid-like (thick hyalinized collagen fibers), loose fascicular, focally myxoid
Microscopic (histologic) images

Contributed by Mark R. Wick, M.D. and AFIP

Adult type

Adult type


Atypical uniform cells in herringbone pattern

Minimal pleomorphism

Grade I tumor


Grade II tumor

Grade III tumor

Positive stains
  • Reticulin stain demonstrates fibers surrounding each cell
  • Phosphotungstic acid-hematoxylin demonstrates abundant cytoplasmic fibrils
  • Also vimentin, type 1 collagen, p53
  • High Ki67
  • May be CD34+ if arises from DFSP or solitary fibrous tumor
Negative stains
  • S100, keratin
  • Smooth muscle markers, histiocytic markers, basal lamina
Electron microscopy description
  • Fibroblasts with prominent rough endoplasmic reticulum but no myofilaments, no external lamina, no intercellular junction
  • No distinct myofibroblasts (if present, call myofibrosarcoma)
Molecular / cytogenetics description
  • Aneuploid
Differential diagnosis
Additional references

Fibrosarcoma-infantile
Definition / general
  • Resembles adult fibrosarcoma morphologically but better prognosis
  • Age cutoff between infantile and adult forms usually varies between 5 and 10 years
Epidemiology
  • Usually presents before age 2 years in axial regions or extremities with vary rapid growth
  • Related to congenital mesoblastic nephroma, which has same translocation
Clinical features
  • 40 - 50% recur but only rarely metastasizes
  • Survival is 90%+
Case reports
Gross description
  • May exceed 30 cm (grotesquely large compared to size of child) with tense erythematous and ulcerated overlying skin
  • Firm to soft cut surface is fleshy, gray-tan with myxoid change, cystic degeneration, hemorrhage and necrosis
Gross images

AFIP images

Fleshy white
mass similar
to adult
fibrosarcoma

Microscopic (histologic) description
  • Poorly circumscribed, lobulated mass of small to large spindled cells in fascicles or herringbone pattern with high cellularity, nuclear atypia and pleomorphism
  • Increased mitotic figures, hemorrhage and necrosis
  • Resembles adult fibrosarcoma
  • May have prominent hemangiopericytoma-like areas, dystrophic calcification, extramedullary hematopoiesis
  • Infiltrates adjacent soft tissue with irregular margins
Microscopic (histologic) images

AFIP images

Biphasic pattern with fibroblastic and cellular myxoid areas

High power of myxoid area

Infiltration of fat

Infiltration of muscle



Spindle cells

Plump cells have granular chromatin

Less cellular tumor which overlaps with infantile fibromatosis, although it almost never metastasizes



Images hosted on other servers:

Leg tumor

Positive stains
Electron microscopy description
  • Fibroblastic and myofibroblastic features
Molecular / cytogenetics description
Differential diagnosis
Additional references

Fibrous hamartoma of infancy
Definition / general
  • A benign soft tissue neoplasm of infants and young children, showing organoid, triphasic morphology with bundles of bland fibroblastic / myofibroblastic cells, nodules of primitive, rounded or stellate cells with myxoid stroma and mature adipose tissue (Mod Pathol 2017;30:474)
Essential features
  • Organoid, triphasic morphology, variable prominence of bundles of bland fibroblastic / myofibroblastic cells, nodules of primitive, rounded or stellate cells with myxoid stroma and mature adipose tissue (Mod Pathol 2017;30:474)
Terminology
  • Subdermal fibrous tumor of infancy (not recommended)
ICD coding
  • ICD-11: LC2Y - other specified hamartomata derived from dermal connective tissue
Epidemiology
  • Very rare; mean age at diagnosis: 15 months (range: birth to 14 years); 20% are congenital
  • M:F = 2.7:1 (Mod Pathol 2017;30:474)
Sites
Pathophysiology
Etiology
  • Unknown
Clinical features
Diagnosis
  • Primarily based on histological findings of characteristic triphasic morphology with bland fibroblastic / myofibroblastic cell bundles, primitive mesenchyme nodules and mature adipose tissue
Radiology description
Radiology images

Images hosted on other servers:

Fatty strands and whirling

Prognostic factors
Case reports
Treatment
  • Complete local excision
Clinical images

Images hosted on other servers:

Massive swelling of thigh

Upper right hemiscrotal mass

Ill defined lesion with hypertrichosis

Axillary tumor in 6 month old boy

Scapular tumor in 7 year old girl

Scapular tumor
at surgery
showing well
developed capsule

Gross description
Gross images

Images hosted on other servers:

Lobulated nodular mass

Frozen section description
  • Fibrous tissue with bland spindle cells (Ocul Oncol Pathol 2017;3:8)
  • Organoid triphasic appearance with fat, fibrocollagenous tissue and immature, myxoid to basophilic, mesenchymal components; mitoses and calcification are rare; necrosis is absent
Microscopic (histologic) description
  • Variable amounts of 3 components arranged in an organoid pattern
    • Mature adipose tissue
    • Intersection fascicles of fibrous tissue with prominent collagen deposition; fibroblastic cells show dark wavy nuclei and scant cytoplasm
    • Myxoid to basophilic areas with haphazardly arranged bland looking primitive spindle to stellate cells
  • Angiectoid areas in 33 - 50% of cases; these areas show densely sclerotic pseudoangiomatous foci with slit-like spaces lined by plump tumor cells, resembling giant cell fibroblastoma (Mod Pathol 2017;30:474)
  • Mitoses are rare, necrosis is absent
  • Inflammatory cell infiltrate with aggregate formation may occupy most of the lesion
Microscopic (histologic) images

Contributed by Nasir Ud Din, M.B.B.S.

Triphasic morphology

Ill defined margins

Predominantly fibrous lesion

Collagenized fascicles


Bland cytology

Myxoid nodule

Smooth muscle actin

Virtual slides

Images hosted on other servers:

Subcutaneous lesion with nodules

Fat dominant lesion

Cytology description
  • Moderately cellular smears with mature adipose tissue admixed with short spindle to plump cells and numerous naked nuclei (Diagn Cytopathol 2003;28:272)
  • Myxoid to collagenized background
  • No cytological atypia or mitosis
Positive stains
Electron microscopy description
  • Cells with features of both fibroblasts (and possibly myofibroblasts) and occasional fusiform banded fibers (Luse bodies) (Hum Pathol 1982;13:586)
  • Cellular myxoid areas showed prominent vasoformative proliferation (Hum Pathol 1984;15:717)
Molecular / cytogenetics description
Videos

Fibrous hamartoma of infancy

Sample pathology report
  • Axilla, excisional biopsy:
    • Fibrous hamartoma of infancy, 3.5 cm (see comment)
    • Comment: Sections show an ill defined, vaguely organoid lesion in the subcutaneous tissue. It is composed of abundant mature adipose tissue, fibrous tissue and myxoid spindle cells nodules. Fibrous areas show dense hyalinization with slit-like spaces, lined by bland spindle cells. Myxoid nodules show haphazardly arranged primitive spindle cells. No hyperchromatic, multinucleated, floret-like giant cells are noted. There is no significant mitotic activity. Necrosis and calcification are also absent. CD34 is positive in fibrous areas and myxoid nodules. The lesion appears to be completely excised.
Differential diagnosis
  • Lipofibromatosis:
    • Peak incidence in first decade
    • Common in hand and extremities
    • Long fascicles of uniform bland spindle cells in a collagenized stroma
    • Lacks the immature mesenchymal component
  • Giant cell fibroblastoma:
    • Mean age at diagnosis is 6 years
    • Most cases occur in trunk, groin and axilla
    • Can mimic, with cases showing prominent angeictoid areas
    • Scattered hyperchromatic, multinucleated, floret-like giant cells are discriminatory
    • Positive for t(17;22), which results in COL1A1::PDGFB fusion
  • Calcifying aponeurotic fibromas:
    • Occurs in the first or second decade of life
    • Commonly involves hand, wrist and foot
    • Infiltrative fascicles of fibroblastic cells
    • Calcified, vaguely chondroid looking nodules surrounded by epithelioid cells
    • Osteoclast-like giant cells adjacent to chondroid nodules
    • FN1::EGF fusion gene has been recently documented
  • Lipoblastoma:
    • Most cases occur during first 3 years of life
    • Commonly involves lower and upper extremities
    • May pose a challenge in cases with prominent nodular myxoid zones of immature mesenchymal tissue
    • Has a more distinctive lobular architecture
    • Nuclear staining for PLAG1
  • Infantile myofibroma:
    • Congenital in 67% of cases
    • 3 subtypes: solitary, multiple and generalized
    • Fascicles of bland spindle cells infiltrating adipose tissue
    • Hemangiopericytoma-like vessels surrounded by primitive round or polygonal cells
    • Lacks abundant fat and often shows a distinctive perivascular growth pattern at the periphery
    • Sometimes shows necrosis or calcification in the center
  • Fibromatosis:
    • Usually deep seated
    • Lacks primitive mesenchymal component
    • Beta catenin positive
  • Cellular schwannoma:
    • Can mimic cases with predominance of immature mesenchymal nodules
    • S100 positive in spindle cells
  • Infantile fibrosarcoma:
    • Can mimic cases with predominance of immature mesenchymal nodules
    • Unusually large, hypercellular and frequent mitoses
    • ETV6::NTRK3 gene fusion
Board review style question #1

An 11 month old boy was incidentally found to have a soft lump in the axilla. An excisional biopsy was performed and a representative section is shown in the image above. CD34 positivity was seen in fibrous and myxoid areas. Which of the following is the most appropriate diagnosis?

  1. Fibromatosis
  2. Fibrous hamartoma of infancy
  3. Giant cell fibroblastoma
  4. Spindle cell hemangioma
  5. Spindle cell lipoma
Board review style answer #1
B. Fibrous hamartoma of infancy. 3 essential elements of fibrous hamartoma of infancy are visible. Presence of immature myxoid spindle cell aggregates is unique to this entity and is not seen in fibromatosis, giant cell fibroblastoma or spindle cell hemangioma. CD34 positive angiectoid areas are not seen in spindle cell lipoma, which otherwise may closely resemble fibrous hamartoma of infancy.

Comment Here

Reference: Fibrous hamartoma of infancy
Board review style question #2
Which of the following genetic alterations has been recently associated with fibrous hamartoma of infancy?

  1. COL1A1::PDGFB gene fusion
  2. CTNNB1 gene mutation
  3. EGFR gene mutation
  4. ETV6::NTRK3 gene fusion
  5. PDGFRB gene mutation
Board review style answer #2
C. EGFR gene mutation. COL1A1::PDGFB gene fusion is associated with giant cell fibroblastoma, while CTNNB1 gene mutation is present in fibromatosis. EGFR gene mutation (EGFR exon 20 insertion / duplication) has been recently described in fibrous hamartoma of infancy. ETV6::NTRK3 gene fusion is seen in infantile fibrosarcoma and PDGFRB gene mutation is associated with myofibroma.

Comment Here

Reference: Fibrous hamartoma of infancy

Focal myositis
Definition / general
  • Inflammatory condition of skeletal muscle with degeneration and regeneration, inflammatory cells and fibrosis
  • Not a WHO diagnosis
Epidemiology
  • Mean age 41 years, range 7 - 94 years
Clinical features
  • Usually healthy patients with no history of trauma
  • Evolves over weeks to a localized, painful soft tissue swelling, usually of lower extremity
  • Solitary, self limited, may be related to denervation
  • May be due to statins (Int J Cardiol 2009;133:e33, Cleve Clin J Med 2011;78:393)
Case reports
Treatment
  • Usually none; spontaneously regresses
Gross description
  • Pale, ill defined
  • Mean 4 cm, range 1 - 20 cm
Microscopic (histologic) description
  • Degeneration and regeneration of muscle fibers associated with interstitial inflammation and fibrosis
  • Also focal neurogenic changes
  • Occasionally prominent eosinophils
  • Markedly inflamed cases have B cells or CD123+ dendritic plasma cells
Microscopic (histologic) images

Images hosted on other servers:

Focal perivascular
chronic inflammatory
infiltrate

Post BCG vaccination

Various images

Cytology description
  • Inflammatory cells, skeletal muscle fibers with degenerative and regenerative changes, fibrous tissue (Acta Cytol 2005;49:653)
Positive stains
Negative stains
Molecular / cytogenetics description
  • No B or T cell rearrangement
Differential diagnosis
  • Poliomyelitis

Gardner-type fibroma
Definition / general
  • Benign soft tissue lesion with thick, haphazard collagen and bland fibroblasts that entrap adjacent tissue
  • 90% associated with FAP / Gardner’s syndrome / APC germline mutation
  • Uncommon
  • Affects infants, children and teenagers
  • May be initial diagnostic clue to Gardner’s syndrome and APC mutations (Am J Surg Pathol 2001;25:645)
  • 45% develop desmoid-type fibromatosis
  • Similar histology to nuchal-type fibroma (Pathol Int 2004;54:523, Am J Surg Pathol 2000;24:1563)
Sites
  • Superficial or deep soft tissue
  • Various sites
Case reports
Gross description
  • 1 - 10 cm, poorly circumscribed, firm, rubbery, plaque-like, white to tan-pink cut surface with trapped fat in yellow areas
Microscopic (histologic) description
  • Thick, haphazardly arranged collagen bundles, hypocellular bland fibroblasts, small blood vessels, plaque-like growth pattern with infiltration of adjacent structures
Positive stains

Gastrointestinal stromal tumor
Definition / general
Gross images

Contributed by @Andrew_Fltv on Twitter
Gastrointestinal stromal tumor Gastrointestinal stromal tumor

Gastrointestinal stromal tumor

Microscopic (histologic) images

Contributed by @Andrew_Fltv on Twitter
Gastrointestinal stromal tumor Gastrointestinal stromal tumor

Gastrointestinal stromal tumor

Important immunohistochemical stains

Giant cell fibroblastoma
Definition / general
Epidemiology
  • Usually children less than 10 years old, 2/3 male
  • Painless nodule of subcutis, usually in trunk, extremities, head and neck
  • 50% recur but recurrences are controllable, no metastases
Case reports
Clinical images

Contributed by Mark R. Wick, M.D.

Mammogram

Gross description
  • Poorly circumscribed, gray to yellow mucoid mass that is difficult to completely excise, usually in subcutis
Microscopic (histologic) description
  • Dermis and subcutis contains hyperchromatic spindle or stellate shaped cells in a collagenous or myxoid matrix with scattered hyperchromatic, multinucleated, floret-like giant cells with prominent nucleoli, similar to those in pleomorphic lipomas
  • Ectatic pseudovascular spaces are lined by a discontinuous row of floret-like cells and tumor cells
  • Honeycomb or parallel pattern of infiltration
  • Also hyalinized area, perivascular lymphocytes in onionskin pattern, intralesional hemorrhage
  • Often foci of DFSP
  • No histiocyte-like cells, no mitotic figures
Microscopic (histologic) images

Contributed by Mark R. Wick, M.D. and AFIP

Ectatic pseudovascular spaces are lined by giant cells


Stroma has giant cells and myxoid stroma

Fibrotic stroma

Multinucleated stromal giant cells

Breast skin

Cytology description
  • Moderately cellular smears with mononuclear cells, usually single but occasionally in clusters
  • Most cells have no/scanty cytoplasm, bland nuclei with small nucleoli
  • Nuclear membranes often have notches, creases or folds
  • Rare multinucleated giant cells with bland oval nuclei
  • No necrosis, no mitotic figures (Diagn Cytopathol 2002;26:398, Arch Pathol Lab Med 2001;125:1091)
Positive stains
Negative stains
Electron microscopy description
  • Myofibroblasts or fibroblasts
Molecular / cytogenetics description
  • t(17,22)(q22;q13) - creates fusion of collagen type 1 alpha 1 gene and platelet derived growth factor B chain gene
  • Also supernumerary ring chromosomes derived from t(17;22)
Differential diagnosis

Giant cell tumor of soft tissue
Definition / general
  • Tumor of low malignant potential that is considered the soft tissue analog of giant cell tumor of bone (Mod Pathol 1999;12:894)
Essential features
  • Giant cell tumor of soft tissue (GCT-ST) is morphologically identical to giant cell tumor of bone
  • Tumor of low grade malignant potential, which is cured by complete surgical resection
  • Lacks mutations in the H3F3A gene, which makes it distinct from giant cell tumor of bone
Terminology
  • Soft tissue giant cell tumor of low malignant potential
  • Giant cell tumor of soft parts
  • Extraskeletal giant cell tumor, osteoclastoma of soft tissue (not recommended or commonly used)
ICD coding
  • ICD-0: 9251/1 - giant cell tumor of soft parts, NOS
  • ICD-10: D48.1 - neoplasm of uncertain behavior of connective and other soft tissue
Epidemiology
  • Rare benign neoplasm, exact incidence unknown
  • M = F
  • Age average ~40 years (range: 1 - 86 years)
  • Can be associated with Paget disease of bone (Diagn Cytopathol 1998;19:352)
Sites
  • Arms, hand and thighs are most common sites (Mod Pathol 1999;12:894)
  • May develop in superficial or deep soft tissue
  • May present multifocal
Clinical features
  • Painless mass
Diagnosis
  • Indistinguishable from giant cell tumor of bone in the absence of radiology or clinical information
Radiology description
  • Soft tissue mass
  • Mineralization in some cases
  • May present as cystic lesion with internal nodules; nonspecific soft tissue changes adjacent
  • Reference: BJR Case Rep 2020;6:20200012
Radiology images

Images hosted on other servers:

T1 and T2 weighted images

Prognostic factors
  • Conservative surgical resection with negative margins is curative in the majority of cases
  • Local recurrence rate ~6 - 10%
  • Lymph node metastasis and distant metastasis very uncommon
  • Single cases of malignant GCT-ST with metastases have been reported (Am J Surg Pathol 2000;24:248, Am J Surg Pathol 2000;24:386)
Case reports
Treatment
  • Surgical resection is curative
Clinical images

Images hosted on other servers:

54 year old man

Firm, polypoid nodule

Gross description
  • Up to 10 cm, usually dermis or subcutis, 30% are below superficial fascia
  • Well circumscribed, nodular, fleshy, red-brown-gray, gritty at periphery due to calcification
Gross images

Images hosted on other servers:

Gross specimen of GCT-ST

Microscopic (histologic) description
  • Tumors present as multinodular lesions diffusely infiltrating soft tissue
  • Tumor nodules consist of a mix of bland, round to oval mononuclear cells, spindle cells and multinucleated giant cells of osteoclast type
  • Spindle cells may be arranged in a storiform or fascicular pattern
  • Tumors commonly have a peripheral rim of metaplastic bone that may extend to the center
  • Hemorrhage and cystic areas, consistent with secondary aneurysmal bone cyst-like changes, are commonly appreciated
  • Significant atypia will exclude this diagnosis, however, mitotic activity is commonly seen and may be brisk
  • Vascular invasion is seen in up to 50% of cases (Mod Pathol 1999;12:894)
  • Necrosis is uncommon
Microscopic (histologic) images

Contributed by Carina Dehner, M.D., Ph.D.

Well circumscribed lesion

Subcutaneous lesion

Minimal atypia

Spindling of cells


Giant, mononuclear and spindle cells

Mononuclear cells

Epithelioid cells

Lobulated lesion

Cytology description
  • Mononuclear stromal cells have oval to spindle shaped nuclei with ill defined cytoplasm lacking significant atypia
  • Osteoclast type giant cells are evenly distributed throughout the lesion with bland, vesicular nuclei
  • Reference: J Cytol 2009;26:33
Cytology images

Images hosted on other servers:

Multinucleated giant cells

Positive stains
Negative stains
Molecular / cytogenetics description
Sample pathology report
  • Soft tissue, left arm, excision:
    • Giant cell tumor of soft tissue, 3.5 cm, involving dermis and subcutis (see comment)
    • Comment: Provided hematoxylin and eosin stained sections show a well circumscribed, multinodular tumor composed of sheets of evenly distributed, multinucleated giant cells of osteoclast type. The latter are intervened by abundant mononuclear cells, some of which show spindling. Immunohistochemical studies (single stain antibody procedure with adequate controls) show that tumor cells are positive for SMA (patchy) and CD68, while negative for S100 protein, CD45 and desmin. An immunohistochemical study for H3K34W is negative, ruling out a metastasis of giant cell tumor primary to the bone. Morphologic and immunohistochemical studies support the diagnosis of giant cell tumor of soft tissue.
Differential diagnosis
Board review style question #1

The tumor shown in the image above presented as a subcutaneous mass in the arm. What is the diagnosis?

  1. Giant cell tumor of bone
  2. Giant cell tumor of soft tissue
  3. Tenosynovial giant cell tumor, localized type
  4. Undifferentiated pleomorphic sarcoma
Board review style answer #1
B. Giant cell tumor of soft tissue presents as subcutaneous lesion rather than a bone lesion (therefore, A is wrong). It has a peripheral shell of metaplastic bone (which distinguishes it from C) and is not as atypical as D.

Comment Here

Reference: Giant cell tumor of soft tissue
Board review style question #2
What genetic aberration may be helpful to distinguish primary giant cell tumor of soft tissue from a metastasis of a giant cell tumor of bone?

  1. Lack of a mutation in H3F3A
  2. Presence of a mutation in H3F3A
  3. Presence of a mutation in CSF1
  4. Presence of a mutation in H3K36M
Board review style answer #2
A. H3F3A mutations can be found in giant cell tumor of bone but not in giant cell tumor of soft tissue. CSF1 is the molecular feature of tenosynovial giant cell tumor and H3K36M is the mutation found in chondroblastoma.

Comment Here

Reference: Giant cell tumor of soft tissue

Glomus tumor
Definition / general
  • Mesenchymal tumor composed of modified smooth muscle cells that arise from the glomus body
Essential features
  • Predominantly cutaneous; most common in subungual region of the finger but may occur anywhere
  • Usually benign with very rare malignant variants
  • Combination of glomus cells, smooth muscle cells and vasculature
  • Round, punched out nuclei with amphophilic to eosinophilic stroma surrounding branching, capillary sized vessels
  • Positive for SMA, MSA, calponin, h-caldesmon, collagen type IV; negative for cytokeratin and S100
Terminology
  • Name differs based on morphology: glomus tumor, glomangioma, glomangiomyoma and glomangiomatosis
  • Malignant glomus tumors are also called glomangiosarcoma
  • Note: certain site specific misnomers like glomus faciale, glomus jugulare, glomus tympanicum, glomus vagale, etc. are actually paragangliomas (N Engl J Med 2002;347:854)
  • Glomangiopericytoma may be used for glomus tumors with prominent hemangiopericytic vasculature; however, these tumors are different from the pericytic site specific sinonasal glomangiopericytoma
ICD coding
  • ICD-10: D21.10 - other benign neoplasms of connective and other soft tissue
Epidemiology
Sites
  • Described throughout the body; most common is upper extremity (62%), particularly the fingers (27%)
  • Subungual region is the most common location on the finger
  • Other sites:
    • Trunk wall (11%)
    • Internal (11%)
    • Lower extremity (9%)
    • Head and neck (7%)
  • Atypical and malignant glomus tumors are more commonly deep seated than benign tumors (Am J Surg Pathol 2001;25:1)
Pathophysiology
  • NF1 associated tumors arise from biallelic inactivation of NF1 (Cancer Res 2009;69:7393)
    • Loss of neurofibromin in glomus cells leads to activation of the RAS-MAPK pathway
  • Familial glomus tumors (glomuvenous malformations) are caused by inactivating mutations in the glomulin gene expressed in vascular smooth muscle cells (Am J Hum Genet 2013;92:188)
  • Some cases show novel translocation resulting in oncogenic activation of NOTCH driven by translocation with the MIR143 promoter (Genes Chromosomes Cancer 2013;52:1075)
Etiology
  • Arises from the Sucquet-Hoyer canal of the glomus body, a specialized arteriovenous anastomosis that regulates heat in the skin and is surrounded by layers of epithelioid, SMA+ glomus cells
  • Cold temperature causes relaxation of the glomus cells to open the anastomosis and divert blood away from the capillary network to conserve body heat (J Med Genet 2010;47:525)
Clinical features
  • Symptoms:
    • Paroxysms of pain radiating away from the lesion, often out of proportion to the size of the neoplasm
    • Pain can be elicited by changes in temperature or tactile stimulation
    • Can be accompanied by hypesthesia, muscle atrophy or osteoporosis
  • Some glomus tumors of the fingers and toes are associated with NF1 (J Med Genet 2010;47:525)
Diagnosis
  • Clinical and location may be highly suggestive
  • Definitive diagnosis requires histologic assessment of the tumor by biopsy or excision
Radiology description
  • MRI: well defined mass, dark on T1 and bright, contrast enhancing on T2 or T1 post gadolinium fat saturation images
  • MR angiography: strong enhancement in the arterial phase and tumor blush, which increases in size in the delayed phase
  • Ultrasound: solid, hypoechoic mass with possible associated erosion of the underlying bone (Radiographics 2010;30:1621)
  • Radiography: may show thinning of the cortical bone in subungual tumors (AJR Am J Roentgenol 2004;182:263)
Radiology images

Contributed by Farres Obeidin, M.D.

Leg mass

Leg mass

Leg mass


Duodenal mass

Wrist mass

Subungual mass

Subungual mass

Prognostic factors
  • Benign tumors have a small chance for recurrence if incompletely excised
  • Atypical features found to correlate with adverse outcome; can be called glomus tumor of uncertain malignant potential:
    • Deep location, size > 2 cm and the presence of atypical mitotic figures
  • Malignant glomus tumors are characterized by (1) marked nuclear atypia and any level of mitotic activity or (2) atypical mitotic figures and metastasize in up to 40% of cases (Am J Surg Pathol 2001;25:1)
Case reports
Treatment
  • Benign tumors cured by complete surgical excision; recurrence rate of 10% likely due to incomplete resection (Arch Pathol Lab Med 2008;132:1448)
  • Direct transungual excision is the standard but can cause nail deformity; lateral subperiosteal approach reduces risk of nail deformity but may slightly raise risk of recurrence (Plast Reconstr Surg 2004;114:1486)
  • No clinical trial data is available for treatment of malignant glomus tumors but doxorubicin and olaratumab have been approved for recurrent / metastatic disease
  • No prospective data exists yet for Notch inhibition or immune checkpoint inhibitors
Clinical images

Images hosted on other servers:
Missing Image

Nail discoloration

Fullness in posterior nail fold region

Glomus tumor during exploration

Missing Image

Tumor resection

Gross description
  • Bulging mass with an irregular, nodular hemorrhagic cut surface
  • Superficial lesions typically < 1.0 cm
Gross images

Contributed by Farres Obeidin, M.D.

Leg mass



Images hosted on other servers:

Liver tumor

Microscopic (histologic) description
  • Well circumscribed mass composed of 3 components: glomus cells, vasculature and smooth muscle cells
  • Solid glomus tumor (75% of cases): predominantly glomus cells, poor vasculature and rare smooth muscle cells
  • Glomangioma (20% of cases): tumors with prominent vascular component
  • Glomangiomyoma (5% of cases): tumors with prominent vascular and smooth muscle components
  • Rare variants have microscopically infiltrative and diffuse growth and are known as glomangiomatosis
  • Benign glomus tumors (Arch Pathol Lab Med 2008;132:1448):
    • Branching capillary sized vessels lined by endothelial cells surrounded by collars of uniform glomus cells forming nests, sheets and trabeculae in a hyalinized or myxoid stroma
    • Glomus cell has a round shape with indistinct borders with a rounded, sharply punched out nucleus in an amphophilic to eosinophilic cytoplasm
    • Chromatin is homogenous and bland with inconspicuous nucleoli
    • Very rare mitoses
  • Symplastic glomus tumor:
    • Marked nuclear atypia with no mitotic activity or other atypical features
    • Does not affect prognosis
  • Malignant glomus tumor (Am J Surg Pathol 2001;25:1):
    • Diagnosis should be reserved for tumors showing (1) marked nuclear atypia and any level of mitotic activity or (2) atypical mitotic figures
    • May be spindled and resemble leiomyosarcoma / fibrosarcoma or may consist of sheets of malignant appearing round cells
    • Benign glomus tumor component can often be found at the periphery and help with diagnosis
  • Glomus tumor of uncertain malignant potential:
    • Tumors that do not fulfill criteria for malignancy but have at least 1 atypical feature other than nuclear pleomorphism
    • Atypical features include deep location, size ≥ 2 cm and mitotic activity > 5 mitoses/50 high powered fields
  • Oncocytic features have been described (Histopathology 1987;11:523)
Microscopic (histologic) images

Contributed by Farres Obeidin, M.D.

Well circumscribed

Sheets of cells

Spindling

Myxoid stroma

Round, bland cells

Nuclear inclusions


Glomangioma

Glomangioma

Glomangioma

Glomangiomyoma


Malignant glomus tumor

SMA

Glomus body



Case #289

Glomangioma

SMA

CD31



Case #238

Symplastic glomus tumor

Symplastic glomus tumor, smooth muscle actin

Cytology description
  • Cohesive clusters of uniform round cells with scanty cytoplasm
  • Background of scattered amorphous magenta colored material, blood and occasional inflammatory cells (J Cytol 2010;27:104)
Cytology images

Contributed by Farres Obeidin, M.D.

Malignant glomus tumor



Images hosted on other servers:

FNA

Positive stains
Electron microscopy description
  • Thick basal lamina surrounds individual glomus cells except at cellular junctions
  • Pinocytic vesicles and myofibrils with dense bodies in the cytoplasm (Cancer 1969;23:1176)
Electron microscopy images

Images hosted on other servers:
Missing Image

Thickened basal lamina

Molecular / cytogenetics description
Molecular / cytogenetics images

Images hosted on other servers:
Missing Image

MIR143-NOTCH2 fusion

Sample pathology report
  • Thumb, mass, excision:
    • Glomus tumor, excised (see comment)
    • Comment: Histologic sections demonstrate a dermal, solid and syncytial proliferation of round cells with punched out nuclei surrounding scattered blood vessels with perivascular hyalinization. No mitotic activity, necrosis or nuclear atypia is noted. SMA and desmin IHC are positive and keratins are negative, supporting the diagnosis.
Differential diagnosis
Board review style question #1

    Which of the following statements about this lesion is true?

  1. Complete surgical excision is curative
  2. It may metastasize in about 30% of cases
  3. It will show diffuse positivity for CD34, CD31 and ERG
  4. The most common location is in the ankle
Board review style answer #1
A. Complete surgical excision is curative. This is a glomangioma, a variant of glomus tumor. It is a benign tumor and if completely excised, surgical excision is essentially curative. The most common location is in the subungual region of the finger. Though it may show focal positivity for CD34, the staining for CD31 and ERG will be negative as the tumor cells are not vascular in origin.

Comment Here

Reference: Glomus tumor
Board review style question #2
    Which of the following molecular aberrations can be seen in glomus tumors?

  1. ETV6-NTRK3
  2. EWSR1-FLI1
  3. MIR143-NOTCH1
  4. SS18-SSX2
Board review style answer #2
C. MIR143-NOTCH1. Over 50% of glomus tumors harbor gene fusions of MIR143 with either NOTCH1 or NOTCH2.

Comment Here

Reference: Glomus tumor

Grading
Definition / general
  • Grading in soft tissue tumors is to assign a level of differentiation
  • Most important factor in prognosis, metastatic risk and management of soft tissue tumors (Arch Pathol Lab Med 2006;130:1448)
Essential features
  • Grading is required in soft tissue sarcomas where possible
  • Management often relies on the grade
  • FNCLCC (Fédération Nationale des Centres de Lutte Contre le Cancer) is the most common grading system; defined by a combination of tumor differentiation, mitotic count and necrosis
Terminology
  • FNCLCC: Fédération Nationale des Centres de Lutte Contre le Cancer
  • NCI: National Cancer Institute
  • POG: Pediatric Oncology Group
Fédération Nationale des Centres de Lutte Contre le Cancer (FNCLCC) grading
  • Based on a combination of tumor differentiation / histology, mitotic count and tumor necrosis
    • Tumor differentiation
      • Given 1, 2 or 3 points based on how closely it resembles normal tissue
      • See table 1 for specific point values by histologic type
    • Mitotic count: 10 successive high power fields (HPFs) in the most mitotically active area
      • 1 point: 0 - 9 mitoses
      • 2 points: 10 - 19 mitoses
      • 3 points: 20 or more mitoses
    • Tumor necrosis
      • 0 points: no necrosis
      • 1 point: < 50% necrosis
      • 2 points: ≥ 50% necrosis
    • Grade: sum total of tumor differentiation, mitotic count and tumor necrosis scores
      • Grade 1: 2 - 3 points
      • Grade 2: 4 - 5 points
      • Grade 3: 6 - 8 points
  • Additional notes on specific tumors:
    • Sarcomas not graded in the FNCLCC system: embryonal and alveolar rhabdomyosarcoma, angiosarcoma, extraskeletal myxoid chondrosarcoma, alveolar soft part sarcoma, clear cell sarcoma and epithelioid sarcoma
    • Malignant peripheral nerve sheath tumor:
      • FNCLCC grading has been controversial, recent data suggests an overall correlation with survival (Am J Clin Oncol 2022;45:28)
      • Tumor differentiation score may be variable based on closeness to benign peripheral nerve sheath tumors:
        • Score 1: resembles benign peripheral nerve sheath tumor with increased cellularity and mitotic activity
        • Score 2: overtly malignant with characteristics of nerve sheath differentiation (i.e. marbling and short fascicle formation)
        • Score 3: overtly malignant lacking nerve sheath differentiation, having heterologous elements or epithelioid morphology
      • In neurofibromatosis type 1 (NF1) associated tumors, the distinction between neurofibroma and low grade malignant peripheral nerve sheath tumor may be difficult and certain criteria have been proposed (Hum Pathol 2017;67:1) (see table 2)
    • Myxofibrosarcoma grading by FNCLCC is controversial; currently graded by level of cellularity and percentage of myxoid versus solid areas
    • Dedifferentiated liposarcoma:
      • By FNCLCC, well differentiated liposarcoma / atypical lipomatous tumor by definition are grade 1 and dedifferentiated liposarcoma may be grade 2 or 3
      • There remains controversy over whether nonlipogenic areas with low grade features should be classified as low grade dedifferentiation or cellular well differentiated liposarcoma (Am J Surg Pathol 2007;31:1)
      • Well differentiated liposarcoma may represent a spectrum of tumors with variable prognosis (J Surg Oncol 2018;117:1799)
    • Solitary fibrous tumors and gastrointestinal stromal tumors are not graded; metastatic risk is given by specific risk stratification nomograms
  • Grading on core needle biopsies
    • Encouraged where possible but may be limited by amount of tissue
    • Separation at least into low (FNCLCC 1) versus high (FNCLCC 2 and 3) grade is recommended as there are implications for neoadjuvant therapy

Table 1: tumor differentiation score by histologic type
Histologic type
Score
Atypical lipomatous tumor / well differentiated liposarcoma 1
Well differentiated leiomyosarcoma 1
Malignant neurofibroma 1
Well differentiated fibrosarcoma 1
Myxoid liposarcoma 2
Conventional leiomyosarcoma 2
Conventional fibrosarcoma 2
Myxofibrosarcoma 2* (see above)
High grade myxoid liposarcoma 3
Pleomorphic liposarcoma 3
Dedifferentiated liposarcoma 3
Pleomorphic rhabdomyosarcoma 3
Poorly differentiated / pleomorphic leiomyosarcoma 3
Synovial sarcoma 3
Mesenchymal chondrosarcoma 3
Extraskeletal osteosarcoma 3
Extraskeletal Ewing sarcoma 3
Malignant rhabdoid tumor 3
Undifferentiated pleomorphic sarcoma 3
Undifferentiated sarcoma, NOS 3


Table 2: NF1 associated malignant peripheral nerve sheath tumor (MPNST) grading
Grade
Description
Atypical neurofibromatous neoplasm of uncertain biologic potential (ANNUBP) Schwann cell neoplasm with ≥ 2 of 4 features: cytologic atypia, loss of neurofibroma architecture, hypercellularity, < 3 mitoses per 10 HPFs
MPNST, low grade Features of ANNUBP but with mitotic count of 3 - 9 mitoses per 10 HPFs and no necrosis
MPNST, high grade MPNST with ≥ 10 mitoses per 10 HPFs or 3 - 9 mitoses per 10 HPFs combined with necrosis
National Cancer Institute (NCI) grading
  • Less commonly used grading system
  • Similar performance to the FNCLCC grading system but more difficult to objectively quantify (J Clin Oncol 1997;15:350)
  • Uses older terminology and defunct diagnoses

Grade
Tumor type
1
  • Well differentiated liposarcoma
  • Myxoid liposarcoma
  • Subcutaneous myxoid malignant fibrous histiocytoma (defunct)
  • Well differentiated malignant hemangiopericytoma with < 1 mitosis per 10 HPFs, no necrosis and no hemorrhagic areas
  • Well differentiated fibrosarcoma or leiomyosarcoma with no pleomorphism, no necrosis and < 6 mitoses per 10 HPFs
  • Malignant peripheral nerve sheath tumor if resembles neurofibroma with high cellularity and mitotic figures < 6 mitoses per 10 HPFs
  • Myxoid chondrosarcoma that is uniformly myxoid and hypocellular with no mitotic activity
2
  • Other histologic types with < 15% necrosis
3
  • Extraskeletal Ewing sarcoma / primitive neuroectodermal tumor (PNET)
  • Extraskeletal osteosarcoma
  • Mesenchymal chondrosarcoma
  • Malignant triton tumor
  • Other histologic types with ≥ 15% necrosis
Pediatric Oncology Group (POG) grading
  • Created as a modification of the NCI system for use in nonrhabdomyosarcomatous sarcomas in the pediatric population
  • Similar performance to FNCLCC but may upgrade certain tumors (Cancer 2010;116:2266)

Grade
Tumor type
1
  • Myxoid and well differentiated liposarcoma
  • Well differentiated or infantile (≤ 4 years old) fibrosarcoma
  • Well differentiated or infantile (≤ 4 years old) hemangiopericytoma
  • Angiomatoid malignant fibrous histiocytoma (defunct)
  • Deep seated dermatofibrosarcoma protuberans (defunct)
  • Myxoid chondrosarcoma
2
  • < 15% of the surface area shows necrosis
  • Mitotic count is < 5 mitoses per 10 HPFs
  • Nuclear atypia is not marked
  • Tumor is not markedly cellular
3
  • Pleomorphic or round cell liposarcoma
  • Mesenchymal chondrosarcoma
  • Extraskeletal osteogenic sarcoma
  • Malignant triton tumor
  • Alveolar soft part sarcoma
  • Any other sarcoma not in grade 1 with > 15% necrosis or ≥ 5 mitotes per 10 HPFs
Treatment
  • Clinical staging for sarcomas of trunk, extremities and retroperitoneum depends on the grade
    • Localized grade 1 sarcomas are staged as stage IA or IB
    • Grade 2 - 3 sarcomas or metastatic sarcomas are staged as stage II - IV
  • Higher grade tumors often require adjuvant therapy to control their higher metastatic risk
Microscopic (histologic) images

Contributed by Farres Obeidin, M.D.
FNCLCC grade 1

FNCLCC grade 1

FNCLCC grade 3

FNCLCC grade 3

Sample pathology report
  • Right leg, mass, radical resection:
    • Undifferentiated pleomorphic sarcoma, FNCLCC grade 3 (see comment)
    • Comment: Histologic sections demonstrate a pleomorphic malignancy without evidence of definitive line of differentiation, consistent with an undifferentiated pleomorphic sarcoma (score 3). The mitotic count is up to 25 mitoses per 10 HPFs (score 3) and there is necrosis encompassing approximately 20% of the tumor volume (score 1).
Board review style question #1
A 65 year old man presents with a 10 cm leg mass. A core needle biopsy is performed which demonstrates a leiomyosarcoma. Which of the following most accurately describes the factors involved in FNCLCC grading of this lesion?

  1. Mitotic activity, size, necrosis
  2. Pleomorphism, architecture, vascular invasion
  3. Size, age, location
  4. Tumor differentiation, mitotic activity, necrosis
Board review style answer #1
D. FNCLCC grading is based on a combined score of tumor differentiation, mitotic activity and necrosis

Comment Here

Reference: Soft tissue grading
Board review style question #2
A 30 year old woman presents with a 5 cm arm mass. After histologic examination and ancillary studies, a diagnosis of synovial sarcoma is made. The tumor has up to 12 mitoses per 10 HPFs and no necrosis is seen. What is the appropriate FNCLCC grade for this tumor?

  1. FNCLCC grade 1
  2. FNCLCC grade 2
  3. FNCLCC grade 3
  4. Not graded by FNCLCC
Board review style answer #2
B. FNCLCC grade 2. The combination of tumor differentiation (score 3), mitotic activity (score 2) and necrosis (score 0) sum to 5, which falls under grade 2 in the FNCLCC grading system.

Comment Here

Reference: Soft tissue grading

Granular cell tumor
Definition / general
  • Soft tissue tumor with neuroectodermal differentiation composed of large cells with eosinophilic, granular cytoplasm
Essential features
  • Very rare, predominantly benign soft tissue tumor involving a wide variety of sites
  • Shows neural differentiation
  • Histology: large cells with eosinophilic, granular cytoplasm; may show pseudoepitheliomatous hyperplasia
  • Immunohistochemistry: S100+, SOX10+, CD68+
Terminology
ICD coding
  • ICD-10: D21.9 - benign neoplasm of connective and other soft tissue, unspecified
  • ICD-10: C49.9 - malignant neoplasm of connective and soft tissue, unspecified
Epidemiology
Sites
  • Wide variety of sites
    • Peripheral soft tissues (38%)
    • Trunk (36%)
    • Head and neck (20%)
      • Predilection for the tongue
    • Internal (16%)
      • Esophagus most common
  • 5 - 15% have multiple lesions
Pathophysiology / etiology
Clinical features
  • Majority are benign (only 0.5 - 2% are malignant)
  • Most are asymptomatic, typically 1 - 2 cm in greatest dimension
  • Slow, indolent growth
Radiology description
  • Mostly nonspecific
  • MRI:
    • Isointense or brighter than muscle on T1
    • High peripheral signal and centrally isointense to muscle on T2
    • May be affected by the amount of stromal component in the tumor
    • Malignant variants show characteristics seen in other aggressive neoplasms (Skeletal Radiol 2005;34:625)
Prognostic factors
  • Majority are benign with excellent outcome after surgical resection
  • Some propensity for local recurrence if incompletely excised (J Surg Oncol 1980;13:301)
  • Malignant granular cell tumors behave similarly to malignant peripheral nerve sheath tumors and have a 50% rate of metastasis
  • Adverse prognostic factors include local recurrence, metastasis, larger tumor size, older patient age, histologic classification as malignant, presence of necrosis, increased mitotic activity, spindling of tumor cells, vesicular nuclei with large nucleoli and Ki67 ≥ 10% (Am J Surg Pathol 1998;22:779)
  • Another study showed size > 5 cm and distant metastases were the factors most negatively affecting survival (J Surg Oncol 2018;118:891)
Case reports
Treatment
  • Benign lesions treated with surgical excision have excellent prognosis
  • Malignant tumors require wider surgical resection
    • Lymph node dissection not recommended unless metastasis is biopsy proven
    • Benefit of adjuvant radiotherapy / chemotherapy is debatable but may be helpful in controlling progression in advanced disease (J Surg Oncol 2018;118:891)
    • Pazopanib showed activity in a single case report (Case Rep Med 2014;2014:794648)
Clinical images

Contributed by Mark R. Wick, M.D.

Arm



Images hosted on other servers:

Esophagus

Cecum

Gross description
  • Uninodular firm mass
  • In the skin, overlying epidermal hyperplasia can be seen
  • Cut surface is yellow with finely granular texture
Gross images

Contributed by Farres Obeidin, M.D.

Uninodular subcutaneous mass

Tan-yellow, gritty nodule

Ill defined borders

Intervening fibrous septae

Sessile subcutaneous nodule

Microscopic (histologic) description
  • Architecture:
    • Poorly defined mass composed of sheets of cells or nests / ribbons separated by thin collagenous bands
  • Cytological features:
    • Cells are round and polygonal to slightly spindled; cell borders can be distinct or show a syncytial pattern
    • Nuclei: range from small and dense to large and vesicular; mild to moderate nuclear atypia can be seen
    • Abundant eosinophilic cytoplasm with coarse granules (representing phagolysosome aggregates)
  • Overlying squamous epithelium may show pseudoepitheliomatous hyperplasia
  • Malignant criteria still debated
    • Fanburg-Smith criteria:
      • Necrosis, tumor cell spindling, vesicular nuclei with large nucleoli, > 2 mitoses/10 high power fields, high nuclear to cytoplasmic ratio and pleomorphism
      • 0: benign; 1 - 2: atypical; ≥ 3: malignant (Am J Surg Pathol 1998;22:779)
    • Nasser-Ahmed-Kowalski criteria:
      • Necrosis and > 2 mitoses/10 high power fields
      • 0: benign; ≥ 1: granular cell tumor of uncertain malignant potential
      • Metastasis was the only criteria to diagnose malignant granular cell tumor (Pathol Res Pract 2011;207:164)
Microscopic (histologic) images

Contributed by Farres Obeidin, M.D. and Christopher Hartley, M.D.

Involving tongue musculature

Pseudoepitheliomatous
hyperplasia

Coarse granular cytoplasm

Sheets of cells with abundant cytoplasm

Interspersed chronic inflammation

Ovoid to spindled appearance


Large cytoplasmic inclusions

S100+

Desmin-

Gastrointestinal tract submucosal lesion

Focal nuclear atypia

Adnexal sparing


Spindled area

Pseudoepitheliomatous
hyperplasia

CD68+

S100+

Fibrotic stroma

Granular cell tumor

Cytology images

Case #434

Papanicolaou

May-Grünwald-Giemsa

Positive stains
Electron microscopy description
  • Cells in small clusters separated by basement membrane material
  • Nuclei have irregular, invaginated contour with dense granulation
  • Cytoplasm is complex and filled with irregular osmophilic granules
  • Occasional unmyelinated axons can be seen among the clusters of granular cells (J Surg Oncol 1980;13:301)
Molecular / cytogenetics description
Sample pathology report
  • Tongue, mass, resection:
    • Granular cell tumor, 1.3 cm (see comment)
    • Surgical margins, negative for tumor
    • Comment: Microscopic sections show an ill defined proliferation of bland round to ovoid cells with prominent eosinophilic, granular cytoplasm. The overlying epithelium shows pseudoepitheliomatous hyperplasia. S100 and CD68 stains are positive, supporting the diagnosis.
Differential diagnosis
Board review style question #1
A 45 year old man presents with a 1.2 cm mass on the right subcutaneous forearm. The mass is diffusely positive for S100 and CD68. Which of the following is true about this lesion?



  1. Cells are filled with PAS+, diastase resistant granules
  2. It's likely reactive in nature and will regress without treatment
  3. Lesion is associated with multifocal lymphadenopathy
  4. Overlying epithelium is usually ulcerated and thinned
Board review style answer #1
A. Cells are filled with PAS+, diastase resistant granules. This is a granular cell tumor. The granules are composed of phagolysosomes and are PASD+. They are not reactive and typically need surgery for treatment. Granular cell tumors are not associated with lymphadenopathy and the overlying epithelium is typically hyperplastic, giving a pseudoepitheliomatous appearance.

Comment Here

Reference: Granular cell tumor
Board review style question #2
Which of the following immunostaining profiles would you expect to see in a granular cell tumor?

  1. Desmin+, myogenin+, MyoD1+
  2. S100+, CD1a+, fascin+
  3. S100+, CD68+, TFE3+
  4. S100+, SOX10+, HMB45+, MelanA+
Board review style answer #2
C. S100+, CD68+, TFE3+. Granular cell tumors stain with S100, CD68 and TFE3. HMB45 and MelanA positivity favors a melanoma. Desmin and more specific myogenic markers are seen in adult type rhabdomyoma. CD1a and fascin are characteristic for Langerhans cell histiocytosis.

Comment Here

Reference: Granular cell tumor

Hemangioma & variants
Definition / general
Essential features
  • Most cases arise in children, with equal gender distribution
  • Composed of small capillary sized blood vessels with a larger feeding vessel commonly present
  • Most cases are treated with topical or systemic beta blockers in isolation or in combination with other modalities like laser therapy, excision, etc.
  • Recurrence is uncommon and only exceptional examples show malignant transformation
Terminology
  • Hemangioma is commonly used with a qualifier (e.g. congenital hemangioma)
ICD coding
  • ICD-O: 9120/0 - hemangioma, NOS
  • ICD-11: 2E81.0Y - other specified neoplastic hemangioma
Epidemiology
Sites
Pathophysiology
  • Specific pathogenesis has not been fully understood
  • Currently regarded as a multifactorial condition, resulting in endothelial proliferation, with uncontrolled angiogenesis and abnormal function of downstream pathways (notably HIF1α, VEGF and PI3K / Akt) (Biomed Res Int 2021;2021:5695378)
Etiology
  • Not known
Clinical features
Diagnosis
  • Requires correlation of detailed history and physical examination with microscopic features, as the latter overlaps for various clinical presentations
Laboratory
  • Transient mild to moderate thrombocytopenia due to consumption may be seen in rapid growth phase (F1000Res 2019;8:F1000)
Radiology description
  • Mostly required for extracutaneous cases
  • Skeletal (bone) hemangiomas: radiology is highly nonspecific (BMC Musculoskelet Disord 2021;22:27)
  • Ultrasound: highly effective; shows well defined mass with high vascularity
  • Contrast enhanced ultrasonography (CEUS): gold standard for hepatic hemangiomas (Korean J Radiol 2000;1:191)
  • High flow in proliferative phase of infantile hemangioma; slow flow in involuted phase
  • Central high flow for NICH; slow flow for RICH
  • Fluttering sign: recently described feature for grayscale ultrasound in hepatic hemangiomas (Ultrasound Med Biol 2021;47:941)
  • MRI: useful in deep locations to estimate extent of tissue involvement
  • Infantile hemangioma: hyperintense on T2; isointense on T1 with postcontrast enhancement
  • Congenital hemangioma: heterogeneous appearance on MRI
  • CT scan: hypodense, with postcontrast enhancement of periphery (World J Gastroenterol 2020;26:11)
Radiology images

Images hosted on other servers:
Hepatic hemangioma Hepatic hemangioma

Hepatic hemangioma

Intrathoracic hemangioma

Intrathoracic hemangioma

Infantile thyroid hemangioma

Infantile thyroid hemangioma

Infantile hepatic hemangioma

Infantile hepatic hemangioma

Choroidal hemangioma (fundoscopy)

Choroidal hemangioma (fundoscopy)


Hemangioma in long bones Hemangioma in long bones

Hemangioma in long bones

Hemangioma in long bones Hemangioma in long bones Hemangioma in long bones

Hemangioma in long bones

Prognostic factors
Case reports
Treatment
Clinical images

Contributed by Nasir Ud Din, M.B.B.S.
Noninvoluting congenital hemangioma

Noninvoluting congenital hemangioma



Images hosted on other servers:
Infantile hemangioma

Infantile hemangioma

Caption

Infantile hemangioma with ulceration

Infantile hemangioma in beard distribution

Infantile hemangioma in beard distribution

Rapidly involuting congenital hemangioma

Rapidly involuting congenital hemangioma

Partially involuting congenital hemangioma

Partially involuting congenital hemangioma

Laryngeal hemangioma (laryngoscopic view)

Laryngeal hemangioma (laryngoscopic view)

Gross description
Gross images

Images hosted on other servers:
Atrial hemangioma

Atrial hemangioma

Hepatic hemangioma

Hepatic hemangioma

Pulmonary hemangioma

Pulmonary hemangioma

Microscopic (histologic) description
  • Lobules of capillary sized vascular channels, lined by single layer of flattened endothelial cells
  • Large feeding vessel is usually seen at the deeper aspect
  • Associated lymphocyte infiltrate may be seen (Cardiovasc Pathol 2017;28:59)
  • Anastomosing hemangioma: anastomosing vascular channels lined by flattened endothelium; deep occurrence
  • Angiomatosis: involvement of multiple tissue planes with irregular and poorly circumscribed edges
  • Cavernous hemangioma: shows predominantly ectatic channels
  • Congenital hemangioma:
    • Solid appearance in rapid growth phase with poorly canalized vessels and mitotically active endothelium
    • Surrounding pericyte layer is present
    • With maturation, the lumina become prominent and blood flow ensues
    • Combination of solid and vascular areas in varying proportions may be seen
    • Noninvoluting congenital hemangioma (NICH) shows well formed capillaries and vascular channels
    • Involuting examples show thickening of basement membranes and fibrosis in the background
  • Epithelioid hemangioma:
    • Well formed small vessels are lined by plump endothelial cells with abundant eosinophilic cytoplasm and round enlarged nuclei, accompanied by abundant eosinophils in the background
    • Lobulated, well demarcated with maturation of vascular lumina at the periphery
  • Glomeruloid hemangioma: resembles glomerular capillaries
  • Hobnail hemangioma: hobnail nuclei protruding into vascular lumina; circumscribed
  • Infantile hemangioma:
    • Proliferation of capillary lobules; has a distinct natural history involving three stages (i.e., proliferation, partial regression and complete regression):
      • Early proliferative stage: lobules of immature dendritic type cells with intervening stroma, large feeding vessels and occasional presence of perineural involvement
      • Early regression: capillaries dilate and eventually start disappearing; apoptotic debris in basement membrane with increased numbers of pericapillary mast cells.
      • Late regression / end stage: ghosts of capillaries, rings of basement membrane with rare endothelial cells having immunophenotype of placental capillaries (GLUT1, LeY, CD15, CCR6, IDO and IGF2 positive)
  • Intramuscular angioma:
    • Arises within skeletal muscle in association with variable amount of mature adipose tissue, phleboliths and metaplastic bone formation
    • Shows a combination of lymphatics, variable sized veins and arteriovenous component
  • Lobular capillary hemangioma / pyogenic granuloma: ulceration with dense mixed inflammation
  • Microvenular hemangioma:
    • Poorly defined, in superficial and deep dermis
    • Small venule-like channels lined by single layer of endothelial cells which lack mitotic activity, surrounded by single layer of pericytes; these venules appear to dissect hyalinized collagen bundles of the dermis but lack multilayering and HHV8 positivity
  • Sinusoidal hemangioma:
    • Well demarcated proliferation of dilated, congested and thin vascular channels anastomosing in a sinusoidal manner
    • Intervening stroma is scant and scant smooth muscle may be present in the wall
    • Mitotic activity is not seen
  • Spindle cell hemangioma: proliferating spindle cells with intraluminal phleboliths
  • Verrucous hemangioma: hyperkeratosis and involvement of several tissue planes
Microscopic (histologic) images

Contributed by Nasir Ud Din, M.B.B.S.
Anastomosing hemangioma Anastomosing hemangioma

Anastomosing hemangioma

Skeletal hemangioma Skeletal hemangioma Skeletal hemangioma

Skeletal hemangioma

Cavernous hemangioma

Cavernous hemangioma


Hobnail hemangioma Hobnail hemangioma

Hobnail hemangioma

Hepatic hemangioma Hepatic hemangioma

Hepatic hemangioma

Infantile hemangioma Infantile hemangioma

Infantile hemangioma


Infantile hemangioma

Infantile hemangioma

Lobular capillary hemangioma Lobular capillary hemangioma Lobular capillary hemangioma

Lobular capillary hemangioma

Spindle cell hemangioma Spindle cell hemangioma

Spindle cell hemangioma


Spindle cell hemangioma

Spindle cell hemangioma

Phleboliths in spindle cell hemangioma

Phleboliths in spindle cell hemangioma

Synovial hemangioma

Synovial hemangioma

Verrucous hemangioma

Verrucous hemangioma

Virtual slides

Images hosted on other servers:
Extensive evolution

Extensive evolution

Involuting infantile hemangioma

Involuting infantile hemangioma

Congenital hemangioma

Congenital hemangioma

Cytology description
  • Aspiration is not advised, as there is high risk of uncontrolled bleeding
Positive stains
Negative stains
Electron microscopy description
  • Not required for diagnosis in routine cases
  • Endothelial cells show cytoplasmic folds on luminal surface, junctional complexes and cytoplasmic pinocytic vesicles (Cancer Res 1990;50:4787)
Electron microscopy images

Images hosted on other servers:
Endothelial differentiation

Endothelial differentiation

Cutaneous hemangioma

Cutaneous hemangioma

Molecular / cytogenetics description
Molecular / cytogenetics images

Images hosted on other servers:
Sanger sequencing for <i>RASA1</i>

Sanger sequencing for RASA1

Videos

Spindle cell hemangioma

Cutaneous vascular tumors

Sample pathology report
  • Liver, lobectomy:
    • Benign vascular lesion, consistent with hemangioma (see comment)
    • Comment: The tumor is completely excised with free margins. Evidence of embolization is seen, with presence of acellular material within the lumina of large caliber vessels, accompanied by areas of infarction within the lesion.
Differential diagnosis
  • Granulation tissue:
    • Sites of trauma, commonly cutaneous or mucosal
    • Shows associated inflammatory infiltrate and edema
    • Feeding vessels are not seen commonly
    • Lobular capillary hemangioma is common mimicker
    • Correlation with clinical history is important
  • Hemangioendothelioma:
    • Mimics proliferating phase of congenital hemangioma
    • Retiform type mimics hobnailing seen in proliferating phase of congenital hemangioma
    • Not well circumscribed
    • Clinical presentation with consumptive coagulopathy is common in both
    • May be associated with other anomalies
    • Requires more aggressive treatment
  • Kaposi sarcoma:
    • Mimics proliferating phase of congenital hemangioma due to compact cellularity and slit-like spaces
    • Eosinophilic hyaline globules are not seen in hemangioma
    • HHV8 association is not seen in hemangioma
    • Requires aggressive treatment
  • Angiosarcoma:
    • Malignant vascular tumor with infiltrative growth
    • Marked nuclear pleomorphism and brisk mitotic activity
    • Visceral location is more common compared to hemangioma
Board review style question #1

A newborn shows a nodular growth on his scalp. Over the next couple of weeks, the lesion starts shrinking. Despite reassurances from the pediatrician, the parents insist on excision. Upon pathological examination, it shows a network of capillary sized vessels with large feeding vessels near the base, shown in the above photomicrograph. The diagnosis in this case is

  1. Infantile hemangioma
  2. Noninvoluting hemangioma
  3. Partially involuting hemangioma
  4. Rapidly involuting hemangioma
Board review style answer #1
A. Infantile hemangioma. This case is an example of infantile hemangioma, which typically shows regression after birth, unlike congenital hemangioma, which grows with the patient, at least to some extent. Within the umbrella of congenital hemangioma, there is partial involution over time with shrinkage and secondary changes. Noninvoluting examples grow with the baby and rapidly involuting ones regress at a much faster rate, ultimately disappearing completely.

Comment Here

Reference: Hemangioma
Board review style question #2

A 9 year old boy presents with a purple nodular / bosselated lesion on the left leg, with involvement of the plantar and dorsal aspects of foot (as shown in the clinical photograph). He has had this lesion since birth and it has grown with him since then. The microscopic examination shows well formed vascular channels of variable size lined by a single layer of endothelial cells. The most likely diagnosis in this case is

  1. Birth mark
  2. Infantile hemangioma
  3. Kaposiform hemangioendothelioma
  4. Lobular capillary hemangioma
  5. Noninvoluting congenital hemangioma
Board review style answer #2
E. Noninvoluting congenital hemangioma. Noninvoluting congenital hemangioma is present at birth and grows with the child. As the name implies, there is no microscopic evidence of involution if a biopsy is performed. Birth marks commonly present with a flat, pigmented / reddish appearance of the skin rather than the appearance given in the clinical photograph, which shows prominent vascular marking, like the appearance of the lesion with some areas showing bosselation. Infantile hemangioma rapidly regresses after birth. Lobular capillary hemangioma presents as a nodular growth, commonly associated with ulceration of the skin or mucosa. Kaposiform hemangioendothelioma clinically presents commonly on the head and neck and trunk region, with a strong association with Kasabach-Merritt syndrome (consumptive coagulopathy). Microscopically, it shows glomeruloid proliferation and spindle cell morphology with slit-like spaces on microscopic examination.

Comment Here

Reference: Hemangioma

Hematoma
Definition / general
  • May clinically resemble a tumor if deep seated
  • Important to exclude a sarcoma with intratumoral hemorrhage
  • Pseudoaneurysm may occur after intravascular procedures
  • Postoperative seromas may persist for years and be confused with recurrent tumor
Sites
  • Usually near tensor fascia lata (upper thigh muscle, see drawing below)
Case reports
Treatment
  • Complete surgical excision, including pseudocapsule
Clinical images

Images hosted on other servers:

Lesion of lateral thigh

Perforating vessel
from tensor fascia lata
appears to run into the mass

Lobulation with dense
fibrous pseudocapsule
and chocolate brown fluid

Microscopic (histologic) images

Images hosted on other servers:

Cystic cavity with necrotic debris, fibrin and blood costs

Cyst wall has hyalinized fibrous tissue, chronic inflammation and granulation tissue

Additional references

Hibernoma
Definition / general
  • Rare, benign, adipocytic tumor composed of variable proportions of brown fat cells admixed with white adipose tissue
Essential features
Terminology
ICD coding
Epidemiology
Sites
Pathophysiology
  • Intraosseous hibernoma: osteoblasts and adipocytes have common origin from mesenchymal stem cells
  • Brown fat cell differentiation is associated with PRD1-BF1-RIZ1 homologous domain containing 16 (PRDM16) underregulated by bone morphogenetic protein 7 (BMP7), which also stimulates bone formation
  • This explains how brown fat cells and sclerotic bony trabeculae are mixed in intraosseous hibernomas (J Pathol Transl Med 2017;51:499)
Etiology
Clinical features
Diagnosis
Radiology description
Radiology images

Images hosted on other servers:
Intraosseous hibernoma (MRI, CT and PET)

Intraosseous hibernoma (MRI, CT and PET)

Posterior cervical triangle hibernoma (MRI)

Well demarcated lesion in right hip joint (MRI)

Mass within the left external oblique muscle (MRI)

Prognostic factors
Case reports
Treatment
Clinical images

Images hosted on other servers:
Lower limb subcutaneous hibernoma

Lower limb subcutaneous hibernoma

Hibernoma in proximal thigh

Hibernoma in proximal thigh

Gross description
Gross images

Contributed by Nasir Ud Din, M.B.B.S. and @Andrew_Fltv on Twitter
Well defined, encapsulated, hibernoma

Well defined, encapsulated hibernoma

Lipoma-like hibernoma

Lipoma-like hibernoma

Hibernoma

Hibernoma

Frozen section description
Microscopic (histologic) description
Microscopic (histologic) images

Contributed by Nasir Ud Din, M.B.B.S. and @Andrew_Fltv on Twitter
Peripheral well defined borders

Peripheral well defined borders

Benign adipocytes

Benign adipocytes

Sheets of brown cells and vasculature

Sheets of brown cells and vasculature

Eosinophilic granulated cells with central nuclei

Eosinophilic granulated cells with central nuclei

Hibernoma with all benign features

Hibernoma with all benign features

Rich vascularization

Rich vascularization


Eosinophilic cells with granular vacuolated cytoplasm

Granular vacuolated cytoplasm

Mixture of multivacuolated and univacuolated cells

Multivacuolated and univacuolated cells

Variable proportions of brown cells and mature adipocytes

Brown cells and mature adipocytes

Mixture of brown and white fat cells

Mixture of brown and white fat cells

Lipoma-like hibernoma Lipoma-like hibernoma

Lipoma-like hibernoma


Lipoma-like hibernoma

Lipoma-like hibernoma

Well defined border

Well defined border

No signs of malignancy

No signs of malignancy

Univacuolated and granular eosinophilic cells

Univacuolated and granular eosinophilic cells

S100 S100

S100


Hibernoma Hibernoma Hibernoma

Hibernoma

Cytology description
  • Uniform, polygonal multivacuolated cells with finely abundant granular cytoplasm, bland round nucleus and inconspicuous to prominent nucleoli
  • Occasional spindle shaped cells can be present
  • No nuclear atypia, mitosis or necrosis
  • Benign granular cell tumor is a close differential on cytology (J Clin Diagn Res 2017;11:ED01)
Cytology images

Images hosted on other servers:
Regular nuclei and granular cytoplasm (Giemsa stain)

Regular / eccentric
nuclei, granular
cytoplasm
(Giemsa / Pap)

Positive stains
Electron microscopy description
  • Hibernoma cells have several large mitochondria with high electron density matrix and dense cristae, lipid droplets, undulating plasmalemmal invaginations, pinocytic vesicles, free ribosomes, high glycogen content and small amount of smooth endoplasmic reticulum (J Clin Diagn Res 2017;11:ED01)
Molecular / cytogenetics description
Videos

Hibernoma: 5 minute pathology pearls

Sample pathology report
  • Thigh lump, excision:
    • Hibernoma (see comment)
    • Microscopic description: The sections reveal a well circumscribed neoplastic lesion comprising large polygonal cells arranged in sheets. These cells have multivacuolated, granular cytoplasm with small central nucleus admixed with variable amount of univacuolated cells. There is no significant cytological atypia, necrosis or mitosis.
    • Comment: Hibernoma is a benign neoplasm with no significant potential for recurrence.
Differential diagnosis
Board review style question #1

A 32 year man presented with a mass in his right thigh; MRI revealed circumscribed lobulated lesion and microscopy is shown in the picture above. What is the diagnosis?

  1. Atypical lipomatous tumor
  2. Fat necrosis
  3. Hibernoma
  4. Lipoblastoma
  5. Lipoma
Board review style answer #1
C. Hibernoma

Comment Here

Reference: Hibernoma
Board review style question #2
Hibernoma has an association with which one of the following syndromes?

  1. Anders syndrome
  2. Gardner syndrome
  3. Lynch syndrome
  4. MEN1 syndrome
  5. MEN2 syndrome
Board review style answer #2
D. MEN1 syndrome

Comment Here

Reference: Hibernoma

Histology-blood vessels
Definition / general
    • The circulatory system facilitates the continuous movement of the body fluid including the blood and the lymph
    • 2 main elements:
      • Blood vascular system
        • Arterial system
        • Microcirculation: capillaries
        • Venous system
      • Lymph vascular system
    • Vasa vasorum: small blood vessels responsible for the supply of the walls of the big blood vessels
    • Normal vessels contain endothelial cells facing lumen and pericytes, smooth muscle cells and glomus cells towards outside of vessel
Essential features
  • 2 main elements:
    • Blood vascular system (active, closed circuit pumping system [heart])
      • Arterial system
      • Microcirculation: capillaries
      • Venous system
    • Lymph vascular system (passive, open one way system)
  • 2 types of new blood vessel formation:
    • Vasculogenesis: vessels from precursor cells angioblasts
    • Angiogenesis from pre-existing vessels via sprouting
  • General structure of the arterial and venous system (thickness depends on size and type of vessel)
    • Tunica adventitia (TA)
    • Tunica media (TM)
    • Tunica intima (TI)
  • Capillaries and sinusoids: single endothelial layer with nuclei bulging into the capillary lumen
Terminology
  • Sinusoid - large diameter capillary
Embryogenesis
  • Development of the vascular system begins in the yolk sac of the embryo with the formation of hemangioblasts, which differentiate into hematopoietic cells and angioblasts, a mesoderm derived precursor of endothelial cells (Development 2011;138:4569, Int J Dev Biol 2011;55:419)
  • Vasculogenesis describes the formation of new blood vessels from precursor cells angioblasts
  • Angiogenesis describes the formation of new blood vessels from pre-existing vessels via sprouting, a coordinated expansion of a pre-existing network.
  • Notch signaling pathway plays a central role in all the processes of vascular development
    • Postnatal angiogenesis and ischemic vascularization / endothelial progenitor cells - endothelium transformation
    • Artery formation by upregulating the expression of arterial markers (lack of Notch will result in venous endothelial cells allows the expression of venous markers) (Stem Cells Int 2012;2012:805602)
Physiology
  • Principal functions are:
    • Transportation of gases (O2 and CO2) and metabolites (nutrients and metabolic waste product) to and from tissue cells
    • Temperature regulation
    • Dispersion of hormones
    • Immune system
    • Blood clotting
  • Blood vascular system:
    • Active pumping system (heart)
    • Closed circuit: arterial system - capillary network - venous system
  • Lymph vascular system:
    • Passive system
    • Open one way system: drains the excess extravascular fluid (lymph) to the blood vascular system
Diagrams / tables

Images hosted on other servers:
Arterial wall

Arterial wall

Pericyte and endothelial interactions Pericyte and endothelial interactions

Pericyte and endothelial interactions

Gross description
  • Arterial system
    • Elastic artery: major distribution vessels
      • Aorta, subclavian, iliac and pulmonary
    • Muscular artery: main distributing branches
      • Radial, femoral, renal cerebral arteries
    • Arteriole: terminal branches supplying the capillaries
Microscopic (histologic) description
  • Normal histology:
    • Structural transition is gradual between the different types of vessels
    • General structure of the arterial and venous system (thickness depends on size and type of vessel)
      • Tunica adventitia (TA):
        • Longitudinally arranged collagen
        • Layer is larger in veins than arteries
      • Tunica media (TM):
        • Concentrically arranged smooth muscle
        • Diameter is altered by autonomic control
        • Smooth muscle may secrete collagen, elastic fibers, elastic lamellae and proteoglycans
        • Larger diameter in arteries than veins of similar size
      • Tunica intima (TI):
        • Inner endothelium
        • Variable outer subendothelial connective tissue (SCT) composed of scattered fibroblasts and myointimal cells producing collagen and elastin fibers
          • With age myoepithelial cell can accumulate lipids causing tunica intima thickening
  • Arterial system:
    • Artery types:
      • Elastic artery
      • Muscular artery
      • Arteriole (lumen < 0.3 mm)

Tunica intima Tunica media Tunica adventitia
Elastic artery
  • Thick
  • Broad layers of fenestrated elastic sheets (40 - 50) + collagen layers + smooth muscle
  • Vasa vasorum are present
  • Vasa vasorum also penetrates the outer layer of tunica media
Muscular artery
  • Thick
  • Internal elastic lamina (IEL)
  • Thick layer of circumferential smooth muscle layer
  • External elastic lamina (EEL)
  • No vasa vasorum
Arteriole (large; lumen < 0.3 mm)
  • Thin, with almost no subendothelial connective tissue
  • Thin internal elastic lamina
  • 3 - 6 layers of smooth muscle
  • No external elastic lamina

  • Microcirculation:
    • Direction of blood flow:
      • Arterioles → capillaries → postcapillary venules → small muscular venules
      • AV shunt: direct connection between the arterioles and venules
    • Arteriole (small):
      • Lumen < 0.1 mm)
      • TI with almost no subendothelial connective tissue
      • No internal elastic lamina
      • < 2 layers of smooth muscle
      • No external elastic lamina
    • Capillaries and sinusoids:
      • 3 - 40 micrometers
      • Single endothelial layer with nuclei bulging into the capillary lumen
      • No muscular layer or elastic lamina
      • 3 types:
        • Continuous capillaries: uninterrupted endothelial layer (brain, muscle, lung, connective tissue)
        • Fenestrated capillaries: capillaries with multiple large pores (kidney, endocrine organ)
        • Sinusoid with discontinuous endothelium: disrupted Interface between the tissue and capillary lumen (spleen sinusoid, liver sinusoids)
      • Pericytes:
    • Postcapillary venules - small muscular venules:
      • Series of postcapillary draining vessels with increasing luminal diameter and wall thickness
  • Venous system:
    • Low pressure draining system returning the blood to the heart
    • Blood flow is maintained by:
      • Negative pressure created in the chest cavity / right atrium during inspiration
      • Compression of veins during contraction of skeletal muscle
      • Venous valves
    • Elastic and muscular components of the veins are less prominent
  • Lymphovascular system:
    • Low pressure draining system returning the lymph to the venous system
    • Lymph flow is maintained by
      • Negative pressure created in the chest cavity during inspiration
      • Compression of vessels during contraction of skeletal muscle
      • Lymphovascular valves
    • Lymphoid capillary:
      • Similar size to the regular capillary
      • No pericytes
      • No or minimal basement membrane
      • Greater permeability compared to regular capillary
    • No lymphatic vessels in CNS, cartilage, bone, teeth, thymus, placenta
    • Lymph nodes:
      • Site of antigen presentation, antibody secretion, activated lymphoid cells entry into lymph
  • Glomus body:
    • Encapsulated glomeriform arteriovenous anastomosis (glomus body) richly innervated by autonomic nervous system fibers
    • Located in dermis
    • High number in finger and toes
    • Involved in the thermal regulation
    • Glomus cells:
      • Specialized modified smooth muscle cell with indistinct borders, pale cytoplasm, uniform nuclei; surround arterial segment of glomeriform arteriovenous anastomosis (glomus body)
      • SMA+, MSA+, type IV collagen (outlines basement membrane), desmin variable / +, S100-, CK-
  • Normal aging related changes:
    • Normal vascular structure changes during life, resulting with progressive vascular stiffness
    • Elastic arteries:
      • Increase in diameter
      • Fibrous intimal thickening
      • Fragmentation of elastic laminae
      • Thickening of vasa vasorum walls
    • Muscular arteries:
      • Dilatation and tortuosity
      • Fibrous intimal thickening
      • Fragmentation and calcification internal of elastic laminae
      • Fibrosis and hyalinization of tunica media
    • Arterioles:
      • Fibrous intimal thickening
      • Hyalinization of tunica media
    • Capillaries:
      • Basement membrane thickening
    • Venules and veins:
      • Fibrous intimal thickening
      • Hypertrophy of the muscle bundles
Microscopic (histologic) images

Contributed by Norbert Sule, M.D., Ph.D.

Muscular type artery and vein

Wall of muscular type artery

Wall of muscular type vein

Positive stains
Electron microscopy description
  • Endothelial cells: cells are joined by tight, adherens or gap junctions; numerous pinocytotic vesicles, cytoplasmic microfilaments, microvilli, continuous basal lamina and Weibel-Palade bodies (membrane bound organelle which contains von Willebrand factor)
  • Capillary endothelium: surrounded by basement membrane, numerous supporting pericytes with its own basement membrane
  • Fenestrated capillary endothelium: fenestration (thin electron dense line), surrounded by basement membrane, rare supporting pericytes
  • Lymphatic endothelium: minimal or no little basement membrane, no pericytes
Electron microscopy images

Images hosted on other servers:
Missing Image

Rat brain capillary with endothelial cells and pericytes

Missing Image

Endothelial tight junction

Missing Image

Pericytes

Board review style question #1
Which of the following is part of the arteriolar wall structure?

  1. External elastic lamina (EEL)
  2. Fenestrated elastic sheets
  3. Internal elastic lamina (IEL)
  4. Muscularis propria
Board review style answer #1
C. Internal elastic lamina (IEL). The arteriolar wall is composed of thin tunica intima with endothelial layers and subendothelial connective tissue, tunica media with thin internal elastic lamina (answer C) and smooth muscle and tunica adventitia. External elastic lamina is present only in muscular type arteries and the fenestrated elastic sheets are present in the elastic type arteries. Muscular wall of a luminal visceral organ is called muscularis propria. The muscular wall of a blood vessel is called tunica media.

Comment Here

Reference: Histology-blood vessels
Board review style question #2

The above section shows a portion of the blood vessels stained with elastin stain. Based on the image, this blood vessel represents

  1. Arteriole
  2. Elastic type artery
  3. Large muscular vein
  4. Muscular type artery
  5. Sinusoid
Board review style answer #2
D. Muscular type artery. The depicted portion of this blood vessel shows 2 (internal and external) elastic lamina. Arterioles and large muscular vein have only internal elastic lamina. Elastic type artery has multiple layers of elastic membranes in the tunica media. Muscularis type artery as shown has 2 distinct internal and external elastic laminae. Sinusoid is a type of capillary and has only a single layer of endothelium without tunica media / adventitia or elastic lamina.

Comment Here

Reference: Histology-blood vessels

Histology-brown and white adipose
Definition / general
  • Brown adipose tissue (BAT)
    • Specialized adipose tissue
    • Primary function is body temperature regulation in newborns and hibernating animals
    • Minimal residual brown fat is present in human adults
  • White adipose tissue (WAT)
    • Most tissues contain fat cells (adipocytes) in isolation, in clumps or may constitute a prominent component or main cell type (in adipose tissue)
    • Compromises 20% of body weight in males, 25% in females
    • Primary function is fat / energy storage
    • Additional role in thermal insulation and mechanical cushion
  • Beige / brite adipose tissue (Endocrinology 2013;154:2992)
    • Cells interspersed within white adipose tissue and capable of transformation into brown-like adipocyte after cold exposure or adrenergic stress
    • Multilocular droplets
    • High mitochondrial density
Essential features
  • 3 types of adipose tissue (white, brown and beige)
  • Primary role of white adipose tissue is energy storage
  • Primary role of brown adipose tissue is heat production
Embryogenesis
  • Brown adipose tissue
    • Brown but not white adipocytes are derived from myogenic factor 5 (Myf5) expressing muscle precursor cells under the control of the PRDM16 switch (J Mol Cell Biol 2010;2:23)
    • Protein tyrosine phosphatase 1B is a modulator of brown fat adipogenesis (PLoS One 2011;6:e16446)
  • White adipose tissue
    • Adipocytes are derived from mesenchymal cells
Terminology
  • Brown adipose tissue
    • Also called normal brown fat
  • White adipose tissue
    • Also called normal white fat
    • Cells are called adipocytes
Sites
  • Brown adipose tissue
    • More conspicuous in infants (5% of body weight) and children; replaced with white fat over time
    • Persists in adults in interscapular region, neck, mediastinum, axilla, retroperitoneum (perirenal)
    • Substantial amounts of metabolically active brown fat are present in healthy adults (N Engl J Med 2009;360:1518)
  • White adipose tissue
    • Subcutaneous, mediastinum, abdomen, retroperitoneum, perirenal
Physiology
  • Brown adipose tissue
    • Main function is heat production (nonshivering thermogenesis)
    • Cold exposure leads to sympathetic stimulation of brown fat via norepinephrine binding to beta adrenergic receptors, then oxidation of fatty acids and heat production
    • Brown fat mitochondria express uncoupling protein (UCP1), which uncouples electron transport from the phosphorylation ADP to generate ATP, deriving from the oxidation of lipids
    • Generated energy is released as heat and conducted to the rest of the body through a dense vascular network of tissue (Physiol Rev 2004;84:277)
    • Also important in animals coming out of hibernation, allowing them to rewarm quickly
  • White adipose tissue
    • Triglycerides circulate in blood as chylomicrons; the enzyme lipoprotein lipase, produced by adipocytes and present on luminal surface of endothelium, converts triglycerides to free fatty acids; free fatty acids are taken up by adipocytes and converted to triacylglycerol, which is stored within cytoplasmic lipid droplet of adipocytes
    • Main function is energy storage as lipid from 3 sources:
      • Dietary fat
      • Triglycerides synthetized by liver
      • Triglycerides synthetized from glucose by adipocytes
    • Additional functions:
      • Endocrine (adipocyte derived peptide)
Pathophysiology
  • Brown adipose tissue
  • White adipose tissue
    • Stored lipid is mobilized to other tissues depending on metabolic needs, influenced by hormones and sympathetic nervous system
    • Adipocytes also convert androstenedione to estrone, the main source of estrogen in men and postmenopausal women
    • White adipose tissue is heterogeneous, different classes of adipocytes have differing metabolism and ability to communicate with other tissues by secretion of peptides, lipids and miRNAs, which affect systemic metabolism differently (J Clin Invest 2019;129:3990)
Clinical features
  • Brown adipose tissue
    • Brown fat thermogenesis is visible with a thermal (infrared) camera in infants over neck and interscapular area
  • White adipose tissue
    • Obesity: increased white adipose tissue mass
    • Depot specific differences of white adipose tissue are due to the different gene expression profile:

      Subcutaneous white adipose tissue Visceral white adipose tissue
      Primary role: energy storage Primary role: energy storage
      Insulin sensitive Insulin resistant
      Low lipolysis High lipolysis

    • Increased body fat is a major contributing factor in insulin resistance and the pathogenesis of type 2 diabetes (J Clin Invest 2019;129:3990)
    • Lipodystrophy (complete or partial loss of body fat)
Radiology images

Images hosted on other servers:
Sites of brown fat

Sites of brown fat

Gross description
Gross images

Contributed by Norbert Sule, M.D., Ph.D.
Perirenal and sinusoidal adipose tissue

Perirenal and sinusoidal adipose tissue

Subcutaneous fibroadipose tissue

Subcutaneous fibroadipose tissue

Microscopic (histologic) description
  • Brown adipose tissue
    • Lobules of adipocytes separated by fibrous septa
    • Adipocytes are smaller (25 - 40 microns) than in white fat and are polygonal, with eccentrically located nucleus displaying fine indentations
    • Multivacuolated cytoplasm results from numerous small lipid droplets
    • Cytoplasm is acidophilic and granular due to large number of mitochondria, multivacuolated and granular cytoplasm and central spherical nucleus
    • Fibrous septae contain capillaries and nerves
  • White adipose tissue
    • Uniform large spherical cells up to 120 microns
    • Small lipid droplets fuse and form a single large intracytoplasmic droplet
    • Nucleus is pushed and compressed to periphery of the cell by the lipid droplet, resulting in a thin crescent shape
    • Adipocytes are clear on a routine H&E stained section because cytoplasmic fat dissolves during tissue processing
    • Nucleus may have central vacuole or Lochkern (derived from German - loch: hole; kern: nucleus) (J Oral Maxillofac Pathol 2016;20:339)
    • Very thin membranes are present between cells
Microscopic (histologic) images

Contributed by Norbert Sule, M.D., Ph.D.
White adipose: Lochkern in nuceli

White adipose: Lochkern in nuceli

Brown fat

Brown fat

Positive stains
Electron microscopy description
  • Brown adipose tissue
    • Numerous mitochondria
    • Numerous smaller lipid droplets
  • White adipose tissue
    • Preadipocytes: spindle cells with abundant endoplasmic reticulum and small spherical mitochondria
    • Mature adipocyte:
      • Large irregular central lipid droplet with multiple minute additional droplets at the periphery, representing ongoing fusion with the main fat droplet
      • No membrane is present around the droplets
      • Thin rim of residual cytoplasm contains cellular organelles, such as mitochondria and endoplasmic reticulum
      • Nucleus is flattened against cytoplasmic membrane
  • References: Young: Wheater's Functional Histology, 6th Edition, 2013, Mills: Histology for Pathologists, 3rd Edition, 2007
Board review style question #1
What is the approximate size of a white fat cell?

  1. 0.1 micrometers
  2. 1 micrometer
  3. 10 micrometers
  4. 100 micrometers
Board review style answer #1
D. 100 micrometers. White adipose tissue cells are one of the largest cells in the body, measuring approximately 100 micrometers.

Comment Here

Reference: Histology-brown and white adipose
Board review style question #2

Which of the following is a characteristic feature of brown fat tissue?

  1. Largest fat depot in the body
  2. Multilocular vacuolization in the cytoplasm
  3. Nucleus is pushed and compressed against the cytoplasmic membrane
  4. Primary role is energy storage
Board review style answer #2
B. Multilocular vacuolization in the cytoplasm. Brown fat represents only 2 - 3% of body fat. The stored fat is present in multiple small droplets within the cytoplasm. The nucleus maintains its round shape and is not compressed against the cytoplasmic membrane. It has a primary role of thermoregulation.

Comment Here

Reference: Histology-brown and white adipose

Histology-fibrous tissue & nerve
Definition / general
  • Fibrous tissue consists of fibroblasts and extracellular matrix
  • Extracellular matrix consists of collagen, elastin and ground substance

Fibrous tissue:
  • Loose or dense
  • Dense fibrous tissue includes tendons (connect muscle to bone), ligaments (connect bones or cartilage to each other), aponeuroses (ribbon-like tendinous expansion)

Fibroblasts:
  • Spindled (along collagen fibers) to stellate (star shaped in myxoid areas)
  • Produce various collagens
  • Positive for vimentin, actin

Fibrocytes:
  • Quiescent stage of fibroblasts

Myofibroblasts:
  • Modified fibroblasts with multiple possible origins (see diagram below), including transition from fibroblasts during tissue repair (J Invest Dermatol 2007;127:526)
  • Features are intermediate between fibroblasts and smooth muscle cells
Nerve
  • Composed of axons, Schwann cells, perineurial cells and fibroblasts in epineurium (outer sheath)
  • Perineurium: surrounds each nerve fascicle, is continuous with pia mater of CNS
  • Perineurial cells: derived from fibroblasts; EMA+, S100-
  • Schwann cells: neuroectodermally derived cells that resemble fibroblasts but strongly S100+, intimately related to axons (by EM), have continuous basal lamina that coats the cell facing the endoneurium
Diagrams / tables

Images hosted on other servers:

Myofibroblast progenitors

Microscopic (histologic) images

AFIP images

Myofibroma: interface
of myofibroblastic
tumor with
normal collagen



Images hosted on other servers:

NIH / 3T3 mouse fibroblasts in cell culture

Extracellular matrix compliance
controls development of the
myofibroblast phenotype in
3 dimensional collagen gels


Histology-skeletal muscle
Definition / general
  • Specialized contractile tissue responsible for the voluntary movements
Essential features
  • Contraction of skeletal muscle is controlled by large motor nerve fibers of the alpha neuron located in the anterior horn of the spinal cord and motor nuclei of the brain
  • Neurotransmitter of the neuromuscular junction is acetylcholine, which binds to the nicotinic receptors
  • Skeletal muscle fiber is an elongated cylindrical, unbranched, multinucleated contractile unit formed by fusion of single cells resulting in multinucleated syncytia
  • Sarcoplasm is filled with myofibrils, a protein structure composed of contractile proteins and organized in a repeating pattern resulting in the light microscopic and electron microscopic appearance of cross striation
  • Biochemically, there are 2 types of fibers: type I (aerobic, slow, red) and type II (anaerobic, fast, white)
Embryogenesis
  • Paraxial mesoderm / somite → dermatomyotome → myoblast → myotubes → muscle fiber
  • Normal skeletal muscle is formed from the primitive mesenchymal element of the paraxial mesoderm (somite) which divides into ventral dorsolateral somite or dermatomyotome
  • Mesenchymal cells of the dermatome differentiate into dermal fibroblast and myotomes differentiate into myoblast (portion of skin and group of skeletal muscles derived from the same segment of dermatomyotome is supplied by a specific segmental spinal nerve)
  • Myoblast proliferation and fusion results in formation of multinucleated cells called myotubes
  • After formation of myotubes, synthesis of fibrillary protein starts in the center of the cells and causes peripheral displacement of the nuclei
  • This process is mostly complete at the time of birth
  • Only a few residual satellite myoblasts are present in the mature muscle that are able to play a role in the repair and regeneration
  • Reference: J Anat 2015;227:673
Terminology
  • Synonyms: striated muscle, voluntary muscle
  • Its basic cellular unit is the muscle fiber (also called myofibers or myocytes)
  • Group of muscle fibers controlled by the same nerve fibers is called the motor unit
    • Excitation of the nerve fiber results in simultaneous contraction of all the muscle fibers in the same motor unit
  • Structural organization of skeletal muscle:
    • Muscle fiber = cellular unit (enwrapped by endomysium)
    • Fasciculi = bundle of muscle fibers (enwrapped by perimysium)
    • Muscle = collection of fasciculi (enwrapped by epimysium)
  • Special terminology to describe the highly specialized cellular components of muscle fibers:
    • Plasma membrane = sarcolemma / plasmalemma
    • Endoplasmic reticulum = sarcoplasmic reticulum
    • Mitochondria = sarcosomes
  • Specialized intramuscular stretch receptors are called neuromuscular spindles
Physiology
  • Contraction of skeletal muscle is controlled by large motor nerve fibers of the alpha neuron located in the anterior horn of the spinal cord and motor nuclei of the brain
  • Intercellular junction between the nerve ending and the muscle fiber is called the neuromuscular junction
  • Stimulation by the motor nerve causes simultaneous contraction
  • Neurotransmitter of the neuromuscular junction is acetylcholine, which binds to the nicotinic receptors
  • Excitation of the muscle cell membrane results in the release of Ca2+ from the sarcoplasmic reticulum
  • Ca2+ binds to troponin tropomyosin complex, unmasking the cross bridge site on the actin molecules
  • Contraction is the result of a sliding mechanism of the thick (myosin) and thin (actin) fibers
  • ATP hydrolysis activates sites on the myosin fibers connecting to actin molecules and forming cross bridges
    • This connection causes configurational changes of the myosin fiber, shortening the sarcomere and resulting in movement of the muscle
Diagrams / tables

Images hosted on other servers:

Organization from
gross muscle fibers to
contractile filaments

T tubules and sarcoplasmic reticulum

Microscopic (histologic) description
  • Skeletal muscle
    • Organization of skeletal muscle:
      • Muscle fiber enwrapped by endomysium, loose connective tissue with reticulin and collagen fibers
      • Fasciculi enwrapped by perimysium, loose connective tissue with elastin and collagen fibers
      • Muscle enwrapped by epimysium, dense collagenous sheath
  • Muscle fiber
    • Elongated cylindrical, unbranched, multinucleated contractile unit formed by fusion of single cells resulting in multinucleated syncytia
    • Size: diameter varies 10 - 100 micrometers; length: up to 35 cm
    • Flattened nuclei are pushed to periphery of the muscle fiber (just underneath the sarcolemma)
    • Sarcoplasm is filled with myofibrils, a protein structure composed of contractile proteins and organized in a repeating pattern resulting in the light microscopic and electron microscopic appearance of cross striation
    • Myofibrils composed of thin (actin) and thick (myosin) filaments
    • Biochemically there are 2 types of fibers
      • Type I (aerobic, slow, red):
        • Small cross section
        • Abundant mitochondria
        • Large amount of myoglobin (oxygen storage)
      • Type II (anaerobic, fast, white): intense, short, rapid but sporadic movements
        • Large cross section
        • Few mitochondria
        • Little myoglobin
        • Rich in glycogen and glycolytic enzymes
  • Neuromuscular spindles
    • Encapsulated, cigar shaped structures (~6 micrometer) embedded in endomysium and perimysium
    • Composed of modified muscle fibers (intrafusal fibers) and associated sensory myelinated and nonmyelinated nerve endings, positioned parallel to the muscle fibers
  • Reference: Ageing Res Rev 2018;47:123
Microscopic (histologic) images

Contributed by Norbert Sule, M.D., Ph.D.
Cross striation

Cross striation

Masson trichrome

Masson trichrome

Special and histochemical stains
  • Masson trichrome: epi, peri and endomysium
  • Iron hematoxylin: cross striation
  • PAS - glycogen content:
    • Type 1 / red fibers: lighter
    • Type 2 / white fibers: darker
  • Histochemical stains (to reveal the metabolic properties of red and white fibers):
    • Succinate dehydrogenase - Kreb cycle enzyme:
      • Type 1 / red fibers: darker
      • Type 2 / white fibers: lighter
    • ATPase pH 4.3 - pattern depends on pH:
      • Type 1 / red fibers: darker
      • Type 2 / white fibers: lighter
    • ATPase pH 9.4 - pattern depends on pH:
      • Type 1 / red fibers: lighter
      • Type 2 / white fibers: darker
Negative stains
Electron microscopy description
  • Electron microscopy reveals structural components (Curr Diagn Pathol 2002;8:225):
    • I (isotropic) band: thin filaments only
    • A (anisotropic) band: overlapping thin and thick filaments
    • H band: thick filaments only
    • Z line: divides center of I band; serves as attachment site for the sarcomere, the repeating individual unit of the muscle fiber
    • T tubule:
      • Tubular plasma membrane system / extension at the level of A and I band junction surrounding the myofibril
      • Lumen is continuous with the extracellular space
    • Sarcoplasmic reticulum:
      • Smooth endoplasmic reticulum derived membrane system located between the T tubules with sac-like dilatation called terminal cisternae
      • Calcium ion concentrated in the lumens
    • Triad: a complex composed of T tubule and 2 terminal cisternae
Electron microscopy images

Images hosted on other servers:

Contractile filament within muscle fiber resulting in band-like arrangement

Board review style question #1

Which of the following statements regarding the connective tissue of skeletal muscle is correct?

  1. Endomysium consists of fine reticular fibers surrounding the muscle fiber
  2. Epimysium separates each muscle fascicle from the others
  3. Perimysium envelopes the whole skeletal muscle
  4. Perimysium enwraps the individual muscle fibers within a fascicle
Board review style answer #1
A. Endomysium consists of fine reticular fibers surrounding the muscle fiber. Skeletal muscle is enwrapped in a dense collagenous sheath called epimysium. The muscle represents a collection of muscle fasciculi separated by perimysium, which is a loose connective tissue of elastin and collagen fibers. The individual skeletal muscle fiber enwrapped by loose connective tissue is called endomysium, which is composed of reticulin and collagen fibers.

Comment Here

Reference: Histology - skeletal muscle
Board review style question #2


A 66 year old woman undergoes resection of a thigh mass composed of large pleomorphic epithelioid cells with eosinophilic cytoplasm, necrosis and increased mitotic activity. The cells are diffusely positive for vimentin, desmin and MyoD1 (subset) and negative for CD45, S100, MelanA, pancytokeratin and MDM2. What differentiation do the pleomorphic cells show?

  1. Smooth muscle
  2. Adipocytic
  3. Epithelial
  4. Skeletal muscle
Board review style answer #2
D. Skeletal muscle. The histology describes a pleomorphic epithelioid cell neoplasm. Based on the morphology alone, no definite differentiation is evident. Considering the age, the differential diagnosis is wide and includes dedifferentiated liposarcoma, pleomorphic leiomyosarcoma, melanoma, malignant peripheral nerve sheath tumor and rhabdomyosarcoma. The performed immunohistochemical stains revealed positive reaction with vimentin and desmin. Considering the morphology and the immunohistochemical profile, the findings are most consistent with muscular differentiation. The positive reaction of MyoD1 is compatible skeletal muscle differentiation (vimentin+, desmin+ and MyoD1+; pleomorphic rhabdomyosarcoma) and rules out smooth muscle differentiation (vimentin+, desmin+ and MyoD1-; pleomorphic leiomyosarcoma). The lack of expression of S100 and MDM2 rules out melanocytic (S100+; melanoma), adipocytic (S100+, MDM2+; pleomorphic liposarcoma) and neural differentiation (S100+ focal; malignant peripheral nerve sheath tumor). The negativity with CD45 and pancytokeratin excludes leukocytic and epithelial differentiation, respectively.

Comment Here

Reference: Histology - skeletal muscle

Histology-smooth muscle
Definition / general
  • Nonstriated muscle that serves diverse functions throughout the body and is responsible for involuntary movements
  • 1 of 3 types of muscle tissue alongside cardiac and skeletal muscle
Essential features
  • Smooth muscle is widely distributed throughout the body and serves diverse functions
  • Physiologically divided into single unit and multi unit fibers
  • Intracellular actin and myosin filaments generate contractile forces by a sliding filament mechanism
  • Contractile filaments are not arranged into sarcomeres, thus giving it a nonstriated (smooth) appearance
  • Awareness of the distribution of smooth muscle helps avoid microscopic misinterpretation
Terminology
  • Synonyms: nonstriated muscle, involuntary muscle
  • Smooth pertains to the lack of striations
  • Muscle fiber is the basic functional unit (also known as myocyte, myofiber, muscle cell)
  • Myofilaments are the contractile apparatus of smooth muscle cells consisting of thick myosin filaments and thin actin filaments along with regulatory proteins
Embryogenesis
Functional anatomy
  • Smooth musculature is ubiquitous in viscera and blood vessels
  • At certain sites, smooth and skeletal muscles come into contact coordinating the contractile force and direction with each other (e.g., esophagus, rectum and pelvic floor) (Anat Sci Int 2023;98:407)
  • Muscle cell spatial arrangements (J Smooth Muscle Res 2021;57:19)
    • Circumferential ring arrangement (e.g., arterioles)
    • Less circumferential and not as tightly packed bundles (e.g., ureter, bile duct)
    • Orthogonal layers (e.g., intestine, vas deferens, vessels)
    • Cord-like (e.g., teniae coli)
    • Bundles running seemingly in all directions (e.g., urinary bladder, myometrium)
    • Laminar sheet (e.g., trachea)
    • Short straight bundles (e.g., arrector pili)
  • Contractile properties serve diverse functions based on
    • Dimensions of the muscle fibers
    • Spatial arrangement of the fibers (above)
    • Types of stimuli
    • Characteristics of innervation
  • Types of intracellular filaments
    • Thin actin, contractile function
    • Thick myosin, contractile function
    • Intermediate desmin and vimentin, maintain structure
  • Visceral smooth muscle: gamma smooth muscle actin and desmin predominate
  • Vascular smooth muscle: abundant alpha smooth muscle actin, vimentin > desmin (Proc Natl Acad Sci U S A 1981;78:298)

Skin
Arrector pili muscles
  • Located in the dermis inserting into hair follicles below sebaceous glands causing piloerection
  • Mediate thermoregulation and contribute to follicular unit integrity and sebum secretion (Int J Trichology 2014;6:88)
Nipple
  • Smooth muscle bundles present in areola and around lactiferous ducts
  • For nipple erection and milk expression
Scrotum
  • Tunica dartos in subdermal layer
  • Assists in scrotal thermoregulation
Vulva

Eye
Iris
  • Smooth muscle fibers function to regulate the pupil diameter controlling the amount of light entering the lens
Ciliary body
  • Ciliary muscle helps to control the shape of the lens (accommodation)
Müller muscle of eyelid
  • Also known as superior tarsal muscle; small bundles of smooth muscle within the upper eyelid adjoining the levator palpebrae superioris to help keep the eyes open (Ann Plast Surg 1985;14:324)
Orbit

Cardiovascular
Elastic arteries
  • Tunica media: more elastic fibers and less smooth muscle relative to muscular arteries
  • Conducting blood and withstanding the high pressure of blood ejected from the heart (e.g., aorta, pulmonary arteries)
Muscular arteries
  • Tunica media: less elastic fibers and more smooth muscle relative to elastic arteries
  • Constrict and dilate as needed to change the amount of blood delivered (e.g., radial artery, femoral artery)
Arterioles
  • Composed chiefly of smooth muscle; critical in maintaining systemic vascular resistance and determining blood pressure
Capillaries
  • Composed of a single endothelial layer and a pericytic layer (see below) allowing for diffusion of nutrients and oxygen
Venules
  • Only 1 or 2 layers of muscle also allow for diffusion
Veins
  • Less smooth muscle and connective tissue than arteries making their walls thinner and allowing them to accommodate a large volume of blood at relatively low pressures
Capillary pericytes
Lymphatics
  • Larger lymphatics have thin wisps of muscular media
Endocardium

Lung
Conducting airways
  • Smooth muscle present in the walls of the airways from trachea to terminal bronchioles, becoming progressively less prominent distally, serving to regulate the bronchomotor tone of the airways (Int J Biochem Cell Biol 2003;35:272)
  • In the rigid trachea, the muscle may have a passive role, completing the circumference of the tube (J Smooth Muscle Res 2021;57:19)
Respiratory airways
  • Bundles of smooth muscle can be seen at the level of the alveolar ducts, interrupted by alveolar openings, which may appear as isolated round or oblong aggregates in cross section

Gastrointestinal
Muscularis mucosae
  • Thin muscular layer between mucosa and submucosa
  • Layers: inner circular and outer longitudinal (inconspicuous on H&E) except in the esophagus where it is only longitudinal
  • Thickest in distal esophagus
  • In the rectum it extends into the upper transition zone of the anal canal
  • Provides a structural foundation for the mucosa and keeps the glands in constant motion to expel their contents
Muscularis propria (externa)
  • Thick muscular layer between submucosa and subserosa / serosa
  • Layers: inner circular and outer longitudinal except in the stomach where there is an additional innermost oblique layer
  • In the esophagus: upper part is entirely skeletal muscle, middle part is a mixture of skeletal and smooth, lower part is entirely smooth
  • Contains the myenteric plexus of Auerbach and interstitial cells of Cajal responsible for peristaltic movement
  • Teniae coli are 3 longitudinal smooth muscle bands in the colonic wall
Anal canal
  • Internal anal sphincter is a thickened extension of the circular smooth muscle in the bowel wall
Pelvic floor and perineum
  • Perineal smooth muscle: the rectourethralis in men and the rectovaginalis in women are considered homologous structures contributing to the stabilization of the pelvic floor (Anat Sci Int 2023;98:407)

Biliary system
Gallbladder
  • Muscularis propria: loosely arranged bundles of circular, longitudinal and oblique fibers, lying directly beneath the lamina propria (no muscularis mucosae or submucosa)
Extrahepatic biliary tract
Sphincter of Oddi
  • Concentric ring of muscle regulating the flow of bile and pancreatic secretions into the duodenum and preventing reflux of duodenal contents

Genitourinary
Bladder
  • Muscularis mucosae: discontinuous wisps of smooth muscle
  • Muscularis propria (detrusor muscle): 3 coats with interwoven orientation, inner and outer longitudinal and a central circular, which seem to be randomly arranged
  • Detrusor contributes to micturition and allows the bladder to stretch for the storage of urine
Ureter
  • Muscularis propria with indistinct layers
Renal pelvis
  • Muscularis propria becomes thinner along the major and minor calyces
Male ducts & seminal vesicles
  • Efferent ductules, epididymis and vas deferens all contain smooth muscle which function to propel sperm and fluids
  • Thickest in vas deferens (inner and outer longitudinal layers and middle circular), which transports spermatozoa from epididymis to ejaculatory duct
  • Seminal vesicles are also surrounded by a prominent layer of smooth muscle
Penis
  • Corpora cavernosa: main erectile tissues of the penis consisting of anastomosing vascular network with intervening smooth muscle and fibrous tissue
  • Corpus spongiosum surrounds the penile urethra with less smooth muscle than that of the cavernosa
  • Penile dartos surrounds the penile shaft and is continuous with the scrotal dartos producing retraction of genital structures when exterior temperature falls
Prostate
  • Anterior fibromuscular stroma constitutes the anterior nonglandular portion of the prostatic surgical capsule, composed of vertically oriented smooth muscle continuous with the bladder detrusor muscle, may play a role in maintaining urethral resistance (Urology 2010;76:511.e10)
  • Prostate itself has abundant fibromuscular stroma
Urethra
  • Internal urethral sphincter: collar of smooth muscle at the neck of the bladder continuous with the detrusor muscle of bladder
  • Maintains urinary continence and prevents reflux of semen into the bladder

Female reproductive
Uterus
  • Smooth muscle within the corpus is more concentrated than the muscle in either the cervix or the lower uterine segment
  • This distribution of muscle is consistent with the passive role of the cervix during parturition
Vagina
  • Smooth muscle musculature is continuous with that of the uterus
Fallopian tube
  • Smooth muscle is thickest in the isthmus and thins as the fimbriated end is approached
  • Remainder of tube wall contains 2 layers of smooth muscle
Ovary
  • Occasional smooth muscle may be identified within the stroma

Physiology
  • 2 major types of smooth muscle
    • Single unit
      • Hundreds to thousands of muscle fibers contract together as a single unit
      • Action potential spreads through gap junctions between fibers
      • Controlled by nervous and nonnervous stimuli
      • Examples: walls of viscera such as gastrointestinal tract, blood vessels
    • Multi unit
      • Each fiber is innervated by a single nerve ending and operates independently for much finer control
      • Controlled by nervous stimuli
      • Examples: ciliary muscle of the eye, arrector pili muscles
  • Types of stimuli
    • Nervous: autonomic
    • Nonnervous: hormones, local tissue chemical factors, stretch of the fibers (mechanical)
  • Mechanism of contraction
    • Initiated by an increase in intracellular Ca2+ ions upon stimulation (as in skeletal muscle)
    • Mediated by actin myosin filament interaction (as in skeletal muscle)
    • Does not involve troponin (unlike skeletal muscle)
    • Source of Ca2+ ions
      • Entry through channels in the caveolae of the cell membrane
      • Release of sequestered Ca2+ from the sarcoplasmic (endoplasmic) reticulum
    • Ca2+ binds to calmodulin (regulatory protein in place of troponin)
      • Calmodulin calcium complex → activation of myosin light chain kinase (MLCK) → phosphorylation of myosin light chain → myosin actin interaction → contraction
  • Some differences from skeletal muscle
    • Involuntary contractions
    • Prolonged and slower yet stronger contractions
    • Less energy consumption
    • Nervous and nonnervous stimuli
  • References: Hall: Guyton and Hall Textbook of Medical Physiology, 13th Edition, 2015, StatPearls: Physiology, Smooth Muscle [Accessed 3 July 2023]
Diagrams / tables

Contributed by Rola H. Ali, M.D.
Smooth muscle fiber structure

Smooth muscle fiber structure

Contractile proteins relevant to IHC

Contractile proteins relevant to IHC

Microscopic (histologic) description
  • Cell size: diameter between 1 - 5 μm and length between 20 - 500 μm (much smaller than skeletal muscle)
  • Cell shape: spindle shaped or fusiform with tapered ends
  • Cytoplasm: eosinophilic fibrillar with distinct cell borders and no cross striations
  • Nucleus
    • Central elongated cigar shaped nucleus with blunted ends
    • Spiral corkscrew appearance in the contracted state
    • Appear round on cross section
  • Spindle cell mimics
    • Fibroblasts in dense regular connective tissue
    • Myofibroblasts: spindle to stellate, amphophilic cytoplasm, single elongated nucleus with 1 or 2 small nucleoli, collagen in between the cells
    • Schwann cells: cells show slightly undulated buckled nuclei, often with 1 blunt and 1 pointed end, described as S shaped, serpentine, comma shaped, bullet shaped or boomerang-like
  • Reference: Young: Wheater's Functional Histology, 6th Edition, 2013
Microscopic (histologic) images

Contributed by Rola H. Ali, M.D.
Relaxed smooth muscle fibers

Relaxed smooth muscle fibers

Contracted smooth muscle

Contracted smooth muscle

Spiral corkscrew nuclei

Spiral corkscrew nuclei

Cross versus longitudinal sections

Cross versus longitudinal sections


Smooth versus skeletal muscle

Smooth versus skeletal muscle

Smooth muscle versus nerve

Smooth muscle versus nerve

Myofibroblasts for comparison

Myofibroblasts for comparison

Virtual slides

Images hosted on other servers:
Gallbladder muscularis beneath lamina

Gallbladder muscularis beneath lamina

Juxtaposition of smooth and skeletal esophagus

Juxtaposition of smooth and skeletal esophagus

Positive stains
Negative stains
Electron microscopy description
  • Cytoplasm
    • Filaments
      • Thin (actin) and thick (myosin) filaments
      • Intermediate filaments
    • Dense bodies
      • Electron opaque structures floating freely within the cytoplasm → analogous to Z lines of striated muscle anchoring actin and myosin
    • Organelles
      • Include sarcoplasmic reticulum, mitochondria and Golgi apparatus
  • Cell membrane
    • Membrane bound dense bodies
    • Pinocytotic vesicles or caveolae → Ca2+ movement across the membrane
    • Gap junctions → consist of connexins facilitating the transfer of metabolites, ions and signaling molecules
  • References: Edwin: Methods in Pharmacology - Smooth Muscle, 1st Edition, 1975, Motta: Ultrastructure of Smooth Muscle, 1st Edition, 1990
Electron microscopy images

Images hosted on other servers:
Smooth muscle longitudinal section

Smooth muscle longitudinal section

Smooth muscle cross section

Smooth muscle cross section

Wall of muscular artery

Wall of muscular artery

Intestinal muscularis propria (externa)

Intestinal muscularis propria (externa)

Smooth muscle cross section

Smooth muscle cross section

Videos

Smooth muscle versus myofibroblasts

Board review style question #1

Which immunohistochemical expression pattern would be expected in the tissue shown below?

  1. Cytoplasmic staining with h-caldesmon
  2. Cytoplasmic staining with myogenin
  3. Membranous staining with smooth muscle actin
  4. Nuclear and cytoplasmic staining with S100
  5. Nuclear staining with MyoD1
Board review style answer #1
A. Cytoplasmic staining with h-caldesmon. Smooth muscle cells contain cytoplasmic caldesmon molecules bound to actin filaments. Answers B and E are incorrect because myogenin and MyoD1 both encode nuclear proteins expressed in skeletal muscle. Answer C is incorrect because membranous staining with smooth muscle actin is seen in myofibroblasts. Answer D is incorrect because S100 staining is seen in Schwann cells.

Comment Here

Reference: Histology-smooth muscle
Board review style question #2
Which of the following statements pertaining to the physiology of smooth muscle contraction is true?

  1. Actin filaments pull the Z lines inward in the contracted state
  2. Ca2+ binding to calmodulin stimulates myosin actin interaction
  3. Consumption of tremendous amounts of ATP is required
  4. Contraction is stimulated by somatic motor neurons
  5. Strong affinity of troponin for Ca2+ ions initiates contractions
Board review style answer #2
B. Ca2+ binding to calmodulin stimulates myosin actin interaction. In smooth muscle, calmodulin acts in place of troponin. Answers A and C - E are incorrect because these statements are true for skeletal muscle.

Comment Here

Reference: Histology-smooth muscle

Hybrid nerve sheath tumor (pending)
Table of Contents
Definition / general
Definition / general
[Pending]

Inclusion body fibromatosis
Definition / general
  • Dermal fibroblastic and myofibroblastic lesion with cytoplasmic eosinophilic inclusions, usually in digits of infants
  • Also called infantile digital fibromatosis, infantile digital fibroma (J Hand Surg Am 1995;20:1014)
  • Distinct lesion from classic fibromatosis (Am J Surg Pathol 2009;33:1)
Sites
Clinical features
  • Rare; lesions usually present at birth or in first 2 years
  • Similar lesions in adults
  • Often are multiple
Case reports
Treatment
Clinical images

Contributed by Mark R. Wick, M.D.


Images hosted on other servers:

Various images

Gross description
  • Nodules with stretched overlying skin, lesions are ill defined, white-tan, usually 2 cm or less
  • No hemorrhage or necrosis
Microscopic (histologic) description
  • Nonencapsulated, dermal proliferation of hypocellular sheets or fascicles of fibroblasts and myofibroblasts with variable collagen
  • Some spindle cells have peculiar eosinophilic (hyaline) cytoplasmic inclusions the size of a lymphocyte nucleus
  • Usually mitotic figures
  • May infiltrate into adjacent tissue
  • No atypia
Microscopic (histologic) images

Contributed by Mark R. Wick, M.D. and AFIP

Proliferation

Fibroblastic cells

Cells are bland and monomorphic

Inclusions resemble red blood cells


Cytoplasmic inclusions

Various images



Images hosted on other servers:

Various images

Cytology description
Positive stains
Negative stains
Electron microscopy description
  • Spindle cells are myofibroblasts with rough endoplasmic reticulum and free lying inclusions composed of compact masses of actin granules and filaments without a limiting membrane (Am J Pathol 1979;94:19)
Differential diagnosis

Inflammatory leiomyosarcoma
Definition / general
  • Inflammatory leiomyosarcoma (ILMS) is a malignant neoplasm showing smooth muscle differentiation, a prominent inflammatory infiltrate and near haploidization
Essential features
Terminology
  • Related terminology: these tumors have significant pathologic and genetic overlap with the recently described inflammatory rhabdomyoblastic tumor (IRT) and low grade inflammatory myogenic tumor (LGIMT), suggesting that ILMS and IRT / LGIMT may belong to 1 entity (Mod Pathol 2021;34:758, Virchows Arch 2020;477:219).
ICD coding
  • ICD-O: 8890/3 - leiomyosarcoma, NOS
  • ICD-10: C49.9 - malignant neoplasm of other connective and soft tissue, unspecified
  • ICD-11: 2B58 & XH7ED4 - leiomyosarcoma, primary site & leiomyosarcoma, NOS
Epidemiology
Sites
  • Deep soft tissue of the extremities and trunk
  • Neck, abdominal cavity and retroperitoneum are less frequent locations
Pathophysiology
  • Near haploidization and NF1 mutations are the 2 most common defining genetic abnormalities
  • May progress to high grade rhabdomyosarcoma in a small subset (Mod Pathol 2021;34:1035)
Etiology
  • Unknown at this time
Clinical features
  • Commonly presents as an enlarging soft tissue mass with indolent behavior
  • In 2 reported cases, the tumor was associated with paraneoplastic inflammatory syndrome (Int J Surg Pathol 2005;13:185)
  • When rhabdomyosarcomatous transformation occurs, pulmonary metastases are frequently present at the time of diagnosis (Mod Pathol 2023;36:100131)
Diagnosis
  • Clinical presentation: enlarging soft tissue mass
  • Imaging: computed tomography (CT), magnetic resonance imaging (MRI)
Radiology description
Radiology images

Contributed by Diego M. Montoya-Cerrillo, M.D.
Xray

Xray

Ultrasound

Ultrasound

Well circumscribed soft tissue mass Well circumscribed soft tissue mass

Well circumscribed soft tissue mass

Prognostic factors
Case reports
Treatment
  • Wide local excision with negative surgical margins
  • Radiation and chemotherapy are reserved to cases where rhabdomyosarcoma is present
Gross description
Gross images

Contributed by Andrew E. Rosenberg, M.D.
Upper arm tumor

Upper arm tumor

Microscopic (histologic) description
  • Well circumscribed tumor surrounded by a fibrous pseudocapsule
  • Calcifications (calcospherites), lymphoid aggregates, hyalinized blood vessels and hemosiderin deposits are commonly present
  • Spindle and epithelioid tumor cells arranged in sheets and fascicles with eosinophilic cytoplasm, vesicular nuclei and conspicuous nucleoli
  • Prominent inflammatory infiltrate of foamy histiocytes and variable lymphocytes, plasma cells or Touton type giant cells
  • Degree of nuclear atypia and pleomorphism varies, as does mitotic count, depending on the histologic grade (Mod Pathol 2021;34:758)
Microscopic (histologic) images

Contributed by Diego M. Montoya-Cerrillo, M.D. and Elizabeth A. Montgomery, M.D.
Fibrous pseudocapsule

Fibrous pseudocapsule

Prominent inflammation and thickened blood vessels

Prominent inflammation and thickened blood vessels

Prominent nucleoli

Prominent nucleoli

Pleomorphic cells

Pleomorphic cells


Foamy macrophages

Foamy macrophages

Desmin

Desmin

MyoD1

MyoD1

Myogenin

Myogenin

Positive stains
Negative stains
Electron microscopy description
  • Lack of filament arrangement pattern characteristic of smooth muscle cells with varying amounts of stromal collagen fibers (Int J Surg Pathol 2005;13:185)
Molecular / cytogenetics description
Molecular / cytogenetics description

Images hosted on other servers:
Loss of heterozygosity

Loss of heterozygosity

Sample pathology report
  • Soft tissue tumor, left thigh, biopsy / resection:
    • Inflammatory leiomyosarcoma (see comment and synoptic report)
    • Comment: There is moderate cytological atypia, limited mitotic activity (x mitoses per 10 HPF) and no foci of necrosis. Immunohistochemistry shows that the tumor cells are positive for desmin, MyoD1 and focally positive for myogenin and SMA. CD68 / CD163 highlights intratumoral histiocytes. The proliferation index by Ki67 is x%. All surgical margins are negative for tumor. No areas concerning for rhabdomyosarcoma are present. No vascular invasion is present.
    • Overall, these pathologic findings are consistent with inflammatory leiomyosarcoma. These tumors have significant pathologic and genetic overlap with the recently described inflammatory rhabdomyoblastic tumor (IRT) and low grade inflammatory myogenic tumor (LGIMT), suggesting that ILMS and IRT / LGIMT may belong to 1 entity (Mod Pathol 2021;34:758, Virchows Arch 2020;477:219)
Differential diagnosis
Board review style question #1

A 35 year old man presented with a slowly growing mass in the right thigh. Magnetic resonance imaging showed a 6 cm well circumscribed tumor in the deep planes. A biopsy was performed. The tumor cells are diffusely positive for desmin and MyoD1. What is the diagnosis?

  1. Inflammatory leiomyosarcoma
  2. Inflammatory myofibroblastic tumor
  3. Pleomorphic rhabdomyosarcoma
  4. Sclerosing rhabdomyosarcoma
Board review style answer #1
A. Inflammatory leiomyosarcoma. The histology shows spindle and epithelioid cells with eosinophilic cytoplasm, moderate pleomorphism but no mitotic activity and prominent nucleoli admixed with inflammatory cells characteristic of inflammatory leiomyosarcoma / inflammatory rhabdomyoblastic tumor. Answer D is incorrect because the cells in sclerosing rhabdomyosarcoma are small, round to spindle and embedded in a prominent hyalinized stroma. Answer C is incorrect because pleomorphic rhabdomyosarcoma is a high grade tumor with marked pleomorphism and increased mitotic activity. Answer B is incorrect because inflammatory myofibroblastic tumor does not show expression of skeletal muscle markers and is ALK positive instead.

Comment Here

Reference: Inflammatory leiomyosarcoma
Board review style question #2
Which of the following genetic alterations is most commonly seen in inflammatory leiomyosarcoma?

  1. 1q21 amplification
  2. Mutation of MYOD1
  3. Near haploidization
  4. VGLL2 rearrangements
Board review style answer #2
C. Near haploidization. Near haploidization with preservation of biparental disomy of chromosomes 5 and 22 is the most common genetic finding in inflammatory leiomyosarcoma / inflammatory rhabdomyoblastic tumor. Genome doubling leading to hyperdiploidy and mutations of NF1 are other recurrent genetic alterations. Answers B and D are incorrect because mutations of MYOD1 and rearrangements of VGLL2 are commonly seen in sclerosing / spindle cell rhabdomyosarcoma. Answer A is incorrect because amplification of 1q21 is one of the most frequent cytogenetic aberrations in multiple myeloma.

Comment Here

Reference: Inflammatory leiomyosarcoma

Inflammatory myofibroblastic tumor
Definition / general
  • Histologically distinctive myofibroblastic spindle cell neoplasm of borderline malignancy, classically featuring an intermixture of plasma cells and lymphocytes
Essential features
  • Myofibroblastic spindle cell neoplasm of borderline malignancy
  • Myofibroblastic spindle cells without overt cytologic atypia, interspersed by more or less prominent lymphoplasmacytic infiltrates
  • Up to 60% of cases show ALK rearrangement and will be positive for ALK1 IHC
  • Patients may present with constitutional symptoms (fever, night sweats, weight loss)
  • Epithelioid inflammatory myofibroblastic sarcoma is a distinct, highly aggressive variant that is predominantly intra-abdominal and is associated with a worse outcome
Terminology
  • Inflammatory pseudotumor; inflammatory myofibrohistiocytic proliferation; plasma cell granuloma; inflammatory fibrosarcoma (obsolete terms, not recommended)
ICD coding
  • ICD-O: 8825/1 - Inflammatory myofibroblastic tumor
  • ICD-10: D48.9 - Neoplasm of uncertain behavior, unspecified
  • ICD-11: 2B53.Y & XH66Z0 - Other specified fibroblastic or myofibroblastic tumor, primary site and myofibroblastic tumor, NOS
Epidemiology
  • Wide age range but most common in children and young adults
  • Extrapulmonary IMTs show predilection for children, with mean age of ~10 years (Semin Diagn Pathol 1998;15:85)
Sites
Clinical features
  • Site specific symptoms
  • Intra-abdominal tumors associated with increased abdominal girth, possibly symptoms of obstruction
  • Some patients (~33%) will present with systemic symptoms, such as fever, night sweats, weight loss (thought to be related to cytokine release) (Cytokine 2008;44:293)
Diagnosis
  • Tissue sampling and histological examination
Laboratory
  • Some patients have laboratory abnormalities (elevated erythrocyte sedimentation rate [ESR], hypochromic, normocytic anemia, thrombocytosis, hypergammaglobulinemia), which resolve soon after surgical resection (Cancer 1993;72:2042, Semin Diagn Pathol 1998;15:85)
Radiology description
  • Nonspecific radiology findings; ultrasound may show hypoechoic or hyperechoic mass(es) with ill or well defined borders; increased vascularity on Doppler studies
  • CT may show a homogeneous or heterogenous lesion with variable enhancement on delayed acquisitions due to fibrosis; calcification(s) seen in 15 - 25% of lung lesions in children, while rare in adult patients (Insights Imaging 2015;6:85, Radiology 1998;206:511)
  • MRI shows low signal intensity on T1 and T2 weighted sequences (Insights Imaging 2015;6:85)
Radiology images

Contributed by Carina Dehner, M.D., Ph.D.

Upper lobe mass

Upper lobe mass cross sectional imaging

Prognostic factors
  • Variable; generally good prognosis but up to 35% may recur and rare cases of distant metastasis have been reported
  • Adverse factors: intra-abdominal location, epithelioid variant
  • No established correlation of tumor size, mitotic rate, cellularity, necrosis and atypia with outcome in conventional IMT; however, ALK positive IMTs are less likely to recur and may have less aggressive behavior (Am J Surg Pathol 2007;31:509)
Case reports
Treatment
  • Surgical resection (re-excision of recurrences)
  • Treatment with specific tyrosine kinase inhibitors, such as crizotinib (N Engl J Med 2010;363:1727)
Gross description
  • Circumscribed, solid mass with rubbery cut surface
  • May present with calcifications
  • May present as multiple nodules (same anatomic site) in 33% of cases
  • Size ranges from 1 to 20 cm (median: 5 - 6 cm)
Gross images

Contributed by Carina Dehner, M.D., Ph.D.

Well circumscribed, firm white

Microscopic (histologic) description
  • Myofibroblastic spindle cell proliferation with mixed inflammation (lymphocytes, plasma cells and eosinophils)
  • Number of mitotic figures varies, atypical figures can be seen
  • Significant pleomorphism should be viewed with concern, may not fit with diagnosis of IMT
  • Variety of histologic patterns sometimes seen
    • Loosely arranged myxoid or hyaline stroma, spindle to stellate cells and admixed inflammatory cells (nodular fasciitis-like)
    • Storiform or fascicular growing elongated spindle cells without overt hyperchromasia or cytologic atypia, associated with prominent lymphoplasmacytic infiltrate
    • Hypocellular, scar-like pattern with occasional metaplastic bone or associated calcifications
  • Some tumors may show ganglion-like cells
  • Epithelioid variant predominantly composed of plump round to epithelioid cells with vesicular chromatin, large, prominent nucleoli and amphiphilic to eosinophilic cytoplasm; a prominent neutrophilic component and abundant myxoid stroma are common (Am J Surg Pathol 2011;35:135)
  • Reference: Goldblum: Enzinger and Weiss's Soft Tissue Tumors: Expert Consult, 7th Edition, 2019
Microscopic (histologic) images

Contributed by Carina Dehner, M.D., Ph.D.

Variable cellularity

Myofibroblasts and inflammatory cells

Short fascicles of spindle cells

Myofibroblasts without overt atypia


Eosinophilic cytoplasm

Ganglion-like cells

ALK1 IHC

Desmin IHC

Positive stains
Negative stains
Electron microscopy description
  • Myofibroblasts and fibroblasts
Molecular / cytogenetics description
Videos

Inflammatory myofibroblastic tumor

Sample pathology report
  • Lung, right upper lobe, pneumonectomy:
    • Inflammatory myofibroblastic tumor, ALK positive, present at bronchial and vascular margins (see comment)
    • Comment: Sections of the right lobe show a circumscribed spindle cell neoplasm with a prominent inflammatory component. The tumor has replaced the lung parenchyma and is continuous with the right upper lobe bronchus with partial destruction of the bronchial cartilage. The bronchial and vascular margins are involved by tumor. Immunohistochemical stains show that tumor cells are diffusely positive for vimentin, SMA and ALK, while negative for IgG and IgG4. In summary, the morphological and immunohistochemical features support the diagnosis of inflammatory myofibroblastic tumor.
Differential diagnosis
Board review style question #1

Which of the following is true about the inflammatory myofibroblastic tumor?

  1. Average age is 60 years or older
  2. It can be easily classified on imaging
  3. It is known to have USP6 rearrangement
  4. It never metastasizes
  5. Its aggressive variant shows perinuclear or nuclear membrane staining for ALK
Board review style answer #1
E. Its aggressive variant (epithelioid inflammatory myofibroblastic tumor) shows distinct, perinuclear or nuclear membrane staining for ALK.

Comment Here

Reference: Inflammatory myofibroblastic tumor
Board review style question #2
Which of the following genes may show a molecular abnormality in inflammatory myofibroblastic tumor?

  1. CTNNB1
  2. MYC
  3. NOTCH3
  4. ROS1
  5. USP6
Board review style answer #2
D. ROS1. ALK negative IMTs may show ROS1 alterations.

Comment Here

Reference: Inflammatory myofibroblastic tumor

Intimal sarcoma
Definition / general
  • Undifferentiated mesenchymal tumors arising from the inner lining (intima) of large arteries are classified as intimal sarcomas (ISA) with MDM2 amplification as their molecular hallmark (Mod Pathol 2021;34:2122)
  • The defining feature is predominantly intraluminal growth with obstruction of the lumen of the vessels of origin and seeding of emboli to the peripheral organs (WHO 5th edition)
  • ~67% of intimal sarcomas arise in the pulmonary circulation, almost always in the pulmonary artery; these are covered at Pulmonary artery intimal sarcoma
Essential features
ICD coding
  • ICD-O: 9137/3 - intimal sarcoma
  • ICD-11: 2B5F.2 & XH36H7 - sarcoma, not elsewhere classified of other specified sites & intimal sarcoma
Epidemiology
  • Rare tumor
  • Estimated prevalence: 0.0003 - 0.001%
  • M = F
  • Median age at diagnosis: 60 years (Intern Med 2016;55:3191)
Sites
  • Medline search on intimal sarcoma performed on September 20, 2023, revealed 411 papers published in the last 50 years describing 716 cases
    • Systemic circulation
      • Arteries
        • Aorta: 140 cases (19.56%)
          • Ascending: 5 cases (0.70%)
          • Arch: 15 cases (2.09%)
          • Descending: 6 cases (0.84%)
          • Thoracic: 29 cases (4.05%)
          • Abdominal: 44 cases (6.15%)
          • Not known: 41 cases (5.73%)
        • Other arteries: 21 cases (2.93%)
      • Veins: 14 cases (1.94%)
        • Inferior vena cava: 7 cases (0.97%)
        • Other veins: 7 cases (0.97%)
    • Pulmonary circulation: 338 cases (47.21%)
      • Pulmonary artery: 336 cases (46.93%)
      • Pulmonary vein: 2 cases (0.28%)
    • Heart: 203 cases (28.36%)
      • Left heart: 128 cases (17.89%)
      • Right heart: 75 cases (10.47%)
Pathophysiology
Etiology
  • Not known
  • 5 - 10% of tumors occur at sites of remote Dacron graft placement
  • In up to 20% of patients, the tumor is an incidental finding during aortic aneurysm repair (Pathology 2014;46:596)
Diagrams / tables

Images hosted on other servers:
Missing Image

Characteristic
histologic features
of 2 most common
patterns

Clinical features
  • Clinical and radiological findings of intimal sarcoma are often similar to those of thromboembolic disease, leading to delays in establishing diagnosis
  • Aortic intimal sarcoma mimics embolic phenom, such as claudication, skin necrosis and abdominal angina, due to the bulky intravascular component and friability
  • In case reports, more than 50% of patients had metastatic disease at the time of presentation; common sites include bone, lung, liver, lymph nodes, extremities, kidneys, skin and adrenal glands
  • Embolization to the brain is usually limited to tumors that involve the aortic arch and occurs in ~25% of patients
  • Additional symptoms are nonspecific and include chest, back or abdominal pain, nausea, dyspnea and systemic hypertension (Radiographics 2021;41:361)
Diagnosis
  • In some cases, the diagnosis of aortic neoplasia is made after endarterectomy for presumed thromboembolus in the systemic circulation (Pathology 2014;46:596)
  • More often, diagnosis of aortic intimal sarcoma (AIS) is delayed or even postmortem because these neoplasms are frequently misdiagnosed as more common nonneoplastic conditions, such as atheroma
  • Diagnosis is made by histology on resection specimens or during autopsy; biopsies of aorta are usually not performed
  • More prompt recognition is essential for timely initiation of aggressive treatment (Radiographics 2021;41:361)
Laboratory
  • D dimer might be elevated (mimics thromboembolism) (Med Clin (Barc) 2022;158:265)
  • Clinical findings of vasculitis and other inflammatory conditions, including elevated C reactive protein (CRP), erythrocyte sedimentation rate (ESR), dilated aorta or symptoms of claudication, may also manifest in aortic intimal sarcoma, increasing the diagnostic challenge (Semin Ultrasound CT MR 2011;32:377)
Radiology description
  • Great vessel sarcomas are often misdiagnosed at initial clinical presentation, due to their extreme rarity and their imaging appearance, which can overlap with those of much more common entities
  • Aortic intimal sarcomas are often misdiagnosed as protuberant atherosclerotic disease or intimal thrombus
  • Localized, near occlusive, lobulated or frond-like, endoluminal aortic lesion of low attenuation lining the vessel wall, evident anywhere in the thoracoabdominal aorta, is suspicious for aortic intimal sarcoma
  • Most useful imaging modalities for assessment of a suspected intimal sarcoma include computed tomography (CT) angiography, fluorine 18 fluorodeoxyglucose positron emission tomography (PET) and magnetic resonance imaging (MRI)
  • None of the above modalities, however, reliably differentiate between a thrombus and a malignancy; the ultimate diagnosis must be based on tissue examination (Radiographics 2021;41:361)
Radiology images

Contributed by Matjaz Sebenik, M.D.
BPD reporting overview

MRI of luminal tumor aortic arch

Prognostic factors
  • While median survival in patients without metastatic disease at the time of diagnosis is 20 months, it is only 6 months when metastases are present (Ann Vasc Surg 2014;28:515)
  • Even in the absence of metastatic disease, outcomes remain poor unless the tumor can be entirely excised (Radiographics 2021;41:361)
  • Age does not appear to be a prognostic factor (Oncol Res Treat 2017;40:353)
Case reports
  • 48 year old man with a history of chronic cardiac disease presented with 3 months of persistent back pain; CT and MRI both revealed a solid fusiform lesion of the descending aorta (Aorta (Stamford) 2019;7:169)
  • 49 year old woman with atherosclerotic plaque and thrombus (Open Med (Wars) 2021;16:1306)
  • 59 year old man with a tumor involving all segments of the thoracic aorta and a large floating thrombus causing acute mesenteric ischemia, treated successfully with embolectomy (Vasc Specialist Int 2021;37:46)
  • 71 year old woman with abdominal angina and an intra-aortic mass at the thoracoabdominal aorta that restricted blood supply to the organs (Ann Vasc Dis 2019;12:225)
  • 74 year old woman with aortic mass in the thoracoabdominal aorta masquerading as a mycotic aneurysm (J Vasc Surg Cases Innov Tech 2019;5:593)
Treatment
  • Treatment of aortic intimal sarcoma depends on the extent of aortic involvement and the presence or absence of metastatic disease
  • Radical surgical resection is the mainstay of treatment but complete surgical resection is rarely an option (Oncol Res Treat 2017;40:353)
  • Neoadjuvant therapy might shrink the tumor and thereby facilitate more efficient surgery; however, patients are often highly symptomatic at diagnosis requiring immediate surgery for symptomatic relief, leaving no time for neoadjuvant chemotherapy (Tex Heart Inst J 2014;41:518)
  • Role of adjuvant therapy is not yet proven, although some authors suggested better outcomes in patients receiving multimodality treatment compared to patients undergoing single modality treatment (Tex Heart Inst J 2009;36:451)
  • If adjuvant chemotherapy is initiated, there is no consensus on which agent or combination of agents could be effective
  • Palliative chemotherapy should be considered for inoperable, advanced cases but the quality and quantity of responses to available systemic agents is poor (Oncol Res Treat 2017;40:353)
  • MDM2, studied extensively in intimal sarcoma of pulmonary artery and heart, has become an area of focus in cancer treatment because anti-MDM2 therapy has become a reality
    • Small molecule inhibitors that block the MDM2 - p53 interaction and thus, reestablish wild type p53 activity, are available
    • First generation small molecule inhibitors of MDM2, nutlins, were identified in 2004
    • Nutlin 3A, a pharmacological inhibitor of the MDM2 - p53 interaction, stabilizes p53; however, its clinical efficacy is very low and severe thrombocytopenia represents a dose limiting toxicity
    • Recent novel second generation molecules that interfere with MDM2 or the MDM2 - p53 interaction, seem to be more promising (Diagnostics (Basel) 2021;11:496)
Clinical images

Contributed by Domenico Galetta, M.D., Ph.D.
CT scan and an intraoperative view

CT scan and an intraoperative view

Gross description
  • Macroscopically, intimal sarcomas are predominantly intravascular polypoid masses bound to the vessel wall, resembling thrombi (Radiographics 2021;41:361)
  • Section shows rubbery, gray, hemorrhagic, necrotic cut surface often associated with thrombus and atherosclerotic plaque material
  • Sizes ranged from 1 cm to very large tumors exceeding 10 cm (Am J Surg Pathol 2005;29:1184)
Gross images

Contributed by Marietta Kintiroglou, M.D. and Case #206
Gross photograph of cross sections

Cross sections

Abdominal aorta

Necrotic tumor infarcts

Microscopic (histologic) description
  • Surgical specimens
    • Intraluminal tumors are largely necrotic, with a narrow layer or rind of preserved cells limited to the luminal surface of a large thrombus or the intimal aspect of the affected vessel (see figures 1 - 6)
    • Low power appearance is often reminiscent of a complex atheroma with extensive fibrin thrombus
    • Tumors are composed of large epithelioid or histiocytoid cells that are poorly cohesive exhibiting a high nuclear to cytoplasmic ratio
    • Nuclei are pleomorphic with macronucleoli and numerous mitotic figures, including abnormal forms; the cytology of individual tumor cells most closely resembles amelanotic malignant melanoma or an anaplastic lymphoma and multinucleated giant tumor cells are infrequent
    • Cytoplasm is sparse and occasionally vacuolated but tumor cells neither exhibit intracytoplasmic lumina nor do they line capillary-like spaces in the primary tumors (see figure 4)
    • In all metastatic sites, the tumor’s appearance is altered and can have a wide variety of histologic patterns; these alterations included classic angiosarcoma (figure 9), pleomorphic tumor resembling undifferentiated pleomorphic sarcoma (UPS) (malignant fibrous histiocytoma [MFH]) (figures 10 - 11) or pleomorphic tumor resembling liposarcoma (figure 12) (Am J Surg Pathol 2005;29:1184)
  • Autopsies
    • In all cases where autopsy material or records were available for review, the tumor had recurred at the primary site
    • Widespread metastases were found in all cases, involving the lung, liver, vertebral bone, pancreas, small bowel, kidney, adrenal glands, diaphragm and skin
    • In local extravascular soft tissue recurrences and in distant metastatic sites, the tumors no longer showed the characteristic histology described in the luminal lesions; similar to the surgical pathology material, extravascular tumors took on the appearance of differentiated sarcomas, such as rhabdomyosarcoma (figure 7), osteosarcoma (figure 8), angiosarcoma (figure 9) and liposarcoma (figure 12)
    • Multiple patterns were seen in a single case and even in a single metastatic site (Am J Surg Pathol 2005;29:1184)
Microscopic (histologic) images

Contributed by Matjaz Sebenik, M.D. and and Case #206
Typical appearance of luminal UIS

Typical appearance of luminal UIS

Typical appearance of luminal UIS

Luminal UIS arising in a large vein

Intraluminal tumor Intraluminal tumor

Intraluminal tumor

Intraluminal tumor, CD31

Intraluminal tumor, CD31

Intraluminal tumor, FLI1

Intraluminal tumor, FLI1


Phenotype alteration of UIS in extravascular sites Phenotype alteration of UIS in extravascular sites Phenotype alteration of UIS in extravascular sites

Phenotype alteration of UIS in extravascular sites

Phenotype alteration of UIS in extravascular sites Phenotype alteration of UIS in extravascular sites Phenotype alteration of UIS in extravascular sites

Phenotype alteration of UIS in extravascular sites


Abdominal aorta with a narrow rind of preserved malignant cells

Abdominal aorta with narrow rind of preserved malignant cells

Preserved large epithelioid poorly cohesive tumor cells

Preserved large epithelioid poorly cohesive tumor cells

Tumor emboli present in liver, spleen, lung and kidney Tumor emboli present in the bone marrow of femur

Tumor emboli

Individual tumor cells in spleen

Individual tumor cells in spleen

Virtual slides

Contributed by Matjaz Sebenik, M.D.
Missing Image

A: huge tumor of entire aorta

Missing Image

A: aortic tumor invading adventitia with osteoid

Missing Image

B: thrombus of abdominal aorta

Missing Image

B: skin metastases with typical angiosarcoma


Missing Image

C: iliac aortas with typical intraluminal appearance

Missing Image

C: femoral bone metastases with pleomorphic liposarcoma

Missing Image

D: thoracic aorta with typical intraluminal appearance

Missing Image

D: pelvic bone metastases with UPS

Missing Image

E: tumor of right posterior lower thigh skin vein



Positive stains
Negative stains
Electron microscopy description
  • Fine structure is nonspecific (Am J Surg Pathol 2005;29:1184)
    • Viable appearing cells are ovoid with a high nuclear to cytoplasmic ratio
    • Nuclei are irregular and deeply convoluted with marginated condensed heterochromatin
    • About 33% of cases show focal perinuclear intermediate filaments
    • Cytoplasm is devoid of specific organelles like melanosomes or Weibel-Palade bodies
Molecular / cytogenetics description
Sample pathology report
  • Mass, aorta, excision:
    • Intimal sarcoma (see comment)
    • Comment: Sections show proliferation of oval to round cells covering thrombus. Nuclei are hyperchromatic, nucleoli are prominent. Cells are predominantly limited to intima (supported by elastic stain) and only very focally invade media. Tumor cells are diffusely positive for CD31 and FLI1, focally positive for factor VIII and CD34, and negative for AE1 / AE3, SOX10 and HMB45. MDM2 immunohistochemistry shows positivity in the tumor nuclei. FISH study shows MDM2 and CDK4 amplification. The histologic, immunohistochemical and molecular findings are consistent with a diagnosis of intimal sarcoma.
Differential diagnosis
  • Intravascular lymphoma:
    • Most common in capillaries to medium sized vessels, especially in brain and skin; occasionally seen in larger systemic vessels, such as aorta
    • Most cases are of B lineage, occasionally of T lineage
      • T cell cases should generally be considered a different disease
    • Positive for LCA and B cell markers; negative for CD31 and FLI1 (BMC Nephrol 2018;19:300)
  • Metastatic carcinoma:
    • Secondary (metastatic) aortic malignant neoplasms are more commonly seen than primary tumors and are more readily diagnosed, possibly owing to the knowledge of a pre-existing primary tumor
    • There appears to be no age predilection (J Vasc Surg Cases Innov Tech 2018;4:160)
    • In the thoracic region, invasion from the lungs, esophagus and thymoma is the usual scenario (Semin Ultrasound CT MR 2012;33:265)
    • Retroperitoneal sarcomas and germ cell tumors are the most common malignant neoplasms invading the abdominal aorta, potentially with resultant aneurysm or pseudoaneurysm formation and rupture (Ann Vasc Surg 2008;22:568)
    • Usually present in vasa vasorum and adventitia, not in the tunica intima (Discoveries (Craiova) 2020;8:e106)
    • Positive for carcinoma markers; negative for CD31 and FLI1
  • Leiomyoma / leiomyosarcoma (LMS):
    • Originates from smooth muscles of tunica media
    • 5 times more common in veins than in arteries
    • 50% of venous LMS originate in inferior vena cava, 25% in saphenous vein, the rest in femoral vein, internal jugular vein or iliac vein
    • Majority of the extremity venous LMS arise from lower extremity
    • Histologically are indistinguishable from their soft tissue counterpart diagnosis (Ann Vasc Surg 2019;57:274.e5)
    • Positive for smooth muscle markers; negative for endothelial markers
  • Angiosarcoma:
    • In theory, primary angiosarcomas of aorta should feature intima based vasoformative histologic pattern typical of soft tissue angiosarcomas
    • To date, no vasoformative cases have been described in the aortic intima
    • Published cases of primary angiosarcoma of the aorta are actually intimal sarcomas, which were called angiosarcomas because of positive staining for endothelial markers or because they featured vasoformative structures outside of tunica intima (i.e., within the native vessel wrap) (J Vasc Surg 1991;14:87)
  • Metastatic melanoma:
    • Melanoma cells are most frequently found embedded in the fibrinous blood clot
    • Positive for melanoma markers; negative for CD31 and FLI1 (J Vasc Surg 2002;36:191)
Board review style question #1

A 45 year old woman presented with a hypertensive crisis. Computed tomography (CT) revealed a thoracic aortic tumor and tissues obtained via endovascular biopsy revealed a thrombus rimmed by atypical cells, which were positive for CD31 and FLI1 and negative for LCA, CD20, SOX10 and HMB45. Despite undergoing surgical resection followed by adjuvant chemotherapy, the patient died from progressive multiple metastasis and severe sepsis only 3 months later. What is the diagnosis?

  1. High grade sarcoma, consistent with intimal sarcoma
  2. Intravascular lymphoma
  3. Metastatic carcinoma
  4. Metastatic melanoma
  5. Thromboembolus
Board review style answer #1
A. High grade sarcoma, consistent with intimal sarcoma. Answer A is correct because malignant cells covering the thrombus are positive for CD31 and FLI1 and negative for LCA, CD20, SOX10 and HMB45. Answer B is incorrect because intravascular lymphoma is stained with endothelial, not lymphatic markers. Answer D is incorrect because metastatic melanoma is stained with endothelial, not melanoma markers. Answer C is incorrect because metastatic carcinoma is stained with endothelial (not carcinoma) markers and because it was intimal (metastatic carcinomas involve adventitia and vasa vasorum, not intima). Answer E is incorrect because the thrombus in this case is covered by malignant cells.

Comment Here

Reference: Intimal sarcoma
Board review style question #2
What is the most common location of intimal sarcoma?

  1. Aorta
  2. Heart
  3. Pulmonary artery
  4. Pulmonary vein
  5. Systemic vein
Board review style answer #2
C. Pulmonary artery. Answer C is correct because the pulmonary artery is involved in ~47% of cases. Answer A is incorrect because the aorta is involved in ~19.5% of cases. Answer E is incorrect because systemic veins are involved in ~1.9% of cases. Answer B is incorrect because the heart is involved in ~28% of cases. Answer D is incorrect because pulmonary veins are involved in ~0.3% of cases.

Comment Here

Reference: Intimal sarcoma

Intramuscular angioma
Definition / general
  • Benign vascular neoplasm arising within skeletal muscle, accompanied by variable amount of adipose tissue
Essential features
  • Variable admixture of arteries, veins, lymphatics of cavernous / capillary or mixed type of vessels
  • Prominent intralesional adipose tissue and infiltrative growth pattern
  • Complete excision is the recommended treatment as recurrence rate is high; therefore, a comment on resection margin is essential
  • Degenerated entrapped skeletal muscle fibers may mimic malignancy due to nuclear hyperchromasia
Terminology
  • Intramuscular hemangioma
  • Intramuscular infiltrating angiolipoma (not recommended)
ICD coding
  • ICD-O: 9132/0 - intramuscular hemangioma
  • ICD-11: 2E81.0Y & XH0553 - neoplastic hemangioma of other specified site & intramuscular hemangioma
Epidemiology
Sites
Pathophysiology
Etiology
Clinical features
  • Pain is a commonly encountered symptom, particularly in the extremities after exercise (Sports Health 2013;5:448)
  • Slow growing nodular swelling in more superficial areas like head and neck, with variable skin discoloration
  • Rare presentation with limited functionality, when located near large joints (Medicine (Baltimore) 2019;98:e14343)
Diagnosis
  • Requires high level of suspicion and characteristic radiological findings, especially on vascular imaging modalities (color Doppler, CT and MR angiography), along with diagnostic histopathological features
Radiology description
Radiology images

Images hosted on other servers:
Nonhomogeneous arm mass

Nonhomogeneous arm mass

Hypoechogenic right parapharyngeal mass

Hypoechogenic right parapharyngeal mass

Medial rectus lesion

Medial rectus lesion

Gluteus medius lesion

Gluteus medius lesion

Phleboliths, right foot

Phleboliths, right foot


Multilobulated mass, right foot

Multilobulated mass, right foot

Phlebolith, biceps mass

Phlebolith, biceps mass

Lobulated calf mass

Lobulated calf mass

Fast flow phenomenon, shoulder lesion

Fast flow phenomenon, shoulder lesion

Right buccal mass

Right buccal mass

Prognostic factors
Case reports
Treatment
  • Complete surgical excision with clear margins is the recommended treatment
  • Preoperative embolization reduces preoperative bleeding (Clin Exp Otorhinolaryngol 2015;8:298)
  • Conservative management (with corticosteroids, sclerotherapy, etc.) sometimes works, depending on the individual case (Sports Health 2013;5:448)
Clinical images

Images hosted on other servers:
Asymmetrical right lateral neck swelling

Asymmetrical right lateral neck swelling

Right masseter swelling

Right masseter swelling

Abductor hallucis mass

Abductor hallucis mass

Gross description
  • Size range: 1 - 29 cm; mean size: 6.5 cm
  • Irregular, ill defined, soft, multilobulated mass (Int J Environ Res Public Health 2021;18:9088)
  • May show solid to hemorrhagic cut surface with thrombosis of vascular channels
  • Yellowish appearance, based on amount of adipose tissue present
Gross images

Contributed by Nasir Ud Din, M.B.B.S.

Circumscribed multilobulated lesion, elbow

Ill defined, diffuse lesion, neck

Intramuscular lobulated lesion, gastrocnemius



Images hosted on other servers:
Plantar intramuscular angioma

Plantar intramuscular angioma

Gluteus lesion

Gluteus lesion

Frozen section description
Microscopic (histologic) description
  • Variable mixture of large and medium arteries and veins with small capillary sized vessels and ectatic lymphatic channels
  • Infiltration into skeletal muscle with ill defined edges
  • Variable intralesional adipose tissue with some examples mimicking angiolipoma; checkerboard pattern is also seen
  • Phleboliths may be seen in longstanding lesions
  • Variable atrophy of native skeletal muscle with degenerative hyperchromatic appearance of sarcolemmal nuclei
  • Perineural proliferation of small vessels, does not indicate aggressive behavior (Goldblum: Enzinger and Weiss’s Soft Tissue Tumors, 7th Edition, 2019)
  • Preoperative embolization shows intravascular foreign material with degenerative changes in the lesion, including necrosis and inflammation (Stockman: Diagnostic Pathology - Vascular, 1st Edition, 2015)
  • Rarely, intralesional bone tissue is seen
  • Traditionally classified into:
    • Capillary type:
      • More common in head and neck region
      • Small capillary sized vessels, with solid areas mimicking infantile hemangioma
      • Plump endothelial cells
    • Cavernous type:
      • More common in trunk locations
      • Proliferating small capillaries at periphery of larger vessels
      • Attenuated bland endothelial cells
      • Thrombosis is frequent
    • Mixed type:
      • Most common
      • May also have lymphatic channels
Microscopic (histologic) images

Contributed by Nasir Ud Din, M.B.B.S.

Lobulated lesion

Cavernous vessels

Variably sized vessels

Intralesional adipose tissue

Angiolipoma-like appearance


Capillary vessels

Slit-like vessels

Infantile hemangioma-like appearance

Lobular proliferation of vessels

Inflammatory cells


Diffuse pattern

Thickened basement membrane

Hyalinization

Virtual slides

Images hosted on other servers:
Nonhomogeneous arm mass

Intramuscular hemangioma

Cytology description
Positive stains
Negative stains
Electron microscopy description
  • Transmission electron microscopy shows composition of a mixture of cavernous and capillary channels within the muscle (Acta Ophthalmol Scand 2002;80:336)
  • Not required for diagnosis
Videos

Hemangioma, capillary and cavernous: clinical features and morphology

Sample pathology report
  • Right thigh, excisional biopsy:
    • Benign vascular lesion infiltrating into skeletal muscle, composed of small to intermediate sized vascular channels lined by bland endothelial cells, favoring diagnosis of intramuscular angioma (see comment)
    • Size: 12 x 10 x 3 cm
    • Margin: painted excision margin is involved
    • Comment: Although benign, these lesions have tendency to recur, if incompletely excised, in 30 - 50% of cases. Follow up with clinical findings and radiographic studies is recommended.
Differential diagnosis
  • Angiomatosis:
    • Usually congenital
    • Involves large areas of body and multiple tissue planes
    • Microscopic features similar to intramuscular angioma; distinction requires clinical input
  • Infantile hemangioma:
    • Involves skin / soft tissue of face, head and neck of infants
    • Lobular arrangement and proliferation of capillaries separated by thin fibrous tissue
    • Cellularity and number of vessels decrease with time
    • GLUT1 positivity by IHC
  • Angiolipoma:
    • Occurs in subcutaneous tissue
    • Small, painful, circumscribed lesions, often multiple
    • Capillary sized vascular channels only
    • Vessels may show fibrin thrombi
  • Angiosarcoma:
    • Extremely rare in deep soft tissues
    • Complex interanastomosing vascular channels with atypia
    • Multiple layers of endothelial cells with nuclear pleomorphism and increased mitotic activity
  • Intramuscular lipoma:
    • Site predilection same as intramuscular angioma
    • Infiltrative lesion with sheets of mature adipocytes and intervening skeletal muscle component
    • Vascular component is sparse
Board review style question #1

A 25 year old man presented with right leg pain, particularly after exercise. Physical examination showed slightly increased bulkiness of the right leg as compared to the left. MRI showed a well defined lesion in the belly of gastrocnemius, which was isointense on T1, hyperintense on T2 and showed an intralesional fat component. A photomicrograph of the excised lesion is given. Which of the following statements regarding this lesion is true?

  1. Association with trauma is well established in these lesions
  2. Capillary proliferation within perineural sheaths is ominous
  3. Examples with adipose tissue represent intramuscular lipoma
  4. Recurrence rate is high after incomplete excision
  5. These tumors represent congenital malformations
Board review style answer #1
D. Recurrence rate is high after incomplete excision, which can be up to 50%. This tumor is an intramuscular angioma, which represents a true neoplasm rather than a congenital malformation and is not associated with a history of trauma. The intralesional adipose tissue component is common and may be so much that it masks the vascular nature of the lesion. Proliferation of small capillaries with the perineural sheaths, mitotic figures or infiltrating nature of this lesion do not underlie an aggressive behavior.

Comment Here

Reference: Intramuscular angioma
Board review style question #2

A 16 year old girl presented with a slowly growing left leg mass. Gross specimen was comprised of skin covered fibroadipose and fibromuscular tissue with an ill defined, diffuse lesion within muscle. Cut surface showed small to intermediate sized blood filled vessels arranged in lobules. Microscopy revealed a proliferation of benign appearing vascular channels of variable sizes with intervening adipose tissue. No atypia, necrosis or increased mitoses were observed. A photomicrograph of the lesion is shown. What is the most likely diagnosis?

  1. Angioleiomyoma
  2. Angiolipoma
  3. Angiosarcoma
  4. Intramuscular angioma
  5. Intramuscular lipoma
Board review style answer #2
D. Intramuscular angioma

Comment Here

Reference: Intramuscular angioma

Intramuscular myxoma
Definition / general
Sites
  • Thigh, shoulder, upper arm
Clinical features
Diagnosis
  • Diagnosis less likely if any of these findings: atypia, mitotic activity, more than occasional vessels, not intramuscular or juxtaarticular
Case reports
Treatment
  • Excision; rarely recurs if adequately excised
  • Cellular cases appear to have same indolent behavior if completely excised (Histopathology 2001;39:287)
Gross description
  • Mucoid / gelatinous, often poorly circumscribed, may have infiltrative borders
  • May be up to 13 cm
Microscopic (histologic) description
  • Hypocellular, composed of bland cells, no mitotic activity, no lipoblasts, scant blood vessels; may have focal histiocytes
  • Slightly more cellular with collagenized capsule at periphery
  • May have areas of increased vascularity or be hypercellular (Am J Surg Pathol 1998;22:1222)
  • Slightly basophilic matrix with usually a few spindle cells at high power with oval nuclei
  • May have central mucinous cyst
  • At periphery, may infiltrate striated muscle and be associated with muscle atrophy
Microscopic (histologic) images

Contributed by Syed T. Hoda, M.D. and Mark R. Wick, M.D.

53 year old woman with subcutaneous thigh mass

Various images



Images hosted on other servers

Mandibular

Site unknown

Cytology description
  • Viscous, gelatinous quality when first applied to glass slide
  • Paucicellular, often finely granular myxoid stroma with few cells, usually macrophages and bland spindle cells (Am J Clin Pathol 2005;123:858)
Positive stains
Negative stains
Electron microscopy description
  • Fibroblast-like cell with prominent Golgi, endoplasmic reticulum, cytoplasmic filaments
Electron microscopy images

Images hosted on other servers:

Conjunctival myxoma

Molecular / cytogenetics description
Differential diagnosis

Ischemic fasciitis
Definition / general
  • Painless pseudosarcomatous fibroblastic proliferation in soft tissue overlying bony prominences subject to intermittent pressure-induced ischemia
  • Also spelled "ischaemic"
  • First described in 1992 as "atypical decubital fibroplasia" (Am J Surg Pathol 1992;16:708)
Epidemiology
Sites
  • Shoulder, chest wall, sacrococcygeal region, greater trochanter
Etiology
  • Similar to decubitus ulcers; pressure-induced ischemia causes mass-producing reactive process, but is insufficient to cause skin ulceration
Case reports
Treatment
  • Local excision is curative, although may recur due to continuation of underlying ischemia and injury
Clinical images

Contributed by Mark R. Wick, M.D.

Decubital

Gross description
  • Usually 1 - 8 cm, poorly circumscribed, often myxoid
  • Usually involves deep subcutis, may involve adjacent skeletal muscle and fascia
  • Ulceration is uncommon (i.e. overlying skin is intact)
Microscopic (histologic) description
  • Zonal pattern of central fibrinoid necrosis with uneven borders staining deep red / violet and prominent myxoid areas surrounded by ectatic, thin walled vessels and proliferating fibroblasts
  • Endothelial cells may be atypical
  • Fibroblasts have degenerative features with abundant, eosinophilic to amphophilic cytoplasm, enlarged nuclei with smudged chromatin and prominent nucleoli (resembling ganglion-like cells in proliferative fasciitis)
  • Variable mitotic activity, but no atypical mitotic figures
  • Fibrin thrombi are common within peripheral vessels, which may show fibrinoid necrosis and recanalization but no true vasculitis
  • May have multivacuolated macrophages in myxoid zones mimicking lipoblasts
Microscopic (histologic) images

Contributed by Mark R. Wick, M.D., AFIP and Ronald Angeles, M.D. (Case #64)

Cellular, fibrin rich proliferation

Hyalinized focus

Foci of ganglion-like cells

Decubital


Hip mass in 55 year old bedridden man

Cytology description
Positive stains
Negative stains
Differential diagnosis
  • Epithelioid sarcoma:
    • Young adults on distal extremities
    • More cellular with central tumor cell necrosis
    • Cells have eosinophilic cytoplasm
    • Atypical mitotic figures
    • Keratin+
  • Myxofibrosarcoma:
    • Marked atypia but no smudgy chromatin or fibrin thrombi
    • Lacks zonation and degenerative features
  • Myxoid liposarcoma:
    • Prominent plexiform vasculature and lipoblasts
    • Small monotonous cells rather than larger myofibroblasts
  • Proliferative fasciitis:
    • Younger patients
    • Lesions not associated with pressure; zonation, myofibroblasts and fibroblasts with tissue culture type growth, also large ganglion cells

Juvenile hyaline fibromatosis
Definition / general
  • Rare hereditary pediatric disorder with extracellular hyaline material deposition in skin, soft tissue and bone
Terminology
  • Molluscum fibrosum in children (original name given by Murray, 1873), mesenchymal dysplasia (Puretic et al, 1962), disseminated painful fibromatosis
  • Also called fibromatosis hyalinica multiplex
  • Called infantile systemic hyalinosis if organ involvement, debatable if this is same disease or a distinct entity (Pediatr Dermatol 2004;21:154, J Am Acad Dermatol 2009;61:695, Hum Mutat 2009;30:583)
  • "Hyaline fibromatosis syndrome" proposed as an umbrella term for juvenile hyaline fibromatosis and infantile systemic hyalinosis since the two syndromes have significant clinicopathologic overlap (J Am Acad Dermatol 2009;61:695)
Epidemiology
  • Infants or children under 5 years
  • No ethnic predisposition (Rheumatol Int 2011;31:273), slight male predominance
  • Occasionally presents in 4th - 5th decade
Sites
  • Skin, soft tissue, bones
Etiology
Clinical features
  • Progressive, chronic, and often debilitating disease
  • Pearly papules and plaques (face, posterior neck, perianal region)
  • Large, subcutaneous nodules (most often on scalp)
  • Gingival hypertrophy
  • Flexure contractures (due to masses in periarticular soft tissue)
  • Osteolytic bone lesions of skull, long bones, phalanges (J Am Acad Dermatol 2009;61:695)
Laboratory
  • Anemia, hypogammaglobulinemia, hypoalbuminemia, electrolyte imbalance
Radiology description
  • Fractures, osteolytic bone lesions, osteoporosis
Case reports
Treatment
Clinical images

Images hosted on other servers:

Recurrent lesions

Gross description
  • Solid, white, waxy nodules
Gross images

Images hosted on other servers:

Nodules with gelatinous surface

Microscopic (histologic) description
  • Poorly circumscribed amorphous or nodular deposits of abundant, hyaline material with embedded fibroblasts in cords
  • Can have retraction artifact around fibroblasts
  • Early lesions are characterized by increased cellularity
  • No atypia, no necrosis
Microscopic (histologic) images

Images hosted on other servers:

Sparse, uniform spindle cells

PAS+ matrix

Cytology description
Cytology images

Images hosted on other servers:

Scant spindle cells

Positive stains
Negative stains
Electron microscopy description
Differential diagnosis

Juxta-articular myxoma
Definition / general
Epidemiology
  • Median age 43 years, range 16 - 83 years
  • 72% male
Sites
  • Knee (88%), also shoulder, elbow, ankle, hip
Case reports
Clinical images

Images hosted on other servers:

Arthroscopic image
of lesion within
suprapatellar pouch

Gross description
  • Myxoid, mucoid or slimy mass with pale white to yellow color
Gross images

Images hosted on other servers:

Glassy appearance

Microscopic (histologic) description
  • Loosely arranged spindle cells in hypovascular myxoid matrix; variable cystic spaces can be seen
Microscopic (histologic) images

Images hosted on other servers:

Myxoid stroma and spindle cells

Bland myxoid
neoplasm without
mesenchymal atypia
or hypercellularity


Kaposi sarcoma
Definition / general
  • Vascular neoplasm caused by human herpesvirus 8 (HHV8)
  • Tends to be indolent but may be locally aggressive
Essential features
  • Malignant vascular neoplasm of intermediate behavior
  • In all forms and cases, the etiologic agent is human herpesvirus 8 (HHV8)
  • Histologic features are similar for all clinical variants
  • Slit-like vascular spaces formed by spindled endothelial cells with minimal to moderate atypia; hemorrhage, hemosiderin deposition and plasma cells are common
Terminology
ICD coding
  • ICD-O: 9140/3 - Kaposi sarcoma
  • ICD-10: C46 - Kaposi sarcoma
  • ICD-11: 2B57 - Kaposi sarcoma, primary site
Epidemiology
  • Depends on the clinical variant of Kaposi sarcoma (4 subtypes) (Nat Rev Dis Primers 2019;5:9, StatPearls: Kaposi Sarcoma [Accessed 6 October 2022])
    • Classic / sporadic (Mediterranean) (Cancers (Basel) 2021;13:5692)
      • Middle aged to elderly; M > F
      • Ethnic groups from regions with high prevalence of HHV8 (Middle East, Eastern Europe and the Mediterranean) are at increased risk
    • Endemic (African)
      • Male adults and children of both sexes
      • Not related to HIV
      • Mirrors HHV8 seropositivity
      • Most commonly in sub-Saharan Africa
    • Iatrogenic (transplant related) (Cancers (Basel) 2021;13:5692)
      • M > F
      • Patients > 50 years
      • Solid organ transplant patients
      • Risk correlates with the level of immunosuppression; hence, the risk is higher in multiorgan transplants and with greater HLA mismatching
    • Epidemic (HIV / AIDS associated) (Cancers (Basel) 2021;13:5692)
      • Patients infected with HIV
      • Risk increases with declining CD4 cell counts and is reduced with the early use of combined antiretroviral therapy (cART)
    • Kaposi sarcoma in men who have sex with men (MSM) without HIV infection (AIDS 2008;22:1163)
      • Very rare
      • MSM without HIV infection who are not immunocompromised
      • Favorable prognosis
Sites
  • Skin is the most common location
  • Visceral involvement includes lymph nodes, lungs, gastrointestinal system and other viscera; respiratory involvement can be fatal
  • Classic / sporadic (Mediterranean)
    • Usually few lesions limited to the lower limbs
    • Visceral and mucosal involvement is uncommon
  • Endemic (African)
    • Children develop visceral involvement including the lymph nodes
    • Skin involvement is not typical in children
    • Adults develop cutaneous lesions akin to classic Kaposi sarcoma; can sometimes be locally aggressive
    • Adults can present with lower limb lymphedema
    • Visceral involvement may occur in some adult patients
  • Iatrogenic (transplant related)
    • Typically affects the lower limbs
    • Mucosal and visceral involvement is relatively common
  • Epidemic (HIV / AIDS associated)
    • Skin lesions on the limbs, trunk and face; lesions can be localized or disseminated
    • Mucosal and visceral lesions are more common in patients with low CD4 count and high viral loads
  • Kaposi sarcoma in men who have sex with men (MSM) without HIV infection
    • Few cutaneous lesions at any site
    • Visceral and mucosal involvement is rare
  • References: Nat Rev Dis Primers 2019;5:9, StatPearls: Kaposi Sarcoma [Accessed 6 October 2022]
Pathophysiology
Etiology
  • HHV8
    • Enveloped, double stranded DNA virus
    • HHV8 virus belongs to the Herpesviridae family of DNA viruses
    • Diseases associated with HHV8 virus infections include Kaposi sarcoma (KS), primary effusion lymphoma (PEL) and multicentric Castleman disease (MCD)
Diagrams / tables

Images hosted on other servers:

Endothelial response to HHV8 infection

Clinical features
  • Progressive, enlarging red to violaceous skin lesions (Arch Pathol Lab Med 2013;137:289)
    • Patch stage
      • Red or purple macules or patches
    • Plaque stage
      • Thickened red, purple or brown plaques
    • Tumor stage
      • Nodule formation
  • Skin lesions may be painful with associated lymphedema and secondary infection
  • All clinical subtypes of Kaposi sarcoma may show these stages
  • Multiple different stages may be present simultaneously
  • Clinical course of the subtypes of Kaposi sarcoma
    • Classic / sporadic (Mediterranean)
      • Usually indolent, rarely aggressive and disseminated
    • Endemic (African)
      • Usually indolent to locally invasive in adults
      • Occasional rapid progression with visceral disease in adults
      • Aggressive in children
    • Iatrogenic (transplant related)
      • May regress with modification of immunosuppression
      • May be aggressive
    • Epidemic (HIV / AIDS associated)
      • Indolent or aggressive
      • May regress with effective HIV treatment
    • Kaposi sarcoma in men who have sex with men (MSM) without an HIV infection
      • Usually indolent, although disseminated disease has been described
  • References: Nat Rev Dis Primers 2019;5:9, StatPearls: Kaposi Sarcoma [Accessed 6 October 2022]
Diagnosis
Laboratory
Radiology description
  • Imaging facilitates the diagnosis, staging and follow up of Kaposi sarcoma
  • Features are dependent upon site of involvement (Abdom Radiol (NY) 2021;46:5297)
Radiology images

Images hosted on other servers:

Pulmonary Kaposi sarcoma

Prognostic factors
  • 20% of patients with the classic form of Kaposi sarcoma will die of disease; a similar percentage may develop secondary malignancies
  • Endemic, HIV related and iatrogenic forms have a variable prognosis
    • Prognosis may depend on CD4 count and the presence of opportunistic infections
    • In patients with iatrogenic Kaposi sarcoma, the prognosis is dependent on their underlying condition and their ability to tolerate a reduction in immunosuppression
    • Worse prognosis is usually found in patients with visceral organ involvement, particularly the lungs
  • Reference: StatPearls: Kaposi Sarcoma [Accessed 6 October 2022]
Case reports
Treatment
  • Treatment varies depending on the clinical subtype (Am J Clin Dermatol 2017;18:529):
    • AIDS related disease responds favorably to HAART in 50% of cases
      • Protease inhibitors may have direct effect against Kaposi sarcoma
      • Combination of HAART and chemotherapy in advanced cases
    • Cutaneous lesions can be treated by local excision, liquid nitrogen and injection of vincristine (StatPearls: Kaposi Sarcoma [Accessed 6 October 2022])
    • Early stage classical Kaposi sarcoma is radiosensitive
    • Endemic and systemic forms are mainly treated with chemotherapy
    • Treatment of iatrogenic disease must balance reduction in transplant immunosuppression with risk of transplant rejection
Clinical images

Contributed by Mark R. Wick, M.D.

Advanced disease



Images hosted on other servers:

Reddish purple nodules

Lesions

Intraoral lesion

Violaceous plaques

Gross description
  • Red to purple lesions
  • Hemorrhagic on cut surface, especially visceral lesions
Gross images

Images hosted on other servers:

Infiltration of lung parenchyma

Reddish nodules on gastric mucosa

Irregular masses in the liver

Microscopic (histologic) description
  • Patch stage
    • Dilated vascular channels dissecting through dermal collagen
      • Promontory sign: tumor vascular channels surround and entrap native vessels (classic but uncommon feature)
    • May be very subtle in early lesions
  • Plaque stage
    • More extensive, compressed, slit-like vascular channels infiltrating deeper dermis
    • Infiltrative proliferation of spindled endothelial cells (may resemble fibroblasts)
      • Spindle cells infiltrating and destroying eccrine coils is very characteristic
  • Tumor (nodular) stage
    • Discrete nodules composed of intersecting fascicles of uniform spindle cells
    • Intervening blood filled spaces between spindle cells
      • Slit-like spaces (longitudinal section)
      • Sieve-like spaces (cross section)
    • Intracytoplasmic hyaline globules may be seen
  • All stages
    • Mitoses are common but pleomorphism is usually minimal
      • Rare poorly differentiated cases may have severe pleomorphism; it's important to confirm the diagnosis with HHV8 IHC in order to exclude angiosarcoma
    • Extravasated erythrocytes and hemosiderin deposits are common
    • Plasma cells are present in most cases
  • Distinction of stage is not required in pathology report (Arch Pathol Lab Med 2013;137:289)
Microscopic (histologic) images

Contributed by Michella Whisman, M.D. and Mark R. Wick, M.D.

Early lesion subtle at low power

Dilated channels visible at higher power

Inflammatory appearance

Lymphoplasmacytic
infiltrate

Hemosiderin deposition

Spindle cell proliferation


Tumor (nodular) stage

Sieve-like pattern

Blood filled channels

Spindle cells without blood

Ill defined fascicles

Hyaline globules


HHV8

ERG

HHV8 can be missed at low power

HHV8 with focal positivity

Virtual slides

Images hosted on other servers:

Eyelid, Kaposi sarcoma

Skin, Kaposi sarcoma

Cytology description
Cytology images

Contributed by Michella Whisman, M.D.

Lymph node touch prep

Positive stains
Videos

Kaposi sarcoma:
5 minute pathology pearls

Sample pathology report
  • Skin, left lower leg, shave biopsy:
    • Kaposi sarcoma (see comment)
    • Comment: Histologic sections contain a dermal based neoplasm composed of sheets of uniform, relatively bland spindle cells with poorly formed slit-like vascular channels. There is extensive extravasation of red blood cells. Mitotic activity is conspicuous and no tumor necrosis is identified. Immunohistochemical stains show diffuse reactivity for ERG and HHV8.
Differential diagnosis
  • Angiosarcoma:
    • Infiltrative vascular channels, usually with marked nuclear atypia
    • HHV8 negative
    • Most common sites include the head / neck of elderly patients and previously irradiated sites (most commonly breast)
    • Visceral lesions have a dismal prognosis
  • Severe vascular stasis (acroangiodermatitis):
    • Fibroblastic spindle cell component rather than endothelial
    • Negative for vascular markers
    • HHV8 negative
  • Hobnail hemangioma (targetoid hemosiderotic hemangioma):
    • Slit-like vascular channels but no spindle cell proliferation
    • HHV8 negative
  • Spindle cell hemangioma:
    • Spindled endothelial cells with slit-like channels but also has epithelioid cells
    • Biphasic: solid spindled / epithelioid zone and zone of dilated cavernous vessels
    • Usually lacks plasma cells and hemorrhage
    • HHV8 negative
  • Kaposiform hemangioendothelioma:
    • Unique whorled and nodular pattern of spindled endothelial cells
    • Hemorrhage and plasma cells uncommon
    • Usually in young children or infants
    • HHV8 negative, GLUT1 positive
Board review style question #1

The tumor pictured above is from the leg of an 80 year old man of Mediterranean descent. The virus associated with the diagnosis also causes which of the following?

  1. Adult T cell leukemia
  2. Burkitt lymphoma
  3. Kaposi sarcoma
  4. Merkel cell carcinoma
Board review style answer #1
C. The tumor is Kaposi sarcoma, which is caused by HHV8. HHV8 may also lead to primary effusion lymphoma and multicentric Castleman disease. Merkel cell carcinoma is caused by polyomavirus. Burkitt lymphoma can be associated with EBV. Adult T cell leukemia is associated with HTLV1.

Comment Here

Reference: Kaposi sarcoma
Board review style question #2

The tumor pictured above has a viral association. What is the causative agent?

  1. Enveloped, double stranded DNA virus
  2. Enveloped, double stranded RNA virus
  3. Enveloped, single stranded DNA virus
  4. Enveloped, single stranded RNA virus
Board review style answer #2
A. Enveloped, double stranded DNA virus. The HHV8 virus is an enveloped, double stranded DNA virus.

Comment Here

Reference: Kaposi sarcoma

Kaposiform hemangioendothelioma & tufted angioma
Definition / general
  • Kaposiform hemangioendothelioma (KHE) is a rare and locally aggressive endothelial neoplasm that often involves the entire dermis and extends into the subcutaneous tissue
    • Characterized by proliferation of spindle shaped endothelial cells and associated capillary hemangioma-like lymphatic vessels
    • Predominantly presents in children, often at birth or shortly after birth
  • Tufted angioma (TA) is a benign vascular neoplasm that often occurs in children and often presents at birth or shortly after birth
  • TA, a more superficial lesion, is otherwise identical to KHE, with KHE representing a more aggressive form with higher morbidity; tufted angioma may spontaneously regress in some cases but KHE does not
  • KHE and TA both have strong association with Kasabach-Merritt phenomenon (KMP)
Essential features
  • Discrete lobules of capillaries in swirling growth pattern (glomeruloid structures) are typical
  • Lymphangioma-like areas, composed of lymphatic vessels at the periphery of nodules of spindle endothelial cells
ICD coding
  • ICD-O:
    • 9130/1 - kaposiform hemangioendothelioma
    • 9161/0 - acquired tufted angioma
  • ICD-11: 2F2Y & XH6PA4 - other specified benign cutaneous neoplasms & kaposiform hemangioendothelioma
Epidemiology
Sites
  • Most common sites are extremities; may involve the skin and subcutaneous tissue
  • Can occur in soft tissue and noncutaneous sites
  • Retroperitoneum and peritoneum are frequent extracutaneous sites involved by KHE
  • Head and neck, mediastinum and trunk involvement has also been reported (Am J Surg Pathol 2004;28:559, Int J Clin Exp Pathol 2015;8:13711)
Pathophysiology
  • Pathogenesis of KHE is poorly understood
  • May derive from the lymphatic endothelium (Am J Surg Pathol 2004;28:559)
    • Expresses the vascular markers CD31 and CD34
    • Expresses vascular endothelial growth factor receptor 3 (VEGFR3), required for lymphangiogenesis
    • Expresses the lymphatic markers D2-40 and PROX1
  • No reported association with human herpesvirus 8 (HHV8) infection
Etiology
  • Etiology is unknown
  • Surgical manipulation of KHE or trauma can stimulate Kasabach-Merritt phenomenon (Ann Plast Surg 2009;62:456, J Pediatr Surg 2002;37:E29)
  • Likely multifactorial with genetic factors contributing, although mutations are likely sporadic rather than germline
Clinical features
  • Usually occurs in infants and children, with 50% occurring in the first year of life
  • Cutaneous lesions present with atypical indurated, red to purple plaques with ill defined borders and occasional telangiectasias, without purpura or petechiae
  • Typically accompanied by hypertrichosis or hyperhidrosis overlying the lesion
  • Most tumors occur in the extremities; less frequently in the trunk, head and neck and mediastinal regions (Am J Surg Pathol 2004;28:559, Int J Clin Exp Pathol 2015;8:13711)
  • Deep lesions in the retroperitoneum or intrathoracic region may present as a bluish, purpuric hue on the overlying skin and may be mistaken for a bleeding disorder
  • Tumors can cause pain or functional impairment of the involved area (Int J Clin Exp Pathol 2015;8:13711)
  • Over 70% of KHE develop Kasabach-Merritt phenomenon, which consists of (J Pediatr 2013;162:142, Pediatr Clin North Am 2010;57:1085):
    • Consumptive coagulopathy
    • Thrombocytopenia
    • Risk is highest in retroperitoneal and intrathoracic tumors and large congenital lesions
  • KHE typically does not regress
  • Regional lymph node metastases are rare; no distant metastases reported to date
  • Mortality is approximately 10%, usually secondary to Kasabach-Merritt phenomenon (Am J Surg Pathol 2004;28:559)
Diagnosis
  • Clinical presentation (J Pediatr 2013;163:285):
    • KHE is suspected in an infant presenting with atypical indurated, red to purple plaques with ill defined borders; most often in the extremities and trunk
    • Lanugo hair hypertrichosis and increased sweating are clues for the diagnosis
  • Imaging (J Pediatr 2013;163:285):
    • Ultrasound is best for superficial and small lesions
    • MRI is generally the first choice assessment due to the deep infiltrating nature of tumors; MRI is also helpful in differentiating KHE from other vascular tumors but mainly is used to investigate the extent of disease and response to treatment
    • MRI is recommended in patients presenting with severe unexplained coagulopathy and thrombocytopenia, with coexisting cutaneous purpura, to identify deep seated KHE
  • Biopsy of the lesion for microscopic examination is possible and safe; biopsy is not recommended in the presence of severe Kasabach-Merritt phenomenon (Br J Dermatol 2018;179:457)
Laboratory
  • Laboratory evaluation includes complete blood count, platelet count, coagulation studies (prothrombin time [PT], partial thromboplastin time [PTT], fibrinogen, D dimer)
  • Kasabach-Merritt phenomenon is suspected when there is severe thrombocytopenia, hypofibrinogenemia and elevated D dimer
  • Liver and kidney function tests performed in acutely ill patients
  • Reference: J Pediatr 2013;163:285
Radiology description
  • On MRIs, KHE often displays ill defined margins and multiplanar extension with diffuse enhancement (Orphanet J Rare Dis 2020;15:39)
    • T1 weighted imaging shows isointensity compared with adjacent muscle
    • T2 weighted imaging reveals hyperintensity
    • Changes in adjacent bone or joint are common, including destruction of the cortex or epiphysis and joint invasion
Prognostic factors
  • Typically does not regress
  • Surgical excision of the tumor is curative in the majority of cases
  • Outcome is strongly associated with sites, tumor size and the progression of consumptive coagulopathy (J Pediatr 2013;162:142)
  • Mortality rate is about 10% in patients with Kasabach-Merritt phenomenon; however, it is decreasing due to better pharmacologic therapy and supportive care (N Engl J Med 2004;350:1764)
  • Most common cause of death includes hemorrhage, sepsis and organ failure
Case reports
Treatment
  • Various treatment modalities, including surgical resection, pulsed dye laser and multiple pharmacological agents have been reported based on limited case series, case reports and clinical experience without large observational studies or randomized trials
  • Surgical resection is considered cured; however, complete resection is not always possible due to poorly defined margins, involvement of vital organs or accompanied Kasabach-Merritt phenomenon
  • Pharmacologic therapy is recommended in KHE with symptomatic Kasabach-Merritt phenomenon and showed positive results, including reducing tumor mass, improving symptoms and resolving coagulopathies
  • Embolization may be useful in decreasing tumor mass before starting pharmacologic therapy
  • Radiation therapy is used as last resort treatment for tumors refractory to other therapies (J Pediatr 2013;163:285, Dermatol Pract Concept 2015;5:91, World J Pediatr 2018;14:322)
Clinical images

Contributed by Thuy L. Phung, M.D., Ph.D.
Tumor masses in the trunk

Tumor masses in the trunk

Tumor masses in the cheek

Tumor masses in the cheek

Gross description
  • Skin: ill defined violaceous plaques and occasional telangiectasias, without purpura or petechiae
  • Deep soft tissue: multiple nodules with associated prominent desmoplasia as tumor infiltrates
Gross images

Contributed by Thuy L. Phung, M.D., Ph.D.
Trunk kaposiform hemangioendothelioma

Trunk KHE

Microscopic (histologic) description
  • Glomeruloid structures are a characteristic feature of the tumor
  • Infiltrating sheets or nodules of spindled tumor cells arranged in short fascicles with slit-like vascular lumina
  • Swirling growth pattern of spindled cells gives a glomeruloid appearance
  • Glomeruloid structures contain red cell fragments, intracellular and extracellular hyaline granules, hemosiderin and microthrombi as evidence of red cells and platelets sequestration and destruction
  • Lymphangioma-like areas with thin walled ectatic vessels at the periphery of nodules of spindled cells; in some extremes, it can present as a discrete lymphangioma
  • Well formed vessels with pericyte component are present
  • Mitoses are present but tumor cells show minimal cytologic atypia
  • Tufted angioma usually involves the dermis but may extend to the fascia and muscle
    • Multiple lobules of tightly packed spindle endothelial cells with slit-like lumina throughout the dermis, creating a cannonball appearance, intermingled with scattered tufts of capillaries
    • Lymphangioma-like areas, composed of lymphatic vessels at the periphery of nodules of spindle endothelial cells
  • References: Am J Surg Pathol 1993;17:321, Am J Surg Pathol 2004;28:559, An Pediatr (Barc) 2015;83:201, Pediatr Dev Pathol 2000;3:556
Microscopic (histologic) images

Contributed by Thuy L. Phung, M.D., Ph.D.
Infiltrative tumor

Infiltrative tumor

Desmoplasia and hemorrhage

Desmoplasia and hemorrhage

Slit-like vascular lumina Spindle tumor cells

Slit-like vascular lumina

Infiltrating sheets growth pattern Infiltrating sheets growth pattern

Infiltrating sheets growth pattern


Atypical spindled cells

Atypical spindled cells

Nodularity appearance

Nodular appearance

Kaposi’s sarcoma-like area s

Kaposi sarcoma-like area

Caption

Capillary proliferation area

Extravasated erythrocytes

Extravasated erythrocytes

Thrombus

Thrombus


CD31

CD31

CD31

CD31

CD34

CD34

CD34

CD34

D2-40

D2-40

D2-40

D2-40


FLI1

FLI1

FLI1

FLI1

GLUT1

GLUT1

GLUT1

GLUT1

Positive stains
  • Expresses endothelial cell markers, including CD31, CD34, ERG and FLI1
  • Lymphatic markers (PROX1 and podoplanin) are expressed in peripheral lymphatic component (Kaposi-like areas), useful in differentiating it from other vascular lesions of infancy; glomeruloid structures lack these antigens (Am J Surg Pathol 2004;28:559)
Negative stains
Electron microscopy description
  • Spindle endothelial cells lining the slit-like channels have plump nuclei outstretching into the lumen; the basement membrane of these channels is thin and contains pericytes
  • Within the slit-like channels are platelets, lymphocytes and macrophages
  • Neoplastic cells contain cytoplasmic phagocytic vesicles (Ultrastruct Pathol 2013;37:452)
Molecular / cytogenetics description
Sample pathology report
  • Soft tissue, left face, biopsy:
    • Consistent with kaposiform hemangioendothelioma
    • Clinical history: facial mass; differential diagnosis: kaposiform hemangioendothelioma
    • Gross description: The specimen is received in a single container labeled with the patient's name and left facial mass. It consists of an aggregate of irregularly shaped, red-tan, hemorrhagic soft tissue measuring 1.5 x 0.7 x 0.2 cm. Portions of tissues are submitted for cytogenetic studies and electron microscopy and are snap frozen for possible future studies. The remaining tissue is wrapped in filter paper and submitted in cassettes 1 and 2 for microscopic examination.
    • Microscopic examination: 4 H&E slides. Immunohistochemical stains: CD31, CD34, GLUT1, D2-40 and FLI1. All controls are appropriate.
    • Sections of the left facial mass biopsy show interconnecting sheets and nodules of plump, spindled endothelial cells lining a network of proliferative capillaries and smaller rounded vessels with occasional slit-like appearance that contain erythrocytes. In some areas, the capillary network is markedly congested, getting a more cavernous appearance. Occasionally, a vague glomeruloid pattern is seen. Few of these smaller rounded vessels contain intravascular fibrin thrombi. In some areas, the interstitium shows hyalinizing changes. Extravasated red blood cells are seen, as well as hemosiderin laden macrophages. Mitotic activity is low. No significant cytologic atypia is seen. The spindle cells are positive for the vascular markers CD31, CD34, D2-40 and FLI1 and are negative for GLUT1.
Differential diagnosis
  • Kaposi sarcoma:
    • Older or immunocompromised patients
    • Very rare in children, except in Africa
    • Nodular stage presents predominantly as a solid eosinophilic spindle cell area that often lacks endothelial lining and associated with red blood cell extravasation
    • Lacking of hemangioma-like areas
    • Positive for HHV8
  • Infantile hemangioma:
    • Distinct lobules of capillary vessels
    • Lacks spindle cells and glomeruloid areas
    • Positive for GLUT1 and Lewis Y antigen
    • Regresses spontaneously
    • Not associated with Kasabach-Merritt phenomenon
  • Spindle cell hemangioma:
    • Occurs in distal extremities of adults
    • Biphasic tumor
    • Admixture of thin walled, congested cavernous vascular spaces and solid areas of spindled to epithelioid endothelial cells
    • Vacuolated (blister) endothelial cells
    • Large, calcified thrombi and thrombosis may be present
    • IDH1 mutations
Board review style question #1

Which of the following immunohistochemical stains is negative in kaposiform hemangioendothelioma?

  1. CD31
  2. CD34
  3. D2-40
  4. FLI1
  5. GLUT1
Board review style answer #1
Board review style question #2
Which of the following is true about kaposiform hemangioendothelioma?

  1. A small proportion of tumors occur in infant and young children
  2. May be associated with fatal consumptive coagulopathy
  3. Tumor is indistinguishable from Kaposi sarcoma and is positive for HHV8
  4. Tumor may regress completely without treatment
  5. Tumor does not metastasize
Board review style answer #2
B. May be associated with fatal consumptive coagulopathy

Comment Here

Reference: Kaposiform hemangioendothelioma

Keratin positive giant cell tumor / xanthogranulomatous epithelial tumor
Definition / general
  • Keratin positive giant cell tumor (KPGCT) is a rare, emerging entity with uncertain biologic potential that can affect soft tissue and bone (Mod Pathol 2022;35:1656)
Essential features
  • Keratin positive giant cell tumor can affect soft tissue and bone
  • Tumors with uncertain malignant potential can locally recur but rarely metastasizes
  • Characterized by the presence of keratin positive eosinophilic cells in a background of histiocytes, osteoclast type giant cells, Touton giant cells and mixed inflammatory cells with or without hemorrhage or hemosiderin deposition
  • It has been shown to harbor HMGA2::NCOR2 fusions
Terminology
  • Xanthogranulomatous epithelial tumor
  • Giant cell rich tumors with HMGA2::NCOR2 fusion
Epidemiology
Sites
  • Tumor can affect both soft tissues and bone
  • Soft tissue lesions typically affect the subcutaneous tissue, most commonly in the extremities
  • Other reported sites include the back and the head and neck area
  • When the bone is involved, it most commonly affects the vertebral bodies
  • References: Mod Pathol 2022;35:1656, Mod Pathol 2020;33:1889
Pathophysiology
  • Most of the reported cases have been found to have recurrent HMGA2::NCOR2 fusions
  • Histiocytes, giant cells and inflammatory cells, as well as the hemorrhage / hemosiderin deposition, may represent a reaction to the keratin positive mononuclear neoplastic cells
Etiology
  • Unknown
Clinical features
  • May present as superficial, deep or bone lesion
Diagnosis
  • Patients present with either subcutaneous soft tissue masses or with pathologic fractures
  • Imaging findings are not entirely specific
  • Needle core biopsy or excisional biopsy is diagnostic
  • References: Mod Pathol 2020;33:1889, Mod Pathol 2022;35:1656
Radiology description
  • For soft tissue lesions
    • Subcutaneous lesion, growing along the fascial plane with well circumscribed, slightly lobulated margins, with heterogenous echogenicity and increased internal vascularity (Mod Pathol 2020;33:1889)
  • For osseous lesions
Radiology images

Images hosted on other servers:
Subcutaneous soft tissue lesion

Subcutaneous soft tissue lesion

Prognostic factors
Case reports
Treatment
  • Surgical excision for soft tissue tumors
  • Curettage of primary bone tumors
  • Denosumab: a RANKL inhibitor that causes giant cell depletion and fibrosis
  • Pexidarnitib: CSF1R inhibitor (Am J Surg Pathol 2023;47:801)
Gross description
  • Average size: 4.0 cm
  • Well circumscribed, can be encapsulated or nonencapsulated
  • Can be slightly lobulated
  • Color ranges from tan-yellow to brown (Mod Pathol 2022;35:1656)
Gross images

Images hosted on other servers:
Well circumscribed, subcutaneous

Well circumscribed, subcutaneous

Frozen section description
  • On frozen section, it will show giant cell rich tumor with evenly distributed giant cells
  • Keratin positive giant cells may not be obvious
  • Differentiation of this lesion from other giant cell rich lesions is deferred for permanent section
Microscopic (histologic) description
  • Well circumscribed, encapsulated with associated lymphocytic cuff, arranged in lobes with evenly distributed osteoclast type giant cells and variable Touton giant cells (Mod Pathol 2022;35:1656)
  • Background shows foamy histiocytes, mononuclear inflammatory cells and eosinophils (Mod Pathol 2022;35:1656)
  • Mononuclear cells with round to oval nuclei and prominent nucleoli with brightly eosinophilic cytoplasm, distributed singly or in small clusters or cords (Mod Pathol 2020;33:1889)
  • Morphologic spectrum can range from xanthoma-like, showing hybrid features of xanthoma and giant cell tumor, to giant cell tumor-like
  • Stromal hemorrhage or hemosiderin can be seen
  • Infarct type necrosis can be seen
  • Mitotic count ranges from 1 to 7/high power field (Mod Pathol 2022;35:1656)
Microscopic (histologic) images

Contributed by Carina Dehner, M.D., Ph.D.
Well circumscribed

Well circumscribed

Osteoclast type giant cells

Osteoclast type giant cells

Touton giant cells are prominent in the xanthogranulomatous tumors

Prominent Touton giant cells

Epithelioid cells Epithelioid cells Epithelioid cells

Mononuclear epithelioid cells


Keratin AE1 / AE3 IHC

Keratin AE1 / AE3 IHC

CD68 IHC

CD68 IHC

Positive stains
Electron microscopy description
  • Mononuclear cells show
    • Intracytoplasmic thin parallel cytoskeletal filaments
    • Basement membrane deposition
  • Reference: Mod Pathol 2022;35:1656
Electron microscopy images

Contributed by Carina Dehner, M.D., Ph.D.
Ultrastructure of mononuclear cells

Ultrastructure of mononuclear cells

Molecular / cytogenetics description
  • HMGA2::NCOR2 fusion can be detected through next generation sequencing but may be hard to detect given the low number of neoplastic cells that are obscured by a large amount of inflammation
Sample pathology report
  • Skin and subcutaneous tissue, left forearm mass, resection:
    • Keratin positive giant cell rich tumor of soft tissue (see comment)
    • Tumor size: 2.5 cm in greatest dimension
    • Resection margin: free of lesion (0.5 cm from closest deep margin and 0.7 cm from closest peripheral margin)
    • Comment: Genetic testing shows a HMGA2::NCOR2 gene fusion, which confirms the above mentioned diagnosis.
Differential diagnosis
  • Soft tissue lesions:
    • Cutaneous fibrous histiocytomas (dermatofibroma):
      • Dermal based, unencapsulated lesion
      • Composed of spindle cell fibroblasts and histiocytes
      • May show scattered multinucleated giant cells
      • Ischemic type necrosis and mitotic figures can be seen
    • Giant cell tumor of soft tissue:
      • Well demarcated, not encapsulated multinodular proliferation of osteoclast type giant cells and mononuclear stromal cells, separated by fibrous septa and surrounded by a rim of metaplastic bone
      • Foamy macrophages can be seen
      • Lacks H3F3A gene mutation (found in giant cell tumor of bone)
    • Xanthogranuloma:
      • Well demarcated dermal based lesion, composed of foamy histiocytes with scattered Touton giant cells that may extend to the subcutaneous tissue
      • Can show infiltration of lymphocytes, plasma cells or eosinophils
  • Bone lesions (Int J Surg Pathol 2023 Jul 17 [Epub ahead of print]):
    • Giant cell tumor of bone:
      • Involves epiphysis
      • Histologically, it is a giant cell rich lesion with evenly distributed osteoclast type giant cells, admixed with mononuclear stromal cells with oval nuclei
      • Characterized by H3F3A gene mutation (positive for H3G34W IHC)
    • Aneurysmal bone cyst:
      • Involves metaphysis of long bone
      • Radiologically, multiloculated cystic lesion
      • Histologically, composed of multicyclic lesion, separated by fibrous septa with fibroblasts and randomly distributed osteoclast type giant cells
      • Characterized by the presence of USP6 gene mutation
    • Nonossifying fibroma:
      • Involves the metaphysis
      • Histologically, spindle cell proliferation, arranged in storiform pattern, admixed with randomly distributed osteoclast-like giant cells
      • KRAS and FGFR mutations have been reported
Board review style question #1

Keratin positive giant cell rich tumor is characterized by which of the following molecular alterations?

  1. GNAS mutation
  2. H3F3A gene mutation
  3. HMGA2::NCOR2 fusions
  4. USP6 gene alteration
Board review style answer #1
C. HMGA2::NCOR2 fusions. Answer A is incorrect because GNAS mutations are seen in fibrous dysplasia. Answer B is incorrect because H3F3A gene mutations are seen in giant cell tumor of bone. Answer D is incorrect because USP6 gene alteration is seen in aneurysmal bone cyst.

Comment Here

Reference: Keratin positive giant cell tumor / xanthogranulomatous epithelial tumor
Board review style question #2
What giant cell rich neoplasm is keratin positive?

  1. Aneurysmal bone cyst
  2. Giant cell tumor of bone
  3. Keratin positive giant cell tumor
  4. Tenosynovial giant cell tumor
Board review style answer #2
C. Keratin positive giant cell tumor. Keratin positive giant cell tumor is positive for keratin; the thought about the underlying cause for this concept is that the neoplastic cells are likely of epithelial origin; however, this is still under investigation. Answers A, B and D are incorrect because these tumors do not express keratin.

Comment Here

Reference: Keratin positive giant cell tumor / xanthogranulomatous epithelial tumor

Leiomyoma
Definition / general
Terminology
  • Cutaneous and pilar tumors are described in the Skin chapter
  • Some tumors previously considered to be leiomyomas are actually GI stromal (GIST) tumors
  • See also colon-tumor, esophagus, uterus chapters
Epidemiology
  • Skin and subcutis; also deep soft tissue, uterus (most common neoplasm in women)
  • Patients with multiple cutaneous leiomyomas may have autosomal dominant disorder (described under Cutaneous leiomyoma)
Case reports
Gross description
  • Deep tumors are well circumscribed, gray-white; usually large at presentation, up to 15 cm; may have myxoid change
Gross images

Contributed by @Andrew_Fltv on Twitter
Leiomyoma of deep soft tissue

Leiomyoma of deep soft tissue



Images hosted on other servers:

Myxoid subcutaneous tumor

Microscopic (histologic) description
  • Bundles or fascicles of spindled cells with eosinophilic and possibly fibrillary cytoplasm
  • Nuclei are blunt ended and elongated with fine chromatin, indistinct nucleolus and variable cytoplasmic vacuole at one end
  • Minimal atypia
  • Few mitotic figures
  • No coagulative tumor necrosis
Microscopic (histologic) images

AFIP images

Oblong nuclei have rounded ends

Bundles of elongate cells



Images hosted on other servers:

Calcified leiomyoma

Negative stains
Differential diagnosis

Leiomyosarcoma-general
Definition / general
  • Leiomyosarcoma (LMS) is a malignant mesenchymal tumor showing smooth muscle differentiation (J Clin Oncol 2018;36:144)
Essential features
  • Fascicles of eosinophilic spindled cells with blunt ended nuclei showing variable pleomorphism (Cancer 1981;48:1022, Am J Surg Pathol 2002;26:14)
  • Immunostaining for SMA, MSA, desmin or h-caldesmon
  • Dedifferentiated areas lack expression of myogenic markers (Histopathology 2011;59:1135)
  • Clinically aggressive neoplasms with frequent local recurrences and distant metastases
ICD coding
  • ICD-O: 8890/3 - leiomyosarcoma, NOS
  • ICD-11:
    • 2B58.0 & XH7ED4 - leiomyosarcoma of retroperitoneum or peritoneum & leiomyosarcoma, NOS
    • 2B58.Y & XH7ED4 - leiomyosarcoma, other specified primary site & leiomyosarcoma, NOS
    • 2B58.Z & XH7ED4 - leiomyosarcoma, unspecified primary site & leiomyosarcoma, NOS
Epidemiology
  • Leiomyosarcoma is one of the most common subtypes of malignant mesenchymal neoplasms and represents ~10 - 20% of all newly diagnosed soft tissue sarcomas (J Clin Oncol 2018;36:144)
  • Overall incidence of leiomyosarcoma increases with age and peaks at the seventh decade of life
  • Sex predilection greatly depends on tumor location, with women comprising a clear majority of patients with retroperitoneal and inferior vena cava leiomyosarcoma and men showing a slight predominance in noncutaneous soft tissue sites
Sites
Pathophysiology
  • Leiomyosarcoma belongs to the group of soft tissue sarcomas with complex and unbalanced karyotypes, which results in severe genomic instability (J Clin Oncol 2018;36:144)
  • Some of the most common changes in leiomyosarcoma occur in the form of loss in chromosomes 10q(PTEN) and 13q (RB1) and gain at 17p (TP53)
Etiology
  • There is no definite, identifiable, causative factor for leiomyosarcoma
  • Prior history of radiotherapy, which is one of the most significant risk factors for development of soft tissue sarcomas, can also lead to the development of leiomyosarcoma (J Natl Cancer Inst 1988;80:233)
  • Patients with genetic syndromes like hereditary retinoblastoma (RB1 gene deletion) and Li-Fraumeni syndrome (mutation in the TP53 gene) can develop leiomyosarcoma, amongst other soft tissue sarcomas (J Clin Oncol 2018;36:144)
Clinical features
  • Clinical presentation of leiomyosarcoma, as with other soft tissue sarcomas, is often associated with nonspecific symptoms caused by the displacement of structures, rather than invasion, in specific anatomic locations of the primary tumor and its metastases (J Clin Oncol 2018;36:144)
Diagnosis
  • Imaging approaches include magnetic resonance imaging (MRI) in soft tissue extremity / truncal tumors and contrast enhanced computed tomography (CT) scan for retroperitoneal lesions
  • Chest and abdominal CT scans are required in the initial workup, as hematogenous spread is a frequent event in leiomyosarcoma, with the lung and liver as 2 common sites of metastases (AJR Am J Roentgenol 1988;150:615)
  • Pretreatment biopsy is mandatory in extrauterine sites, with core biopsy as the preferred technique (J Clin Oncol 2018;36:144)
  • Detailed pathologic evaluation is typically performed after complete resection
  • Fine needle aspiration is inadequate to establish a diagnosis (J Surg Oncol 2010;102:523)
Laboratory
Radiology description
  • CT findings (Radiology 1984;152:133)
    • Generally heterogeneous
    • Commonly demonstrate central low attenuation representing necrosis
    • Calcification exceedingly rare
  • MRI findings (Diagn Interv Radiol 2015;21:4)
    • T1: isointense to muscle
    • T2 non-fat suppressed: intermediate to hypointense to neighboring fat
    • T2 FS: predominantly hyperintense
Radiology images

Contributed by Borislav A. Alexiev, M.D.

MRI of right forearm

MRI of abdomen

Prognostic factors
Case reports
Treatment
  • Staging of soft tissue sarcomas, including leiomyosarcoma, is important in guiding treatment
  • Treatment is best carried out in a specialized center with expertise in sarcoma care (Surg Oncol Clin N Am 2022;31:527)
  • Treatment planning begins with a multidisciplinary review of the patient’s history, all available radiographic images and the pathologic results from biopsy
  • Treatment plan is formulated upon the input from orthopedic and general surgeons, musculoskeletal radiologists, pathologists, medical oncologists and radiation oncologists
  • Goal of treatment is to control the symptoms, decrease tumor bulk and prolong survival (J Clin Oncol 2018;36:144)
  • Surgery (Adv Surg 2015;49:107)
    • Local control of soft tissue leiomyosarcoma is usually achieved with surgical resection
    • Achieving wide surgical margins is important in preventing local recurrence
  • Radiation therapy
    • Radiation therapy is an important additional treatment for improving rates of local control
    • Many tumors involve or are directly adjacent to vital structures and in these cases achieving a wide surgical margin is impossible
    • Radiation therapy can be delivered either preoperatively (neoadjuvant) or postoperatively (adjuvant)
      • Perioperative radiation therapy for soft tissue sarcoma is the gold standard of treatment for localized disease in extremities, trunk and head / neck region (J Clin Oncol 2018;36:144)
      • At this point, no consensus exists on the timing or benefit of perioperative radiation therapy for patients diagnosed with retroperitoneal soft tissue sarcoma
    • Radiation therapy can also be utilized as a means of palliative local control when extensive metastases have already occurred
  • Chemotherapy
    • Leiomyosarcoma is characterized by severe genomic instability, which results in multiple genetic aberrations; as a result, leiomyosarcoma is considered moderately sensitive to chemotherapy (Sarcoma 2010;2010:506182)
    • Neoadjuvant chemotherapy can help shrink the tumor, hence improving resectability, achieving negative margins and earlier control of metastatic disease
    • Adjuvant chemotherapy after surgery significantly improves the time to local and distant recurrence and overall recurrence free survival (Lancet 1997;350:1647)
    • Chemotherapy is the first line treatment in metastatic or unresectable leiomyosarcoma
Clinical images

Images hosted on other servers:

12 cm tibial tumor

Oral cavity tumor

25 cm epididymis mass

Internal jugular vein

Gross description
Gross images

Contributed by Borislav A. Alexiev, M.D.

Soft tissue mass

Inferior vena cava mass

Frozen section description
  • Cellular neoplasm that is composed of intersecting fascicles of spindle cells with bright eosinophilic cytoplasm and elongated, blunt ended (cigar shaped) nuclei (Eur J Cancer 2002;38:1218, Am J Clin Pathol 2006;125:555)
  • Nuclear pleomorphism, tumor necrosis and mitotic activity
Microscopic (histologic) description
  • Classic leiomyosarcoma (Eur J Cancer 2002;38:1218, Am J Clin Pathol 2006;125:555)
    • Spindle shaped cells with plump, blunt ended nuclei and moderate to abundant, pale to brightly eosinophilic fibrillary cytoplasm
    • Cells are set in long intersecting fascicles parallel and perpendicular to the plane of section
    • Some tumors show areas with storiform or palisaded patterns
    • Moderate nuclear pleomorphism is usually noted, although pleomorphism may be focal
    • Mitotic figures, including atypical ones, are easy to find
    • Tumors usually show diffuse hypercellularity
    • Focal fibrosis, myxoid change and hyalinized hypocellular areas can be seen
    • Tumor cell necrosis is often present
    • Unusual features in soft tissue leiomyosarcoma include multinucleated osteoclast-like giant cells, granular cytoplasmic change and epithelioid morphology
  • Pleomorphic leiomyosarcoma (Am J Surg Pathol 2001;25:1030)
    • Composed of pleomorphic cells with or without abundant eosinophilic or fibrillary cytoplasm in > 66% of the maximum cut surface of the tumor, accompanied by an ordinary leiomyosarcoma fascicular area covering < 33%
    • Storiform pattern in > 50% of cases
    • Stromal hyalinization
    • Chronic inflammatory infiltrate
    • Myxofibrosarcoma-like (myxoid malignant fibrous histiocytoma-like) areas
  • Myxoid leiomyosarcoma (Am J Surg Pathol 2000;24:927)
    • Extensively myxoid stroma (> 50% of the tissue examined)
    • Tumor cells are predominantly spindled
    • 3 major histologic architectures: fascicular, reticular / microcystic and myxofibrosarcoma-like
    • Areas of conventional leiomyosarcoma are usually present at least focally
  • Dedifferentiated leiomyosarcoma (Histopathology 2011;59:1135)
    • Tumor showing features of low grade leiomyosarcoma associated with a discrete undifferentiated component lacking morphological or immunophenotypic features of myogenic differentiation
  • Diagnostic threshold for diagnosing low grade leiomyosarcoma is much lower outside the uterus than in the uterus
  • Histologic grade (French Federation of Cancer Centers Sarcoma Group [FNCLCC]) (J Clin Oncol 1997;15:350)
Microscopic (histologic) images

Contributed by Borislav A. Alexiev, M.D.

Intersecting fascicles

Blunt ended nuclei

Nuclear pleomorphism

Mitotic activity

Necrosis

Osteoclast-like giant cells


Pleomorphic tumor cells

Dedifferentiation

Desmin

Caldesmon

Muscle specific actin

Loss of myogenic differentiation

Cytology description
  • Leiomyosarcoma, classical variant (Diagn Cytopathol 2003;28:119)
    • Various proportions of spindle shaped, cohesive, small or large sized cells arranged in parallel alignment
    • Blunt ended nuclei
    • Cytoplasm varies from fibrillary to granular to vacuolar
  • Leiomyosarcoma, epithelioid variant
  • Leiomyosarcoma, pleomorphic variant
    • Variably sized and shaped cells, often including multinucleated giant cells with atypical nuclei
  • Leiomyosarcoma, myxoid variant
    • Large amounts of background myxoid matrix containing large spindle shaped and giant cells
  • Intranuclear inclusions and mitotic figures are occasionally seen, as well as stromal fragments
Cytology images

Contributed by Taylor Bronson, M.D.

Spindle cells

Nuclear atypia

Negative stains
Molecular / cytogenetics description
  • Complex karyotypes with numerous chromosomal gains and losses (JCO Precis Oncol 2020;4:PO.20.00040)
  • Losses involving tumor suppressors TP53 (17p13.1), RB1 (13q14.2) and PTEN (10q23.31) (Am J Pathol 2010;177:2080)
  • TP53 is mutated in as many as 50% of sporadic leiomyosarcomas
  • Homozygous copy loss of CDKN2C at chromosome 1p32.3b (JCO Precis Oncol 2020;4:PO.20.00040)
    • CDKN2C null leiomyosarcoma defines a genomically distinct tumor that may have prognostic or therapeutic clinical implications, including the possible use of specific cyclin dependent kinase inhibitors
Sample pathology report

  • Right thigh mass, core biopsy:
    • Leiomyosarcoma, FNCLCC grade 2 (see comment)
    • Comment: The neoplasm is composed of cellular, intersecting fascicles of spindle cells with bright eosinophilic cytoplasm and elongated blunt ended (cigar shaped) nuclei. Moderate nuclear pleomorphism and mitotic activity (up to 13 mitoses/10 high power fields) are noted. There is focal tumor necrosis (< 5%). Immunohistochemical stains for SMA, desmin and h-caldesmon are positive in tumor cells. These pathologic findings support the diagnosis above.
    • Leiomyosarcomas of soft tissue are aggressive neoplasms with frequent local recurrence and distant metastases. The most important prognostic factors are histologic grade, tumor location and size.
Differential diagnosis
Board review style question #1

A 57 year old woman presented with a mass in the soft tissue of the left thigh. H&E stains show a tumor composed of spindle shaped cells with plump, blunt ended hyperchromatic nuclei and moderate pale to brightly eosinophilic fibrillary cytoplasm. The cells are set in long intersecting fascicles parallel and perpendicular to the plane of the section. Mitotic figures, including atypical ones, are present.

Immunohistochemical stains for h-caldesmon, SMA and desmin are positive in tumor cells, while all of the following are negative: TLE1, CD99, HMB45, MelanA, S100, SOX10, EMA and AE1 / AE3.

Which of the following is most likely the correct diagnosis?

  1. Leiomyoma
  2. Leiomyosarcoma
  3. Malignant peripheral nerve sheath tumor
  4. PEComa
  5. Synovial sarcoma (monophasic)
Board review style answer #1
B. Leiomyosarcoma

Comment Here

Reference: Leiomyosarcoma
Board review style question #2
What is the most common gene mutation in soft tissue leiomyosarcoma?

  1. EWSR1
  2. FUS
  3. NF1
  4. PLAG1
  5. TP53
Board review style answer #2
E. TP53

Comment Here

Reference: Leiomyosarcoma

Lipoblastoma / lipoblastomatosis
Definition / general
  • Benign, rapidly growing neoplasm composed of embryonal white fat with variable degrees of myxoid changes (Mod Pathol 2021;34:584, J Clin Med 2022;11:1938)
  • Occurs predominantly in infancy, early childhood and very rarely in adulthood (J Clin Med 2022;11:1938)
  • Lipoblastoma: superficial, localized, well circumscribed; complete excision easy; common type
  • Lipoblastomatosis: deep, diffuse, irregular borders; complete excision difficult; rare type (J Clin Med 2022;11:1938)
Essential features
  • Benign, rapidly growing neoplasm that is commonly superficial and well defined
  • 75 - 90% are seen before the age of 3 years, with male predilection (M:F = 2.8:1)
  • Mostly occurs in trunk and extremities
  • Lobular architecture with intersecting fibrovascular septa, myxoid changes and a spectrum of fat cell maturation
  • Positive stains are S100, CD34 (adipocytes and vascular network), desmin (mesenchymal cells)
Terminology
  • Lipoblastoma: well circumscribed and superficial (J Clin Med 2022;11:1938)
  • Lipoblastomatosis: deep seated with ill defined borders (J Clin Med 2022;11:1938)
  • Not recommended: fetal lipoma, fetal fat tumor, fetocellular lipoma, embryonal lipoma, congenital lipomatoid tumor, lipoblastic tumor of childhood
ICD coding
  • ICD-O: 8881/0 - lipoblastomatosis
  • ICD-10: D17.9 - benign lipomatous neoplasm, unspecified
  • ICD-11: 2E80.1 & XH8L55 - lipoblastoma / lipoblastomatosis
Epidemiology
Sites
Pathophysiology
  • Exhibits pseudodiploid or hyperdiploid karyotypes combined with structural alteration of 8q11-q13, which leads to PLAG1 rearrangement (J Clin Med 2022;11:1938)
  • Numerical change in one or more extra copies of chromosome 8, with or without concurrent rearrangement of 8q11-q13 (J Clin Med 2022;11:1938)
Etiology
Clinical features
Diagnosis
Radiology description
Radiology images

Contributed by Nasir Ud Din, M.B.B.S.
Lipomatous lesion with septations

Lipomatous lesion with septations



Images hosted on other servers:
Circumscribed, lobulated hyperechoic mass

Circumscribed, lobulated hyperechoic mass

Localized mass of soft tissue, without calcification

Localized mass of soft tissue, without calcification

T1 fat saturation (MRI)

T1 fat saturation (MRI)

Cervical mass, greasy density

Cervical mass, greasy density

Prognostic factors
Case reports
Treatment
Clinical images

Images hosted on other servers:
Right lower leg mass

Right lower leg mass

Intraoperative aspect of lipoblastoma

Intraoperative aspect of lipoblastoma

Gross description
Gross images

Contributed by Nasir Ud Din, M.B.B.S.
Gross appearance

Round mass

Cut surface

Cut surface

Gross appearance

Multilobulated mass

Microscopic (histologic) description
Microscopic (histologic) images

Contributed by Nasir Ud Din, M.B.B.S.
Lobulation Lobulation

Lobulation

Vasculature

Vasculature

Myxoid background Myxoid background

Myxoid background

Lipoblasts in various stages

Lipoblasts in various stages


Lipoblasts and mesenchymal cells

Lipoblasts and mesenchymal cells

Septation

Septation

Zonation

Zonation

Primitive mesenchymal cells

Primitive mesenchymal cells

Desmin IHC stain

Desmin IHC stain

CD34 IHC stain

CD34 IHC stain

Virtual slides

Images hosted on other servers:
Lipoblastoma / lipoblastomatosis

Lipoblastoma / lipoblastomatosis

Cytology description
Cytology images

Images hosted on other servers:
Mature adipocytes and lipoblasts

Mature adipocytes and lipoblasts

Myxoid stromal with capillaries

Myxoid stromal with capillaries

Univacuolated and multivacuolated lipoblasts

Univacuolated and multivacuolated lipoblasts

Positive stains
Negative stains
Molecular / cytogenetics description
  • Clonal rearrangements involving chromosomal region 8q11 > q13 (8q12) is the hallmark (BMC Res Notes 2018;11:42)
  • Oncogene PLAG1 (pleomorphic adenoma gene 1) is located on band 8q12 (BMC Res Notes 2018;11:42)
  • Other rare fusion partners of PLAG1 are COL1A2 (7q21.3), HAS2 (8q24.13), RAD51B (14q24.1), COL3A1 (2q32.2), RAB2A (8q12.1-q12.2) and BOC (3q13.2) (Mod Pathol 2021;34:584)
  • Novel reported fusions are HNRNPC::PLAG1, SRSF3::PLAG1, PCMTD1::PLAG1, YWHAZ::PLAG1, CTDSP2::PLAG1 and PPP2R2A (Mod Pathol 2021;34:584)
Molecular / cytogenetics images

Images hosted on other servers:
PLAG1 RNA expression

PLAG1 RNA expression

Molecular confirmation of fusions

Molecular confirmation of fusions

FISH / CISH of PLAG1 gene FISH / CISH of PLAG1 gene

FISH / CISH of PLAG1 gene

Videos

Lipoblastoma histopathology

Sample pathology report
  • Soft tissue, thigh, left, excisional biopsy:
    • Benign adipocytic neoplasm with features favoring lipoblastoma (see comment)
    • Tumor size: 7 x 6 x 4 cm
    • Margins are free of tumor
    • Comment: Histological examination shows an adipocytic neoplasm with a lobular architecture by intersecting fibrous septa and zonation along with myxoid areas. A spectrum of fat cell maturation, from primitive stellate to spindled mesenchymal cells, multivacuolated lipoblasts to mature fat cells are seen. Immunohistochemically, the tumor cells are showing positivity for S100 and CD34 and are negative for MDM2, CKD4 and p16. This constellation of morphological and immunohistochemical features strongly supports the diagnosis of lipoblastoma. This is a benign, rapidly growing soft tissue neoplasm prone to recur unless completely excised.
Differential diagnosis
Board review style question #1

A 12 month old boy presents with a large, painless mass at lateral part of the left thigh. Excisional biopsy shows a well defined lobulated mass with fibrovascular septa, myxoid areas, aggregates of spindle cells and a mixture of mature and immature adipocytes. The adipocytic cells are positive for CD34 and CD56. The desmin is positive in spindle cells. The tumor cells are negative for MDM2 and p16. Which of the following is true regarding the above scenario?

  1. Adipocytic tumor with low malignant potential
  2. Findings are compatible with myxoid liposarcoma
  3. Radiological examinations are the main diagnostic tool
  4. The tumor cells are also positive for CKD4 IHC stain
  5. The tumor mainly exhibits PLAG1 rearrangement
Board review style answer #1
E. The tumor mainly exhibits PLAG1 rearrangement

Comment Here

Reference: Lipoblastoma / lipoblastomatosis
Board review style question #2
Which of the following is true about lipoblastoma / lipoblastomatosis?

  1. It is a highly infiltrative lesion, which leads the majority to recur
  2. It is common in women and complete resection is the treatment of choice
  3. It is the most common benign neoplasm with no chance of recurrence
  4. It usually appears as a superficial, round, lobulated mass with male predilection
  5. Negative CD34 IHC stain confirms the diagnosis of lipoblastoma
Board review style answer #2
D. It usually appears as a superficial, round, lobulated mass with male predilection

Comment Here

Reference: Lipoblastoma / lipoblastomatosis

Lipofibromatosis
Definition / general
  • Pediatric tumor with predilection for hands and feet, composed predominantly of mature adipose tissue with proliferative fibroblasts in adipose septa (Stanford University)
  • Previously called infantile fibromatosis of non-desmoid type
  • First described in 2000 using "lipofibromatosis" (Am J Surg Pathol 2000;24:1491)
Epidemiology
  • Rare childhood tumor (first surgery usually at age 1), 2/3 male, often of distal extremities
Clinical features
  • Associated with macrodactyly of foot (Foot Ankle 1991;12:40)
  • Clinically resembles lymphatic malformation or lymphedema
  • Recurs locally due to lack of securing clean margins at resection, no metastases
Case reports
Gross description
  • White-tan or yellow with obvious fatty component, 1 - 3 cm
Microscopic (histologic) description
  • Lobules of mature adipose tissue with fibroblastic foci consisting of bland fibroblasts involving adipose septa with a preserved lobular architecture
  • Often have minute small univacuolated cells at interface between fibroblasts and adipose
  • No atypia, no/rare mitotic figures
  • Rarely pigmented cells are associated with fibroblastic element; resemble those in Bednar tumor, pigmented neurofibroma, nevi
Microscopic (histologic) images

Contributed by Mark R. Wick, M.D.
Positive stains
Negative stains
Molecular / cytogenetics description
Differential diagnosis

Lipoma
Definition / general
  • Benign tumor composed of mature adipocytes
Essential features
  • Most common soft tissue tumor
  • Mostly subcutaneous, < 5 cm
  • Radiologically, grossly and microscopically same as normal fat
  • In large (> 10 cm) and deep seated / retroperitoneal tumors, exclusion of MDM2 amplification is required for diagnosis
ICD coding
  • ICD-O: 8850/0 - lipoma, NOS
  • ICD-11: 2E80.0Y - lipoma, other specified site
Epidemiology
Sites
  • Superficial (subcutaneous) soft tissue
  • Upper back, proximal extremities and abdominal region
  • Subset deep seated (intramuscular, parosteal, visceral)
Pathophysiology
Etiology
  • Unknown at this time
Clinical features
Diagnosis
Radiology description
Radiology images

Contributed by Salvatore Lorenzo Renne, M.D.
Isointense mass on MRI

Isointense mass on MRI

Discrete incapsulated mass on CT

Discrete incapsulated mass on CT

Prognostic factors
  • Recurrence < 5%
  • Incomplete surgery, higher local recurrence
Case reports
Treatment
  • Simple excision
Clinical images

Contributed by Mark R. Wick, M.D.

Superficial

Gross description
  • Well circumscribed
  • Nodular
  • Cut surface homogeneous
  • Fatty appearance
  • Usually small (< 5 cm)
  • Osteolipoma and chondrolipoma can be recognized grossly
Gross images

Contributed by Salvatore Lorenzo Renne, M.D. and AFIP images
Lipoma gross appearance

Lipoma gross appearance

Yellow adipose tissue within skeletal muscle

Microscopic (histologic) description
  • Proliferation of mature adipocytes
  • Paucicellular fibrous septa can be present
  • Fat necrosis is often found in larger tumor
  • Skeletal muscle fibers are infiltrated in intramuscular lipoma
  • Subtypes
    • Intramuscular lipoma
    • Chondrolipoma
  • Visceral sites
    • Submucosal lipoma of the colon
    • Endobronchial lipoma
Microscopic (histologic) images

Contributed by Salvatore Lorenzo Renne, M.D., Mark R. Wick, M.D., Charanjeet Singh, M.D. and AFIP images
Superficial, well circumscribed neoplasm

Superficial, well circumscribed neoplasm

Mature adipose tissue

Mature adipose tissue

Paucicellular fibrous septa

Paucicellular fibrous septa

Mature adipose proliferation

Mature adipose proliferation

Endobronchial lipoma

Endobronchial lipoma

Osteolipoma

Osteolipoma


Intramuscular type

Lipoma

Normal adult fat cells

Intramuscular lipoma

Adipocytes and atrophic muscle

Virtual slides

Images hosted on other servers:

Lipoma with fat necrosis

Intussusception by submucosa lipoma

Sigmoid polyp lipoma

Cytology description
  • Mature adipocytes with single lipid large droplet
  • Inconspicuous nucleus
Cytology images

Images hosted on other servers:

Adipocytes

Positive stains
Molecular / cytogenetics description
Molecular / cytogenetics images

Contributed by Salvatore Lorenzo Renne, M.D.
<i>MDM2</i> nonamplified

MDM2 nonamplified

Videos

Lipoma and its differential diagnosis

Sample pathology report
  • Soft tissue, subcutaneous trunk, excision:
    • Lipoma
  • Soft tissue, deep (deltoid), excision:
    • Lipoma, intramuscular (see comment)
    • Comment: Sections show a mature adipocytic neoplasm. No cytologic atypia or mitotic activity is identified. Immunohistochemical stain shows negativity for MDM2. Fluorescence in situ hybridization does not show amplification of MDM2 gene.
Differential diagnosis
  • Angiolipoma:
    • Typically composed of two elements: mature adipocytes and branching capillary sized vessels, which often contain fibrin thrombi
    • Relative proportions of adipocytes and vessels varies and some lesions are almost completely composed of vascular channels
  • Spindle cell / pleomorphic lipoma:
    • Head, neck, back
    • Male
    • Spindle and pleomorphic cells
    • Ropy collagen
    • CD34 strong IHC positivity
    • Rb loss by IHC (positive internal control of blood vessels)
  • Atypical lipomatous tumor / well differentiated liposarcoma:
    • Size > 10 cm
    • Deep seated
    • Atypical cells
    • MDM2 IHC positivity
    • MDM2 gene FISH cluster amplification
  • Mobile encapsulated adipose tissue (MEAT)
Board review style question #1

The tumor shown above is found in chest subcutis of a 58 year old woman. What is the most likely diagnosis?

  1. Angiolipoma
  2. Atypical lipomatous tumor
  3. Lipoma
  4. Normal fat
  5. Spindle cell lipoma
Board review style answer #1
C. Lipoma. The mass is in the subcutis, small (< 5 cm), well circumscribed and homogenously composed of adipocytes.

Comment Here

Reference: Lipoma
Board review style question #2
Which of the following is true about a lipoma?

  1. Diagnosis is often made without immunohistochemistry
  2. Highly infiltrative lesion, which leads the majority to recur
  3. It is the most common benign neoplasm
  4. It is usually large (> 10 cm) and located in deep soft tissue or retroperitoneum
  5. Patient always reports a history of trauma
Board review style answer #2
A. Diagnosis is often made without immunohistochemistry. Diagnosis can be done on H&E in most cases and in the appropriate clinicopathological setting (i.e. small, superficial, homogeneous lesion, composed of uniform adipocytic cells without atypia).

Comment Here

Reference: Lipoma

Lipomatosis
Definition / general
  • Diffuse overgrowth of mature adipose tissue
Epidemiology
  • Rare disorder of children under 2 years or adults
  • Associated with obesity, Cushing’s disease, steroid therapy, protease inhibitors for HIV
  • Familial multiple lipomatosis: rare, usually autosomal dominant; multiple lipomas of trunk and extremities with relative sparing of the head and shoulders; due to translocation involving high-mobility-group protein isoform I-C on chromosome 12 and the lipoma preferred partner gene on chromosome 3 (An Bras Dermatol 2012;87:324)
Sites
  • Limb (may cause massive enlargement) or trunk
Case reports
Treatment
  • Palliative surgical removal of excess fat
  • May recur
  • May lead to amputation of extremity due to distortion or loss of function
Gross description
  • Poorly circumscribed aggregates of normal appearing fat in subcutaneous and skeletal muscle (but not confined to muscle)
  • Does not affect nerves
Microscopic (histologic) description
  • Sheets and lobules of white adipose tissue that may infiltrate skeletal muscle
  • May involve bone
Positive stains
Additional references

Lipomatosis of nerve
Definition / general
  • Infiltration of epineurium of a major nerve by adipose and fibrous tissue
Terminology
  • Also called fibrolipoma of nerve, fibrolipomatous hamartoma of nerve, macrodystrophia lipomatosa
Epidemiology
  • May be noted at birth
  • Almost always age 30 years or less
  • Associated with macrodactyly (abnormal enlargement of digits) innervated by affected nerve in 30 - 67%
Sites
  • 85% have involvement of median nerve and its digital branches in hand, wrist and forearm (Histopathology 1994;24:391)
  • Also ulnar nerve
Radiology description
  • T1 weighted images on MR imaging reveal a fatty mass that is evenly distributed between nerve bundles and is seen running along individual nerves
  • Often described as having a "coaxial cable-like" appearance on axial scans (Acta Radiol 2003;44:326)
Radiology images

Contributed by Mark R. Wick, M.D.

Xray



Images hosted on other servers:

Lipomatosis of
the median nerve
in a patient with
macrodactyly

Lipomatosis of
the median nerve
in a patient without
macrodactyly

Case reports
Treatment
  • Benign but often no effective treatment as resection causes sensory and motor deficits (J Neurosurg 1998;89:683)
  • Carpal tunnel release may relieve symptoms of median nerve involvement
  • Amputation if severe deformity
  • May recur in 33 - 60% if incomplete resection
Clinical images

Images hosted on other servers:

Median nerve

Gross description
  • Fusiform enlargement of nerve by yellow adipose tissue, confined within epineurium
Gross images

Images hosted on other servers:

Lipomatosis of
the median nerve
in a patient with
macrodactyly

Lipomatosis of
the median nerve
in a patient without
macrodactyly

Microscopic (histologic) description
  • Infiltration of epineurium and perineurium by adipose and fibrous tissue (collagen), causing enlargement of nerve
  • Concentric perineurial fibrous tissue and pseudo-onion bulb formation
  • Occasionally metaplastic bone
Microscopic (histologic) images

Contributed by Charanjeet Singh, M.D., Mark R. Wick, M.D., Saroona Haroon, M.D. and Geoffrey A. Talmon, M.D. (Case #158)

Lipoma of nerve

Various images

Various images


Lipomatosis of nerve



Images hosted on other servers:

Lipomatosis of
the median nerve
in a patient without
macrodactyly

Angiolipoma in the
subcutaneous thigh
of a 21 year old man

Differential diagnosis

Low grade fibromyxoid sarcoma
Definition / general
  • Low grade sarcoma with fibrous and myxoid areas, whorled growth pattern, low cellularity, bland fibroblastic cells and curvilinear or arcuate vessels
Essential features
Terminology
  • Also called Evans tumor, hyalinizing spindle cell tumor with giant rosettes (a unique morphologic pattern seen in some low grade fibromyxoid sarcomas) (Am J Surg Pathol 2003;27:1229)
Epidemiology
  • Usually occurs in young to middle aged adults (median age 34 years, range 3 - 78 years)
  • Slight male predominance
Sites
  • Trunk and deep extremities (thigh most common site), also intrathoracic (Hum Pathol 2008;39:623)
  • Rarely in retroperitoneum and mediastinum
Clinical features
  • Prolonged preclinical stage
  • Slow growing, painless soft tissue mass (1 - 18 cm)
  • Earlier studies suggested only a small subset metastasized but a more recent study with long term follow up reported recurrence in 64%, metastasis in 45%, death from disease in 42%; patients with positive / uncertain margins more likely to have recurrence (up to 15 years later; median 3.5 years); metastases usually to lungs, pleura, chest wall (up to 45 years later; median 5 years) (Am J Surg Pathol 2011;35:1450)
Radiology description
Prognostic factors
Case reports
Treatment
  • Complete excision with wide margins
  • Resection of pulmonary metastases (metastasectomy) may be considered
  • Long term clinical follow up is prudent, due to potential of very late recurrence or metastasis (decades after initial diagnosis)
Gross description
  • Often large (median ~9 cm), well circumscribed, fibromyxoid cut surface, may be grossly infiltrative
Gross images

Contributed by Mark Rodacker, M.D.

Low grade fibromyxoid sarcoma



Images hosted on other servers:

Tumor of falciform ligament

Well circumscribed tumor of leg

Well demarcated tumor

Microscopic (histologic) description
  • Low to moderately cellular, bland fusiform or spindled cells with focal to diffuse whirling in heavily collagenized stroma with abrupt transition to myxoid areas
  • 45% have epithelioid areas
  • 40% contain poorly formed but large collagen rosettes
  • Often infiltrates adjacent skeletal muscle
  • Occasionally has areas of increased cellularity, atypia, necrosis or mitotic activity characteristic of intermediate to high grade sarcoma
  • Recurrences may show increased cellularity and mitotic activity
Microscopic (histologic) images

Contributed by Raul S. Gonzalez, M.D., Mark Rodacker, M.D., AFIP and @JMGardnerMD on Twitter
Low grade fibromyxoid sarcoma Low grade fibromyxoid sarcoma Low grade fibromyxoid sarcoma Low grade fibromyxoid sarcoma

Low grade fibromyxoid sarcoma

Low grade fibromyxoid sarcoma with transformation

Low grade fibromyxoid sarcoma with transformation


Low grade fibromyxoid sarcoma


Low grade fibromyxoid sarcoma with giant rosettes

Low grade fibromyxoid sarcoma with giant rosettes

Swirling growth pattern is storiform

Fibrous and myxoid areas

Relatively linear cell arrangement


Low grade fibromyxoid sarcoma Low grade fibromyxoid sarcoma Low grade fibromyxoid sarcoma Low grade fibromyxoid sarcoma Low grade fibromyxoid sarcoma

Low grade fibromyxoid sarcoma

Cytology description
Cytology images

Images hosted on other servers:

Ovoid to spindle cells

Hypercellular, 3D tissue fragment

Bland spindle cells

Cytologic features

Positive stains
Electron microscopy description
  • Fibroblastic differentiation
Electron microscopy images

Images hosted on other servers:

Elongated irregular nucleus

Molecular / cytogenetics description
Sample pathology report
  • Soft tissue, left thigh, resection:
    • Low grade fibromyxoid sarcoma (8.3 cm) (see comment)
    • Margins of resection unremarkable.
    • Comment: This uncommon soft tissue tumor requires long follow up, as it sometimes metastasizes many years after initial presentation. An immunohistochemical stain for MUC4 is positive.
Differential diagnosis
  • Desmoid type fibromatosis:
    • Usually lacks myxoid areas (sometimes can be myxoid, though), fibrous cells are aligned in broad sweeping fascicles, straighter, cells appear more like reactive fibroblasts, distinct ectatic vessels present, diffuse or occasionally focal nuclear beta catenin staining (Am J Surg Pathol 2005;29:653)
  • Fibrosarcoma:
    • No myxoid component; herringbone fascicular pattern, a diagnosis of exclusion that should be made with much hesitation (Histopathology 2006;49:152)
  • Myxofibrosarcoma:
    • More myxoid and less fibrous, more nuclear pleomorphism and hyperchromatism (in contrast to low grade fibromyxoid sarcoma, which is almost always bland and monomorphic with little to no pleomorphism), more developed vascular network, tumor cells aggregate around vessels (Histopathology 2004;45:29)
  • Myxoid neurofibroma:
    • Wavy nuclei, background of thick collagen bundles, S100+
  • Nodular fasciitis:
    • Tissue culture histology, extravasated erythrocytes, myxoid cystic degeneration
Board review style question #1

Which of the following is true about low grade fibromyxoid sarcoma?

  1. It appears related to synovial sarcoma
  2. It has a characteristic gene fusion
  3. It is often metastatic at presentation
  4. The fibrous and myxoid areas are intimately intermixed on histology
Board review style answer #1
B. It has a characteristic gene fusion - t(7;16)(q32-34;p11) FUS-CREB3L2

Comment Here

Reference: Low grade fibromyxoid sarcoma
Board review style question #2
Which of the following immunohistochemical stains is considered sensitive and specific for low grade fibromyxoid sarcoma?

  1. h-caldesmon
  2. MNF116
  3. MUC4
  4. TLE1
Board review style answer #2

Low grade myofibroblastic sarcoma
Definition / general
  • Rare tumor composed predominantly of malignant myofibroblasts
Essential features
  • Deep intramuscular tumor of head and neck (tongue and oral cavity) and extremities (1.5 - 17 cm), rarely abdominopelvic (J Clin Pathol 2008;61:301)
  • Commonly recurs, only rarely metastasizes; recurrences may be higher grade
  • No corresponding high grade myofibroblastic sarcoma; such a tumor would fit under the designation of undifferentiated pleomorphic sarcoma
  • No uniformly accepted criteria for diagnosis; some authors suggest electron microscopy appearance is the gold standard (Int J Surg Pathol 2013;21:29)
Terminology
  • Also called myofibrosarcoma
Epidemiology
  • Any age but predominantly patients in their 30s or 40s; slight male predominance
Case reports
  • 49 year old woman with laryngeal mass (Case #310)
Treatment
  • Complete excision
Gross description
  • Firm, pale, fibrous cut surface, ill defined margins
Microscopic (histologic) description
  • Circumscribed to diffusely infiltrative with fascicles or storiform growth of spindled tumor cells
  • Cells have ill defined pale eosinophilic cytoplasm, fusiform nuclei that are elongated or wavy with evenly distributed chromatin or round and vesicular with indentations and small nucleoli
  • At least focal moderate nuclear atypia with hyperchromasia and irregular nuclear membranes
  • Collagenous matrix with prominent hyalinization
  • May have numerous thin walled capillaries
  • 1 - 6 mitoses/10 high power fields
  • No histiocytic giant cells or prominent inflammation
Microscopic (histologic) images

Contributed by Mark R. Wick, M.D.

Various images



Case #310

Desmin

SMA



Various images



Images hosted on other servers:

SMA+, vimemtin+, fibronectin+, type IV collagen-

Femur, hypocellular area with a proliferation of spindle cells, SMA

Positive stains
Negative stains
Electron microscopy description
  • Myofibroblastic features of indented and clefted nuclei, variable rough endoplasmic reticulum, discontinuous basal lamina, fibronexus junctions (Ultrastruct Pathol 2008;32:97)
Electron microscopy images

Contributed by Mark R. Wick, M.D.
Molecular / cytogenetics description
  • Gains at 1p11 → p36.3 (66%), 12p12.2 → p13.2 (45%), 5p13.2 → p15.3 (31%), +22 (28%), loss at 15q25 → q26.2 (24%) (Am J Clin Pathol 2009;131:701)
Sample pathology report
  • Tongue, resection:
    • Low grade myofibroblastic sarcoma (3.8 cm) (see comment)
    • Margins of resection unremarkable.
    • Comment: Immunohistochemical stains for SMA and desmin are positive. The overall histologic and immunohistochemical findings are most consistent with low grade myofibroblastic sarcoma.
Differential diagnosis
  • Fibromatosis:
    • Minimal nuclear atypia, usually negative for myogenic markers (but may sometimes express them focally)
  • Fibrosarcoma:
    • May have focal (not diffuse) myofibroblastic differentiation; herringbone fascicular pattern, a diagnosis of exclusion that should be made with much hesitation (Histopathology 2006;49:152)
  • Inflammatory myofibroblastic tumor:
    • Tumor of myofibroblasts with prominent lymphoplasmacytic infiltrate scattered among tumor cells, usually less atypia than myofibroblastic sarcoma, often ALK+ (Hum Pathol 2008;39:846)
  • Leiomyosarcoma:
    • Alternating fascicular pattern, more eosinophilic cytoplasm, more strong and diffuse staining for myogenic makers, usually dense cytoplasmic actin staining rather than tram track pattern
  • Myofibroma / myofibromatosis:
    • Often multiple and often in children (although solitary lesions and lesions in adults are both common as well), usually zonated with peripheral myoid nodules with intervening central cellular spindle areas with ectatic "hemangiopericytic" vessels; more diffuse or atypical forms of myofibroma may be very difficult to distinguish from myofibroblastic sarcoma (zonation is helpful feature)
  • Nodular fasciitis:
    • Not infiltrative, not deep, < 3 cm usually, lacks chromosomal anomalies seen in myofibroblastic sarcoma (although recently a recurrent translocation resulting in MYH9-USP6 gene fusion has been described in many cases of nodular fasciitis) (Lab Invest 2011;91:1427)
  • Solitary fibrous tumor (SFT):
    • Diffusely positive for CD34, usually patternless haphazard arrangement of cells, usually no atypia (except in malignant SFT)
Board review style question #1

Low grade myofibroblastic sarcoma most commonly arises in the

  1. Abdominal cavity
  2. Extremities
  3. Head and neck
  4. Upper back
Board review style answer #1
Board review style question #2
Which of the following is true about low grade myofibroblastic sarcoma?

  1. It is more common than high grade myofibroblastic sarcoma
  2. It is paradoxically negative for muscle markers by immunohistochemistry
  3. It often metastasizes to the liver
  4. It should show at least focal nuclear atypia
Board review style answer #2
D. It should show at least focal nuclear atypia

Comment Here

Reference: Low grade myofibroblastic sarcoma

Lymphangioendothelioma
Definition / general
  • Proliferation of D2-40+ endothelial cells
  • Uncommon benign vascular lesion that may mimic well differentiated angiosarcoma or patch stage Kaposi's sarcoma (Am J Surg Pathol 2000;24:1047)
Terminology
  • Also called acquired progressive lymphangioma
Epidemiology
  • Rare
  • No gender preference, median age 54 years, range 17 - 90 years
  • Usually not associated with other vascular anomalies or HIV infection
Clinical features
  • Solitary red or bruise-like slow growing plaque present for median 5.5 years
  • Often in head and neck, but variable sites
  • May resemble actinic keratosis (Cutis 2001;67:29)
Prognostic factors
  • Benign
  • Occasional local recurrence
Case reports
Clinical images

Contributed by Dr. Mark R. Wick

Progressive acquired lymphangioma



Images hosted on other servers:

Arm

Erythematous nodule

Thigh

Gross description
  • Median 1.5 cm, range 0.3 cm to 10 cm
Microscopic (histologic) description
  • Delicate, thin walled, endothelium lined dilated vascular spaces involving the superficial dermis
  • Intravascular papillary stromal projections resembles papillary endothelial hyperplasia
  • Deeper portion of lesions have vascular space collapse and dissect collagen bundles, mimicking patch stage Kaposi's sarcoma
  • Preexisting vessels and adnexal structures of the dermis also appear dissected by newly formed vascular channels
  • Smooth muscle often focally present around vascular spaces
  • Endothelial cells may hobnail, may form morula resembling giant cells
  • Crowding of endothelial cells present, but no endothelial atypia
  • Vascular spaces lack erythrocytes and hemosiderin deposits
  • No mitotic figures
Microscopic (histologic) images

Contributed by Dr. Mark R. Wick

Breast

Podoplanin, breast

Progressive acquired lymphangioma



Images hosted on other servers:

Thigh

Lymphatic endothelium is podoplanin+

Site unknown

Negative stains
Differential diagnosis
  • Atypical or benign vascular proliferations of breast: history of radiation therapy
  • Kaposi sarcoma: patch stage usually has widespread multiple lesions in HIV+ patients or extensive lesion of lower extremities in elderly patients of Jewish or Mediterranean origin; usually lymphoplasmacytic infiltrate, with inflammatory cells aggregating around vessels, commonly extravasated red blood cells, often other forms of Kaposi’s sarcoma present
  • Lymphangioma circumscriptum
  • Angiosarcoma - well differentiated: elderly patients, reddish blue plaques or nodules, more endothelial atypia, multilayering and micropapillary tufting, often epithelioid or spindle cell component, inflammatory response common (Am J Dermatopathol 2000;22:151)

Lymphangioma
Definition / general
  • Benign localized or diffuse vascular proliferation composed of lymphatic vessels
Essential features
  • Developmental proliferation composed of lymphatic vessels of varying sizes
  • May be either microcystic or macrocystic in nature
  • Wide age range but predominant in children and young adults
  • Histology: dilated, thin walled lymphatic channels with flat endothelial cells; may show prominent stromal lymphoid tissue and proteinaceous fluid
  • IHC: PROX1, CD31, D2-40 and VEGFR3 positive
Terminology
  • Updated 2018 ISSVA classification for vascular anomalies recommends using the term lymphatic malformation for all lymphatic anomalies
  • Related / previous terminology: cystic hygroma, lymphangioma, lymphangioma circumscriptum, cavernous lymphangioma, intra-abdominal cystic lymphangioma, hemangiolymphangioma
ICD coding
  • ICD-10: D18.1 - lymphangioma, any site
Epidemiology
  • Wide age range
  • Majority are in children and young adults (90% diagnosed by age 2)
  • Mean age is 3.3 years (J Pediatr Surg 1999;34:1164)
  • Slight male predominance
Sites
  • May be superficial or deep (Semin Intervent Radiol 2017;34:225)
  • Macrocystic
    • Often in skin, subcutaneous tissue or deeper
    • Commonly neck and groin in infants with chromosomal abnormalities
  • Microcystic
    • Most commonly in skin and mucosal sites
Pathophysiology
  • Multiple theories suggested for the pathogenesis, including
    • Sequestration of lymphoid tissue during development
    • Abnormal budding of lymphatic vessels during development
    • Lack of fusion of the lymphatic system to the venous system
    • Trauma / infection
  • Various vascular growth factors are associated with the genesis of lymphangiomas (Virchows Arch 2008;453:1)
Etiology
  • Many lymphatic malformations show somatic mutations in PIK3CA (J Pediatr 2015;166:1048)
  • Macrocystic lymphatic malformations (cystic hygroma) may be associated with both euploid and aneuploid fetuses, including trisomy 21, trisomy 18, trisomy 13, Turner syndrome and Noonan syndrome (Acta Obstet Gynecol Scand 2007;86:1442)
Diagrams / tables

Table 1: Nomenclature for vascular anomalies
Vascular tumors Vascular malformations


Benign
Locally aggressive or borderline
Malignant
Simple Combined
Capillary malformations
Lymphatic malformations
Venous malformations
Arteriovenous malformations
Arteriovenous fistula
Combination of capillary, venous,
lymphatic or arterial vessels; for example
  • Capillary venous malformation
  • Capillary lymphatic malformation
  • Lymphatic venous malformation
Clinical features
  • In the skin, pale and pink vesicles
  • Deep lesions are circumscribed, soft, fluctuant and painless swellings
  • Large cystic lesions may cause compression of adjacent structures
Diagnosis
  • Clinical history and imaging studies can suggest a lymphangioma
  • Usually diagnosed on simple excision or incidentally on biopsy
Radiology description
  • Considered a low flow vascular malformation
  • Usually cystic in appearance but hemorrhage or infection may give a more solid appearance
  • Ultrasound (AJR Am J Roentgenol 2004;182:1485)
    • Multilocular cystic masses with anechoic cyst contents or echogenic debris
    • Can have wide variation
  • CT
    • Homogenous and cystic with low attenuation
    • Often does not show the intrinsic septations
  • MRI (AJR Am J Roentgenol 2004;182:1485)
    • Septated, lobulated mass with fluid-fluid levels; may infiltrate tissue planes
    • Low signal intensity on T1, high signal intensity on T2
    • Macrocystic tumors may have rim and septal enhancement that are not seen in microcystic tumors
Radiology images

Images hosted on other servers:
Retroperitoneal lymphatic malformation CT

Retroperitoneal lymphatic malformation CT

Mesenteric mass MRI

Mesenteric mass MRI

Prognostic factors
Case reports
Treatment
  • Depends heavily on the clinical presentation, size and location
  • Small or localized lesions are generally amenable to surgical resection
  • Large lesions involving multiple deep structures or tissue planes are often not surgically resectable
  • Sclerotherapy has been used to some success; particularly in macrocystic lymphatic malformations (Semin Pediatr Surg 2014;23:178)
  • Sirolimus may be used to improve prognosis in lymphatic malformations that are life threatening or refractory to other therapies (J Vasc Surg 2020;71:318)
Clinical images

Images hosted on other servers:
Lymphangioma circumscriptum

Lymphangioma circumscriptum

Multiple vesicular papules

Multiple vesicular papules

Cystic hygroma

Cystic hygroma

Gross description
  • Multicystic and spongy, containing watery / milky chylous fluid
Gross images

Contributed by Nasir Ud Din, M.B.B.S.
Cystic lymphangioma

Cystic lymphangioma

Mesenteric lymphangioma

Mesenteric lymphangioma



Images hosted on other servers:
Multiloculated soft tissue mass

Multiloculated soft tissue mass

Microscopic (histologic) description
  • Divided into categories of macrocystic, microcystic or combined based on the size of the lymphatic channels
  • Macrocystic includes cystic hygroma / cystic lymphangioma
  • Microcystic includes lymphangioma circumscriptum, cutaneous and small mucosal lymphatic malformations
  • Thin walled, dilated lymphatic channels with or without intraluminal proteinaceous material and lymphocytes
  • Composed of bland, flat endothelial cells
  • Stroma / lymphatic walls may show aggregates of lymphocytes
  • Cutaneous tumors may have overlying epidermal hyperplasia
  • Larger lymphatic channels can have smooth muscle in the walls
  • Longstanding lesions have stromal inflammation, fibrosis, xanthogranulomatous inflammation and myofibroblastic proliferation, particularly in mesenteric and retroperitoneal locations (Hum Pathol 2005;36:426)
  • Can cross tissue planes, dissect around normal structures and have anastomosing growth pattern (lymphangiomatosis)
Microscopic (histologic) images

Contributed by Farres Obeidin, M.D.
Colonic lymphatic malformation Colonic lymphatic malformation

Colonic lymphatic malformation

Skin lymphatic malformation Skin lymphatic malformation

Skin lymphatic malformation


Dilated lymphatic channels

Dilated lymphatic channels

Lymphoid stroma

Lymphoid stroma

Stromal fibrosis Stromal fibrosis

Stromal fibrosis


Lymphaticovenous malformation Lymphaticovenous malformation

Lymphaticovenous malformation

Lymphaticovenous malformation Lymphaticovenous malformation

Lymphaticovenous malformation

D2-40

D2-40

Positive stains
Molecular / cytogenetics description
  • Can show somatic mutations in PIK3CA or association with PIK3CA related overgrowth syndromes (J Pediatr 2015;166:1048)
Sample pathology report
  • Abdominal mass, excision:
    • Lymphatic malformation, mixed macrocystic and microcystic (see comment)
    • Comment: Microscopic sections demonstrate small to large caliber vascular spaces filled with proteinaceous fluid and are surrounded by extensive lymphocytic inflammation. The endothelial cells are flat without nuclear atypia and are highlighted with a D2-40 immunostain, supporting the diagnosis.
Differential diagnosis
  • Benign vascular neoplasms / malformations:
    • Lymphatic vessels may be present alone in a lymphatic malformation or with other vessel types
    • Artery is typically thicker walled with an internal elastic lamina
    • Vein has a muscular wall without an elastic lamina
    • Arteries and veins stain with ERG, CD34 and CD31 but not PROX1 or D2-40
  • Papillary intralymphatic angioendothelioma:
    • Typically arises in a background of lymphatic malformation
    • Defined by tufting and hobnailing endothelial cell proliferation into the lymphatic channels
  • Retiform hemangioendothelioma:
    • Positive for vascular markers and PROX1, usually negative for D2-40
    • Retiform / arborizing architecture with hobnail luminal cells
  • Angiosarcoma:
    • Cytologic atypia with anastomosing, infiltrative architecture and mitotic activity
Board review style question #1

A 55 year old woman is undergoing a routine screening colonoscopy, which demonstrates a 0.3 cm sessile nodule in the ascending colon. A biopsy is taken and the histology is represented by the picture above. Which of the following immunohistochemical panels would be expected to stain the lesional cells?

  1. D2-40, PROX1, CD31
  2. ERG, FLI1, CD34
  3. HHV8
  4. p53, AE1 / AE3
Board review style answer #1
A. D2-40, PROX1, CD31. The picture represents a benign lymphatic malformation / lymphangiomatosis in a colon polyp. These tumors may be infiltrative throughout the lamina propria but overall have a benign cytology and stain with lymphatic endothelial cell markers (D2-40, PROX1, CD31, LYVE1).

Comment Here

Reference: Lymphangioma
Board review style question #2
Macrocystic lymphatic malformations (cystic hygroma) are commonly seen in neonates with which of the following syndromes?

  1. Beckwith-Wiedemann syndrome
  2. Fragile X syndrome
  3. Meckel-Gruber syndrome
  4. Noonan syndrome
Board review style answer #2
D. Noonan syndrome. Cystic hygromas are commonly seen in aneuploidy syndromes like trisomies and Turner syndrome; however, they may also commonly be seen in some euploid fetuses, particularly those with Noonan syndrome.

Comment Here

Reference: Lymphangioma

Malignant melanotic nerve sheath tumor
Definition / general
  • Rare aggressive peripheral nerve sheath tumor uniformly composed of Schwann cells showing melanocytic differentiation, usually arising in association with spinal or visceral autonomic nerves (Adv Anat Pathol 2021;28:44)
  • Previously classified as a benign tumor in the 2013 WHO classification; however, 2020 WHO classification includes this entity as malignant melanotic nerve sheath tumor (MMNST) due to its malignant behavior (World J Clin Cases 2022;10:8735, BMJ Case Rep 2022;15:e252107)
  • Approximately 50% of cases are associated with Carney complex (Adv Anat Pathol 2021;28:44)
Essential features
  • Peripheral nerve sheath tumor neoplasm comprised of Schwann cells showing melanocytic differentiation, commonly in association with spinal or visceral autonomic nerves
  • Fascicular to sheet-like proliferation of heavily pigmented, relatively uniform plump spindle cells
  • Co-expression of S100 / SOX10 and melanocytic markers (HMB45, MelanA)
  • The behavior of MMNST is difficult to predict and metastases can occur in the absence of morphologically malignant features
  • Loss of PRKAR1A expression suggests a link to Carney complex (Am J Surg Pathol 2014;38:94)
Terminology
  • Preferable: malignant melanotic nerve sheath tumor
  • Acceptable: melanotic schwannoma; psammomatous melanotic schwannoma; malignant melanotic Schwannian tumor
ICD coding
  • ICD-O: 9540/3 - malignant peripheral nerve sheath tumor
  • ICD-11: 2B5E & XH2XP8 - malignant nerve sheath tumor of peripheral nerves or autonomic nervous system, primary site & malignant peripheral nerve sheath tumor
Epidemiology
Sites
Pathophysiology
  • Poorly understood, most tumors are sporadic
  • Characterized by a complex karyotype with recurrent monosomy of band 22q, variable whole chromosomal gains and recurrent losses involving chromosomes 1, 21 and 17p
  • 50 - 55% of psammomatous MMNST cases are associated with Carney complex, which shows mutation within chromosome 17 (affecting PRKAR1A / 17q24 band) (Nat Genet 2000;26:89, Genes Chromosomes Cancer 2015;54:463)
  • Recent cytogenetic studies demonstrated trisomy 6p and ring chromosome 11 in MMNST, suggesting that the tumor may share some genetic aberrations with malignant melanoma; however, it lacks BRAF V600E as in malignant melanoma (Arch Pathol Lab Med 2018;142:1517)
  • A missense DDR2 mutation of Q231K has been identified in 1 case report (Case Rep Oncol 2021;14:826)
Clinical features
  • Local mass with associated pain and neurologic symptoms, depending on site and rate of growth or incidental
  • Tumors associated with nerves (35.5% of patients) include motor and sensory abnormalities
  • Pressure symptoms with large tumors
  • Cutaneous lesions can manifest features similar to melanomas
  • Asymptomatic presentations have been noted in 29% of cases
  • Reference: Arch Pathol Lab Med 2018;142:1517
Diagnosis
  • Clinical history
  • CT scan / MRI remains the investigation of choice
  • Fluorodeoxyglucose (FDG) PET / CT one stop examination can detect tumors and determine whether they are malignant based on their glucose metabolism before morphological changes occur (Medicine (Baltimore) 2021;100:e24803)
  • Biopsy followed by histopathology
Radiology description
Radiology images

Images hosted on other servers:

PET / CT: extraspinal tumor with strong FDG

Well circumscribed tumor in the Meckel cavum

MET PET / MRI: recurrence with hypermetabolic activity

MRI of cystic mass at the para-aortic region

CT showing left knee irregular mass

Prognostic factors
Case reports
Treatment
Gross description
  • Most MMNSTs are > 5 cm in diameter (Arch Pathol Lab Med 2018;142:1517)
  • Well circumscribed and covered by a thin fibrous membrane and arising in relation to a nerve
  • Solid cysts can be found on the surface; cut surface has the consistency of tar and varies from gray to pitch black
  • Erosion of the adjacent bone may be seen (Arch Pathol Lab Med 2018;142:1517)
Gross images

Images hosted on other servers:

Well encapsulated, black and fibrotic tumor

Microscopic (histologic) description
  • Circumscribed, unencapsulated lesion with plump spindle and epithelioid cells arranged in interlacing fascicles or nests
  • Accumulation of melanin in neoplastic cells and associated melanophages
  • Round, ovoid or elongated nuclei which contain delicate, evenly distributed chromatin and small distinct nucleoli; some areas may show large and prominent nucleoli
  • Rare mitoses
  • Tumors associated with Carney complex may show sheets of adipose-like cells and psammoma bodies
  • Degenerative nuclear atypia, markedly enlarged, hyperchromatic nuclei, smudgy chromatin and cytoplasmic nuclear inclusions may occasionally be seen
  • Psammoma bodies can be seen; these can be isolated foci or innumerable calcified structures (J Int Med Res 2020;48:300060520947919, Am J Surg Pathol 2014;38:94, Arch Pathol Lab Med 2018;142:1517)
  • Necrosis, when present, is often geographic in pattern
  • Strict criteria of malignancy in MMNST are not well developed, although a combination of worrisome histologic features (large, vesicular nuclei with macronucleoli; brisk, mitotic activity; necrosis) raises concern of aggressive behavior (Arch Pathol Lab Med 2018;142:1517)
Microscopic (histologic) images

Contributed by Nasir Ud Din, M.B.B.S.
Fascicles and nests of pigmented spindle to epithelioid cells

Fascicles, nests of pigmented spindle to epithelioid cells

Cells with remarkable brown pigment

Cells with remarkable brown pigment

Accumulation of melanin in neoplastic cells and melanophages

Melanin in neoplastic cells and melanophages

Epithelioid cell clusters

Epithelioid cell clusters

Ovoid cells, regular nuclei and small nucleoli

Ovoid cells, regular nuclei and small nucleoli


Degenerative nuclear atypia

Degenerative nuclear atypia

Foci of calcifications Foci of calcifications

Foci of calcifications

No significant mitosis and necrosis

No significant mitosis or necrosis

Positive stains
Electron microscopy description
  • Epithelial Schwann cells and pigmented spindle cells are bundle shaped, interleaved and wheel shaped, with round and oval nuclei, obvious nucleoli and rare nuclear division (Medicine (Baltimore) 2021;100:e24803)
  • Numerous elongated tumor cell processes, duplicated basement membrane and melanosomes are observed in all developmental stages (Arch Pathol Lab Med 2018;142:1517)
Molecular / cytogenetics description
Videos

MMNST by Dr. Jerad Gardner

Sample pathology report
  • Lumbar spine mass, excision:
    • Malignant melanotic nerve sheath tumor (see comment)
    • Comment: Histology showed a circumscribed, unencapsulated lesion containing plump spindle to epithelioid cells arranged in interlacing fascicles. Individual cells are round to ovoid with regular nuclei and evenly distributed chromatin. No significant mitotic activity or necrosis is identified. These cells exhibit melanin pigment. These tumors can be sporadic or part Carney complex or neurofibromatosis type 1. Clinical correlation is suggested.
Differential diagnosis
  • Malignant melanoma:
    • Occurs in older patients
    • Predilection for cutaneous, mucosal and extracutaneous sites
    • Composed of epithelioid or spindle shaped cells with nuclear pleomorphism, hyperchromasia, coarse irregular chromatin and prominent eosinophilic nucleoli
    • Variable dusty pigmented cytoplasm, frequent mitosis
    • Lacks psammoma bodies, adipose-like cells and pericellular basement membrane synthesis
    • Overlapping positive stains (S100 [nuclear and cytoplasmic], SOX10 [nuclear], MelanA / MART1 [cytoplasmic], HMB45 [cytoplasmic], tyrosinase [cytoplasmic]), Ki67 / MIB1 > 10%
    • Molecular testing for BRAF V600E is positive
  • Pigmented neurofibroma:
    • Occurs in 2 - 61 year old patients (median age of 28 years) (Am J Surg Pathol 2000;24:331)
    • Localized neurofibromas are superficial and evenly disturbed over the body surface
    • Diffuse neurofibromas are usually in the head and neck
    • Plexiform neurofibromas are localized to a major nerve trunk
    • Histologically cells are short and spindle shaped, with wavy scant cytoplasm and occasionally granular, dark brown pigment, small elongated nuclei
    • Intratumoral fat in the form of scattered adipocytes may be seen
    • Lacks psammoma bodies
    • Positive for melanocytic / Schwannian cell markers and CD34
  • Pigmented dermatofibrosarcoma protuberans:
    • Occurs over a wide age range but most commonly in early to mid adulthood
    • Can involve any area of skin but the trunk and extremities are the most common location
    • Histologically plump spindle cells in a storiform pattern, diffuse infiltration of the dermal stroma and subcutis with pigmented dendritic cells dispersed among neoplastic cells
    • Positive for CD34
    • Negative for S100 and other melanocytic markers
    • Dendritic pigmented cells show positivity for Fontana-Masson stain; express S100 protein, MelanA and HMB45; contain premelanosomes and mature melanosomes
    • Characterized by a t(17;22) translocation resulting in the fusion of exon 2 of PDGFB to COL1A1 gene
  • Pigmented epithelioid melanocytoma:
    • Age: median age of 20 - 27 years (Surg Pathol Clin 2021;14:285)
    • Site: extremities (38.3%), head and neck (30.8%) and trunk (30.8%) (Surg Pathol Clin 2021;14:285)
    • Melanocytoma are benign lesions, intermediate grade tumors
    • Histologically heavily pigmented, elongated or polygonal cells with vesicular nuclei and distinct nucleoli
    • Lacks psammoma bodies and pericellular basement membrane and adipose-like cells
    • Often associated with Carney complex or other banal nevi
    • Immunoprofile is similar to MMNST: positive for S100, SOX10, HMB45, MelanA
    • Negative stain: loss of cytoplasmic PRKAR1A
    • Fusions in protein kinase C alpha (PRKCA) or inactivating alterations in protein kinase A regulatory subunit R1 alpha gene (PRKAR1A) with a preceding mutation of BRAF or less frequently, NRAS (J Cutan Pathol 2019;46:878)
  • Reference: Arch Pathol Lab Med 2018;142:1517
Board review style question #1

A 55 year old patient presented with a mass in the lumbar region. After radiologic evaluation, the lesion was excised and histological features are shown in the image. What is the most likely diagnosis?

  1. Germ cell tumor
  2. Lymphoma
  3. Malignant melanotic nerve sheath tumor
  4. Meningioma
  5. Schwannoma
Board review style answer #1
C. Malignant melanotic nerve sheath tumor

Comment Here

Reference: Malignant melanotic nerve sheath tumor
Board review style question #2
Malignant melanotic nerve sheath tumor is associated with which of the following?

  1. Carney syndrome
  2. McCune-Albright syndrome
  3. Neurofibromatosis type 2 (NF2)
  4. Peutz-Jeghers syndrome
  5. Tuberous sclerosis
Board review style answer #2

Malignant peripheral nerve sheath tumor (MPNST)
Definition / general
  • Malignant neoplasm arising from peripheral nerve
  • May arise from a preexisting nerve sheath tumor in neurofibromatosis type 1 (NF1) or in the setting of prior radiation therapy
  • In the absence of association with NF1 or radiation, diagnosis is challenging and based on histologic and immunohistochemical features suggesting Schwannian differentiation (Am J Surg Pathol 2016;40:896)

  • Variants:
Essential features
  • Sarcoma with peripheral nerve sheath differentiation with typically aggressive behavior
  • Can occur in the following settings:
  • Morphology: marbling at low magnification (alternating areas of hypocellularity and hypercellularity) with perivascular accentuation, uniform cytologic features
  • Heterologous differentiation in 10 - 15% of cases
  • Rhabdomyoblastic differentiation associated with adverse clinical behavior (Eur J Surg Oncol 2013;39:46)
  • SOX10 and S100 IHC only seen in 50% of cases
Terminology
  • Malignant peripheral nerve sheath tumor (MPNST)
  • Obsolete: neurofibrosarcoma, malignant schwannoma, neurogenic sarcoma
ICD coding
  • ICD-O: 9540/3 - malignant peripheral nerve sheath tumor
  • ICD-11:
    • 2B5E & XH2XP8 - malignant nerve sheath tumor of peripheral nerves or autonomic nervous system, primary site, malignant peripheral nerve sheath tumor
    • 2B5E & XH4V81 - malignant nerve sheath tumor of peripheral nerves or autonomic nervous system, primary site, malignant peripheral nerve sheath tumor, epithelioid
    • 2B5E & XH2VV8 - malignant nerve sheath tumor of peripheral nerves or autonomic nervous system, primary site, malignant peripheral nerve sheath tumor with rhabdomyoblastic differentiation
Epidemiology
Sites
  • Can arise in virtually any anatomic location
  • Most common sites are the trunk and extremities, followed by head and neck (Am J Surg Pathol 2016;40:896)
Pathophysiology
  • Germline mutations in NF1 predispose to the development of peripheral nerve sheath neoplasms in patients with type 1 neurofibromatosis (Acta Neuropathol 2012;123:349)
  • In the setting of NF1, lesions often arise from plexiform neurofibroma (Acta Neuropathol 2012;123:349)
    • Transformation process is accompanied by progressive genomic changes involving NF1, CDKN2A / CDKN2B and PRC2 (Hum Pathol 2017;67:1)
    • Radiation therapy predisposes to the development of secondary sarcomas through repeated DNA damage and defective repair
Etiology
Clinical features
  • No sex predilection
  • Patients with NF1 are typically younger than their sporadic and radiation associated counterparts (J Med Genet 2002;39:311)
Diagnosis
  • Histologic evaluation is necessary but not always specific and requires correlation with clinical and radiologic findings
  • Helpful features include close association with peripheral nerves and history of NF1 or precursor lesions
Radiology description
Radiology images

Contributed by Jose G. Mantilla, M.D.

Forearm lesion T1

Forearm lesion T2

Cerebellopontine angle mass coronal view

Cerebellopontine angle mass sagittal view

Prognostic factors
Case reports
Treatment
Gross description
  • Fusiform to globoid, pseudoencapsulated tumor often with gross evidence of necrosis
    • If the tumor is arising from a nerve, an attached medium or large nerve is evident
Gross images

Contributed by Jose G. Mantilla, M.D.

Gross findings


Frozen section description
  • In high grade tumors, overt features of malignancy are readily identified, including nuclear pleomorphism, brisk mitotic activity and areas of geographic necrosis
  • Diagnosis of low grade lesions is difficult in frozen sections
  • In NF1, mitotic activity, increased cellularity and nuclear atypia within a neurofibroma raise concern for MPNST
Microscopic (histologic) description
  • Low power: marbled appearance due to alternating hypocellular and hypercellular areas with perivascular accentuation
  • Uniform spindle cells with hyperchromatic, thin, wavy, or focally buckled nuclei
  • May have uniform cellularity with fibrosarcoma-like fascicular growth, raising the differential diagnosis of synovial sarcoma
  • Can have foci of myxoid stroma and hyalinization
  • Epithelioid morphology can be present
  • Precursor lesion, such as neurofibroma, may be identifiable
  • Nuclear palisading is uncommon
  • Heterologous differentiation may include chondrosarcomatous, osteosarcomatous and rhabdomyosarcomatous components (malignant triton tumor)
  • Glandular elements are exceedingly rare (Int J Surg Pathol 2017;25:310)
  • Proposed nomenclature for the spectrum of NF1 associated nerve sheath tumors:
    • Atypical neurofibromatous neoplasm of uncertain biologic potential (ANNUBP):
      • At least 2 of the following features: cytologic atypia, loss of neurofibroma architecture, hypercellularity, > 1/50 HPF and < 3/10 HPF
    • MPNST, low grade: features of ANNUBP but with mitotic index of 3 - 9/10 HPF and no necrosis
    • MPNST, high grade: MPNST with at least 10 mits/10 HPF or 3 - 9 mits/10 HPF combined with necrosis (Hum Pathol 2017;67:1)
Microscopic (histologic) images

Contributed by Jose G. Mantilla, M.D.
Marbling

Marbling

Fascicular growth

Fascicular growth

Rhabdomyo-sarcomatous elements

Rhabdomyo-
sarcomatous elements

Epithelioid type, low power

Epithelioid type

Epithelioid type, higher power

Epithelioid type



Contributed by @MirunaPopescu13 and @JMGardnerMD on Twitter
Malignant peripheral nerve sheath tumor (MPNST) Malignant peripheral nerve sheath tumor (MPNST) Malignant peripheral nerve sheath tumor (MPNST) Malignant peripheral nerve sheath tumor (MPNST)

Malignant peripheral nerve sheath tumor (MPNST)

Virtual slides

Images hosted on other servers:

Marbling, hyperchromatic nuclei

Perivascular accentuation

Cytology description
  • Highly cellular smears of uniform spindled cells singly and in clusters (Cancer Cytopathol 2012;120:334)
  • Cytomorphologic overlap with other sarcomas is significant
Cytology images

Contributed by Jose G. Mantilla, M.D.

Uniform spindle cells

Positive stains
Negative stains
Molecular / cytogenetics description
Sample pathology report
  • Right hand mass, excision (consult case):
    • Spindle cell sarcoma, consistent with malignant peripheral nerve sheath tumor (see comment)
    • Tumor size: 3.8 cm in greatest dimension, per outside report
    • Inked tissue edges involved by sarcoma
    • Comment: Histologic sections contain a dermal based neoplasm with a plexiform architectural pattern. It is composed of variably pleomorphic spindle cells forming fascicles and storiform structures with a marbling pattern. Mitotic activity is present in up to 7/10 high power fields and no necrosis is identified. The periphery of the lesion is relatively hypocellular with a single area composed of cytologically bland spindle cells, which morphologically resembles neurofibroma (slide A3). This portion of the lesion shows immunoreactivity for S100 and CD34. The remainder of the lesion is has focal S100 reactivity. Negative stains include EMA, CK AE1 / AE3, desmin, SMA, MART1, MUC4, CD56, neurofilament and ER. Overall, the findings are consistent with malignant peripheral nerve sheath tumor.
Differential diagnosis
Board review style question #1

    Which of the following is true about malignant peripheral nerve sheath tumors?

  1. Characteristic histology is of a marbled appearance at low power with alternating hypocellular and hypercellular areas
  2. Glandular elements are never seen in this entity, which can help distinguish it from synovial sarcoma
  3. Lesion always has diffuse and strong S100 positivity due to its nerve sheath origin
  4. Most of these tumors are radiation associated as opposed to arising de novo or in a preexisting neurofibroma
Board review style answer #1
A. Characteristic histology is of a marbled appearance at low power with alternating hypocellular and hypercellular areas

Comment Here

Reference: MPNST
Board review style question #2
    Which of the following is true about the immunophenotypic and molecular characteristics of MPNST?

  1. Loss of expression of H3K27me3 has been described in high grade appearing tumors
  2. All cases have some degree of SOX10 expression
  3. Germline mutations have no influence on the development of MPNST
  4. MPNST is defined by recurrent cytogenetic alterations
Board review style answer #2
A. Loss of expression of H3K27me3 has been described in high grade appearing tumors

Comment Here

Reference: MPNST

Morton neuroma
Definition / general
  • Morton neuroma is a nonneoplastic, degenerative neuropathy with a strong predilection for the third interdigital nerve of the foot
  • One of the most common disorders encountered in the foot (Clin Radiol 2021;76:235.e15)
Essential features
  • Morton neuroma is a degenerative fibrotic neuropathy
  • Typically affects the interdigital nerve that innervates the third webspace (Radiographics 1999;19:1253)
  • Strong predilection for middle aged women (Foot Ankle 1983;3:238)
  • Characterized microscopically by nerve fiber degeneration and excessive intraneural and perineural fibrosis
  • Original series by Thomas Morton in 1876 (Am J Med Sci 1876;71:37)
Terminology
  • The term neuroma is a misnomer
  • Synonyms: interdigital neuroma, intermetatarsal neuroma, Morton metatarsalgia, plantar neuroma
ICD coding
  • ICD-10: G57.6 - lesion of plantar nerve, applicable to Morton metatarsalgia
Epidemiology
Sites
Pathophysiology
Etiology
  • Hyperextension of the toes by pointed, heeled shoes increases pressure on the forefoot and is implicated in nerve injury (J Foot Ankle Surg 1996;35:112)
Diagrams / tables

Contributed by Rola H. Ali, M.D.
Relevant anatomy for pathologists

Relevant anatomy

Clinical features
  • Sharp burning pain in the plantar aspect of the forefoot around the metatarsal heads and metatarsophalangeal joints that radiates to the toes (metatarsalgia) (Foot Ankle 1983;3:238)
  • Pain that is aggravated by walking and wearing tight shoes or high heels and alleviated by rest and removing the shoe
  • May be associated with numbness and the feeling of a pebble in the shoe (J Clin Orthop Trauma 2020;11:406)
  • Symptoms associated with larger size (AJR Am J Roentgenol 2000;175:649)
  • Smaller asymptomatic lesions may be relatively common with no gender predilection (Radiology 1997;203:516)
Diagnosis
Radiology description
Radiology images

Images hosted on other servers:
Morton neuroma on ultrasound

Ultrasound

Morton neuroma on MRI

MRI

Prognostic factors
Case reports
  • 30 year old man with history of traumatic little fingertip amputation and complicated surgeries developed fusiform enlargements of ulnar and radial proper digital nerves (Plast Reconstr Surg Glob Open 2022;10:e4035)
  • 46 year old woman with a 5 cm neuroma in the third webspace associated with macrodactyly and Raynaud phenomenon (Joints 2020;7:127)
  • 55 year old woman with neuroma of the proper digital branch of the fourth toe presented with isolated extreme fourth toe pain (Cureus 2020;12:e8920)
Treatment
Clinical images

Images hosted on other servers:
Exposing the neuroma - plantar approach

Exposing the neuroma - plantar approach

Exposing the neuroma - dorsal approach

Exposing the neuroma - dorsal approach

Large Morton neuroma

Large Morton neuroma

Gross description
  • Fusiform swelling of the nerve particularly at its bifurcation with adherence to fibrofatty tissue (Foot Ankle Int 2004;25:79)
Gross images

Images hosted on other servers:
Neuroma at the nerve bifurcation

Neuroma at the nerve bifurcation

Large resected neuroma

Large resected neuroma

Microscopic (histologic) description
Microscopic (histologic) images

Contributed by Rola H. Ali, M.D.
Nerve bundles

Nerve bundles

Perineurial fibrosis

Perineurial fibrosis

Arterial changes

Arterial changes

Perineurial and endoneurial fibrosis

Perineurial and endoneurial fibrosis

Concentric fibrosis

Concentric fibrosis

Vascular proliferation

Vascular proliferation


Nerve destruction

Nerve destruction

Hyalinization of endoneurial vessels

Hyalinization of endoneurial vessels

Accompanied synovial cyst

Accompanied synovial cyst

Synovial cyst close up

Synovial cyst close up

S100

S100

Trichrome stain

Trichrome stain


Virtual slides

Images hosted on other servers:
Morton neuroma

Morton neuroma

Positive stains
Electron microscopy description
Sample pathology report
  • Foot, excision:
    • Morton neuroma (see comment)
    • Comment: The histology shows degenerative fibrosing neuropathy consistent with Morton neuroma.
Differential diagnosis
  • Traumatic neuroma:
    • M = F, history of trauma or surgery
    • Location: may occur anywhere
    • Histology: numerous small nerve twigs
  • Localized neurofibroma:
    • M = F
    • Solitary lesions, mostly sporadic; multiple may be in NF1
    • Location: may occur anywhere, deep and superficial
    • Gross: when arising from a large nerve may look fusiform similar to Morton neuroma while cutaneous examples are not
    • Histology: mixed population of Schwann cells, fibroblasts, perineurial cells and occasional mast cells along with scattered axons and shredded collagen usually in a myxoid stroma
  • Plexiform neurofibroma:
    • M = F; arises early in life
    • Strongly associated with NF1
    • Location: may occur anywhere
    • Gross: tortuous nerve with a bag of worms appearance
    • Histology: expanded nerve branches with cytological features of conventional neurofibroma
  • Pacinian corpuscles and neuroma:
    • Corpuscles often encountered in tissue sections of Morton neuroma
    • Neuroma typically occurs on fingertips
    • Histology: hyperplasia of Pacinian corpuscles usually following trauma
  • Clinically, the differential may include intermetatarsal bursitis, rheumatoid arthritis, stress fracture and metatarsal / soft tissue tumors (Radiol Case Rep 2023;18:2416, Cureus 2022;14:e25305, Acta Biomed 2019;90:214)
Board review style question #1

A patient has been experiencing bouts of burning pain and numbness localized to the plantar aspect of the left forefoot at the level of the metatarsal heads. The pain is exacerbated by walking and relieved by rest and removal of the shoe. Excision shows the histology above. What is the diagnosis?

  1. Morton neuroma
  2. Pacinian neuroma
  3. Perineurioma
  4. Plexiform neurofibroma
  5. Traumatic neuroma
Board review style answer #1
A. Morton neuroma. The clinical presentation is typical. The histology image shows degenerative neuropathy with marked fibrosis and loss of nerve architecture. In contrast, all of the other answers are proliferative lesions. Answer B is incorrect because although a Pacinian corpuscle is seen on the right, there is no corpuscle proliferation. Answer C is incorrect because perineurioma of soft tissue is characterized by a proliferation of slender spindle cells with whorling. Answer D is incorrect because plexiform neurofibroma is composed of nerve bundles that are tortuous and expanded (as opposed to shrunken and fibrotic) by spindle cell proliferation. Answer E is incorrect because traumatic neuroma shows a haphazard proliferation of numerous nerve twigs.

Comment Here

Reference: Morton neuroma
Board review style question #2

A 45 year old woman has been experiencing bouts of pain in her left foot and has undergone a surgical procedure to remove the lesion. The histology of the lesion is shown above. Where did the lesion most likely arise from?

  1. First common plantar digital nerve
  2. Lateral plantar nerve
  3. Medial plantar nerve
  4. Proper plantar digital nerve
  5. Third common plantar digital nerve
Board review style answer #2
E. Third common plantar digital nerve. The picture shows a Morton neuroma, which classically affects the third common plantar digital nerve supplying the webspace between the third and fourth metatarsal heads. Answers A - D are incorrect because these nerves are not usually affected.

Comment Here

Reference: Morton neuroma

Myoepithelioma / myoepithelial carcinoma / mixed tumor
Definition / general
  • Myoepithelioma, myoepithelial carcinoma and mixed tumor of soft tissue are a group of uncommon neoplasms that share morphological, immunophenotypic and genetic features with their counterparts in salivary gland and skin
Essential features
  • Trabecular, reticular, nested or solid growth of variably spindled or epithelioid cells
  • Frequent myxoid or hyalinized stroma
  • Mixed tumors show ductal differentiation
  • Myoepithelial carcinoma shows moderate to severe nuclear atypia, increased mitotic activity and necrosis
  • Positivity for S100, EMA, keratin or GFAP
Terminology
  • Not recommended: parachordoma
ICD coding
  • ICD-O:
    • 8982/0 - Myoepithelioma, NOS
    • 8982/3 - Myoepithelial carcinoma
    • 8940/0 - Mixed tumor, NOS
    • 8940/3 - Mixed tumor, malignant
  • ICD-11: 2F7C & XH3CQ8 - Neoplasms of uncertain behavior of connective or other soft tissue and myoepithelioma
Epidemiology
Sites
Pathophysiology
Etiology
  • Tumors appear to be sporadic and of unknown etiology
Clinical features
Diagnosis
Radiology description
Radiology images

Contributed by Borislav A. Alexiev, M.D.

MRI

Prognostic factors
Case reports
  • 4 year old girl and 6 year old girl with molecularly confirmed primary renal myoepithelial carcinoma (Am J Surg Pathol 2016;40:386)
  • 36 year old woman with a molecularly confirmed myoepithelioma of the metatarsal bone (Pathol Int 2019;69:42)
  • 40 year old man with a molecularly confirmed myoepithelial carcinoma with a rhabdoid morphology arising in the neck (Head Neck Pathol 2015;9:273)
  • 45 year old woman with a molecularly confirmed soft tissue myoepithelial carcinoma of the abdominal wall diagnosed on fine needle aspirate (Diagn Cytopathol 2015;43:421)
  • 54 year old man with molecularly confirmed myoepithelioma arising in the sacrum (Histopathology 2014;65:917)
Treatment
Gross description
Gross images

Contributed by Borislav A. Alexiev, M.D.

Soft tissue mass

Microscopic (histologic) description
Microscopic (histologic) images

Contributed by Borislav A. Alexiev, M.D. and William B. Laskin, M.D.

Soft tissue mass

Myxoid stroma

Vacuolated cytoplasm

Spindle cell morphology

Clear cytoplasm

Hyalinized stroma


Cytologic atypia

S100

AE1 / AE3

GFAP

Calponin

Cytology description
  • Moderately cellular smears showing loosely arranged clusters, sheets, isolated cells, rosette-like arrangement with fibrillary material within (Cytojournal 2017;14:14)
  • Round to oval, spindle, epithelioid and plasmacytoid cells in myxoid background (Diagn Cytopathol 2016;44:152)
  • Bland uniform, round to ovoid nuclei with finely distributed chromatin and eosinophilic or pale cytoplasm (Diagn Cytopathol 2016;44:152)
Positive stains
Negative stains
Electron microscopy description
Molecular / cytogenetics description
Molecular / cytogenetics images

Images hosted on other servers:

FISH

Sample pathology report
  • Left leg, biopsy:
    • Myoepithelioma (see comment)
    • Comment: The neoplasm is composed of epithelioid and spindle cells with eosinophilic cytoplasm arranged in anastomosing cords and nests. Nuclei are monomorphic, ovoid or round, with minimal atypia. Interspersed between the cells there is abundant myxohyaline stroma. Occasional mitotic figures are identified (2 mitoses/10 high power fields). The cells are positive for SMA, S100, keratin AE1 / AE3 and GFAP. The findings support the above diagnosis. Although the majority of morphologically benign myoepithelial neoplasms of soft tissue behave in a benign fashion, there is an approximate 20% risk for local recurrence.
Differential diagnosis
Board review style question #1
The most common gene rearranged in myoepithelial tumors is

  1. EWSR1
  2. FUS
  3. SS18
  4. ZNF444
Board review style answer #1
Board review style question #2

A 45 year old man presented with a left thigh mass. Hematoxylin-eosin stains demonstrated a nested and trabecular growth of small to medium sized spindled to epithelioid cells with uniform, round to ovoid nuclei and eosinophilic or pale cytoplasm in myxoid background. Occasional mitotic figures were identified (2 mitoses/10 high power fields). Immunohistochemical stains for S100, keratin AE1 / AE3, GFAP and calponin (focal) were positive in tumor cells while all of the following were negative: desmin, CD34, ERG and h-caldesmon. SMARCB1 / INI1 expression was preserved. Which of the following is most likely the correct diagnosis?

  1. Extraskeletal myxoid chondrosarcoma
  2. Ossifying fibromyxoid tumor
  3. Metastatic carcinoma
  4. Epithelioid sarcoma
  5. Myoepithelioma
Board review style answer #2

Myolipoma
Definition / general
Terminology
  • Called lipoleiomyoma in uterus
  • Note: myelolipoma is tumor with hematopoietic (including myeloid) elements, often in adrenal gland
Epidemiology
  • Very rare tumor of adults in abdomen, retroperitoneum or abdominal wall
  • Other rare sites include eyelid, pericardium, base of tongue
Radiology images

Contributed by Drs. Arno Vanstapel and Raf Sciot, Case #452

CT abdomen frontal

CT abdomen transverse

Case reports
Treatment
  • Excision
  • Does not recur, metastasize or transform
Gross description
  • Often 9 cm or more, completely or partially encapsulated, yellow-white
Gross images

Contributed by Drs. Arno Vanstapel and Raf Sciot, Case #452



Images hosted on other servers:

Fatty tissue with bands and nodules of firm white tissue

Microscopic (histologic) description
  • Mature adipose tissue and mature (well differentiated) smooth muscle in short fascicles (Stanford University)
  • Variable fibrosis and inflammation
  • No floret cells, no thick walled vessels, no necrosis, no mitotic figures
Microscopic (histologic) images

Contributed by Drs. Arno Vanstapel and Raf Sciot, Case #452

5x

10x

20x








Images hosted on other servers:

Mature fat and smooth muscle fibers

Low magnification view of myolipoma

Smooth muscle fibers are SMA+ (left), desmin+ (right)

Positive stains
Differential diagnosis
Board review style question #1
Which of the following immunohistochemical stains would help distinguish retroperitoneal myolipoma from a well differentiated liposarcoma invading smooth muscle?

A. CDK4
B. HMB45
C. S100
D. Smooth muscle actin
E. Vimentin
Board review style answer #1
A. CDK4

Comment Here

Reference: Myolipoma

Myopericytoma / myofibroma
Definition / general
  • Myopericytoma is a distinctive perivascular myoid neoplasm that forms a morphological spectrum with myofibroma
Essential features
  • Myopericytoma: bland, myoid spindled cells growing in a concentric pattern around numerous small blood vessels
  • Myofibroma: biphasic growth pattern, with primitive cellular zones often showing mitotic activity, necrosis and calcification, surrounded by hyalinized nodules of myoid spindled cells
  • PDGFRB mutations in myopericytoma and myofibroma
  • SRF-RELA gene fusions in cellular myofibroma / myopericytoma
Terminology
  • Not recommended: infantile hemangiopericytoma
ICD coding
  • ICD-O: 8824/0 - myopericytoma
  • ICD-11: 2E84.Y & XH0953 - benign fibrogenic or myofibrogenic tumor of other specified sites & myofibroma
Epidemiology
  • Myopericytoma may occur at any age; most cases are seen in adults
  • Myofibroma may be present at birth, appear in the first 2 years of life or arise in adults, with male predominance (Am J Surg Pathol 2006;30:104)
Sites
Pathophysiology
Etiology
Clinical features
Diagnosis
  • Tissue sampling is the gold standard for a definitive diagnosis
Radiology description
  • At magnetic resonance imaging, tumors are most commonly superficial, may be well defined (myopericytoma) or ill defined (myopericytomatosis) and demonstrate highly vascularized, avidly enhancing soft tissue, often with areas of internal hemorrhage (Radiol Case Rep 2017;13:275)
Radiology images

Contributed by Borislav A. Alexiev, M.D.
Myopericytoma, MRI

Myopericytoma, MRI

Prognostic factors
  • Most myopericytomas and myofibromas do not recur
  • Small subset of myofibromas / myopericytomas shows atypical features (diffuse hypercellularity, infiltrative growth pattern and increased mitotic figures) that do not adversely affect outcome (Am J Surg Pathol 2014;38:1649)
  • Malignant myopericytomas are associated with aggressive clinical behavior (Histopathology 2002;41:450)
  • Multicentric myofibroma (referred to as infantile myofibromatosis) is a life threatening disease (JAMA Dermatol 2019;155:946)
Case reports
Treatment
Gross description
  • In superficial locations, myopericytoma tends to be well circumscribed, with nodules measuring < 2 cm
  • Larger neoplasms may be seen in deep soft tissue (Am J Surg Pathol 2006;30:104)
  • Tan-white, firm cut surface
Gross images

Contributed by Borislav A. Alexiev, M.D.
Soft tissue mass

Soft tissue mass

Frozen section description
  • Concentric, perivascular growth of plump spindled, myoid cells
Microscopic (histologic) description
  • Myopericytoma
    • Well circumscribed, nodular or lobular lesions
    • Bland, oval to spindle shaped, myoid cells
    • Characteristic multilayered, concentric growth around numerous small vessels
    • Variable cellularity, ranging from cellular and solid appearing to hypocellular and collagenous / myxoid (Am J Surg Pathol 2006;30:104)
    • Numerous blood vessels and variable in size; branching, hemangiopericytoma-like blood vessels may be present
    • In some cases, a more prominent fascicular arrangement of neoplastic cells is present
    • Some myopericytomas may show degenerative changes (symplastic nuclear atypia, stromal hyalinization and cystic change)
    • Myopericytomatosis is a rare, diffuse benign variant of myopericytoma that typically involves superficial soft tissue in adults with innumerable microscopic myopericytomatous nodules (Am J Surg Pathol 2017;41:1034)
  • Myofibroma
    • Well circumscribed, nodular neoplasms characterized by a distinctive biphasic growth pattern (Oral Surg Oral Med Oral Pathol Oral Radiol Endod 2000;89:57)
    • Center of the lesion is composed of immature appearing, plump, spindled tumor cells associated with hemangiopericytoma-like branching blood vessels
    • Periphery of the lesion consists of nodules and fascicles of variably hyalinized, myoid appearing cells
    • Pattern of zonation may be more haphazard or even reversed (Histopathology 1993;22:335)
    • Mitotic figures are variable in number
    • Cellular zones may undergo necrosis and calcification
    • In infants, myofibromas may be composed almost entirely of primitive, cellular zones; such cases were historically labeled infantile hemangiopericytoma (Am J Surg Pathol 1994;18:922)
    • Morphological features of the cellular zones of myofibroma share some features with myopericytoma, suggesting that these are related entities
    • Myofibromatosis is defined by the presence of multiple myofibromas
  • Myofibroma / myopericytoma with cellular / atypical features
  • Malignant myopericytoma
    • Highly mitotic myoid appearing ovoid to spindle cells
    • Marked nuclear atypia / pleomorphism
    • Tumor necrosis
    • At least focally striking perivascular orientation resembling that seen in benign myopericytoma (Histopathology 2002;41:450)
Microscopic (histologic) images

Contributed by Borislav A. Alexiev, M.D., Hillary Rose Elwood, M.D. and Mark R. Wick, M.D.
Well circumscribed lesion

Well circumscribed lesion

Multilayered, concentric growth

Multilayered, concentric growth

Hemangiopericytoma-like vessels

Hemangiopericytoma-like
vessels

Perivascular proliferation

Perivascular proliferation

Smooth muscle actin

Smooth muscle actin

h-caldesmon

h-caldesmon


Well circumscribed nodule

Well circumscribed nodule

Biphasic proliferation Biphasic proliferation

Biphasic proliferation

Biphasic morphology

Biphasic morphology

Increased cellularity

Increased cellularity

Stromal hyalinization

Stromal hyalinization

Molecular / cytogenetics description
Sample pathology report
  • Right foot mass, excision:
    • Myopericytoma (see comment)
    • Comment: Histologically, the neoplasm has a well circumscribed margin and is composed of numerous thick walled vessels, with a concentric, perivascular arrangement of ovoid, plump spindled to round myoid tumor cells with an abundant cytoplasm and spindled or round nuclei with evenly distributed chromatin. Areas with hemangiopericytoma type growth pattern, characterized by dilated branching thin walled vessels surrounded by evenly distributed spindled neoplastic cells are also noted. Degenerative stromal changes, such as stromal fibrosis, hyalinization and myxoid change, are seen. Mitotic figures, tumor necrosis and vascular invasion are not identified. Immunohistochemically, the tumor stains positively for SMA and h-caldesmon, while all of the following are negative: S100, desmin, keratin AE1 / AE3 and HMB45. The morphological features and immunohistochemical profile support the above diagnosis.
    • Myopericytoma is a benign tumor that rarely recurs following marginal or even intralesional excision. Simple surgical excision is adequate treatment.
Differential diagnosis
Board review style question #1
Which of the following is true about myofibroma / myofibromatosis?

  1. Multicentric infantile myofibromatosis is more common in males
  2. Multiple lesions with visceral involvement can affect virtually any internal organ and are associated with a poor prognosis
  3. Solitary infantile myofibromatosis is more common in females
  4. Tumor is positive for keratin
  5. Tumor is positive for NOTCH gene fusions
Board review style answer #1
B. Multiple lesions with visceral involvement can affect virtually any internal organ and are associated with a poor prognosis

Comment Here

Reference: Myopericytoma / myofibroma
Board review style question #2

A 38 year old man presented with a left thigh mass. Hematoxylin eosin stains demonstrate a well circumscribed tumor in the dermis and subcutaneous fat composed of bland, oval to spindle shaped cells with multilayered concentric growth around numerous small thick walled blood vessels. Neither necrosis nor mitotic activity is identified. Immunohistochemical stains for SMA and h-caldesmon are positive in tumor cells, while all of the following are negative: keratin AE1 / AE3, CD34, desmin, HMB45, S100 and STAT6.

Which of the following is most likely the correct diagnosis?

  1. Leiomyoma
  2. Myofibroblastoma
  3. Myopericytoma
  4. PEComa
  5. Solitary fibrous tumor
Board review style answer #2
C. Myopericytoma

Comment Here

Reference: Myopericytoma / myofibroma

Myositis ossificans and fibro-osseous pseudotumor of digits
Definition / general
  • Myositis ossificans and fibro-osseous pseudotumor of digits are self limited, benign neoplasms composed of spindle cells and osteoblasts
  • Myositis ossificans, fibro-osseous psedotumor and soft tissue aneurysmal bone cyst belong to the same neoplastic spectrum
Essential features
  • Benign, reactive, ossifying soft tissue mass lesion, associated with trauma and characterized by zonal pattern
Terminology
  • Not recommended:
    • Pseudomalignant osseous tumor of soft tissue, myositis ossificans circumscripta, myositis ossificans traumatica
  • Related entities:
    • Panniculitis / fasciitis ossificans: similar histologic features but involves subcutis and tendons / fascia rather than muscle
ICD coding
  • ICD-10: M61.00 - myositis ossificans traumatica, unspecified site
Epidemiology
  • Usually physically active young males (second and third decade) with rapid growth of mass (Clin Case Rep 2021;9:e04608)
  • 60 - 75% have history of trauma in prior 4 - 6 weeks (J Med Case Rep 2010;4:270)
  • May also occur after elective surgery, severe burns, neurological injury
Sites
Pathophysiology
  • Pathogenesis is poorly understood
  • Thought to develop from differentiation of fibroblast to osteoblast secondary to inflammatory cytokine activity, particularly bone morphogenetic protein 1 (BMP1), BMP2 and transforming growth factor (TGF)
  • These cytokines promote differentiation of perivascular mesenchymal cells into osteoblasts and chondroblasts, which then undergo endochondral ossification (J Am Acad Orthop Surg 2015;23:612)
Etiology
Diagrams / tables

Images hosted on other servers:
Different stages of myositis ossificans

Different stages

Pathogenesis of myositis ossificans

Pathogenesis

Clinical features
Diagnosis
Laboratory
  • Usually not required for diagnosis
  • Early stage (0 - 4 weeks):
    • Serum alkaline phosphatase (SAP): normal
    • C reactive protein (CRP) and erythrocyte sedimentation rate (ESR): elevated
    • Creatinine phosphokinase (CPK): elevated
  • Intermediate stage (4 - 8 weeks):
    • SAP: elevated
    • CRP and ESR: normal to mildly elevated
    • CPK: elevated
  • Mature, late stage (> 8 weeks):
    • SAP: elevated
    • CRP and ESR: normal
    • CPK: normal to mildly elevated
  • Reference: Curr Sports Med Rep 2018;17:290
Radiology description
  • Early stage:
    • Xray: normal to faint calcification (flocculent radiopacities: dotted veil pattern)
    • CT scan: soft tissue edema
    • MRI: isodense on T1 and hyperintense on T2
  • Intermediate stage:
    • Xray: peripheral calcified rim with central lucency
    • CT scan: peripheral mineralization with central low attenuation (calcification proceeds from periphery to center)
    • MRI: isodense / hypodense to adjacent skeletal muscle on all images
  • Mature, late stage:
    • Xray: diffuse soft tissue calcification
    • CT scan: diffuse ossification pattern (even in late lesions, the central core may remain uncalcified)
    • MRI: well defined soft tissue mass and isodense to fat on all images
  • References: Curr Sports Med Rep 2018;17:290, J Clin Orthop Trauma 2021;17:123
Radiology images

Contributed by Nasir Ud Din, M.B.B.S.
Early stage

Xray of early stage

Xray of mid to late stage

Xray of mid to late stage

MRI buttock for soft tissue mass

MRI buttock for soft tissue mass



Images hosted on other servers:
Radiology of different stages

Different stages

Prognostic factors
Case reports
Treatment
Clinical images

Images hosted on other servers:
Clinical photograph (a) and chest radiograph (b)

Large right pectoral mass and chest radiograph

Gross description
Gross images

Images hosted on other servers:
Excised and bisected mass

Excised and bisected mass

Microscopic (histologic) description
  • Histologically, zonal pattern is characteristic with different degrees of cellular differentiation (inner zone, intermediate zone and peripheral zone)
    • Inner central zone:
      • Composed of fibroblastic / myofibroblastic proliferation, which is richly vascular, rich in inflammatory cells and resembles nodular fasciitis; some multinucleated giant cells may also be seen
      • Cells show mild degree of pleomorphism and brisk mitosis
      • Areas of hemorrhage, fibrin, endothelial proliferation and entrapped atrophic muscle fibers are noted
    • Intermediate zone:
      • There is a mixture of fibroblasts and osteoblasts along with erratic osteoid separated by small sized vessels
      • Scattered chondrocytes may be appreciated
    • Peripheral zone:
      • Osteoid undergoes calcification and leads to lamellar bone formation
      • Islands of mature or immature cartilage may be present
      • Extreme periphery / margin shows mature bone with osteoblastic rimming and little to no pleomorphism
      • Lesion is separated from the normal tissue (muscle) by a zone of loose, myxoid fibrous tissue
  • Early stage (0 - 4 weeks):
    • Mass shows central zone morphology with only rare foci showing osteoid
    • Zonal pattern is not very much appreciated
  • Intermediate stage (4 - 8 weeks):
    • Zonal pattern is appreciated with central zone, intermediate zone and peripheral zone
  • Late, mature stage (> 8 weeks):
    • Mass is mostly composed of mature bone
    • Very old lesions show only lamellar bone separated by fibrovascular stroma, mimicking osteoma
  • References: Curr Sports Med Rep 2018;17:290, Goldblum: Enzinger and Weiss's Soft Tissue Tumors, 7th Edition, 2019, Exp Ther Med 2021;21:531, Mol Clin Oncol 2018;8:749, Autops Case Rep 2021;11:e2021316
Microscopic (histologic) images

Contributed by Ghazi Zafar, M.B.B.S., Nasir Ud Din, M.B.B.S. and @JMGardnerMD on Twitter
Periphery

Periphery

Circumscription

Circumscription

Zonation

Zonation

Central zone

Central zone

Foci of cartilage

Foci of cartilage

Osteoid production

Osteoid production


Multinucleated giant cells

Multinucleated giant cells

Myositis ossificans and fibro-osseous pseudotumor of digits Myositis ossificans and fibro-osseous pseudotumor of digits Myositis ossificans and fibro-osseous pseudotumor of digits Myositis ossificans and fibro-osseous pseudotumor of digits

Myositis ossificans and fibro-osseous pseudotumor of digits

Cytology description
Cytology images

Images hosted on other servers:
Cytological features

Cytological features

Positive stains
Negative stains
Electron microscopy description
  • Fibroblasts and myofibroblasts have dilated rough endoplasmic reticulum and aggregates of cytoplasmic filaments variably associated with dense bodies; osteoblasts have numerous mitochondria and abundant, dilated rough endoplasmic reticulum
Molecular / cytogenetics description
Videos

Myositis ossificans (benign mimic of sarcoma) bone / soft tissue pathology & radiology correlation

Sample pathology report
  • Thigh mass, excision:
    • Myositis ossificans (see comment)
    • Comment: The sections from the mass show a circumscribed lesion with characteristic zonal pattern, with inner zone showing tissue culture-like background, against which are seen plump fibroblasts and myofibroblasts. This area is rimmed by immature bone in the middle and mature lamellar bone at the periphery.
Differential diagnosis
  • Extraskeletal osteosarcoma:
    • Usually occurs in sixth or seventh decade
    • No zoning pattern is seen (backward zonation is appreciated; that is, most mature bone in the center with woven bone at the periphery) (Czerniak: Dorfman and Czerniak's Bone Tumors, 2nd Edition, 2015, Clin Case Rep 2019;7:2260)
    • There is a mixture of atypical spindle cells with moderate to marked degree of pleomorphism and increased mitoses
    • Osteoid is seen directly produced by these atypical cells
  • Parosteal osteosarcoma:
    • Can cause diagnostic difficulty, especially when myositis ossificans is located near the surface of a major long tubular bone
    • Radiology shows central heavy mineralization as opposed to myositis ossificans
    • Connection with underlying bone is seen in parosteal osteosarcoma
    • Parosteal osteosarcoma shows well developed tumor bone, unlike zonal pattern of myositis ossificans
  • Periosteal osteosarcoma:
    • Connection with the bone surface is identifiable with periosteal reaction
    • Direct tumor bone formation identified
    • No zonal pattern
  • Fibrodysplasia ossificans progressiva:
    • Multifocality and specific hypoplasia of the first metacarpal and metatarsal bones
    • Rare genetic disorder that spreads along the muscle planes
  • Calcified / ossified synovial sarcoma:
    • Can become extensively calcified with secondary bone formation
    • Lacks zoning pattern
    • Highly hyperchromatic tumor cells express SS18::SSX and TLE1 immunostains
Board review style question #1

A 36 year old boxer presented with a painful mass in the right arm for the last 1.5 months. On radiology, an intramuscular mass was seen with peripheral mineralization and central latency. It was biopsied, which showed the morphology in the image above. What is the most likely diagnosis?

  1. Extraskeletal osteosarcoma
  2. Myositis ossificans
  3. Periosteal osteosarcoma
  4. Synovial sarcoma with bone formation
Board review style answer #1
B. Myositis ossificans. The given history in this case shows that the patient is physically active and provides short duration history of the lesion. The radiological findings suggest a lesion with maturation at periphery. The microscopic image shows characteristic zonation. These are all features of myositis ossificans.

Comment Here

Reference: Myositis ossificans
Board review style question #2
Biopsy from a thigh mass in a 31 year old man shows a circumscribed lesion rimmed entirely by skeletal muscle and comprised of a haphazard arrangement of spindle cells with mild nuclear pleomorphism, prominent nucleoli and increased mitotic figures. The background is highly fibrovascular and shows extravasated red blood cells, inflammatory cells and myxoid change. At the advancing edge of the lesion, there is focal, mature lamellar bone present. What is the likely diagnosis?

  1. Extraskeletal osteosarcoma
  2. Myositis ossificans
  3. Nodular fasciitis
  4. Ossified hemangioma
Board review style answer #2
B. Myositis ossificans. The question describes a detailed microscopic description of myositis ossificans. The lesion is not osteosarcoma because it is circumscribed and shows zonation in the form of mature periphery and immature center. This is contrary to osteosarcoma. Nodular fasciitis does not show heterotopic ossification and zonal pattern, although the center of myositis ossificans is reminiscent of nodular fasciitis.

Comment Here

Reference: Myositis ossificans

Myospherulosis
Definition / general
  • Iatrogenic benign mass composed of fungi-like spherules that are actually erythrocytes damaged by endogenous and exogenous fat
Etiology
  • Due to erythrocyte damage from endogenous and exogenous fat
  • Also due to endogenous membranocystic degeneration of fat that occurs in lupus erythematosus and in membranous lipodystrophy with dermal atrophy due to local application of steroid ointment (Arch Dermatol 1991;127:88)
  • In the gluteal region, this entity is described in relation to old injections of petrolatum based hormones and penicillin (Diagn Cytopathol 1988;4:137, J Am Acad Dermatol 1989;21:400)


History:
  • First described by McClatchie (E Afr Med J 1969;46:625), who reported 7 patients from Kenya with unusual soft tissue nodules in arm, legs and subcutaneous tissue of buttock
  • Called myospherulosis due to the involvement of skeletal muscle in some patients (Am J Clin Pathol 1969;51:699)
  • Five patients were subsequently reported by Hutt in Uganda (Trans R Soc Trop Med Hyg 1971;65:182)
  • Initially these structures were thought to be a fungus, but the usual stains for fungi were negative
  • Kyriakos (Am J Clin Pathol 1977;67:118) reported non African cases in paranasal sinuses, nasal cavity and middle ear; most patients had undergone surgery and the surgical wound was packed with gauze impregnated with petrolatum and tetracycline ointment, suggesting an iatrogenic etiology
  • De Schriver and Kyriakos confirmed this etiology by inducing similar lesions in experimental animals (Am J Pathol 1977;87:33)
  • Rosai (Am J Clin Pathol 1978;69:475) and Wheeler (Arch Otolaryngol 1980;106:272) demonstrated that the spherules were erythrocytes damaged by endogenous and exogenous fat
  • Travis (Arch Pathol Lab Med 1986;110:763) and Shimada (Am J Surg Pathol 1988;12:427) confirmed the presence of damaged erythrocytes by immunostaining for hemoglobin
  • Kakizaki (Am J Clin Pathol 1993;99:249) demonstrated that the wall of the spherules was due to the physical emulsion phenomenon that occurs between lipid containing materials and blood
  • The damaged erythrocytes are enclosed by a lipid membrane and later phagocytosed by histiocytes as part of the lipogranulomatous reaction that takes place in adipose tissue
Clinical features
Case reports
Treatment
Treatment and prognosis:
  • Benign process; no treatment needed other than for symptomatic relief
Gross description
  • Large saccular cyst-like lesion, surrounded by fat
  • May contain oily substance with a yellow color
Gross images

Case #173

Buttock tumor



Images hosted on other servers:

Large gluteal mass

Microscopic (histologic) description
  • Cyst composed of spherules which are damaged erythrocytes
  • Wall of the cyst made of fibrous tissue, accompanied by a lipogranulomatous reaction
  • Many eosinophilic spherules containing red blood cells are within histiocytes lining the cyst wall
  • Some larger spherules resemble a bag of marbles
  • Dermal nodules are cystic cavities with a fibrous wall lined by histiocytes and multinucleated foreign body giant cells, with lipogranulomatous inflammation in the adipose tissue adjacent to the cavities
Microscopic (histologic) images

Case #173

Buttock tumor



Images hosted on other servers:

Gluteal mass

Negative stains

Myxofibrosarcoma
Definition / general
  • Myxofibrosarcoma includes a range of malignant fibroblastic tumors with myxoid stroma, variable pleomorphism and characteristic curvilinear vasculature (Surg Oncol Clin N Am 2016;25:775)
Essential features
  • Most common sarcoma of elderly patients
  • Painless, superficial, subcutaneous mass of limbs
  • Multinodular, gelatinous, lobulated tumor with infiltrative margins
  • Spindle to stellate cells in a myxoid stroma with curvilinear vessels and pleomorphic cells
  • Hypercellularity and severe pleomorphism in high grade tumors (Surg Oncol Clin N Am 2016;25:775)
  • Rare epithelioid variant is characterized by nests and sheets of epithelioid cells with focal conventional areas
Terminology
  • Myxoid malignant fibrous histiocytoma (not recommended)
ICD coding
  • ICD-O: 8811/3 - fibromyxosarcoma
  • ICD-10: C49.9 - malignant neoplasm of connective and soft tissue, unspecified
  • ICD-11: 2B53.0 - myxofibrosarcoma, primary site
Epidemiology
  • Affects elderly in the sixth to eighth decades of life (Cancer 2012;118:518)
  • Rare under 30 years
  • Slight male predominance
  • Dermal / subcutaneous location: > 50% of cases
  • Deep / fascial in 33% of cases
Sites
  • Painless, slow growing mass
  • Arises in limbs and limb girdles (lower limb > upper limb) (J Surg Oncol 2021;123:489)
  • Rare on trunk, head and neck, hands and feet
  • Location in body cavities, like abdomen and retroperitoneum, is very rare
    • Most cases of these sites represent dedifferentiated liposarcoma with myxofibrosarcoma-like features
Pathophysiology
  • Complex karyotype with intratumoral heterogeneity and aneuploidy
  • Nondistinct numerical and structural chromosomal abnormalities
  • Higher grade and recurrent tumors show more complex cytogenetic aberrations (Mod Pathol 2006;19:407)
Etiology
  • Unknown
Clinical features
Diagnosis
  • Diagnosis is based primarily on histologic findings in correlation with age, site and clinical features
  • Any undifferentiated pleomorphic sarcoma arising in an extremity of an elderly patient requires thorough sampling to find low grade areas of myxofibrosarcoma
Laboratory
  • Nonspecific
Radiology description
  • CT: low accentuation (AJR Am J Roentgenol 2007;188:W193)
  • MRI:
    • Myxoid component shows low to intermediate signal on T1 weighted images
    • Solid and myxoid components show high signal on T2 / short tau inversion recovery (STIR)
    • Myxoid component does not usually enhance, while the solid component shows enhancement on T1 C+ (Gd)
    • Abnormal signal infiltration along fascial planes (tail sign) (Skeletal Radiol 2013;42:809)
  • Ultrasound: heterogeneous mass, variable presentations (Skeletal Radiol 2022;51:691)
Radiology images

Contributed by Nasir Ud Din, M.B.B.S.
Left shoulder

Left shoulder

Right axillary mass

Right axillary mass



Images hosted on other servers:
Infiltrating buttock tumor

Infiltrating buttock tumor

Tail sign

Tail sign

Prognostic factors
  • Prognosis is good when tumor is superficial and low grade
  • Better prognosis than undifferentiated pleomorphic sarcoma
  • Tumor grade affects metastasis and tumor related mortality (30 - 35%) (Anticancer Res 2019;39:2985)
  • Low grade tumors usually do not metastasize (< 5%)
  • High grade tumors have metastatic rate of 20 - 35% (Orthop Traumatol Surg Res 2020;106:1059)
  • The lesser the myxoid area, the poorer the prognosis
  • Epithelioid myxofibrosarcoma is highly aggressive with local recurrence rate of 70% and metastatic rate of 50%
  • Common metastatic sites are lung, intra-abdominal organs and retroperitoneum, also seen in bone and lymph nodes
  • Inadequate surgery leads to local, often repeated recurrence (30 - 40%), irrespective of grade (Ann Surg Oncol 2013;20:80)
  • Tumor grade tends to increase with recurrence
  • Predictors of survival include tumor size, grade and margins (Ann Surg Oncol 2011;18:720)
  • Survival rate, after transformation into an undifferentiated pleomorphic sarcoma-like condition (< 10% myxoid area), approaches that of undifferentiated pleomorphic sarcoma (Am J Surg Pathol 2020;44:87)
  • Immunostain CD44 expression is associated with increased risk of metastasis (Ann Transl Med 2021;9:1322)
  • Immunostain CD109 expression is associated with poor prognosis (J Surg Oncol 2015;111:975)
Case reports
Treatment
Clinical images

Images hosted on other servers:
Superficial leg mass

Superficial leg mass

Left leg swelling

Left leg swelling

Gross description
  • Superficial: multinodular with infiltrative margins, usually < 10 cm
  • Cut surface is lobulated, gelatinous and myxoid with gray-white, firm nodules
  • Deep: infiltrative single mass, usually large (Jpn J Clin Oncol 2000;30:458)
  • Hemorrhage and necrosis can be present
Gross images

Images hosted on other servers:
Multilobulated mass

Multilobulated mass

Thigh tumor

Thigh tumor

Lower extremity tumor

Lower extremity tumor

Subcutaneous tumor Subcutaneous tumor

Subcutaneous tumor

Frozen section description
  • Usually not performed for diagnostic purpose
  • Used for margin assessment (J Surg Oncol 2021;123:489)
  • Histologic features depend upon the area sampled: low grade versus high grade
  • Atypical spindle cell proliferation (Oral Oncology 2013;49:S123)
  • May show myxoid stroma
Microscopic (histologic) description
  • Lobulated tumor with multinodular growth and incomplete fibrous septae
  • Alternate hypocellular and hypercellular areas
  • 3 tier grading system / FNCLCC grading system used (Oncol Res Treat 2020;43:189)
  • Low grade tumor:
    • Noncohesive, spindle to stellate cells with moderate cytoplasm and atypical, hyperchromatic nuclei
    • Occasional pleomorphic and touton type giant cells
    • Abundant myxoid stroma
    • Elongated, curvilinear blood vessels
    • Perivascular condensation of tumor cells / inflammatory cells (lymphocytes and plasma cells)
    • Pseudolipoblasts (vacuolated fibroblasts) with cytoplasmic mucin and peripherally placed nuclei may be present
    • Infrequent mitoses (Zhonghua Bing Li Xue Za Zhi 2017;46:170)
  • High grade tumor:
    • Cellular fascicles and sheets of spindled to pleomorphic cells with moderate cytoplasm, hyperchromatic nuclei and variable prominent nucleoli
    • Bizarre, multinucleated giant cells with abundant eosinophilic cytoplasm and irregular nuclei
    • Prominent inflammatory infiltrate, more conspicuous at periphery
    • Brisk mitoses with atypical ones, hemorrhage and necrosis
    • Focal conventional low grade areas
  • Intermediate grade tumor:
    • More cellularity and pleomorphism than low grade tumors
    • Lacks solid sheets, marked pleomorphism and necrosis
    • Presence of any hypercellular nonmyxoid area in an otherwise myxoid tumor should be regarded as intermediate grade
  • Epithelioid myxofibrosarcoma (subtype):
    • Predominant atypical epithelioid cells in an otherwise conventional myxofibrosarcoma
    • Cells arranged in nests and sheets
    • Have abundant glassy eosinophilic cytoplasm, vesicular nuclei and prominent nucleoli
    • Diagnosis requires exclusion of carcinoma, melanoma, lymphoma and malignant epithelioid neoplasms as well as presence of conventional myxofibrosarcoma areas, which may be very focal (Am J Surg Pathol 2007;31:99)
Microscopic (histologic) images

Contributed by Nasir Ud Din, M.B.B.S.
Lobulated tumor

Lobulated tumor

Myxoid background

Myxoid background

Curvilinear vessels

Curvilinear vessels

Pleomorphic cells

Pleomorphic cells

Touton type giant cells

Touton type giant cells


Perivascular condensation

Perivascular condensation

Pseudolipoblasts

Pseudolipoblasts

Prominent vacuolation

Prominent vacuolation

Hyperchromatic atypical spindle cells

Hyperchromatic atypical spindle cells

Transition

Transition


Infiltration into skeletal muscle

Infiltration into skeletal muscle

Lobulated cellular tumor

Lobulated cellular tumor

Epithelioid myxofibrosarcoma

Epithelioid myxofibrosarcoma

Epithelioid cells

Epithelioid cells

Cytology description
  • Diagnostic in nearly 65% of cases
  • Spindled cells in a variable amount of myxoid stroma; occasional pleomorphic cells
  • Arborizing capillaries
  • High grade tumor shows pleomorphic and spindled cells in variable myxoid stroma (J Am Soc Cytopathol 2021;10:300)
Cytology images

Images hosted on other servers:
Spindle and pleomorphic cells in myxoid matrix

Spindle and pleomorphic cells in myxoid matrix

Positive stains
Electron microscopy description
  • Spindle shaped, fibroblast-like cells: smooth nuclei with abundant parallel membranes and wide cisternae of endoplasmic reticulum
  • Irregular histocyte-like cells: indented nuclei with large nucleoli and coarse chromatin; cytoplasm with conspicuous systems of vesicles and vacuoles
  • Myofibroblast-like cells: abundant cytoplasmic microfilaments with dense body-like structures
  • Occasional cells: intermediate forms and undifferentiated cells
  • Multinucleated tumor cells in high grade tumors (Semin Diagn Pathol 2003;20:72, Ultrastruct Pathol 2013;37:9)
Electron microscopy images

Images hosted on other servers:
Ultrastructure of myxofibrosarcoma

Ultrastructure

Distended cisternae with granular material

Distended cisternae with granular material

Molecular / cytogenetics description
  • Somatic copy number alterations:
    • Gains of chromosome 5p
  • Genetic aberrations of p53 signaling and cell cycle G1/S checkpoint:
    • More common: TP53, RB1, CDKN2A / CDKN2B mutations (Sci Rep 2017;7:44700)
    • Less common: CDK6, CCND1, MDM2 amplifications
  • Mutations in components of driver regulomic pathways:
    • NTRK1 mutation and amplification / NF1 alterations in RTK / RAS / MAPK pathway
    • VGLL3 amplification in Hippo pathway
    • TRIO / RICTOR amplifications in RAC / PAK and Akt / mTOR pathways
  • Reference: Mod Pathol 2017;30:1698
Molecular / cytogenetics images

Images hosted on other servers:
Genetic analysis

Genetic analysis

Altered genetic pathways

Altered genetic pathways

Videos

Myxofibrosarcoma explained by a sarcoma pathologist

Fibromyxo versus myxofibro sarcoma made simple

Sample pathology report
  • Thigh, wide margin excision:
    • Features are consistent with high grade myxofibrosarcoma, FNCLCC grade 3 (see comment)
    • Tumor size: 8 x 8 x 6 cm
    • Margins:
      • Superior margin: 2 cm
      • Inferior margin: 2 cm
      • Medial margin: 2.1 cm
      • Lateral margin: 2.2 cm
      • Anterior margin: 1.8 cm
      • Posterior margin: 3 cm
    • pTNM stage: pT2, N not assigned
    • Comment: Histology shows a lobulated infiltrative tumor with hypo and hypercellular areas. The hypocellular areas show spindle cells in a myxoid background with curvilinear vessels. The hypercellular areas show fascicles and sheets of moderate to severely pleomorphic, plump spindle to epithelioid cells with interspersed bizarre tumor and touton type giant cells. Brisk mitoses and necrosis are present. Tumor cells show focal SMA and CD34 positivity, whereas desmin, caldesmon, S100 and pancytokeratin are negative. Morphological features and immunoprofile strongly support the diagnosis of high grade myxofibrosarcoma. It is a malignant soft tissue sarcoma with high risk of local recurrence if inadequately excised. Metastatic potential in high grade tumors approaches 20 - 35%.
Differential diagnosis
Board review style question #1

A 55 year old man presented with a painless lump in the left thigh. Gross specimen showed a subcutaneous multinodular tumor of 9 x 8 cm. Histology showed a lobulated and focally infiltrative myxoid tumor with alternating hypo and hypercellular areas. The hypocellular areas showed spindle cells in myxoid stroma with curvilinear blood vessels. The hypercellular areas showed pleomorphic cells in sheets. Brisk mitoses and necrosis were present. Immunostains SMA and CD34 were focal positive and CK AE1 / AE3, caldesmon, STAT6, desmin and S100 were negative. What is the most likely diagnosis?

  1. Cellular myxoma
  2. Malignant peripheral nerve sheath tumor
  3. Myxofibrosarcoma
  4. Myxoid liposarcoma
  5. Myxoinflammatory fibroblastic sarcoma
Board review style answer #1
C. Myxofibrosarcoma

Comment Here

Reference: Myxofibrosarcoma
Board review style question #2

Which vasculature pattern is classic of myxofibrosarcoma?

  1. Chicken wire vessels
  2. Curvilinear vessels
  3. Ectatic vessels
  4. Gaping vessels
  5. Staghorn vessels
Board review style answer #2
B. Curvilinear vessels

Comment Here

Reference: Myxofibrosarcoma

Myxoid liposarcoma
Definition / general
  • Malignant tumor composed of primitive nonlipogenic mesenchymal cells, signet ring lipoblasts and prominent myxoid stroma with a highly characteristic branching vascular pattern
Essential features
  • Prominent myxoid stroma with branching vasculature (so called chicken wire vasculature)
    • Majority of the tumor can be nonlipogenic with only scattered lipoblasts that often have a characteristic signet ring morphology
  • Recurrent molecular alteration with either t(12;16)(q13;p11.2) FUS::DDIT3 or very rarely (~2%) t(12;22)(q13;q12) EWSR1::DDIT3 rearrangements
  • Includes a spectrum of disease including high grade lesions, which were formerly regarded as round cell liposarcoma (see Terminology)
  • Has an unusual propensity to present with a first metastasis to another soft tissue or bony site (such as from one leg to the contralateral leg or to the retroperitoneum or spine)
Terminology
  • Although myxoid liposarcoma and round cell liposarcoma were initially described separately, both tumors have identical molecular alterations and clear evidence of cases with transition between the 2 morphologic patterns are easily found
    • Both tumors are now classified as myxoid liposarcoma with a secondary designation of high or low grade
  • Percent of round cell component needed for this distinction is debated in the literature with most pathologists using either > 25% or > 5% indicating a high grade lesion
    • WHO currently lists 5%
  • Cases with borderline round cell component (< 5%) are regarded as having areas of transition, a designation of unclear diagnostic significance
ICD coding
  • ICD-O: 8852/3 - myxoid liposarcoma
  • ICD-11: 2B59.Y & XH3EL0 - liposarcoma, other specified primary site & myxoid liposarcoma
Epidemiology
  • Peaks in the fourth and fifth decades and represents ~5% of adult sarcomas (Int J Surg Oncol 2018;2018:8928706)
  • M = F
  • May occur in children or very rarely in the elderly
Sites
  • Usually in the extremities, proximal thigh is quite common
  • Primary retroperitoneal involvement is rare, although metastasis to the retroperitoneal soft tissue is not uncommon
  • Propensity to present with multifocal disease, seemingly from hematogenous spread that has spared the lungs (J Surg Oncol 2002;80:89)
  • Primary subcutaneous tumors have been reported (J Cutan Pathol 2014;41:907)
Diagnosis
  • Core needle biopsy is minimally invasive and can provide sufficient tissue sampling for definitive diagnosis, although it may not allow for accurate tumor grading due to sampling bias
  • Surgical resection is the gold standard procedure for ensuring accurate diagnosis and prognostication
Radiology description
Radiology images

Images hosted on other servers:

Well circumscribed
intermuscular
mass within left
thigh on MRI

MRI of myxoid liposarcoma postradiotherapy

Prognostic factors
  • Predilection for metastasis, especially when there is a significant round cell component
  • Metastases can occur decades later, necessitating continued long term follow up (World J Surg Oncol 2008;6:62)
  • Pure low grade tumors have less aggressive behavior but still share a propensity for recurrence and can metastasize in 5 - 10% of cases
  • Unfavorable outcome has been associated with TP53 and CDKN2A gene mutations, although this seems rare (Tumori 1998;84:571)
  • Some evidence suggests that high grade components can be identified via MRI and that metastases can be identified very early with imaging studies (Acta Radiol 2014;55:952, Skeletal Radiol 2018;47:369)
Case reports
Treatment
  • Surgical resection with negative margins is the treatment of choice
  • Trabectedin may be useful in nonresectable tumors
  • Neoadjuvant or adjuvant radiation therapy may be considered since myxoid liposarcoma has been found to be more radiosensitive than many other soft tissue tumor types (Cancer 2009;115:3254)
Gross description
  • Typically these lesions are well circumscribed and multinodular
  • Low grade tumors will have a gelatinous cut surface, with higher grade tumors showing a more solid fleshy appearance
Gross images

AFIP images

Well differentiated myxoid liposarcoma

Thigh tumor postchemotherapy



Images hosted on other servers:

Recurrence of adipose mass with kidney involvement

Well circumscribed white tumor

Lobulated gray white tumor within skeletal muscle

Microscopic (histologic) description
  • Low grade
    • Paucicellular with monomorphic, stellate or fusiform shaped cells without atypia; striking in their blandness, so much so that any significant pleomorphism should cause one to pause
    • Prominent plexiform vasculature (delicate thin walled arborizing and curving capillaries that form a network reminiscent of chicken wire fencing)
      • These are striking because of the overall background paucicellularity and are still present in high grade tumors but are much less obvious
    • Numerous signet ring lipoblasts, particularly at periphery of lobules
    • Mucoid matrix is rich in hyaluronic acid that may form large mucoid pools (so called pulmonary edema pattern)
    • Metaplastic cartilage or bone can rarely be seen
    • Typically there is no significant mitotic activity
    • There are many rare morphologic variants (Am J Clin Pathol 2012;137:229)
  • High grade
    • Hypercellular solid sheets of back to back cells with round cell or primitive cytomorphology in > 5% of the sampled tumor
    • Cells can have a small amount of hypereosinophilic cytoplasm, a finding of no clinical significance but of significant diagnostic confusion, especially in a limited sample
  • Pitfalls and tips
    • High grade tumors are so cellular that you can typically walk across nuclei in a high power field without stepping in matrix
    • When in doubt, especially in a small sample, pursue molecular testing (typically fluorescent in situ hybridization) for FUS gene rearrangement
      • If no FUS gene rearrangement is detected and your suspicion is high that the lesion represents myxoid liposarcoma, proceed to EWSR1 fusion testing, as EWSR1::DDIT3 fusions are detected in rare cases (see Molecular / cytogenetics description)
    • Sample these tumors extensively; you likely will not see small amounts of round cell progression grossly
    • Can contain large areas of mature adipocytic differentiation
      • If the clinical or radiologic picture is concerning, sample additional tumor or do molecular testing
    • Location (e.g., extremity) and age of the patient (e.g., young adult) can be helpful clues in the differential diagnosis
    • Most of the diagnostic clues are helpful in the appropriate context but individually can be seen in many other tumor types
      • Plexiform vasculature and cells that look like signet rings can be found in a diverse variety of tumors
      • Combination of a number of clinical, radiologic, histologic and if needed, molecular features will make the diagnosis
Microscopic (histologic) images

Contributed by Jesse J. Jenkins, M.D. and AFIP

Bland stellate shaped cells

High power

Pulmonary edema pattern

Signet ring lipoblasts

Postchemotherapy


Sheets of primitive round cells

Cords of cells resembling myxoid chondrosarcoma

Hyalinized collagenous stroma


Delicate capillary network

Large (mature) and small fat cells


Pulmonary edema pattern

Hypocellular zones


Signet ring lipoblasts

Alcian blue

S100 protein

Vimentin

Cytology description
Negative stains
Molecular / cytogenetics description
  • t(12;16)(q13;p11.2) FUS::DDIT3 in most cases
  • t(12;22)(q13;q12) EWSR1::DDIT3 very rarely (2 - 5%)
  • Rearrangements can be complex and involve other chromosomes (Am J Clin Pathol 1995;103:20)
  • DDIT3 was formerly known as CHOP (early literature will use the terminology FUS::CHOP)
  • Can be identified via FISH, PCR, classic cytogenetics and sequencing
Videos

Myxoid liposarcoma

High grade myxoid liposarcoma

Sample pathology report
  • Soft tissue mass, left thigh, resection:
    • Myxoid liposarcoma, low grade (5.5 cm) (see comment)
    • Surgical resection margins are negative for tumor
    • Comment: Histologic sections of this soft tissue mass demonstrate a hypocellular lesion composed of bland round to stellate cells evenly dispersed in a myxoid stroma with delicate arborizing vasculature. No areas of high grade disease are identified. FISH studies are positive for a DDIT3 gene rearrangement, supporting the diagnosis.
Differential diagnosis
  • Low grade:
    • Atypical lipomatous tumor / well differentiated liposarcoma (ALT / WDL):
      • Can be myxoid and focally indistinguishable from myxoid liposarcoma
      • Usually has some degree of stromal atypia, however, and will lack the plexiform vasculature
      • Has amplification of chromosome 12q14 (including the MDM2 gene) versus the FUS rearrangement seen in myxoid liposarcoma
      • Myxoid liposarcomas are also more likely to show a predilection for signet ring lipoblasts
    • Extraskeletal myxoid chondrosarcoma (EMC):
      • Composed of cords of epithelioid malignant cells set in a similar chondromyxoid matrix
      • No cytoplasmic fat vacuoles and less prominent vasculature
      • Immunohistochemical staining is not helpful (both are S100 positive)
      • Cytogenetics can be helpful but care must be taken
      • EMC has t(9;22)(q22;q12) gene rearrangements in most cases that result in an EWSR1::NR4A3 fusion
      • EWSR1 FISH will be positive but can lead to confusion with the 2 - 5% of myxoid liposarcomas that rarely have EWSR1 rearrangements
      • PCR applications can be particularly helpful in this setting
      • In the lung, primary pulmonary myxoid sarcoma should be considered (Pathology 2017;49:792)
    • Lipoblastoma / lipoblastomatosis:
      • Can show similar histology but usually are present in patients < 5 years old
      • Will have PLAG1 gene rearrangements instead of FUS or EWSR1 rearrangements
    • Lipoblastoma-like tumor of the vulva:
      • Recently described entity that occurs in the vulva and shares remarkable histologic overlap with lipoblastoma and myxoid liposarcoma
      • These lesions have been shown to have Rb loss like the spindle cell lipoma family of tumors
      • Does not have PLAG1 or FUS rearrangements (Int J Gynecol Pathol 2019;38:204, Am J Surg Pathol 2015;39:1290)
      • There is a significant difference in treatment and clinical outcome (only a limited ability to locally recur) and these lesions should be distinguished when possible
    • Myxoid dermatofibrosarcoma protuberans (DFSP):
      • Typically these are located superficially, which is uncommon in myxoid liposarcoma
      • Look carefully to distinguish between entrapped fat versus true signet ring lipoblasts
      • Immunohistochemical staining and molecular testing can help
      • DFSP will be CD34 positive, S100 negative, with the inverse seen in myxoid liposarcoma and DFSP will harbor the t(17;22)(q22;q13) COL1A1::PDGFB gene fusion
      • Some large reference labs offer PDGFB as a break apart FISH assay
    • Myxoma:
      • Extremely paucicellular, lacks a prominent vascular component and no lipoblasts are found
      • Associated with GNAS mutations (Mod Pathol 2009;22:718)
  • High grade:
    • Myxofibrosarcoma:
      • Older adults, often superficial, infiltrative and with no true cytoplasmic fat vacuoles (although they can contain pseudolipoblasts)
      • Significantly more nuclear atypia, thicker curvilinear vessels and frequent mitotic figures
    • Pleomorphic liposarcoma (PLS):
      • Entirely different entity that shares similarity in name only
      • High grade pleomorphic sarcoma with scattered atypical lipoblasts
      • Atypia far in excess of what is seen in round cell liposarcoma, which retains its monotony (a clue to a tumor driven by a translocation)
      • Typically has a complex karyotype
    • Round cell sarcomas (Ewing, BCOR::CCNB3, CIC::DUX4, etc.):
      • Numerous round cell sarcomas that may morphologically resemble high grade myxoid liposarcoma
      • Low grade areas and lipoblasts can be particularly informative
      • Immunohistochemical and molecular differences can also be exploited (Ewing sarcoma has different partner genes than does myxoid liposarcoma, a feature that can be taken advantage of via sequencing or PCR)
Board review style question #1

The microscopic image above depicts the histologic appearance of a soft tissue mass resected from the thigh of a 37 year old man and is representative of the entire lesion. Which of the following is true regarding the diagnosis?

  1. Immunohistochemical studies for MDM2 will show strong diffuse positive nuclear staining in the tumor cells
  2. Majority of cases harbor t(7;16) FUS::CREB3L2 gene fusion
  3. Majority of cases harbor t(9;22) EWSR1::NR4A3 gene fusion
  4. Majority of cases harbor t(12;16) FUS::DDIT3 gene fusion
  5. The lesion is classified as high grade if hypercellular solid sheets of back to back cells with round cell cytomorphology compose > 1% of the sampled tumor
Board review style answer #1
D. Majority of cases harbor t(12;16) FUS::DDIT3 gene fusion. The microscopic image depicts a low grade myxoid liposarcoma characterized by a proliferation of bland stellate tumor cells embedded in a myxoid matrix with a prominent plexiform capillary network and scattered signet ring lipoblasts. There is no mitotic activity or pleomorphism. The majority of cases of myxoid liposarcoma harbor a t(12;16) FUS::DDIT3 gene fusion. Answers B and C are incorrect because the t(9;22) EWSR1::NR4A3 gene fusion is characteristic of extraskeletal myxoid chondrosarcoma and the t(7;16) FUS::CREB3L2 gene fusion is characteristic of low grade fibromyxoid sarcoma. Answer E is incorrect because classification of myxoid liposarcoma as high grade requires > 5% of the sampled lesion to consist of a hypercellular round cell component according to the WHO. Cases with borderline round cell component (< 5%) are regarded as having areas of transition, a designation of unclear diagnostic significance. Answer A is incorrect because immunohistochemistry for MDM2 is negative in myxoid liposarcomas.

Comment Here

Reference: Myxoid liposarcoma
Board review style question #2
Which of the following features is characteristic of myxoid liposarcomas?

  1. Hyalinized, thick walled vessels
  2. Poor response to radiation therapy
  3. Primary tumor in the retroperitoneum
  4. Propensity to skip the lung and metastasize to other soft tissue sites
  5. Marked nuclear atypia
Board review style answer #2
D. Propensity to skip the lung and metastasize to other soft tissue sites. Myxoid liposarcomas have a unique propensity to metastasize to other soft tissue sites. Answers A and E are incorrect because these tumors tend to have bland nuclear features and arborizing plexiform vasculature. Answer B is incorrect because they have been found to be more radiosensitive than many other soft tissue tumor types. Answer C is incorrect because although myxoid liposarcomas can commonly metastasize to the retroperitoneum, primary retroperitoneal tumors are quite rare.

Comment Here

Reference: Myxoid liposarcoma

Myxoid pleomorphic liposarcoma
Definition / general
  • Rare and aggressive variant of liposarcoma that predominantly affects young patients and shows hybrid morphological features of myxoid liposarcoma and pleomorphic liposarcoma
Essential features
  • Aggressive soft tissue tumor with poor overall survival
  • Frequently affects young patients (< 30 years old) but some cases occur in older adults
  • Predominantly arises in the mediastinum
  • Most cases consist of a variable mixture of myxoid liposarcoma and pleomorphic liposarcoma-like areas
  • FUS / EWSR1-DDIT3 fusions and MDM2 amplification are absent
Terminology
  • Pleomorphic myxoid liposarcoma
ICD coding
  • ICD-O: 8859/3 - myxoid pleomorphic liposarcoma
  • ICD-11:
    • 2B59.Y - liposarcoma, other specified primary site
    • XH3EL0 - myxoid liposarcoma
    • XH25R1 - pleomorphic liposarcoma
Epidemiology
Sites
Pathophysiology
  • Losses of chromosome 13, including the RB1 loci, are frequent and have been postulated as an important hallmark in myxoid pleomorphic liposarcoma tumorigenesis (Histopathology 2016;69:141)
Clinical features
Diagnosis
  • Imaging features are poorly characterized; diagnosis depends on histologic findings and molecular techniques (selected cases)
Radiology description
  • Poorly characterized radiological findings (data restricted to isolated case reports)
  • MRI: large deep mass with low signal on T1 and high signal on T2, favoring a myxoid component (Histopathology 2016;69:141)
Prognostic factors
Case reports
Treatment
  • Complete surgical excision with negative margins is the only potentially curative treatment
  • Specific target therapies are currently unavailable
  • Utility of adjuvant therapies (chemotherapy or radiotherapy) is not well established (Am J Surg Pathol 2009;33:645, Mod Pathol 2021;34:2043)
  • Rare associations with Li-Fraumeni syndrome warrant prudence when considering treatment with radiotherapy
Gross description
  • Macroscopic findings are poorly characterized in the literature
  • Large and grossly infiltrative, to nodular masses with a variably myxoid cut surface (Int J Surg Pathol 2020;28:225)
Microscopic (histologic) description
  • Most cases consist of a variable mixture of myxoid liposarcoma and pleomorphic liposarcoma-like areas (Am J Surg Pathol 2009;33:645, Mod Pathol 2021;34:2043)
  • Myxoid liposarcoma-like areas:
    • Poorly to moderately cellular and composed of spindle cells with inconspicuous cytoplasm and round to oval hyperchromatic nuclei, embedded in a myxoid matrix with abundant ramified capillaries
    • Nuclear atypia and pleomorphism are low but usually are more prominent than in classic myxoid liposarcoma
    • Myxoid pools are frequently seen
  • Pleomorphic liposarcoma-like areas:
    • Highly cellular and composed predominantly of pleomorphic cells with irregular hyperchromatic nuclei
    • Variable adipocytic differentiation (frequent lipoblasts)
    • Mitotic activity is generally high and tumor necrosis is frequent
Microscopic (histologic) images

Contributed by Raul Perret, M.D., M.Sc.

Lobulated architecture

Mucin pools

Myxoid liposarcoma-like areas

Lipoblasts


High grade areas

RB1 loss

CD34 expression

Negative stains
Molecular / cytogenetics description
Molecular / cytogenetics images

Contributed by Raul Perret, M.D., M.Sc.

aCGH profile

Sample pathology report
  • Mediastinal mass, excision:
    • Myxoid pleomorphic liposarcoma (see comment)
    • Comment: Microscopic examination shows a biphasic tumor comprising myxoid liposarcoma and pleomorphic liposarcoma-like areas. The myxoid liposarcoma-like areas contain small, poorly proliferative spindle cells with mildly pleomorphic hyperchromatic nuclei and rare univacuolated lipoblasts. The cells are dispersed in a myxoid background rich in ramified capillaries. The pleomorphic liposarcoma-like areas are densely cellular and made of sheets of pleomorphic cells (including lipoblasts) showing high mitotic activity and areas of necrosis.
    • Immunohistochemically, the tumor cells are focally positive with CD34, negative with MDM2 and show loss of RB1 expression. FISH studies show the absence of DDIT3 rearrangement. Altogether, these findings support the above diagnosis of myxoid pleomorphic liposarcoma. This recently described sarcoma shows a high rate of local and distant spread (Am J Surg Pathol 2009;33:645, Mod Pathol 2021;34:2043).
Differential diagnosis
Board review style question #1

A 25 year old man presented with a rapidly evolving mediastinal mass that shows lipomatous areas on MRI. Histology shows a richly vascular myxoid tumor with adipocytic and spindle cell components. Cellular density and nuclear pleomorphism are variable, ranging from low to high. Mitotic activity is brisk, and tumor necrosis is present. On immunohistochemistry, the tumor cells stain for CD34, show RB1 loss and negativity for MDM2. FISH analyses show no rearrangement of DDIT3. Which of the following is most likely the correct diagnosis?

  1. Atypical myxoid liposarcoma
  2. Atypical spindle cell / pleomorphic lipomatous tumor
  3. Dedifferentiated liposarcoma
  4. Myxoid pleomorphic liposarcoma
  5. Pleomorphic liposarcoma
Board review style answer #1
D. Myxoid pleomorphic liposarcoma

Comment Here

Reference: Myxoid pleomorphic liposarcoma
Board review style question #2
Which of the following is true about myxoid pleomorphic liposarcoma?

  1. It is an intermediate malignancy tumor
  2. It predominantly arises in children and young adults
  3. MDM2 amplification is present in a subset of cases
  4. Most cases are associated with Li-Fraumeni syndrome
  5. Most cases show DDIT3 gene rearrangements
Board review style answer #2
B. It predominantly arises in children and young adults

Comment Here

Reference: Myxoid pleomorphic liposarcoma

Myxoinflammatory fibroblastic sarcoma
Definition / general
Essential features
  • Rare, low grade sarcoma of the distal extremities with prominent myxoid stroma, mixed inflammation, virocyte-like cells, pseudolipoblasts and emperipolesis (Am J Surg Pathol 2014;38:1)
  • Immunohistochemistry is relatively nonspecific, although an immunoprofile showing positivity for BCL1, CD10 and factor XIIIa may help strengthen the diagnosis
  • Most common molecular alteration is VGLL3 amplification; a small number of cases demonstrate a t(1;10) translocation or BRAF rearrangements
  • Diagnosis is primarily based on microscopic appearance and clinical characteristics
Terminology
  • Not recommended: inflammatory myxohyaline tumor of the distal extremities with virocyte or Reed-Sternberg-like cells, acral myxoinflammatory fibroblastic sarcoma, inflammatory myxoid tumor of the soft parts with bizarre giant cells
ICD coding
  • ICD-0: 8811/1 - myxoinflammatory fibroblastic sarcoma (MIFS)
  • ICD-10: C49.9 - malignant neoplasm of connective and soft tissue, unspecified
  • ICD-11: 2B5F.2 & XH2D15 - sarcoma, not elsewhere classified of other specified sites and myxoinflammatory fibroblastic sarcoma
Epidemiology
  • Rare lesion; men and women with a wide age range (mean age 40 - 49 years) presenting with a slow growing mass (Am J Surg Pathol 2014;38:1)
Sites
Pathophysiology
  • Some studies have shown morphologic overlap with hemosiderotic fibrolipomatous tumor and pleomorphic hyalinizing angiectatic tumor but the precise classification of these tumors and their relationship is debated
  • Complex karyotypes with multiple reported recurrent genetic alterations
  • Reference: Mod Pathol 2020;33:2520
Etiology
  • Currently classified under tumors of unknown etiology
Clinical features
Diagnosis
  • Clinicopathologic diagnosis: a combination of the appropriate clinical setting along with the characteristic histopathologic findings
  • Immunohistochemistry pattern is not specific, although positivity for factor XIIIa, BCL1 and CD10 may support the diagnosis
  • Demonstration of the VGLL3 amplification in the appropriate clinical setting supports the diagnosis (Am J Surg Pathol 2014;38:1)
Laboratory
  • Unknown at this time
Radiology description
Prognostic factors
Case reports
Treatment
Clinical images

Contributed by Mark R. Wick, M.D.

Subcutaneous soft tissue mass

Gross description
  • Multinodular tan-white lesion with gelatinous areas corresponding to fibrotic and myxoid areas
  • Usually small tumors (~ 3 cm) but may attain larger sizes
  • Located in subcutaneous tissue of dermis; may be deeply infiltrative or involve joints and tendons
  • Reference: Am J Surg Pathol 2014;38:1
Gross images

Images hosted on other servers:
Macroscopic appearance of MIFS

Macroscopic appearance of MIFS

Frozen section description
Microscopic (histologic) description
  • Multinodular lesion composed of variable myxoid and fibrous / hyalinized areas with a dense associated inflammatory infiltrate
  • Infiltrative growth pattern usually confined to subcutaneous tissue; may sometimes show involvement of deeper structures, including skeletal muscle
  • Acute or chronic inflammatory cells, including neutrophils, lymphocytes, plasma cells, histiocytes and eosinophils, that blend through myxoid and fibrous areas
  • Tumor cell population composed of epithelioid to spindled tumor cells
  • Scattered large cells with bizarre nuclei and prominent nucleoli resembling viral inclusions (virocyte-like cells) or Reed-Sternberg cells
  • Multivacuolated cells resembling pleomorphic lipoblasts (pseudolipoblasts - tumor cells with cytoplasmic vacuoles containing myxoid material)
  • Giant tumor cells, predominantly within the myxoid areas, display prominent emperipolesis
  • Low mitotic rate despite increased atypia; may display some areas of necrosis
  • Can show focal areas that resemble hemosiderotic fibrolipomatous tumor (hemosidern laden histiocytic cells) or pleomorphic hyalinizing angiectatic tumor (pleomorphic cells and dilated hyalinized vessels)
  • High grade variant: increased cellularity, increased mitotic activity and necrosis, retains characteristic virocyte / Reed-Sternberg-like cells, pseudolipoblastic cells (Am J Surg Pathol 2013;37:1627)
  • Reference: Mod Pathol 2020;33:2520
Microscopic (histologic) images

Contributed by David Suster, M.D.
Lobulated appearance

Lobulated appearance

Solid appearance

Solid appearance

Myxoid and solid areas

Myxoid and solid areas

Prominent myxoid areas

Prominent myxoid areas

Admixed inflammatory cells

Admixed inflammatory cells

Inflammatory cells

Inflammatory cells


Fibroblastic area

Fibroblastic area

Atypical cells

Atypical cells

Reed-Sternberg-like cells

Reed-Sternberg-like cells

Pseudolipoblastic cells

Pseudolipoblastic cells

Emperipolesis

Emperipolesis

High grade tumor

High grade tumor


Necrosis

Necrosis

BCL1

BCL1

CD10

CD10

Factor XIIIa

Factor XIIIa

Virtual slides

Images hosted on other servers:
MIFS Hand

MIFS hand

MIFS Skin

MIFS skin

High Grade MIFS

High grade MIFS

Cytology description
Cytology images

Contributed by Mark R. Wick, M.D.
Toe, fine needle aspiration

Toe, FNA



Images hosted on other servers:
FNA of left upper back mass

FNA of left upper back mass

Positive stains
Negative stains
Electron microscopy description
  • Fibroblastic features of abundant rough endoplasmic reticulum and mitochondria, intermediate filaments
  • Lipoblast-like cells have cytoplasmic pseudoinclusions with extracellular mucin
  • Reference: Am J Surg Pathol 1998;22:911
Molecular / cytogenetics description
  • Complex and heterogeneous karyotypes described with multiple genetic aberrations, including aneuploidy, ring chromosomes and a t(2;6) (Virchows Arch 2007;451:923, Cancer Genet Cytogenet 2004;152:61, Cancer Genet Cytogenet 2007;177:139)
  • Recurrent t(1;10)(p22;q24), resulting in gene rearrangements of TGFBR3 and MGEA5 has been described; initially thought to occur more commonly, subsequent studies have shown this rearrangement to be incredibly rare (~ 5%) in conventional myxoinflammatory fibroblastic sarcoma (Am J Surg Pathol 2010;34:1723)
  • BRAF rearrangements have been described in a subset of myxoinflammatory fibroblastic sarcoma, although the largest molecular study examining BRAF rearrangement was able to identify BRAF in only ~ 5% of cases studied (Am J Surg Pathol 2014;38:1)
  • Most common recurrent molecular alteration is amplification of VGLL3 on chromosome 3, which occurs in approximately 40% of cases (Am J Surg Pathol 2014;38:1)
Molecular / cytogenetics images

Images hosted on other servers:
t(1;10) FISH analysis in MIFS

t(1;10) FISH analysis in MIFS

BRAF alterations in FISH

BRAF alterations in FISH

<i>TOM1L2</i>-<i>BRAF</i> fusion in RNA sequencing and reverse transcription PCR

TOM1L2-BRAF
fusion in RNA
sequencing and
real time PCR

Videos

Whole slide review

Sample pathology report
  • Foot, right, resection:
    • Myxoinflammatory fibroblastic sarcoma (see comment)
    • Comment: Sections show a multilobulated, infiltrative tumor within the subcutaneous soft tissues. There is a prominent mixed inflammatory cell population scattered throughout the tumor. Fibroblastic and myxoid areas are identified with a spindled to epithelioid neoplastic cell population with scattered prominent atypical cells (Reed-Sternberg-like cells), pseudolipoblastic cells and giant cells displaying emperipolesis. Mitotic activity is low. The tumor cells are positive for CD10, factor XIIIa and BCL1 and negative for SMA, desmin, cytokeratin AE1 / AE3, FLI1, CD45, MDM2, S100 and SOX10. Although nonspecific, the immunohistochemical profile is compatible with the above diagnosis.
Differential diagnosis
  • Hemosiderotic fibrolipomatous tumor:
    • Can show a spectrum of changes that resemble myxoinflammatory fibroblastic sarcoma
    • Can also harbor the t(1;10) translocation, although debated
    • Some authors believe these tumors may be related
  • Pleomorphic hyalinizing angiectatic tumor:
    • Spindled and epithelioid atypical cell population that may resemble myoxinflammatory fibroblastic sarcoma
    • Prominent dilated vessels with peripheral hyalinization
  • Epithelioid sarcoma:
  • Hodgkin lymphoma:
    • Lacks giant cells and lipoblast-like cells
    • Tumor cells have different staining patterns
    • Positive for many hematologic markers, including CD15 and CD20
    • Classical Hodgkin lymphoma shows loss of expression of OCT2 / BOB1
  • Myxofibrosarcoma:
    • Rare in soft tissues of hands and feet
    • More frequent mitotic figures with atypical forms
    • No inflammatory infiltrate
  • Dedifferentiated liposarcoma:
    • Generally occurs in older patients
    • Can occur in the proximal extremities, particularly proximal lower extremities
    • Can show variable features, including atypical cells, myxoid areas, solid areas and some lipoblastic cells that may resemble myxoinflammatory fibroblastic sarcoma
    • Can be separated by identifying MDM2 amplification or residual areas of well differentiated liposarcoma
  • Rosai-Dorfman disease:
    • Emperipolesis but no intranuclear viral-like inclusions
    • S100+
    • Usually not myxoid
  • Inflammatory myofibroblastic tumor:
    • Can show spindled to epithelioid neoplastic cells with prominent admixed inflammatory infiltrates, particularly plasma cells
    • Characterized by ALK rearrangements
Board review style question #1

A 43 year old woman presents with a 4.5 centimeter, slowly enlarging, dorsally located foot mass. Resection of the specimen shows a multilobulated lesion located in the subcutaneous soft tissue with prominent myxoid areas (as shown in the picture above). Microscopic examination shows a dense inflammatory infiltrate with scattered atypical tumor cells with prominent nucleoli that resemble Reed-Sternberg cells. Which of the following is the most likely diagnosis?

  1. High grade osteosarcoma
  2. Hodgkin lymphoma
  3. Inflammatory myofibroblastic tumor
  4. Myxoinflammatory fibroblastic sarcoma
  5. Undifferentiated pleomorphic sarcoma
Board review style answer #1
D. Myxoinflammatory fibroblastic sarcoma. Of the other entities listed, only Hodgkin lymphoma has Reed-Sternberg cells; however, it would not present as a subcutaneous foot mass.

Comment Here

Reference: Myxoinflammatory fibroblastic sarcoma
Board review style question #2
Which of the following is the most common genetic alteration identified in myxoinflammatory fibroblastic sarcoma?

  1. BRAF rearrangements
  2. MDM2 amplification
  3. t(1;10)(p22;q24)
  4. t(2;6)
  5. VGLL3 amplification
Board review style answer #2
E. VGLL3 amplification. VGLL3 is the most common genetic alteration identified; approximately 40% of cases.

Comment Here

Reference: Myxoinflammatory fibroblastic sarcoma

Necrotizing fasciitis
Definition / general
  • Uncommon fulminant soft tissue infection characterized by extensive fascial necrosis
  • Usually due to group A streptococci, also community-acquired MRSA (Surg Infect (Larchmt) 2008;9:469)
  • May be due to Clostridium perfringens and cause gas gangrene (Cases J 2008;1:252)
Clinical features
Case reports
Treatment
Clinical images

Images hosted on other servers:

Gas gangrene

Right shoulder

Left leg

Microscopic (histologic) images

Images hosted on other servers:

Infected tissue
with gas inclusion
between the
muscle fibers

Gram positive rods


Neurofibroma-general
Definition / general
  • Benign peripheral nerve sheath tumor with classic identifiable features including the presence of a neuronal component comprising transformed Schwann cells and a nonneoplastic fibrous component that includes fibroblasts
Essential features
  • Interlacing bundles of elongated cells with wavy darkly stained nuclei
  • Interspersed variably sized collagen bundles
  • Interspersed mast cells
  • Stroma contains variable mucin and collagen
  • CD34 positivity in cells of unclear histogenesis
  • S100 positivity in neural cells
  • Monomorphic comma shaped nuclei
  • Divergent differentiation, including melanin pigmented cells, may occur rarely
Terminology
Epidemiology
Sites
  • Localized neurofibromas are superficial and evenly disturbed over the body surface
  • Diffuse neurofibromas are usually in the head and neck region (Int J Trichology 2010;2:60)
  • Plexiform neurofibromas are localized to a major nerve trunk
Etiology
  • Neurofibromas are caused by a biallelic inactivation of the tumor suppressor gene neurofibromatosis type 1 which is located on 17q11.2
  • Non myelinating p75+ Schwann cell progenitors are the candidate cell for neurofibromatosis type 1 loss in plexiform neurofibroma (Cancer Cell 2008;13:117)
  • Dermal neurofibromas may have a non Schwannian precursor of a neural stem cell / progenitor
Diagrams / tables

Image hosted on other servers:
Missing Image

Common sites

Clinical features
  • Sporadic (localized variant)
    • Occurs in individuals who do not have neurofibromatosis type 1
    • Painless, slowly growing, solitary, skin colored, soft, flaccid, rubbery to firm papule or nodule with a smooth surface measuring up to 2 cm
    • Lesion invaginates with pressure
  • Cutaneous and subcutaneous
  • Inherited (diffuse and plexiform variants)
    • Close associations with neurofibromatosis type 1
  • Symptoms in patients with neurofibromatosis type 1 include chronic pain, disfigurement, stigma and anxiety (Am J Med Genet A 2017;173:79)
  • Rarely, neurofibromatous neuropathy can occur due to endoneurial fibrosis due to an altered relationship between Schwann cells and collagen fibrils (Ultrastruct Pathol 2018;42:312)
  • Diagnostic criteria of neurofibromatosis type 1 are met if 2 or more of the following are present:
    • ≥ 6 café au lait patches > 0.5 cm in prepubertal individuals or > 1.5 cm in postpubertal individuals
    • ≥ 2 neurofibromas of any type or 1 plexiform neurofibroma
    • Axillary or inguinal freckling
    • ≥ 2 Lisch nodules
    • Optic glioma
    • Sphenoid dysplasia or thinning of long bone cortex with or without pseudoarthrosis
    • First degree relative diagnosed with neurofibromatosis type 1
Neurofibromatosis type 1
Definition / general
  • Also called von Recklinghausen disease, NF1
  • Defect in neurofibromin gene at 17q11.2; protein is widely expressed tumor suppressor gene that downregulates p21 ras oncoprotein; highest levels in neural tissue; gene has numerous sites of mutation; variable phenotypic expression
  • 1 per 3,000 individuals, 50% from autosomal dominant inheritance, 50% are new mutations
  • Adrenomedullin (ADM) is serum biomarker of NF1 (Clin Cancer Res 2010;16:5048)

Clinical features
  • Multiple neurofibromas (plexiform, solitary); plexiform are relatively specific
  • ≥ 6 cafe au lait spots over nerve trunks, ≥ 1.5 cm (cafe au lait spot: increase in melanin in epidermal basal layer, may overlie a neurofibroma, smooth delicate margins; solitary café au lait spots are normal)
  • Lisch nodules (pigmented iris hamartomas, 94% by age 6)
  • 2 - 4x increased risk of other tumors (childhood CML, ganglioneuroma, meningioma, pheochromocytoma, rhabdomyosarcoma); 5 - 13% develop MPNST; also acoustic neuroma (schwannoma), astrocytoma, gastric carcinoid, GIST, glomus tumor, lipoma, optic nerve glioma, Wilm tumor (Hum Genomics 2011;5:623)
  • Nontumors: congenital malformations, fibrosing alveolitis, megacolon, skeletal lesions (30% have spinal deformities [kyphoscoliosis], bone cysts)

Clinical images

Images hosted on other servers:

Cafe au lait spots



Differential diagnosis
  • Albright syndrome:
    • Polyostotic fibrous dysplasia of bone, patchy dermal pigmentation, endocrine dysfunction
Neurofibromatosis type 2
  • Also known as NF2, acoustic neurofibromatosis
  • Autosomal dominant, incidence of 1 per 40,000
  • Mutation in merlin gene at 22q12; function unknown but widely distributed and similar to cytoskeletal protein
  • Nonsense mutations usually more severe than missense mutations
  • Signs / symptoms: bilateral acoustic neuromas or multiple meningiomas, spinal cord ependymomas; also schwannosis (ingrowth of Schwann cells into cord), meningioangiomatosis (meningeal cells and blood vessel proliferation into the brain), glial hamartia (microscopic nodular collections of glial cells in cerebral cortex); cafe au lait spots but no Lisch nodules
  • Reference: Acta Otorrinolaringol Esp 2010;61:306
Radiology description
  • On CT, appears as a well defined hypodense mass with minimal or no contrast enhancement
  • On MRI, appears as a T1 hypointense and T2 hyperintense lesion with heterogeneous contrast enhancement
  • On MRI of a superficial neurofibroma, the signal characteristics are usually homogeneous or heterogeneous without targets (AJR Am J Roentgenol 2005;184:962)
  • Hybrid PET / MRI for the whole body of neurofibromatosis type 1 patients may be used for the detection of malignant transformation to malignant peripheral nerve sheath tumors (World J Nucl Med 2018;17:241)
  • Does not reliably differentiate between neurofibroma and schwannoma (Semin Musculoskelet Radiol 2019;23:76)
Radiology images

Images hosted on other servers:

CT mouth tumor

MRI mouth tumor

Ultrasound intrascrotal mass

Prognostic factors
  • Malignant transformation to malignant peripheral nerve sheath tumor in deeply seated neurofibromas in a subset of neurofibromatosis type 1 patients
  • Nuclear atypia (focal or diffuse) are accepted in neurofibromas
  • Low grade malignant peripheral nerve sheath tumor may be diagnosed if there is diffuse nuclear atypia, high cellularity and low level of mitotic activity
  • Nuclear atypia includes nuclear enlargement (nuclear size ≥ 3 times normal Schwann cells) and hyperchromasia
  • p16 can help to differentiate neurofibroma with atypical features from low grade malignant peripheral nerve sheath tumor (Am J Pathol 1999;155:1879)
Case reports
Treatment
  • Sporadic lesions not associated with neurofibromatosis type 1
    • Superficial neurofibromas respond well to marginal excision and deep-seated neurofibromas are treated conservatively
    • Sometimes it is difficult to be separated from the parent nerve, which may require sacrificing of the parent nerve to ensure complete excision
  • Inherited lesions associated with neurofibromatosis type 1
Clinical images

Images hosted on other servers:

Mouth

Skin

Gross description
  • Skin colored with glistening tan-white cut section
  • Localized neurofibroma can appear as a fusiform growth with myxoid and cystic areas (J Neurosci Rural Pract 2016;7:346)
  • Those arising from major nerves show fusiform expansion of the affected nerves and may appear encapsulated
  • Those arising from small nerves are well circumscribed but not encapsulated
  • Deeper tumors may cause tortuous enlargement of peripheral nerves (plexiform neurofibromas) (Acta Neuropathol 2012;123:295, Am J Med Genet 1999;89:23)
Gross images

Contributed by Engy Abdellatif, M.B.B.Ch., M.D., Ph.D.
Missing Image Missing Image

Neurofibromatosis type I



Image hosted on other servers:

Large inguinal mass

Missing Image

Esophageal mass

Microscopic (histologic) description
  • Proliferation of all elements of peripheral nerves
  • Schwann cells with wire-like collagen fibrils (wavy serpentine nuclei and pointed ends), stromal mucosubstances, mast cells, Wagner-Meissner corpuscles, Pacinian corpuscles, axons (highlight with silver or acetylcholinesterase stain, NSE, neurofilament), fibroblasts and collagen
  • Perineurial cells in plexiform types, mitotic figures are rare
  • May be infiltrative, have myxoid areas, contain melanin pigment, have epithelioid morphology
  • Rarely has skeletal differentiation
  • No Verocay bodies, no nuclear palisading, no hyalinized thickening of vessel walls (Acta Neuropathol 2012;123:295, Am J Med Genet 1999;89:23)
  • Subtypes
    • Plexiform: irregularly expanded nerve bundles with nodular appearance, prominent myxoid matrix; associated with neurofibromatosis type 1
    • Diffuse cutaneous: traps adnexa, infiltrates into fat
    • Focal cutaneous
    • Intraneural
Microscopic (histologic) images

Contributed by Engy Abdellatif, M.B.B.Ch., M.D., Ph.D.
Missing Image Missing Image

Hypocellular spindle cells

Missing Image

Mast cells

Missing Image Missing Image

Spindle cells and interspersed collagen bundles


Missing Image

Neurofibromatosis type I skin nodule

Missing Image

S100 IHC

Virtual slides

Images hosted on other servers:
Missing Image

Abdominal skin lesion

Missing Image

Polypoid skin lesion in arm

Missing Image

Nodular skin lesion in shoulder

Cytology description
  • Loosely arranged small groups of spindle cells
  • Cells have scant cytoplasm and oval elongated and regular nuclei
  • Nucleoli are not seen
Cytology images

Contributed by Engy Abdellatif, M.B.B.Ch., M.D., Ph.D.
Missing Image Missing Image

Bland spindle cells

Positive stains
Negative stains
Electron microscopy description
Videos

Different types of neurofibroma

Differential diagnosis
Schwannoma  Neurofibroma   
Epidemiology Age 20 - 50 Age 20 - 40
May occur in younger age 
Etiology Sporadic but may occur
in neurofibromatosis type 2 >
neurofibromatosis type 1
Sporadic, some in neurofibromatosis type 1
Macroscopic Encapsulated Softer, usually lacks capsule
Microscopic Antoni A and Antoni B
Alternating hypercellular
and hypocellular areas
Spindle cells, shredded
carrot collagen, mast cells,
hypocellular, myxoid areas
without hypercellular areas
Plexiform variant Less common More common
Immunohistochemistry S100: almost diffuse
Calretinin: stronger
CD34: scattered
(Korean J Pathol 2011;45:30)
Factor XIIIa: negative / focal 
S100: weaker
Calretinin: focal
CD34: stronger
(Korean J Pathol 2011;45:30)
Factor XIIIa: stronger 
Malignant transformation  Extremely rare Rare but can occur in
2 - 3% of neurofibromatosis type 1 patients


Neurofibroma  Malignant peripheral nerve
sheath tumor 
Microscopic features Smaller nuclear size, minimal hyperchromasia,
wavy nuclei, abundant shredded carrot type
collagen, rare fascicular growth pattern, no
necrosis and rare mitosis 
Larger nuclear size, marked hyperchromasia,
less evident wavy nuclei, rare shredded carrot
type collagen, obvious fascicular growth pattern, 
shows necrosis and conspicuous mitosis
S100 ++/+++ +/++
Collagen type IV ++/+++ +/++
EMA + -
CD34 +++ ++
Neurofilament protein  ++ +/+++
Podoplanin + +
SOX10 +++ +/++
Hyaluronan Lower levels Higher levels (Clin Exp Metastasi 2014;31:715)

This table was modified from (Acta Neuropathol 2012;123:295)

Board review style question #1
    How can you differentiate between desmoplastic melanoma and neurofibroma using IHC?

  1. CD34
  2. EMA
  3. HMB45
  4. MelanA
  5. S100
Board review style answer #1
A. CD34 is rarely positive in desmoplastic melanoma and is often patchy when it is.

Comment Here

Reference: Neurofibroma-general
Board review style question #2
    Which one of the following immunohistochemical stains is positive in neurofibroma?

  1. CD34
  2. CK5 / 6
  3. EMA
  4. HMB45
  5. MelanA
Board review style answer #2

Neuroma
Definition / general
Essential features
  • Benign, circumscribed, often painful lesion developing after trauma or surgical amputation of the limb
  • Composed of disorganized spindle cell proliferation of nerve components
ICD coding
  • ICD-10: T87.30 - neuroma of amputation stump, unspecified extremity
Epidemiology
  • No predilection for a specific age, gender, or geographic location
  • Usually associated with history of trauma, amputation or chronic friction
Sites
Pathophysiology
Etiology
Laboratory
Radiology description
  • Ultrasound: circumscribed, marginated, homogeneously hypoechoic fusiform mass with echogenic strands inside; bulbous end morphology; and in continuity with a normal nerve proximally (Acta Biomed 2020;91:122)
  • MRI: well defined, ovoid lesion with an intermediate signal intensity on T1 weighted images; intermediate - high signal intensity with fascicular pattern on fast spin echo (FSE) T2 weighted images (Acta Biomed 2020;91:122)
Radiology images

Images hosted on other servers:

2 oval, well circumscribed parallel nodules

Sciatic neuroma: T1 sagittal plane, T2 coronal plane

Homogeneous hypoechoic fusiform mass in continuity with nerve

MRI: well defined ovoid subcutaneous mass

Prognostic factors
Case reports
Treatment
Clinical images

Images hosted on other servers:

Circumscribed subepithelial lesion

Bulbous portion of sciatic nerve

Fusiform portion of nerve

Gross description
Gross images

Images hosted on other servers:

Resected hard duodenal wall lesion

Microscopic (histologic) description
Microscopic (histologic) images

Contributed by Nasir Ud Din, M.B.B.S.
Unencapsulated bundles of spindle cells Unencapsulated bundles of spindle cells

Unencapsulated bundles of spindle cells

Variable sized bundles of nerve components Variable sized bundles of nerve components

Variable sized bundles of nerve components


Bundles of spindle cells Bundles of spindle cells

Bundles of spindle cells

Clusters of nerve components Clusters of nerve components

Clusters of nerve components

Cytology description
  • FNAC of traumatic neuroma yields scant aspirate, hypocellular smears showing scattered spindle cells in single or clustered fashion (Patholog Res Int 2014;2014:678628)
Positive stains
Videos

Nerve sheath tumors, etc.
by Dr. Jerad Gardner

Sample pathology report
  • Upper limb, excision:
    • Neuroma (see comment)
    • Microscopy: The sections examined show a circumscribed, unencapsulated lesion composed of variable sized, closely packed nerve bundle along with scar tissue in the background.
    • Comment: It is a benign condition and has been completely excised.
Differential diagnosis
  • Localized interdigital neuritis (Morton neuroma / Morton metatarsalgia):
    • Occurs in female adults
    • Location: interdigital planter nerve between third and fourth toes
    • Histology: markedly distorted nerve with extensive perineurial fibrosis arranged in concentric circles
    • IHC: S100 positive
  • Neurofibroma:
    • In skin, deeply situated medium sized nerve, common in patient with type 1 neurofibromatosis
    • Location: no particular anatomical distribution
    • Histology: haphazard proliferation of Schwann cells, fibroblasts, perineurial cells, few axons and mast cells, shredded collagen fibers and myxoid stroma
    • IHC: S100, SOX10, collagen IV, CD34 positive, EMA positive at periphery
  • Palisaded encapsulated neuroma:
    • Age: middle age
    • Location: small, solitary asymptomatic papule in face skin
    • Histology: dermal encapsulated lesion exhibiting Schwann cells and numerous axons with perineurium
    • IHC: S100, neurofilament, EMA
  • Schwannoma:
    • Age: occurs in all ages with peak incidence in fourth to sixth decade
    • Few sporadic cases associated with type 2 neurofibromatosis
    • Location: flexor surface of extremities, neck, mediastinum, retroperitoneum, spinal roots and cerebellopontine angle
    • Histology: encapsulated lesion with hypocellular and hypercellular areas of spindle cells, Verocay bodies present
    • IHC: S100, H3k27me3
  • Mucosal neuroma:
    • Associated with multiple endocrine neoplasia type 2 (MEN2) caused by RET proto-oncogene, known as mucosal neuroma syndrome
    • Variant of traumatic neuroma with distinctive clinical features; however, not associated with trauma or surgery
    • Syndrome is characterized by presence of medullary carcinoma thyroid, pheochromocytoma, marfanoid features and mucosal neuromas of the tongue, lips, inner cheeks and inner eyelids (J Oral Maxillofac Pathol 2020;24:339)
Board review style question #1

A patient presented with a painful nodule that developed after an amputation of the left first finger. On examination, the nodule is painful. Excision of the lesion is performed which shows the histology above. Which of the following is the most likely diagnosis?

  1. Granular cell tumor
  2. Neurofibroma
  3. Neurothekoma
  4. Traumatic neuroma
  5. Schwannoma
Board review style answer #1
D. Traumatic neuroma

Comment Here

Reference: Neuroma

Nodular fasciitis
Definition / general
  • Neoplasm of fibroblastic / myofibroblastic derivation that is typically benign and self limited
  • Virtually all cases contain fusion genes; MYH9::USP6 is the most common fusion product, although other USP6 partners have been identified (Lab Invest 2011;91:1427, Mod Pathol 2017;30:1577)
Essential features
Terminology
  • Pseudosarcomatous fasciitis (obsolete)
  • Cranial fasciitis
ICD coding
  • ICD-10: M72.4 - pseudosarcomatous fibromatosis
Epidemiology
Sites
Pathophysiology
  • Virtually all cases contain fusions genes; MYH9::USP6 is the most common fusion product, although many other genes may partner with USP6 (Mod Pathol 2017;30:1577)
Etiology
  • Historically tumors were thought to result from trauma, although this is now considered unlikely
  • Tumors appear to be sporadic and of unknown etiology
Clinical features
Diagnosis
  • Tumors are morphologically distinctive and typically amenable to classification on the basis of H&E
  • There is a limited role for immunohistochemistry as tumors show a myofibroblastic immunophenotype
  • USP6 rearrangement can be confirmed by FISH, PCR or next generation sequencing
Radiology description
Prognostic factors
  • Not applicable; nodular fasciitis is considered benign
Case reports
Treatment
  • Simple excision is generally curative
  • Occasionally may spontaneously regress (e.g., following biopsy)
  • Rarely recurs after incomplete excision
Clinical images

Images hosted on other servers:
Digital nerve

Digital nerve

Gross description
Gross images

Images hosted on other servers:
Breast

Breast

Microscopic (histologic) description
  • Variable cellularity
  • Extracellular matrix ranges from myxoid to collagenous
  • Older lesions may be more collagenous
  • Areas of cystic degeneration may be identified
  • Spindle stellate cells with a loose fascicular to storiform pattern (so called tissue culture-like and feathery growth)
  • Bland ovoid nuclei
    • Mitotic activity may be conspicuous; however, atypical figures are not a feature
  • Scattered lymphocytes, histiocytes and osteoclast type giant cells often present
  • Exceptionally rare tumors may have malignant morphologic features, including pleomorphism or atypical mitotic figures; in this context, a relationship with nodular fasciitis can only be made following molecular confirmation of USP5 rearrangement (Virchows Arch 2021;479:623, Int J Surg Pathol 2021;29:642, Virchows Arch 2021;479:1007)
Microscopic (histologic) images

Contributed by Brendan C. Dickson, M.D., M.Sc.
Cellular

Cellular

Storiform

Storiform

Myxoid

Myxoid

Extravasated erythrocytes

Extravasated erythrocytes

Collagenous Collagenous

Collagenous

Electron microscopy description
  • Ultrastructural features typical of fibroblasts; in addition, there are peripheral, longitudinal, myofilaments and hemidesmosome-like structures (Cancer 1976;38:2378)
Molecular / cytogenetics description
  • Majority contain MYH9::USP6 fusion genes (Lab Invest 2011;91:1427)
  • USP6 rearrangement can be established by FISH, PCR or next generation sequencing techniques
Molecular / cytogenetics images

Images hosted on other servers:
<i>USP6</i> FISH

USP6 FISH

Sample pathology report
  • Soft tissue, neck, biopsy:
    • Nodular fasciitis (see comment)
    • Comment: This is a spindle stellate cell neoplasm arranged in short bundles and fascicles. The nuclei are ovoid and monomorphic, with occasional mitotic activity. The stroma is variably myxoid to collagenous, with scattered lymphocytes and extravasated erythrocytes. The cells are positive for smooth muscle actin; they are negative for desmin, S100, CD34 and epithelial membrane antigen.
Differential diagnosis
Board review style question #1
Most cases of nodular fasciitis are characterized by which of the following gene fusions?

  1. ARL17A::USP6
  2. COL1A1::USP6
  3. MYH9::USP6
  4. PPP6R3::USP6
Board review style answer #1
C. MYH9::USP6

Comment Here

Reference: Nodular fasciitis
Board review style question #2

Which of the following is generally true about the entity seen in the image above?

  1. Benign and self limiting
  2. Benign with frequent local recurrence
  3. Malignant and frequently metastasizing
  4. Malignant with frequent local recurrence
Board review style answer #2
A. Benign and self limiting

Comment Here

Reference: Nodular fasciitis

NTRK rearranged mesenchymal tumor (pending)
[Pending]

Nuchal type fibroma
Definition / general
  • Bundles of thick collagen fibers in posterior neck
  • Also called collagenosis nuchae, nuchal fibrocartilaginous pseudotumor, nuchal fibroma
  • Nuchal: nape (posterior) of neck
  • Related entities: Gardner associated fibroma (extra-nuchal location but essentially identical histology)
Epidemiology
  • Rare benign lesion of dermis and subcutis in posterior neck (70%), upper back or other regions
  • More common in men, mean age 40 years (age range 3 - 74 years)
  • Associated with diabetes mellitus in 44% (Cancer 1999;85:156)
Case reports
Treatment
  • Excision, may recur locally but does not metastasize
Gross description
  • Usually 3 cm or less, hard and white, unencapsulated, poorly circumscribed
Microscopic (histologic) description
  • No capsule, hypocellular, thick collagen fibers with delicate elastic fibers (Am J Surg Pathol 1995;19:313, Stanford University)
  • Entrapped adipose tissue and entrapped nerves (may resemble traumatic neuroma)
  • May infiltrate into skeletal muscle and adipose tissue, may have scattered lymphocytes
Microscopic (histologic) images

Contributed by Saima Absar, M.D., M.P.H. (Case #469) and AFIP

Nuchal type fibroma at extranuchal location


CD34

CD99

Strands of acellular collagen mixed with fat

Hypocellular lesion



Images hosted on other servers:

Figures 2 - 7

Positive stains
Negative stains
Differential diagnosis
  • Elastofibroma: prominent abnormal elastic fibers, subscapular location and often bilateral
  • Fibrolipoma: circumscribed, different location
  • Fibromatosis: deep soft tissue, not back of neck, more cellular with broad fascicles of fibroblasts
  • Gardner fibroma: same histologic features but extra-nuchal location (may be related or identical entity)
  • Solitary fibrous tumor: patternless pattern, more cellular, staghorn type vessels
Board review style question #1
Nearly half of patients with nuchal type fibroma will also carry what diagnosis?

A. Diabetes mellitus
B. Hypertension
C. Proteinuria
D. Turcot syndrome
Board review style answer #1
A. Diabetes mellitus

Comment Here

Reference: Nuchal type fibroma

Ossifying fibromyxoid tumor
Definition / general
  • Ossifying fibromyxoid tumor (OFMT) is a distinctive mesenchymal neoplasm of uncertain differentiation, with cords, nests, clusters and sheets of uniform ovoid cells embedded in a variable myxoid, fibromyxoid or hyalinized stroma, often with an incomplete peripheral shell of bone (Am J Surg Pathol 1989;13:817)
  • Has potential for local recurrence and metastasis (especially when showing malignant features) (Am J Surg Pathol 2003;27:421)
Essential features
ICD coding
  • ICD-O: 8842/0 - Ossifying fibromyxoid tumor, NOS
  • ICD-11: 2F7C & XH1DA7 - Neoplasms of uncertain behavior of connective or other soft tissue and ossifying fibromyxoid tumor
Epidemiology
Sites
Pathophysiology
Etiology
  • Unknown
Clinical features
Diagnosis
Radiology description
Radiology images

Contributed by Borislav A. Alexiev, M.D.

MRI

Prognostic factors
Case reports
Treatment
Clinical images

Images hosted on other servers:
Missing Image

Tongue mass

Missing Image

Thigh mass

Gross description
Gross images

Contributed by Borislav A. Alexiev, M.D.

Foot mass

Microscopic (histologic) description
Microscopic (histologic) images

Contributed by Borislav A. Alexiev, M.D.

Peripheral shell of metaplastic bone

Nodular pattern

Fibrous stroma

Fibromyxoid stroma

Mitotic activity

Satellite nodule


Lung metastasis

CD10

Desmin

AE1 / AE3

S100

GFAP

Cytology description
  • Paucicellular to moderately cellular smears (Diagn Cytopathol 2020;48:396)
  • Clusters, cords or small aggregates of round, polygonal or spindle cells in myxoid background with osteoid-like or calcified material (Diagn Cytopathol 2004;30:41, J Cytol 2012;29:205)
  • Round to oval nuclei with fine granular chromatin and inconspicuous nucleoli
  • Scant cytoplasm
  • Malignant cases may show significant nuclear pleomorphism with coarse chromatin, irregular contours and 1 - 2 distinct nucleoli (Acta Cytol 2001;45:745)
Cytology images

Contributed by Borislav A. Alexiev, M.D.

FNA

Electron microscopy description
Molecular / cytogenetics description
Sample pathology report
  • Soft tissue, excision:
    • Ossifying fibromyxoid tumor (OFMT), atypical subtype (see comment)
    • Comment: The neoplasm is well circumscribed, multinodular, with thick fibrous capsule and incomplete peripheral shell of woven and lamellar bone. Invasion through the capsule and satellite nodules in the surrounding fat are noted. The neoplasm is composed of uniform, round to spindle shaped cells with bland round to oval nuclei with fine chromatin and indistinct nucleoli and scant, pale eosinophilic cytoplasm. Neoplastic cells are arranged in cords, nests and sheets in fibromyxoid stroma. The neoplasm demonstrates moderate cellularity, low nuclear grade and increased mitotic activity (9 mitoses/10 high power fields). The neoplastic cells are positive for CD10, desmin and keratin AE1 / AE3 (focal) and negative for S100, SOX10, MUC4 and CD34. The diagnosis of OFMT is supported by a fusion between MEAF6 and PHF1 gene. OFMT has potential for local recurrence and metastasis.
Differential diagnosis
Board review style question #1

The most common gene rearranged in ossifying fibromyxoid tumor is

  1. EWSR1
  2. FUS
  3. PHF1
  4. ZNF444
  5. FOS
Board review style answer #1
Board review style question #2

A 53 year old man presented with a right foot mass. Hematoxylin eosin stains demonstrate a well circumscribed tumor with thick fibrous capsule and incomplete peripheral shell of metaplastic bone. Invasion through the capsule and extracapsular nodules in the surrounding fat are noted. The tumor is composed of lobules of uniform, round to spindle shaped cells with bland round to oval nuclei with inconspicuous nucleoli and eosinophilic or pale cytoplasm in abundant fibromyxoid background. Occasional mitotic figures are identified (2 mitoses/10 high power fields). Immunohistochemical stains for S100, CD10, desmin and keratin AE1 / AE3 (focal) are positive in tumor cells while all of the following are negative: CD34, SOX10, ERG, GFAP, h-caldesmon and MUC4.

Which of the following is most likely the correct diagnosis?

  1. Extraskeletal myxoid chondrosarcoma
  2. Ossifying fibromyxoid tumor
  3. Epithelioid schwannoma
  4. Low grade fibromyxoid sarcoma
  5. Myoepithelioma
Board review style answer #2
B. Ossifying fibromyxoid tumor

Comment Here

Reference: Ossifying fibromyxoid tumor

Papillary endothelial hyperplasia
Definition / general
  • Benign, reactive proliferation of endothelial cells (nonneoplastic)
  • First described in 1923 by French physician Pierre Masson, who described an atypical papillary endothelial proliferation in an ulcerated hemorrhoid, which was originally considered to be a vascular neoplasm, in a 68 year old man
  • Recognized as a reactive process in 1932 by Henschen
  • Term intravascular papillary endothelial hyperplasia was first used by Clearkin and Enzinger in 1976 (Arch Pathol Lab Med 1976;100:441)
  • 3 types: primary (pure form) arising in a dilated vessel, secondary (mixed form) arising in a pre-existing vascular lesion and rare extravascular form typically arising in hematoma (Am J Dermatopathol 1983;5:539)
Essential features
  • Benign intravascular proliferation of reactive endothelial cells, forming numerous papillae most often lined by a single layer of plump endothelial cells
  • Associated thrombotic material is frequently present
  • Small, well circumscribed lesion without necrosis, marked cytologic atypia or infiltrative growth pattern
Terminology
  • Also known as Masson tumor
  • Intravascular papillary endothelial hyperplasia (IPEH)
Epidemiology
Sites
  • Intravascular location is most common
  • Can occur in any blood vessel
  • Most commonly in subcutis of fingers (Am J Dermatopathol 1983;5:539)
  • Frequently in subcutis of head, neck and upper extremities
  • Does not tend to involve the overlying skin
Pathophysiology
  • Pathogenesis still unknown
  • Favored to be an exuberant form of organizing thrombus rather than a primary tumor, arising in the context of venous stasis
  • Development takes place in several stages, including endothelial cell growth within a thrombus, production of collagenases by the endothelial cells, partial digestion of the thrombus and the formation of papillary structures (Cir Esp 2017;95:235)
  • Hypoxia inducible factor 1 alpha (HIF1α) and vascular endothelial growth factor (VEGF), proteins associated with thrombus remodeling and angiogenesis, are highly expressed in cellular cores (Int J Clin Exp Pathol 2013;6:2912)
  • Elevation of basic fibroblast growth factor (bFGF), which is released by macrophages due to traumatic vascular stasis, stimulates endothelial cell proliferation (J Hand Surg Am 1994;19:559)
Etiology
  • May represent an alteration in the process of thrombosis
  • Thrombus hypothesis: thought to result from hyperplastic endothelial proliferation caused by development and recanalization of a thrombus within a blood vessel
  • Slight female predominance may have hormonal influence (Case Rep Dent 2014;2014:934593)
  • Preceding trauma has been reported in occasional patients (Am J Dermatopathol 1983;5:539)
Clinical features
  • Slow growing mass lesion or swelling
  • Firm, subcutaneous nodule or papule
  • Ovoid, rounded or lobulated shape
  • Superficial location
  • Bluish or reddish discoloration of the overlying skin
  • May be tender or painless (Dermatol Online J 2016;22:13030)
  • Often incidental finding within thrombosed dilated blood vessels or vascular tumors
Diagnosis
  • Nonaggressive features on imaging studies
  • Final diagnosis on histologic examination of excision specimen
  • Assess for pre-existing vascular lesion (secondary form) (Arch Pathol Lab Med 1993;117:259)
Radiology description
  • Ultrasound
    • Well circumscribed mass
    • Lobulated
    • Hypoechoic
  • MRI
Radiology images

Images hosted on other servers:

Ultrasound

CT scan

Prognostic factors
  • Benign clinical course
  • Secondary form follows course of pre-existing vascular lesion
  • No patient with pure form had a recurrence (Am J Dermatopathol 1983;5:539)
Case reports
Treatment
  • Treated by simple local excision
Clinical images

Contributed by Mark R. Wick, M.D.
Tongue lesion

Tongue lesion



Images hosted on other servers:

Oral mucosa

Gross description
  • Well circumscribed lesion
  • Purplish red multicystic mass
  • Associated hemorrhage and clotted blood
  • May be surrounded by a fibrous pseudocapsule
  • Small lesion (usually < 5 cm) (Dermatol Online J 2016;22:13030)
Gross images

Images hosted on other servers:

Submandibular mass

Small bowel

Microscopic (histologic) description
  • Papillary structures within vascular spaces (Case Rep Pathol 2020;2020:4348629)
  • Numerous papillae with cores of fibrin or fibrous connective tissue
  • Papillae lined by a single layer of plump endothelial cells
  • Piling up of endothelial cells is unusual
  • Slight nuclear atypia and rare mitotic figures may be seen
  • Associated thrombi of different sizes and stages of organization often present
  • Absence of tissue necrosis
  • No atypical mitotic figures
  • No extensive high grade cytologic atypia
  • Mixed form arises in vessels with abnormalities such as arteriovenous malformations, hemangiomas and pyogenic granulomas (Cir Esp 2017;95:235)
  • Extravascular form associated with trauma related hematomas
Microscopic (histologic) images

Contributed by Laura Warmke, M.D.
Cavernous hemangioma

Cavernous hemangioma

Organized thrombus

Organized thrombus

PEH involving subcutaneous vessels

Subcutaneous vessels

Dilated vessel

Dilated vessel

Top left vessel

IPEH involving vessel


Calcification

Calcification

Papillary structures

Papillary structures

Numerous papillae

Numerous papillae

Reactive endothelial hyperplasia

Reactive endothelial hyperplasia

Papillary endothelial hyperplasia

Numerous papillary structures


CD31 CD31

CD31

CD34

CD34

Virtual slides

Images hosted on other servers:

Hemangioma with IPEH

Subcutaneous abdominal wall lesion

Cytology description
  • Cytologic features are varied and not diagnostic (Acta Cytol 2012;56:199)
  • Hemorrhagic and fibrinous background
  • Scattered spindle cells with capillary network
  • Hyaline cores surrounded by lesional cells (Acta Cytol 1990;34:175)
  • No necrosis or marked cytologic atypia
  • Rare mitotic figures may be present but no atypical forms
Electron microscopy description
  • Papillae lined by nonstratified layer of plump endothelial cells
  • Fibroblasts and pericytes in underlying stroma
  • Ultrastructure and organization closely resembles granulation tissue (Cancer 1978;42:2304)
Electron microscopy images

Images hosted on other servers:

Plump endothelium

Videos

Masson "tumor"

Sample pathology report
  • Skin and subcutaneous tissue, left index finger, mass, excision:
    • Intravascular papillary endothelial hyperplasia (see comment)
    • Comment: Sections show a dilated vessel centered in the superficial subcutaneous tissue with an associated intravascular proliferation of reactive endothelial cells, forming numerous papillary structures lined by a single layer of plump endothelial cells. The lesion is well circumscribed with a fibrous pseudocapsule and completely excised. No necrosis, no areas of infiltrative growth, no mitotic figures and no marked cytologic atypia is identified. Immunohistochemical stains for CD31 and ERG highlight the reactive endothelial cells, while CK cocktail is negative. These results support the diagnosis above. All control slides stained appropriately.
Differential diagnosis
  • Pyogenic granuloma (lobular capillary hemangioma):
    • Often arises in oral mucosa
    • May be associated with pregnancy
    • Proliferation of granulation tissue
    • Inflammatory infiltrate, often composed of neutrophils, is present
  • Arteriovenous malformation:
    • Admixture of different vessels, including capillaries, veins and arteries
    • Usually forms a larger mass involving deeper soft tissue
    • Often have associated benign adipose tissue
    • Extends beyond the confines of a single vessel (not intravascular)
  • Thrombus:
    • Composed of layers of platelets and fibrin
    • Alternating layers of red blood cells can form lines of Zahn
    • No papillary structures within vascular space
  • Angiosarcoma:
    • Solid areas of tumor growth
    • Tumor necrosis common
    • Stacking or piling up of endothelial cells
    • Marked cellular pleomorphism
    • Infiltrative growth pattern
    • Irregular, anastomosing vascular channels
    • Mitotic figures common, including atypical forms
Board review style question #1

What is another name for intravascular papillary endothelial hyperplasia?

  1. Arteriovenous malformation
  2. Hemangioma
  3. Masson tumor
  4. Organizing thrombus
Board review style answer #1
C. Masson tumor. Masson tumor is another name for intravascular papillary endothelial hyperplasia, which is currently favored to represent a reactive process as opposed to a true neoplasm. Hemangioma and arteriovenous malformation represent a true neoplasm or malformation respectively. Most cases of Masson tumor are associated with an organizing thrombus.

Comment Here

Reference: Papillary endothelial hyperplasia
Board review style question #2

What is the most common anatomic location for papillary endothelial hyperplasia to arise?

  1. Bone
  2. Deep soft tissue
  3. Heart
  4. Subcutaneous tissue of fingers
Board review style answer #2
D. Subcutaneous tissue of fingers. The most common anatomic location for papillary endothelial hyperplasia is the subcutaneous tissue of the fingers. The heart, deep soft tissue and bone are all unusual locations for this entity.

Comment Here

Reference: Papillary endothelial hyperplasia

Papillary intralymphatic angioendothelioma
Definition / general
  • Also known as Dabska's tumor
  • Very rare tumor of children in skin or soft tissue
  • Good prognosis, with only rare nodal metastases
Case reports
Gross images

Images hosted on other servers:

Testicular tumor

Microscopic (histologic) description
  • Papillary tufts lined by plump endothelial cells (epithelioid- or histiocytic-like) within dilated vascular lumina
  • May have glomeruloid appearance
Microscopic (histologic) images

Contributed by Mark R. Wick, M.D.

Dabska tumor



Images hosted on other servers:

Testicular tumor

VEGFR3+

Positive stains

Perineurioma
Definition / general
  • Perineuriomas are soft tissue neoplasms almost exclusively composed of cells resembling normal perineurium
  • 2 main types of perineurioma are recognized: intraneural perineuriomas and soft tissue perineuriomas, including cutaneous and sclerosing perineuriomas
Essential features
  • Perineurioma are soft tissue neoplasms almost exclusively composed of cells resembling normal perineurium
  • 2 main types of perineurioma are recognized: intraneural perineuriomas and soft tissue perineuriomas, including cutaneous and sclerosing perineuriomas
  • Perineurial cells have typical slender, fibroblast-like appearance with long, delicate cytoplasmic processes
  • Perineuriomas express antigens that are identical to normal perineurium (EMA, GLUT1, claudin1)
Terminology
  • Localized hypertrophic neuropathy: old designation of intraneural perineuriomas (not recommended)
  • Hybrid benign peripheral nerve sheath tumors: rare tumors consisting of 2 intermingled neoplastic cell populations with neural differentiation, including hybrid perineurioma schwannoma and hybrid perineurioma neurofibroma
ICD coding
  • ICD-O: 9571/0 - perineurioma
  • ICD-O: 9571/3 - malignant perineurioma
Epidemiology
Sites
  • Intraneural perineurioma
    • Predilection for upper limbs (radial, ulnar, median nerves) and lower limbs (sciatic nerve)
  • Soft tissue perineurioma
    • Predilection for superficial soft tissues of the extremities and trunk
    • 30% of cases: deep soft tissues / visceral organs
    • Sclerosing perineurioma: predilection for hands (fingers, palms) (Am J Surg Pathol 1997;21:1433)
Pathophysiology
  • Intraneural perineurioma
    • Intraneural clonal (neoplastic) proliferation of perineurial cells
    • TRAF7 mutations (60% of cases) (Ann Neurol 2017;81:316)
  • Soft tissue perineurioma
Clinical features
  • Intraneural perineurioma
    • Muscle weakness, progressive loss of sensory function, muscle atrophy if untreated
  • Soft tissue perineurioma
    • Solitary, slowly growing nodule / tumor
    • Painful in a minority of cases
  • Reference: Arch Pathol Lab Med 2007;131:625
Diagnosis
  • Intraneural perineurioma
  • Soft tissue perineurioma
Radiology images

Images hosted on other servers:

Intraneural perineurioma
MRI showing enlarged brachial plexus MRI showing enlarged brachial plexus

MRI showing enlarged brachial plexus

MRI showing enlarged median nerve

MRI showing enlarged median nerve


Soft tissue perineurioma
MRI of the retriperitoneum

MRI of the retriperitoneum

Prognostic factors
  • Intraneural perineurioma
  • Soft tissue perineurioma
    • Typically benign behavior
    • Atypical perineuriomas: overall benign behavior
    • Malignant perineurioma: exceptionally rare, regarded as variant of malignant peripheral nerve sheath tumor
Case reports
Treatment
  • Intraneural perineurioma
    • No standard treatment guidelines
    • Complete resection with nerve grafting (but variable loss of neural function may be inevitable)
  • Soft tissue perineurioma
    • Complete excision (recurrences are uncommon)
Clinical images

Images hosted on other servers:

Intraneural perineurioma
Preoperative photos of enlarged sciatic nerve

Preoperative photos of enlarged sciatic nerve

Preoperative photo of enlarged brachial plexus

Preoperative photo of enlarged brachial plexus


Soft tissue perineurioma
Swelling of the scalp

Swelling of the scalp

Endobronchial nodule

Endobronchial nodule

Gross description
  • Intraneural perineurioma
    • Fusiform expansion of affected nerve, extending several centimeters in length
  • Soft tissue perineurioma
    • Circumscribed white masses (1 - 20 cm in diameter)
  • Reference: Am J Surg Pathol 2005;29:845
Gross images

Images hosted on other servers:

Intraneural perineurioma
Resected specimen from brachial plexus

Resected specimen from brachial plexus


Soft tissue perineurioma
Well circumscribed, whitish, solid mass

Well circumscribed, whitish, solid mass

Solid, whitish, homogeneous mass

Solid, whitish, homogeneous mass

Microscopic (histologic) description
  • Intraneural perineurioma
    • Multiple small "onion bulbs" expanding the affected nerve, consisting of concentric layers of perineurial cells ensheathing a central axon and Schwann cell
  • Soft tissue perineurioma
    • Nonencapsulated, well demarcated spindle cell tumor (Am J Surg Pathol 2005;29:845)
    • Perineurial cells have typical slender, fibroblast-like appearance with long, delicate cytoplasmic processes
    • Variable degree of cellularity, from paucicellular to densely cellular
    • Fibrotic / sclerotic stroma but frequent foci of myxoid degeneration
    • Metaplastic ossification (rare)
    • Storiform, whorled, lamellar or short fascicular patterns may be observed
    • Sclerosing perineurioma: spindled to rounded cells with pale cytoplasm with indistinct cell borders, arranged in cords / trabeculae / chains within a densely sclerotic stroma (Am J Surg Pathol 1997;21:1433)
    • Reticular perineurioma: lace-like arrangement of cells, resulting in formation of microscopic cysts (Virchows Arch 2005;447:677)
    • Rare histologic variants:
    • Atypical perineurioma: variable combination of scattered cytological atypia, mildly increased mitotic activity, increased cellularity or infiltration of muscle (Am J Surg Pathol 2005;29:845)
    • Malignant perineurioma (malignant peripheral nerve sheath tumor variant): diffuse cytological atypia, increased mitotic activity (> 13 figures/30 high power fields), necrotic foci (Clin Neuropathol 2012;31:424)
Microscopic (histologic) images

Contributed by Andrea Saggini, M.D. and Mark R. Wick, M.D.
Spindle cell tumor

Spindle cell tumor

Hemangiopericytoma-like pattern

Hemangiopericytoma-
like pattern

Storiform pattern

Storiform pattern

Pseudovascular pattern

Pseudovascular pattern

Fibrous histiocytoma-like pattern

Fibrous histiocytoma-like pattern

Perineurial cells

Perineurial cells


Spindled morphology

Spindled morphology

Sclerosing variant

Sclerosing variant

Reticular variant

Reticular variant

Cytology

Cytology

Vascularization

Vascularization

Myxoid changes

Myxoid changes


EMA

EMA

GLUT1

GLUT1

Claudin1

Claudin1

Cutaneous

EMA

Positive stains
Negative stains
Electron microscopy description
  • Soft tissue perineurioma
    • Slender, nontapered processes containing large numbers of pinocytotic vesicles and partial encasement by basal lamina
Electron microscopy images

Contributed by Mark R. Wick, M.D.

Perineurioma

Molecular / cytogenetics description
Sample pathology report
  • Soft tissue of arm, incisional biopsy:
    • Benign spindle cell tumor with morphological and immunophenotypical features (EMA+, claudin1+, S100-, pancytokeratin-, alpha smooth muscle actin-) consistent with soft tissue perineurioma, extending to all the specimen margins. Despite the apparent lack of cytological atypia, complete excision of the neoplasm is advisable.
Differential diagnosis
Board review style question #1

A 34 year old man presented with a fibroma-like nodule on the left thigh. The tumor was completely excised. Histologic details are shown in the image above. Regarding this entity, which of the following statements is true?

  1. EMA positivity is sufficiently sensitive and specific for the diagnosis
  2. Positivity for claudin1 is often observed and may be helpful for diagnostic purposes
  3. S100 may be diffusely positive due to the neural differentiation of the neoplasm
  4. SOX10 is always positive
Board review style answer #1
B. Positivity for claudin1 is often observed and may be helpful for diagnostic purposes

The picture shows a nonencapsulated, well demarcated tumor consisting of a population of benign spindle cells with slender, fibroblast-like appearance and long, delicate cytoplasmic processes; the degree of cellularity is intermediate and the vascularity is increased at the periphery of the tumor.

EMA positivity is typically observed in the normal perineurium as well as in neoplastic cells with perineurial differentiation but EMA positivity alone is not specific for perineurial differentiation and requires concomitant positivity for at least another perineurial marker among GLUT1 and claudin1. S100 and SOX10 are typically negative in perineuriomas, due to the lack of cells with Schwannian differentiation; an exception is represented by hybrid neural tumors with concomitant perineurial and schwannomatous / neurofibromatous differentiation.

Comment Here

Reference: Perineurioma
Board review style question #2
A 36 year old woman presented with a subcutaneous nodule on the left buttock. The nodule was completely excised and the final histopathologic report rendered a diagnosis of soft tissue perineurioma. Regarding this entity, which of the following statements is true?

  1. Atypical soft tissue perineuriomas exhibit clinically indolent behavior
  2. EMA is a useful immunohistochemical marker in the differential diagnosis with meningioma
  3. FISH plays no role in the diagnostic process of soft tissue perineurioma
  4. Immunohistochemical expression of alpha smooth muscle actin virtually rules out a diagnosis of soft tissue perineurioma
Board review style answer #2
A. Atypical soft tissue perineuriomas exhibit clinically indolent behavior

Atypical soft tissue perineuriomas are defined by presence (in variable combination) of scattered cytological atypia, mildly increased mitotic activity (> 13 figures/30 high power fields), increased cellularity or infiltration of muscle. Despite such partially worrisome histopathologic features, they normally exhibit clinically indolent behavior.

Immunohistochemical expression of alpha smooth muscle actin (patchy or rarely, diffuse) may be observed in up to 20% of soft tissue perineuriomas, while expression of desmin should be never observed. FISH may be employed to rule out the presence of PDGFB rearrangements in cases characterized by strong positivity for CD34 coupled with a storiform pattern suspicious for dermatofibrosarcoma protuberans. EMA is also positive in meningiomas; the differential diagnosis with the latter may be aided by immunohistochemical staining for SSTR2A and PR (which are negative in perneurioma but positive in most meningiomas).

Comment Here

Reference: Perineurioma

Phosphaturic mesenchymal tumor
Definition / general
  • Benign tumor of bone or soft tissue associated with rickets and osteomalacia (Am J Surg Pathol 1989;13:588)
  • Most cases of tumor associated oncogenic osteomalacia are due to phosphaturic mesenchymal tumor, which produces fibroblast growth factor-23, a protein that inhibits renal tubular phosphate reabsorption or dentin matrix protein 1, causing low serum phosphate and resulting oncogenic osteomalacia; also low serum 1,25 dihydroxyvitamin D (Am J Surg Pathol 2004;28:1, Mod Pathol 2004;17:573, Pediatr Dev Pathol 2000;3:61)
Epidemiology
  • Extremely rare, median age 53 years, range 9 - 80 years, slight female predominance
Laboratory
  • Low serum phosphate, renal phosphate wasting, low 1,25-dihydroxy Vitamin D3
Case reports
Treatment
Clinical images

Images hosted on other servers:

Tumor of ethmoid sinus

Gross description
  • 2 - 14 cm, arises in soft tissue and bone
Microscopic (histologic) description
  • Hypocellular tumor of bland spindle cells with small nuclei, indistinct nucleoli
  • Has hemangiopericytoma-like vasculature, osteoclast-like giant cells, distinctive grungy calcified matrix, fat, microcysts, hemorrhage, incomplete rim of membranous ossification, metaplastic bone
  • Infiltrative
  • No / rare mitotic activity, no atypia
  • Malignant cases: rare cases with nuclear atypia, 5+ mitotic figures / 10 HPF, high cellularity, resembles MFH
Microscopic (histologic) images

Contributed by Sharon Bihlmeyer, M.D. (Case #63) and AFIP

Various images

Giant cells



Images hosted on other servers:

Tumor of craniofacial sinuses



Tumor of ethmoid sinus

Mandible tumor

Positive stains

Pleomorphic hyalinizing angiectatic tumor
Definition / general
  • Rare and underrecognized soft tissue tumor of intermediate malignancy (locally aggressive) characterized by the presence of pleomorphic cells, low mitotic activity, ectatic blood vessels with damaged walls and no necrosis
Essential features
  • Soft tissue tumor of intermediate malignancy (locally aggressive), with no metastatic potential
  • Characterized by the presence of pleomorphic cells, low mitotic activity, ectatic blood vessels with damaged walls and no necrosis
  • Most cases are positive for CD34 and lack expression of S100, desmin, pankeratin and STAT6
  • Recommended treatment is complete surgical excision with negative margins
Terminology
  • Initially characterized by Smith et al. as pleomorphic hyalinizing angiectatic tumor of soft parts (Am J Surg Pathol 1996;20:21)
  • Controversial terminology: early pleomorphic hyalinizing angiectatic tumor (also known as hemosiderotic fibrolipomatous tumor) is considered in the spectrum of pleomorphic hyalinizing angiectatic tumor (PHAT) by some experts (Am J Surg Pathol 2004;28:1417)
ICD coding
  • ICD-O: 8802/1 - pleomorphic hyalinizing angiectatic tumor
  • ICD-11: 2F7C & XH2193 - neoplasms of uncertain behavior of connective or other soft tissue and pleomorphic hyalinizing angiectatic tumor
Epidemiology
  • Exact epidemiology is uncertain due to the controversial diagnostic criteria and the lack of contemporary series
  • Most cases seem to affect adults (median 51 years, range 10 - 79 years), with a slight female predominance (Am J Surg Pathol 2004;28:1417)
Sites
  • Most arise in the superficial soft tissues of the distal lower extremities, particularly the leg, ankle and foot (Am J Surg Pathol 2004;28:1417)
  • Other areas of the body rarely affected include upper extremities, trunk and head and neck
  • Visceral or intracavitary locations have not been thoroughly documented
Pathophysiology
  • Currently unknown
Etiology
  • Currently unknown
Clinical features
Diagnosis
  • Imaging features are poorly characterized, diagnosis depends on histologic findings and molecular features (selected cases)
Radiology description
  • Imaging findings are poorly characterized and nonspecific (Oncol Lett 2018;15:4720)
  • MRI:
    • Well circumscribed to infiltrative lesions with heterogeneous T1 and T2 signal intensity
    • Heterogeneous contrast enhancement
  • Ultrasound:
    • Hypoechoic lesions with internal punctate hyperechogenic areas
Radiology images

Images hosted on other servers:
MRI: buttock mass MRI: buttock mass

MRI: buttock mass

Ultrasound: buttock mass

Ultrasound: buttock mass

Prognostic factors
Case reports
Treatment
Clinical images

Images hosted on other servers:
Intraoperative: hand mass

Intraoperative: hand mass

Gross description
  • Well circumscribed to infiltrative masses with tan to maroon cut surfaces (Am J Surg Pathol 1996;20:21)
  • Cystic, gelatinous or hemorrhagic areas can be present
Gross images

Contributed by Raul Perret, M.D., M.Sc.
Breast mass

Breast mass

Microscopic (histologic) description
Microscopic (histologic) images

Contributed by Raul Perret, M.D., M.Sc.
Ectatic and thrombosed vessels

Ectatic and thrombosed vessels

Hemosiderin

Hemosiderin

Damaged vessels and pleomorphic cells

Damaged vessels and pleomorphic cells

Inflammatory infiltrate

Inflammatory infiltrate

Nuclear pleomorphism

Nuclear pleomorphism


Intranuclear pseudoinclusion

Intranuclear pseudoinclusion

Virocyte-like cell

Virocyte-like cell

CD34 expression

CD34 expression

Desmin expression

Desmin expression


Relatively well circumscribed lesion

Relatively well circumscribed lesion

Ectactic blood vessels and pleomorphic cells

Ectactic blood vessels and pleomorphic cells

Pleomorphic cells

Pleomorphic cells

CD34

CD34 expression

Virtual slides

Images hosted on other servers:
Left foot mass Left foot mass

Left foot mass

Cytology description
  • Tumor cells of varying size and shape with pleomorphic nuclei, pseudoinclusions and intracytoplasmic hemosiderin pigment (Diagn Cytopathol 2015;43:407)
  • Cytological findings alone are nonspecific and should not be used for diagnostic confirmation
Positive stains
Negative stains
Electron microscopy description
  • Bizarre nuclei, nuclear cytoplasmic inclusions and no intercellular junctions
  • Abundant cytoplasmic intermediate filaments and granular endoplasmic reticulum network (Ultrastruct Pathol 2006;30:59)
Molecular / cytogenetics description
Videos

PHAT: pleomorphic hyalinizing angiectatic tumor

Sample pathology report
  • Knee mass, excision:
    • Pleomorphic hyalinizing angiectatic tumor (see comment)
    • Comment: Microscopic examination reveals a relatively well circumscribed lesion with jagged borders centered in the hypodermis. The lesion comprises sheets and fascicles of spindle and pleomorphic cells associated with hemosiderin deposits and clusters of variably thrombotic and ectatic blood vessels. The tumor cells have hyperchromatic nuclei, which are irregular in size and shape. Mitotic activity is low and tumor necrosis is absent. Immunohistochemically, the tumor cells are positive with CD34 and negative with S100, pancytokeratin, MDM2 and STAT6.
    • These histological and immunohistochemical findings are in keeping with pleomorphic hyalinizing angiectatic tumor. If incompletely excised, these intermediate malignancy mesenchymal tumors tend to recur locally, while distant spread is not typically seen.
Differential diagnosis
Board review style question #1

A 53 year old woman presented with a knee mass that had been evolving for 2 years. Histology shows a relatively well circumscribed hypodermal mass composed of bizarre looking pleomorphic cells and abundant clusters of ectatic blood vessels with damaged walls. Mitotic activity is very low (< 1/10 high power fields) and necrosis is absent. On immunohistochemistry, the tumor cells stain for CD34, while S100, pancytokeratin, MDM2 and STAT6 are negative. Which of the following is most likely the correct diagnosis?

  1. Atypical dermatofibroma
  2. Dermatofibrosarcoma protuberans
  3. Pleomorphic hyalinizing angiectatic tumor
  4. Solitary fibrous tumor
  5. Undifferentiated pleomorphic sarcoma
Board review style answer #1
C. Pleomorphic hyalinizing angiectatic tumor

Comment Here

Reference: Pleomorphic hyalinizing angiectatic tumor
Board review style question #2
Which of the following is true about pleomorphic hyalinizing angiectatic tumor (PHAT)?

  1. Desmin is usually diffusely and strongly positive
  2. It is an intermediate malignancy tumor (locally recurring)
  3. Mitotic rate is generally high
  4. Most cases show sarcomatous transformation
  5. OGA (MGEA5) or TGFBR3 rearrangements are present in all cases
Board review style answer #2
B. It is an intermediate malignancy tumor (locally recurring)

Comment Here

Reference: Pleomorphic hyalinizing angiectatic tumor

Pleomorphic liposarcoma
Definition / general
  • Pleomorphic, high grade sarcoma with variable numbers of pleomorphic lipoblasts, without areas that resemble atypical lipomatous tumor / well differentiated liposarcoma (or other lines of differentiation) and absence of MDM2 gene alterations by cytogenetic and molecular studies
Essential features
  • Pleomorphic spindle / epithelioid cell sarcoma with variable number of pleomorphic lipoblasts
ICD coding
  • ICD-O: 8854/3 - pleomorphic liposarcoma
  • ICD-11: 2B59.Y & XH25R1 - liposarcoma, other specified primary site & pleomorphic liposarcoma
Epidemiology
Sites
Etiology
  • Unknown
Clinical features
  • Rapidly growing, painless mass
  • Some report pain or other symptoms related to tumor location
Diagnosis
  • Requires histological examination on core needle biopsy or excision
Laboratory
  • No specific laboratory abnormality is known
Radiology description
  • Soft tissue mass with heterogeneous areas of necrosis and hemorrhage, containing less / no fatty tissue (Radiographics 2005;25:1371)
Radiology images

Images hosted on other servers:

MRI of breast mass

CT heterogeneously enhanced mass

Prognostic factors
  • Aggressive, often exhibiting local recurrence and metastatic rates of 30 - 50%
  • 5 year overall survival of ~60%
  • Most common site of metastasis are lungs; however, may occur in various other organs, including pleura, liver and bone
  • Poor prognostic factors: central location, increased tumor depth, greater size and higher mitotic count (Am J Surg Pathol 2002;26:601, Am J Surg Pathol 2004;28:1257)
Case reports
Treatment
Gross description
  • White-yellow, relatively firm, often large (median 8 - 10 cm), multinodular tumors
  • Tumors are usually well demarcated but not encapsulated
  • Often demonstrate myxoid changes with foci of necrosis and hemorrhage
  • Can show foci or scattered areas of cystic degeneration (Mod Pathol 2001;14:179)
Gross images

Contributed by David Suster, M.D.

Well circumscribed tumor mass

Frozen section description
  • Diagnosis typically not made on frozen section; however, generally a diagnosis of high grade sarcoma may be rendered based on degree of cytologic atypia, mitotic activity and whether necrosis is present
Microscopic (histologic) description
  • Varying proportion of pleomorphic lipoblasts in a background of a high grade, usually pleomorphic, undifferentiated sarcoma (Surg Pathol Clin 2019;12:63, Histopathology 2014;64:38)
  • Well circumscribed but nonencapsulated with infiltrative borders
  • May display significant morphologic overlap with myxofibrosarcoma, undifferentiated pleomorphic sarcoma and high grade dedifferentiated liposarcoma
  • Cytologically, tumors are composed of high grade cells with varying numbers of pleomorphic and often bizarre, multinucleated tumor cells
    • These tumor cells will often show subtle to prominent cytoplasmic vacuolization with some forms appearing as classic lipoblasts with scalloped nuclei
    • Signet ring lipoblasts may also be scattered throughout tumor in association with malignant cells
  • Epithelioid morphology is seen in about 25% of cases
  • Tumor necrosis common
  • Median 25 mitotic figures/10 high power fields
  • May have neutrophils within giant cells (emperipolesis-like finding), extra and intracellular eosinophilic hyaline droplets (not specific)
Microscopic (histologic) images

Contributed by David Suster, M.D.

Sheets of atypical cells

Extensive necrosis

Bizarre cell shapes

Pleomorphic lipoblast

Bone metastasis


Pleomorphic lipoblasts

Epithelioid morphology

S100 positivity

Negative MDM2

Virtual slides

Images hosted on other servers:

Epithelioid,
hemangiopericytic
pleomorphic
liposarcoma

High grade pleomorphic liposarcoma

Treated pleomorphic liposarcoma

Cytology description
  • Pleomorphic spindle cells of high grade sarcoma with occasional lipoblasts
Positive stains
Electron microscopy description
  • Abundant coalescing lipid droplets, numerous cytoplasmic organelles
Electron microscopy images

Images hosted on other servers:

Lipid vacuoles

Ultrastructure

Molecular / cytogenetics description
Molecular / cytogenetics images

Contributed by Bei You, Ph.D.

Complex karyotype

Videos

Pleomorphic liposarcoma: 5 minute pathology pearls

Pleomorphic liposarcoma (epithelioid variant)

Sample pathology report
  • Thigh, left mass, wide resection:
    • Pleomorphic liposarcoma (19 x 12 x 13 cm), high grade (see synoptic report)
    • Tumor necrosis is present, ~30%
    • No lymphovascular or perineural invasion seen
    • Margins of resection, no tumor seen
    • Pathologic stage: pT3 N0 (no lymph nodes submitted or found)
Differential diagnosis
  • Dedifferentiated liposarcoma:
    • May show a high grade component with pleomorphic features
    • Defined by amplification of 12q13-15 region (MDM2 / CDK4 genes) and will sometimes show an adjacent well differentiated component
  • Metastatic carcinoma:
    • May grow as sheets of poorly differentiated pleomorphic cells
    • Usually lacks adipocytic differentiation and stains positive with cytokeratins
  • Undifferentiated pleomorphic sarcoma and other high grade pleomorphic sarcomas:
    • May show prominent tumor cell pleomorphism but lacks evidence of adipocytic differentiation / lipoblastic cells
  • Pleomorphic lipoma / spindle cell lipoma:
    • May show scattered pleomorphic cells; however, degree of pleomorphism is significantly less than in pleomorphic liposarcoma
    • No overt features of malignancy, such as necrosis, high mitotic activity
    • Characterized by loss of RB1
  • Pleomorphic rhabdomyosarcoma:
    • High grade sarcoma with prominent pleomorphism that may resemble areas of pleomorphic liposarcoma
    • However, usually shows evidence of rhabdomyoblastic differentiation rather than adipocytic differentiation
    • Positive for myogenic markers, such as MyoD1, myogenin and desmin
Board review style question #1

A 77 year old woman presents with a 19 cm deep soft tissue mass in the left lower extremity. A core needle biopsy shows the high grade sarcoma shown in the image above. Which is the most likely diagnosis?

  1. Dedifferentiated liposarcoma
  2. Pleomorphic liposarcoma
  3. Pleomorphic rhabdomyosarcoma
  4. Undifferentiated pleomorphic sarcoma
  5. Well differentiated liposarcoma
Board review style answer #1
B. Pleomorphic liposarcoma. The image shows a high grade sarcoma with bizarre pleomorphic tumor cells that show evidence of adipocytic differentiation in the form of cytoplasmic vacuolization.

Comment Here

Reference: Pleomorphic liposarcoma
Board review style question #2
What is pleomorphic liposarcoma molecularly characterized by?

  1. Amplification of 12q13-15
  2. Amplification of MDM2 and CDK4
  3. Complex cytogenetic abnormalities without specific, recurrent molecular alterations
  4. Deletions of SMARCB1 gene and loss of INI1 expression
  5. Gene fusion between FUS and DDIT3
Board review style answer #2
C. Complex cytogenetic abnormalities without specific, recurrent molecular alterations. Pleomorphic sarcoma is a high grade sarcoma characterized by complex molecular alterations. The tumors show complex karyotypes with numerous cytogenetic abnormalities. Specific, recurrent alterations have not been identified. The tumors are negative for amplification of MDM2 / CDK4 (12q-13-15 region).

Comment Here

Reference: Pleomorphic liposarcoma

Pleomorphic rhabdomyosarcoma
Definition / general
  • High grade pleomorphic sarcoma composed of bizarre eosinophilic, round and spindle cells with skeletal muscle differentiation
  • Exceedingly rare category of rhabdomyosarcoma (RMS) in adults
  • Highly aggressive tumor associated with poor prognosis (Anticancer Res 2015;35:6213)
Essential features
  • High grade sarcoma composed of atypical cells that display skeletal muscle differentiation, confirmed with immunohistochemistry for myogenin or myoD1
Terminology
  • Pleomorphic rhabdomyosarcoma
ICD coding
  • ICD-O: 8901/3 - pleomorphic rhabdomyosarcoma, NOS
  • ICD-11: 2B55 & XH5SX9 - rhabdomyosarcoma, primary site & pleomorphic rhabdomyosarcoma, NOS
Epidemiology
Sites
Pathophysiology
  • Complex karyotype but no signature genetic alteration
Etiology
  • Unknown
Clinical features
  • Rapidly growing, painful deep mass, most commonly of lower extremity
  • Nodal involvement uncommon (< 10%)
  • Majority of patients (70%) with localized disease at presentation (Anticancer Res 2015;35:6213)
Diagnosis
  • Pleomorphic cells with brightly eosinophilic cytoplasm (Mod Pathol 2001;14:595)
  • Skeletal muscle differentiation as confirmed by desmin, myogenin or MyoD1 (Ann Diagn Pathol 2018;36:50)
  • Thorough histologic sampling and evaluation are necessary for diagnostic confirmation
Radiology description
  • In MRI T1 weighted sequences, the tumor appears iso or hypointense to muscle
  • In T2 weighted and STIR sequences, the tumor is hyperintense with surrounding edema
  • In postcontrast sequences, the mass shows heterogeneous enhancement; necrosis is usually present within
  • Reference: AJR Am J Roentgenol 2007;189:371
Radiology images

Contributed by Erdener Özer, M.D., Ph.D.
MRI

MRI



Images hosted on other servers:
MRI

MRI

Adductor brevis

Adductor brevis

Prognostic factors
  • High propensity for metastatic spread
  • Lungs are the most common site for metastases (77%)
  • Positive margins associated with disease relapse (Anticancer Res 2015;35:6213)
  • Tumors with superficial location (~20%) have a more favorable outcome
Case reports
Treatment
  • Clinical behavior and responsiveness to chemotherapy are similar to adult high grade soft tissue sarcomas
  • Poor prognosis with median survival of 7 months
  • Surgical resection with wide margins is mainstay of treatment
  • Stereotactic body radiation therapy (SBRT) has been used for lung oligometastases (Cancer Treat Res Commun 2021;26:100282)
Gross description
Gross images

Contributed by Debra Zynger, M.D.

Seemingly circumscribed thigh mass

Variable hemorrhage

Variable necrosis



Images hosted on other servers:

Paratesticular tumors

Resected tumor

Microscopic (histologic) description
  • Sheets of large, atypical and frequently multinucleated polygonal, spindled or rhabdoid eosinophilic cells (Am J Surg Pathol 2009;33:1850)
  • Cross striations are seldom detected
Microscopic (histologic) images

Contributed by Burcin Tuna, M.D. and AFIP

Bizarre, eosinophilic multinucleated cells

Rhabdomyoblasts

Fascicular pattern

Storiform pattern


Patternless pattern

Spindle cells

Pleomorphic and polygonal cells

Epithelioid cells with abundant eosinophilic cytoplasm

Desmin

MyoD1

Positive stains
Electron microscopy description
  • Skeletal muscle differentiation with rudimentary sarcomeres containing Z bands or Z band material with thick and thin filaments (Mod Pathol 2001;14:595)
Molecular / cytogenetics description
Sample pathology report
  • Soft tissue, biopsy:
    • Pleomorphic sarcoma with rhabdomyoblastic differentiation (see comment)
    • Comment: These findings may represent a pleomorphic rhabdomyosarcoma, although heterologous differentiation cannot be excluded.
Differential diagnosis
  • Embryonal or alveolar rhabdomyosarcoma with anaplastic features:
    • No evidence of embryonal or alveolar component which may have scattered pleomorphic cells or solid foci of pleomorphic cells (Am J Surg Pathol 1993;17:601)
      • Embryonal RMS has ovoid to spindle cells, typically occurs in the pediatric population but can be in adults, and has a predilection for the head and neck and genitourinary system (Pediatr Blood Cancer 2021;68:e28798)
    • Alveolar RMS tumor cells are arranged in nests separated by fibrovascular septa, which frequently exhibit loss of cellular cohesion in the center, conferring a pattern of irregular alveolar spaces; the presence of either a PAX3::FOXO1 or a PAX7::FOXO1 fusion gene is detected in the majority of alveolar RMS cases
    • Pleomorphic RMS prefers older adults and occurs in the extremities (Am J Surg Pathol 2009;33:1850)
  • Undifferentiated pleomorphic sarcoma:
    • Diagnosis of exclusion
    • No evidence of skeletal muscle differentiation
  • Pleomorphic liposarcoma:
    • No evidence of skeletal muscle differentiation
  • Carcinomas with rhabdomyosarcomatous differentiation, such as malignant mixed Müllerian tumor:
    • Carcinomatous component present
  • Melanoma with rhabdomyosarcomatous dedifferentiation (Am J Surg Pathol 2016;40:181):
    • Presence of BRAF mutation, history of melanoma with current presentation of tumor in a characteristic site of metastasis (lymph node, lung, etc.)
  • Sarcomas with heterologous rhabdomyoblastic differentiation, such as dedifferentiated liposarcoma or MPNST (malignant Triton tumor):
    • Histologic or molecular evidence of another tumor (i.e., MDM2 amplification present for dedifferentiated liposarcoma or a well differentiated liposarcoma component present), loss of H3K27me3 in MPNST or areas of more classic appearing MPNST arising from a nerve
Board review style question #1

Which of the following is true for pleomorphic rhabdomyosarcoma?

  1. A t(2;13)(q36;q14) translocation is found in the majority of alveolar rhabdomyosarcoma
  2. Composed of an embryonal component
  3. May express pankeratin
  4. Tumor cells often display cross striations
  5. Usually are tumors of children
Board review style answer #1
A. A t(2;13)(q36;q14) translocation is found in the majority of alveolar rhabdomyosarcoma

Comment Here

Reference: Pleomorphic rhabdomyosarcoma
Board review style question #2
Which rhabdomyosarcoma type is more predominant in adults in the sixth to seventh decades and is characterized by the presence of pleomorphic cells with a significant amount of bright eosinophilic cytoplasm?

  1. Alveolar rhabdomyosarcoma
  2. Embryonal rhabdomyosarcoma
  3. Pleomorphic rhabdomyosarcoma
  4. Spindle cell / sclerosing rhabdomyosarcoma
Board review style answer #2
C. Pleomorphic rhabdomyosarcoma

Comment Here

Reference: Pleomorphic rhabdomyosarcoma

Plexiform fibrohistiocytic tumor
Definition / general
  • Dermal or subcutaneous, plexiform or multinodular proliferation of fibrohistiocytic cells and osteoclast-like giant cells with chronic inflammatory infiltrate
Terminology
Epidemiology
  • Usually children and young adults, 80% female
Clinical features
Case reports
Treatment
Gross description
  • Multinodular, poorly circumscribed, firm, dermal or subcutaneous, 3 cm or less
Microscopic (histologic) description
  • Deep dermal or subcutaneous tumor with ray like extension into skeletal muscle or adipose tissue
  • Overlying epidermis and dermis are usually normal
  • Plexiform or multinodular proliferation of fibrohistiocytic cells with minimal atypia plus osteoclast-like giant cells and chronic inflammatory infiltrate
  • Nodules or clusters are interconnected in characteristic plexiform arrangement
  • Prominent dilated vessels; more sclerotic than MFH
  • Often hemorrhage and hemosiderin
  • Vascular invasion in 10 - 20%
  • Usually 0 - 2 MF / 10 HPF, no necrosis
  • Subtypes are fibrohistiocytic (histiocyte-like cells and giant cells), fibroblastic (fibroblast-like cells) and mixed
Microscopic (histologic) images

Contributed by David Cohen, M.D. (Case #180), Mark R. Wick, M.D. and AFIP

Subcutaneous proliferation of bland fibrous tissue

Nodules of histiocyte-like cells

Histiocyte-like cells


Various images


Cells with elongated nuclei

Tumor with nodules and fibrous tissue

Resemblance to desmoid fibromatosis

Various images


Various images



Images hosted on other servers:

Various images

12 year old girl with upper arm lesion

Cytology description
  • Plump fibroblastic cells and histiocyte-like cells within a finely granular myxoid background
  • Also osteoclast-like giant cells (Acta Cytol 1999;43:867)
Positive stains
Negative stains
Electron microscopy description
  • Resembles myofibroblasts, fibroblasts and undifferentiated cells
Differential diagnosis

Plexiform neurofibroma
Definition / general
  • Benign, peripheral nerve sheath tumor that arises from Schwann cells; generally surrounds multiple nerve fascicles
  • Demonstrates irregularly thickened and distorted, tortuous configurations
  • Plexiform: complex; in the form of a plexus or network
Essential features
  • Benign tumor arising from nerve sheath cells
  • Most likely congenital and strongly associated with neurofibromatosis 1 (NF1)
  • May be locally aggressive and infiltrate into surrounding tissue causing symptoms
  • Immunohistochemistry shows scattered S100 reactivity and EMA can highlight perineurial cells
  • Carries risk of developing into malignant peripheral nerve sheath tumor (MPNST)
  • Surgical excision can be curative; however, recurrence rate is high
    • Medications such as MEK inhibitors have also shown promising results in NF1 patients
Terminology
  • Plexiform neurofibroma (PN, PNF)
ICD coding
  • ICD-10: D36.10 - benign neoplasm of peripheral nerves and autonomic nervous system, unspecified
Epidemiology
Sites
Pathophysiology
  • Neural crest stem cell (variable NF1) → neuro / glial lineage or Schwann cell lineage (variable NF1) → plexiform neurofibroma (negative NF1)
  • Cell of origin: Schwann cell precursors (Neurooncol Adv 2019;2:i13)
  • Normally, activated Ras (GTP) is dephosphorylated by neurofibromin to inactivated Ras (GDP)
  • Loss of heterozygosity → inactive neurofibromin
  • Inactive neurofibromin → activated Ras stays constitutively on to activate signaling pathways downstream → tumor development (Nat Rev Cancer 2015;15:290)
  • PN formation involves tumor cells and immune cells, such as mast cells and macrophages, to participate in growth through inflammation (Neurooncol Adv 2019;2:i23, J Clin Invest 2018;128:2848)
  • Mast cells are recruited by chemoattractant stem cell factors within tumors (Stem Cell Res 2020;49:102068)
  • Inhibition of JAK / STAT3 pathway was shown to reduce inflammatory cytokine expression and number of macrophages within tumor → reduced tumor growth (Oncogene 2019;38:2876)
Etiology
Clinical features
Diagnosis
  • Typically a clinical diagnosis with biopsy demonstrating classic histologic characteristics
  • MRI or PET / CT may be utilized based on the size and location of the lesion
  • F-FDG PET / CT may be used to discriminate between benign and malignant PN and monitor tumor progression (Medicine (Baltimore) 2018;97:e10648)
Laboratory
Radiology description
Radiology images

Images hosted on other servers:

Neck mass (ultrasound)

Face and neck tumor (CT)

Face and neck tumor (MRI)

Wrist mass (MRI)

Prognostic factors
  • Increased mortality risk in pediatric NF1 populations (J Pediatr 2012;160:461)
  • Recurrence following surgery is common (Neurol Sci 2022;43:1281)
    • Partial resection (< 50% excision): 55 - 68% recurrence
    • Subtotal resection (50 - 90% excision): 29 - 45% recurrence
    • Near total resection (> 90% excision): 40% recurrence
  • Greatest risk of recurrence postsurgery is in children < 10 years old with head and neck PN (Arch Otolaryngol Head Neck Surg 2005;131:712)
  • PNs are 20x more likely to have malignant transformation into MPNST compared to cutaneous neurofibromas (Neurology 2005;65:205)
Case reports
Treatment
Clinical images

Images hosted on other servers:

Periorbital swelling with hypertrichosis

Facial mass

Thigh mass

Hyperpigmentation overlying tumor

Palmar mass

Gastric mass

Gross description
Gross images

Contributed by @JMGardnerMD on Twitter
Plexiform neurofibroma

Plexiform neurofibroma



Images hosted on other servers:

Soft irregular mass

Multiple tubular structures

Myxoid-like white tan tissue

Irregular, encapsulated, nodular mass

Microscopic (histologic) description
  • Multinodular, plexiform or tortuous bundles of bland spindle cells consistent with expanded nerve branches
  • On high power, cytologic characteristics are often analogous to those of conventional neurofibromas
  • Can be relatively hypocellular with myxoid type changes, comprised of Schwann cells, fibroblasts and mast cells
  • Outer area of the proliferation is lined by perineurium
  • Rarely is pigmented due to melanocytes (J Am Acad Dermatol 2007;56:862)
  • Can display nuclear atypia; care should be exercised when viewing lesions with increased cellularity and increased atypia as these lesions can undergo malignant transformation
  • Small cutaneous lesions showing a microscopic plexiform pattern may not necessarily be associated with NF1
Microscopic (histologic) images

Contributed by Brandon Umphress, M.D.

Plexiform, tortuous architecture

Detached fragments of plexiform growth

Cytologic and background features

Adjacent peripheral nerve


Cytologic characteristics

Plexiform growth

Virtual slides

Images hosted on other servers:

Vague plexiform growth

Deep subcutaneous lesion

Dermal and subcutaneous involvement

Positive stains
Electron microscopy description
  • Tumor contains neuronal axons, Schwann cells, fibroblasts
  • Peripheral nerves with mucinous degeneration (Eur J Med Res 2010;15:84)
Molecular / cytogenetics description
  • NF1 loss appears to frequently be the driver of plexiform neurofibroma tumorigenesis in NF1 patients (Oncogene 2017;36:3168)
  • Epidermal growth factor receptor erb-b2 receptor tyrosine kinase 2 (ERBB2) and ERBB3 were significantly upregulated in PN (Anticancer Res 2022;42:2327)
  • MicroRNA-155 has demonstrated a role in PN growth and Schwann cell proliferation; microRNA-155 deletion in vivo significantly decreased tumor number and volume in mouse models (Oncogene 2021;40:951)
  • Deletion of chemokine Cxcl10 receptor Cxcr3 prevents PN formation in mouse models, demonstrating T cell and dendritic cell role; Cxcr3 expression in both of those cells is required to maintain elevated macrophages and the inflammatory microenvironment (JCI Insight 2019;4:e98601)
  • CXCR4 gene expression is increased 3 - 120 fold and CXCL12 gene expression is increased 33 - 512 fold (Childs Nerv Syst 2018;34:877)
  • IL8, GRO1 / CXCL1, IL1B, IL6, TNF and leukemia inhibitory factor are overexpressed (Clin Cancer Res 2004;10:3763)
  • Upregulation of AP1 transcription factor family including FOS, FOSB, JUN and JUNB are implicated in cell transformation, invasive growth, angiogenesis and metastasis (Clin Cancer Res 2004;10:3763)
  • Upregulation of apoptotic genes including TNFRSF10A / TRAILR1 and TNFRSF10B / TRAILR2 (Clin Cancer Res 2004;10:3763)
  • KIS gene is overexpressed in PN and MPNSTs compared to dermal neurofibromas (Brain Res Mol Brain Res 2003;114:55)
Videos

Plexiform neurofibroma

Sample pathology report
  • Left thigh, subcutaneous tissue, excision:
    • Plexiform neurofibroma (see comment)
    • Comment: The findings are those of an expansile proliferation within the subcutaneous fat, which demonstrates tortuous bundles of bland cells with wavy and elongated nuclei. Subtle background features of myxoid change are appreciated. Furthermore, overt evidence of malignancy (atypia, necrosis and mitoses) is not appreciated. In the context of the patient's reported history of neurofibromatosis type 1 (NF1), the histologic findings would be most consistent with those of a plexiform neurofibroma. In this regard, clinicopathologic correlation is recommended.
Differential diagnosis
  • Plexiform schwannoma:
    • S100+ (strong and diffuse)
    • Histologically demonstrates multiple nodules of varying sizes with a very thin capsule and otherwise typical features of schwannoma (hypercellular and hypocellular areas, Verocay bodies)
  • MPNST:
    • Histologically demonstrates hypercellularity, significant atypia, increased mitotic figures and a marbled appearance
      • Loss of nuclear H3K27me3 in high grade and radiation associated tumors
      • Loss of INI1 can be seen in epithelioid MPNSTs
  • Perineurioma:
    • S100-, EMA+, claudin1+, GLUT1+
    • Nonencapsulated, well demarcated spindle cell tumor composed of cells with a slender, fibroblast-like appearance with long, delicate cytoplasmic processes
    • Histologically demonstrates fibrotic stroma, frequent myxoid generation, variable celluarity
  • Dermatofibroma:
    • Exhibits collagen trapping
    • Typically FXIIIa+ and CD34-
  • Plexiform fibrohistiocytic tumor:
    • Can exhibit plexiform growth, however, demonstrates nodular fibrous tissue with associated histiocytes and possibly osteoclast-like giant cells
    • S100-
Board review style question #1

A 16 year old boy presents with a mass on his neck. Physical exam reveals multiple café au lait macules on the trunk and axillary freckling. Biopsy of the mass demonstrates multiple torturous nodules with cylindrical enlargement of nerves, scattered mast cells and variable S100 reactivity. When did the patient likely develop the mass?

  1. Birth
  2. 1 - 5 years of age
  3. 6 - 10 years of age
  4. 11 - 16 years of age
Board review style answer #1
A. Birth. A is the correct answer because the diagnosis is plexiform neurofibroma. Diagnosis is supported with clinical features of NF1 and histological findings. PNs are congenital tumors that may take several years to manifest clinically due to slow growth. B, C and D are not the correct answers because the tumor was more than likely present at birth.

Comment Here

Reference: Plexiform neurofibroma
Board review style question #2
Which of the following stains is positive in plexiform neurofibroma?

  1. Cytokeratin AE1 / AE3
  2. Mucicarmine
  3. SMA
  4. S100
Board review style answer #2
D. S100. Plexiform neurofibromas stain with S100 in scattered cells. Answers A - C are incorrect because PN does not demonstrate cytokeratin AE1 / AE3, mucicarmine or SMA positivity.

Comment Here

Reference: Plexiform neurofibroma

Proliferative fasciitis
Definition / general
  • Proliferative fasciitis is a pseudosarcomatous proliferation of myofibroblastic / fibroblastic cells with large ganglion-like cells involving subcutaneous tissue
  • First formally recognized as a distinct entity from nodular fasciitis in 1975 (Cancer 1975;36:1450)
Essential features
  • Myofibroblastic / fibroblastic proliferation with scattered ganglion-like cells
  • Variable collagenous to myxoid stroma
  • Subcutaneous / fascial involvement
ICD coding
  • ICD-O: 8828/0 - proliferative fasciitis
  • ICD-11: FB51.Y - other specified fibroblastic disorders (index term: fasciitis, NOS)
Epidemiology
Sites
  • Subcutaneous by definition (whereas proliferative myositis is intramuscular)
  • Usually located between the subcutis and underlying muscle, in the region of the superficial fascia (Cancer 1975;36:1450)
  • Majority (74%) originate in the extremities (Cancer 1975;36:1450)
  • Most commonly involves the subcutaneous tissue of upper extremity, particularly the forearm, followed by lower extremity and trunk
  • May also arise in dermis (J Cutan Pathol 2011;38:59)
Pathophysiology
Etiology
Clinical features
  • Small, firm subcutaneous mass that is moveable
  • Often demonstrates rapid growth
  • Usually excised within 2 months from the time it is first noticed
  • Almost always measures < 5 cm (most < 3 cm)
  • Tenderness or pain is present in approximately 67% of patients (Cancer 1975;36:1450)
Diagnosis
  • Small, palpable rapidly growing nodule
  • Subcutaneous / fascial involvement (Cancer 1975;36:1450)
  • Myofibroblastic / fibroblastic proliferation with ganglion-like cells
Radiology description
  • Ultrasound (J Clin Ultrasound 2017;45:445):
    • Ill defined subcutaneous lesion
    • Hyperechoic relative to subcutaneous fat
    • Hypoechoic striae may be present
    • Color Doppler reveals mild vascularity
  • MRI (Skeletal Radiol 2004;33:300):
    • Isointense to muscle on T1 weighted image
    • Hyperintense on T2 weighted image
Prognostic factors
  • Benign clinical course
  • Usually self limited, disappearing within 1 - 12 weeks (Acta Cytol 2002;46:1049)
  • Rarely recurs after conservative local excision
  • Does not metastasize
Case reports
Treatment
  • Spontaneous resolution can occur in 1 - 12 weeks (mean: 4.7 weeks) (Acta Cytol 2002;46:1049)
  • Initially may be managed nonsurgically with follow up (Acta Cytol 2002;46:1049)
  • Adequately treated by simple / conservative local excision
  • No indication for radical surgery
Gross description
  • Solitary, mobile mass in the subcutaneous tissue
  • Poorly demarcated, stellate and firm mass that extends along preexisting fibrous septae or fascial planes (Arch Pathol Lab Med 2008;132:579)
  • Grayish white, yellowish tan or reddish pink firm tissue (Cancer 1975;36:1450)
  • Variable infiltration into surrounding adipose tissue
  • May extend horizontally along fascia and fibrous septa of subcutaneous tissue
  • Rare childhood type is often better circumscribed and less infiltrative
  • Ranges in size from 1.0 - 6.9 cm in diameter, usually < 4 cm (Arch Pathol Lab Med 2008;132:579)
Microscopic (histologic) description
  • Similar morphologic features to nodular fasciitis
  • Plump myofibroblastic / fibroblastic spindle cells
  • Large ganglion-like cells with round nuclei, prominent nucleoli and abundant amphophilic cytoplasm (Cancer 1975;36:1450)
  • Density of ganglion-like cells (modified fibroblasts) varies
  • Mitotic figures are found in both spindle and ganglion-like cells and may be numerous
  • No atypical mitotic figures
  • Loose tissue culture, myxoid to collagenous stroma
  • Older lesions may have abundant hyalinized collagen
  • Extravasated red blood cells and stromal lymphocytes are common
  • Borders of lesion are typically infiltrative or ill defined
  • Often grows along fibrous connective tissue septa
  • Childhood subtype generally has better delineated borders, greater cellularity, less collagen production, increased mitotic activity, less myxoid change and a predominance of ganglion-like cells (Am J Surg Pathol 1992;16:364)
  • Focal necrosis and acute inflammation may be present in pediatric cases (Am J Surg Pathol 1992;16:364)
  • Focal metaplastic bone is rare
Microscopic (histologic) images

Contributed by Jeanne Meis, M.D. and AFIP

Stellate configuration

Evenly distributed chromatin

Proliferative myositis

Prominent hemorrhage

Mixture of cells


Ganglion-like cells

Thin walled vascular channel

Trichrome stain



Contributed by Meggen Walsh, D.O., M.S., P.A. and Dana Richards, M.D.

Childhood variant

Ganglion-like cells

Ganglion-like cells with infiltrate

Lobulated growth pattern

Virtual slides

Images hosted on other servers:

Proliferative fasciitis

Cytology description
  • Spindle cells with long cytoplasmic processes to plumb cells with round to oval nuclei
  • Collagen fragments and large ganglion-like cells with dense cytoplasm and prominent nucleoli (Am J Clin Pathol 2009;132:857)
  • Ganglion-like cells with 1 - 2 large eccentric nuclei
  • Giant ganglion-like cells are at least twice the size of plump fibroblasts (Acta Cytol 2002;46:1049)
  • Nuclei of ganglion-like cells are cytologically benign with thin, smooth nuclear membranes and fine chromatin (Acta Cytol 2002;46:1049)
Cytology images

Images hosted on other servers:

Spindle and plump cells

Positive stains
Electron microscopy description
  • Numerous cytoplasmic thin filaments with occasional dense bodies, abundant rough endoplasmic reticulum and cellular membrane based vesicles (Arch Pathol Lab Med 1981;105:542)
  • Ganglion-like cells are modified myofibroblasts with abundant rough endoplasmic reticulum, thin filaments and intracytoplasmic collagen production
  • Extracellular matrix consists of collagen fibrils (Am J Surg Pathol 1992;16:364)
Electron microscopy images

Contributed by Jeanne Meis, M.D.

Rough endoplasmic reticulum

Intermediate filaments

Collagen

Polygonal cell

Molecular / cytogenetics description
Molecular / cytogenetics images

Images hosted on other servers:

FOS breakapart FISH in proliferative fasciitis

FOS::VIM fusion

Videos

Proliferative fasciitis: 5 minute pathology pearls

Proliferative fasciitis & myositis explained by a soft tissue pathologist

Sample pathology report
  • Soft tissue, right forearm, excision:
    • Proliferative fasciitis (see comment)
    • Comment: This excision specimen shows a small subcutaneous nodule (1.5 cm) composed of proliferating myofibroblasts with scattered large ganglion-like cells. Several mitotic figures are present; however, no atypical mitotic figures and no areas of necrosis are identified. Immunohistochemical stains show that the lesional cells are positive for vimentin and SMA, while they are essentially negative for desmin, keratin and S100 protein. These results support the above diagnosis. All control slides stained appropriately.
Differential diagnosis
  • Nodular fasciitis:
    • More common in younger adults
    • Histologically similar
    • Lacks ganglion-like cells
    • Well circumscribed
    • USP6 gene rearrangement (most commonly MYH9::USP6)
  • Reactive myofibroblastic proliferations:
    • Intra-abdominal adhesions or incarcerated hernias
    • Often have a prominent component of adipose tissue
    • Clinical presentation is different
  • Inflammatory myofibroblastic tumor (IMT):
    • Myofibroblastic nature of tumor cells
    • Usually arises in lungs or abdominal soft tissue of children and young adults
    • Occasional ganglion-like cells are common in conventional IMT (Am J Surg Pathol 2011;35:135)
    • Epithelioid variant has more prominent epithelioid / round cell morphology
    • 50% harbor ALK gene rearrangements
    • Diffuse cytoplasmic or perinuclear (in epithelioid variant) staining pattern with ALK
    • Inflammatory infiltrate composed of plasma cells, lymphocytes or neutrophils (in epithelioid variant)
    • May show strong expression of desmin
  • Ganglioneuroblastoma:
  • Embryonal rhabdomyosarcoma:
    • Usually involves genital region or head / neck of infants and young children
    • Cambium layer often present
    • Malignant cytologic features
    • Positive for desmin, myogenin and MyoD1
    • Cross striations are usually present
    • Rarely occurs in adults
  • Pleomorphic rhabdomyosarcoma:
    • Older adults
    • Usually large, intramuscular tumors
    • Markedly pleomorphic cells
    • Numerous atypical mitotic figures and necrosis
    • Positive for desmin, myogenin and MyoD1
  • Undifferentiated pleomorphic sarcoma:
    • Older adults
    • Usually large, intramuscular tumors
    • Markedly pleomorphic cells
    • More irregular nuclear features and chromatin pattern
    • Numerous atypical mitotic figures and necrosis
  • Fibrosarcoma:
    • Predominantly composed of spindle cells
    • Usually lacks pleomorphic cells
    • Deep, large mass of longer duration
  • Osteosarcoma:
    • True tumor osteoid production, as opposed to stromal hyalinization
    • Epithelioid osteoblasts may resemble ganglion-like cells
    • Osteoblasts positive for SATB2
Board review style question #1

Based on ultrastructural studies, both the spindled and ganglion-like cells in proliferative fasciitis are modified versions of what type of cell or origin?

  1. Adipocyte
  2. Myofibroblast
  3. Neural cell
  4. Smooth muscle cell
Board review style answer #1
B. Myofibroblast

Comment Here

Reference: Proliferative fasciitis
Board review style question #2

Similar to epithelioid hemangioma and osteoid osteoma / osteoblastoma, cases of proliferative fasciitis have been reported to have gene rearrangements involving which of the following genes?

  1. FOS
  2. GNAS
  3. MYH9
  4. USP6
Board review style answer #2
A. FOS. A subset of cases of proliferative fasciitis have been described as having FOS gene rearrangements, similar to epithelioid hemangioma and osteoid osteoma / osteoblastoma. USP6 gene rearrangements, including MYH9::USP6 fusion, have been described in nodular fasciitis, while GNAS gene rearrangements have been described in fibrous dysplasia and intramuscular myxoma.

Comment Here

Reference: Proliferative fasciitis

Proliferative myositis
Definition / general
  • Infiltrative poorly demarcated intramuscular mass resembling nodular fasciitis but with large basophilic cells resembling ganglion cells; histologically almost identical to proliferative fasciitis except located in muscle
Epidemiology
  • Mean age 50 years, rare in children
Sites
  • Muscles of trunk, shoulder, chest or thigh
Pathophysiology
  • Painless mass that grows rapidly in 1 to 6 weeks
Etiology
  • May be related to a reactive process, occasional history of trauma noted
Treatment
  • Conservative surgery is curative, may have spontaneous resolution (Head Neck 2007;29:416), recurrence suggests diagnostic error
Gross description
  • Poorly circumscribed, scar-like induration of muscle, usually 3 - 4 cm, can occur under fascia, decreases the central portion of muscle in a wedge fashion
Microscopic (histologic) description
  • Cellular with plump fibroblasts and myofibroblasts surrounding individual muscle fibers creating a checkerboard pattern (proliferative fibroblasts alternating with atrophic muscle)
  • Also large ganglion-like cells with abundant amphophilic to basophilic cytoplasm, vesicular nuclei and prominent nucleoli
  • Stroma is collagenous or myxoid
  • Variable mitotic figures but no atypical ones
  • Ill defined margins
  • May have metaplastic bone
Microscopic (histologic) images

AFIP images

Characteristic checkerboard pattern

Metaplastic bone

Large ganglion cells


Abundant amphophilic cytoplasm

Spindle cell sarcoma



Images hosted on other servers:

Ganglion cells

Cytology description
  • Loose clusters of uniform fibroblast-like spindle cells and large, ganglion-like cells with eccentric nuclei, prominent nucleoli and abundant cytoplasm (Acta Cytol 1995;39:535)
  • Cytologic diagnosis not recommended due to limited sampling
Negative stains
Electron microscopy description
  • Fibroblasts and myofibroblasts, ganglion-like cells are fibroblasts or myofibroblasts with abundant dilated rough endoplasmic reticulum but no neuronal characteristics (Am J Surg Pathol 1991;15:654)
Differential diagnosis
  • Desmoid fibromatosis:
    • 3 cm or larger, completely replaces muscle
    • Spindle cells organized into broad sweeping fascicles, stroma is collagenous, skeletal muscle at periphery is often entrapped
    • No ganglion type cells
  • Ganglioneuroblastoma:
    • S100+
    • Similar histology but lacks the fibrillary background
  • Nodular fasciitis:
    • Completely obliterates muscle when extends deeper than fascia
    • No or few ganglion type cells
  • Proliferative fasciitis:
    • Almost identical except subcutaneous rather than intramuscular location
  • Rhabdomyosarcoma:
    • Desmin+, myogenin+
    • Ganglion-like cells of proliferative myositis lack cross striations and are more basophilic than rhabdomyoblasts
  • Sarcoma:
    • Large mass, marked atypia, atypical mitoses, necrosis

Pseudomyogenic hemangioendothelioma
Definition / general
  • Rarely metastasizing vascular tumor with histology mimicking a myoid tumor or epithelioid sarcoma
Essential features
  • Locally aggressive but rarely metastasizing vascular tumor occurring most commonly in young adults < 40 years old
  • Histology: fascicles or sheets of plump spindled to epithelioid cells with bright eosinophilic cytoplasm
  • Immunohistochemistry: keratin+, ERG+, FLI1+, CD31+, FOSB+, CAMTA1-, INI1 retained
  • Molecular: t(7;19) SERPINE1-FOSB
Terminology
  • Synonyms: epithelioid sarcoma-like hemangioendothelioma, fibroma-like variant of epithelioid sarcoma
ICD coding
  • ICD-10: D49.2 - neoplasm of unspecified behavior of bone, soft tissue and skin
Epidemiology
Sites
  • Most common location is the lower extremities
    • Lower limbs: 54%
    • Upper limbs: 24%
    • Trunk: 18%
    • Head and neck: 4%
  • Often multifocal (~67% of reported cases)
  • Tissues affected (Am J Surg Pathol 2011;35:190)
    • Dermis: 31%
    • Subcutaneous tissue: 20%
    • Muscle: 34%
    • Bone: 14%
  • Tumors with only bone involvement have similar demographics (BMC Cancer 2019;19:872)
Pathophysiology / etiology
  • Driven by overexpression of FOSB, a protein involved in the AP-1 transcription complex that plays a role in cell growth and proliferation, cellular differentiation and apoptosis
  • SERPINE1 is highly expressed in vascular tissues and acts as the promoter for FOSB expression when fused (Cancer Genet 2011;204:211)
Clinical features
Diagnosis
  • Evaluate soft tissue masses with CT or MRI imaging followed by imaging guided biopsy
  • Diagnosis typically made on biopsy prior to definitive surgical treatment or adjuvant therapy
Radiology description
  • Xray and CT: lytic, lobulated and well defined lesion
  • MRI:
    • Hypo to isointense on T1 weighted imaging
    • Variably hyperintense on T2 and STIR weighted imaging
  • Fluorodeoxyglucose (FDG) and methylene diphosphonate (MDP) avidity variable but may be used to follow for regression (Radiol Case Rep 2019;14:1228, Am J Surg Pathol 2016;40:587)
Radiology images

Contributed by Farres Obeidin, M.D. and Borislav Alexiev, M.D.

Left wrist


Gastrocnemius muscle

T9 vertebrae

Prognostic factors
Case reports
Treatment
  • Optimal treatment still unclear due to rarity
  • Wide excision is recommended as first line ± adjuvant radio or chemotherapy because of the high risk of local recurrence
  • Amputation may be needed in rare progressive cases (Am J Surg Pathol 2011;35:190)
  • Responses have been seen in isolated case reports with gemcitabine / taxane cytotoxic chemotherapy or mTOR inhibitors (everolimus, sirolimus) (Clin Sarcoma Res 2015;5:22, Pediatr Blood Cancer 2018;65:1)
  • Complete response to telatinib in one case report; other VEGFR / PDGFR inhibitors have been proposed as well (Clin Cancer Res 2018;24:2678)
Gross description
  • Ill defined, usually multifocal
  • White-brown cut surface, rarely with necrosis or hemorrhage
  • Most tumors are 1 - 2.5 cm; mean size: 1.9 cm (Am J Surg Pathol 2011;35:190)
Microscopic (histologic) description
  • Architecture (Arch Pathol Lab Med 2019;143:763, Am J Surg Pathol 2011;35:190):
    • Vaguely nodular or plexiform; infiltrates surrounding soft tissues
    • Desmoplastic reaction may be seen
    • Sheets or short fascicles; often with variably prominent inflammatory (usually neutrophilic) infiltrate
    • Sometimes has a myxoid background
    • In cutaneous lesions, epidermal hyperplasia or ulceration may be present
  • Cytologic features:
    • Plump spindle to epithelioid cells with vesicular nuclei, variably prominent nucleoli and abundant deeply eosinophilic cytoplasm
    • May resemble rhabdomyoblasts
    • Usually only mild nuclear atypia
    • Mitotic activity: typically < 5 - 10 mitoses/high power field (average of 2 mitoses/high power field)
    • Necrosis, marked nuclear pleomorphism and intracytoplasmic vacuoles are rare
Microscopic (histologic) images

Contributed by Farres Obeidin, M.D. and Borislav Alexiev, M.D.

Fascicular to plexiform architecture

Spindled to epithelioid cells

Bright eosinophilic cytoplasm

Infiltrative borders


Background inflammatory infiltrate

Rare cases with marked atypia

CD31 positive

CD34 negative


AE1 / AE3 positive

ERG positive

Cytology description
  • Cells range from plump spindled to round, epithelioid
  • Abundant pink cytoplasm, reminiscent of rhabdoid or plasmacytoid cells
Cytology images

Contributed by Farres Obeidin, M.D. and Borislav Alexiev, M.D.

Spindled to round / epithelioid cells with abundant pink cytoplasm

Rare tumors have marked atypia

Positive stains
Electron microscopy description
  • Only very rare cases have been evaluated by electron microscopy; no definitive Weibel-Palade bodies seen
  • Prominent rough endoplasmic reticulum and intermediate filaments
  • Submembranous pinocytic vesicles and scattered desmosome-like junctions (Am J Surg Pathol 2011;35:190)
Molecular / cytogenetics description
  • Recurrent t(7;19)(q22;q13) translocation resulting in the fusion SERPINE1 and FOSB (J Pathol 2014;232:534)
Sample pathology report
  • Right leg, mass, resection:
    • Pseudomyogenic hemangioendothelioma (see comment)
    • Comment: Microscopic sections show a multinodular spindle cell proliferation with prominent brightly eosinophilic cytoplasm. There is mild to moderate cytologic atypia and only rare mitotic activity. Stains for keratin AE1 / AE3, ERG, CD31 and FOSB are positive while CAMTA1 is negative and INI1 is retained, supporting the above diagnosis.
Differential diagnosis
Board review style question #1


A 30 year old man presents with multiple foot nodules. The largest nodule is biopsied and shows the histology pictured above. Stains for keratin and ERG are positive while CAMTA1 and CD34 are negative. INI1 is retained. Which of the following is true about this lesion?

  1. This tumor is aggressive and has high propensity for metastasis
  2. It typically harbors a YAP1-TFE3 translocation
  3. The cells have a myogenic immunophenotype
  4. FOSB immunostain is often positive
Board review style answer #1
D. FOSB immunostain is often positive. This tumor is an example of a pseudomyogenic hemangioendothelioma (PHE). While locally aggressive, these tumors only rarely metastasize. YAP1-TFE3 translocations are seen in a small subset of epithelioid hemangioendothelioma. Pseudomyogenic hemangioendothelioma is typically negative for smooth muscle and skeletal muscle stains. Immunohistochemistry for FOSB is typically positive as these tumors harbor a recurring t(7;19) translocation involving SERPINE1 and FOSB.

Comment Here

Reference: Pseudomyogenic hemangioendothelioma
Board review style question #2
Which of the following translocations may be seen in pseudomyogenic hemangioendothelioma?

  1. t(7;19) SERPINE1-FOSB
  2. t(1;3) WWTR1-CAMTA1
  3. t(7;16) FUS-CREB3L2
  4. t(2;13) PAX3-FOXO1
Board review style answer #2
A. t(7;19) SERPINE1-FOSB. Pseudomyogenic hemangioendothelioma is characterized by a t(7;19) SERPINE1-FOSB fusion in the majority of cases. The t(1;3) WWTR1-CAMTA1 translocation is seen in epithelioid hemangioendothelioma. Low grade fibromyxoid sarcoma harbors the t(7;16) FUS-CREB3L2 fusion and alveolar rhabdomyosarcoma shows the t(2;13) PAX3-FOXO1 fusion.

Comment Here

Reference: Pseudomyogenic hemangioendothelioma

Retiform hemangioendothelioma
Definition / general
  • Vascular neoplasm with locally aggressive behavior and characteristic morphology among other vascular lesions; exhibits a high recurrence rate and rarely has metastatic potential
Essential features
  • Locally aggressive, infiltrating vascular neoplasm composed of elongated arborizing vascular channels lined by endothelial cells with prominent hobnail nuclei with hyperchromasia but no atypia and rare mitotic activity
  • Presents as plaque-like areas of discoloration and high rate of local recurrence after incomplete excision
Terminology
  • Hobnail hemangioendothelioma (also used for papillary intralymphatic angioendothelioma / Dabska tumor)
ICD coding
  • ICD-O: 9136/1 - retiform hemangioendothelioma
  • ICD-11: 2B56.1 & XH64U8 - angiosarcoma of skin & retiform hemangioendothelioma
Epidemiology
Sites
Pathophysiology
  • Not known
Etiology
  • Not known
  • Case reports of lesions arising after low dose radiation and in association with lymphedema (Milroy disease)
Clinical features
  • Plaque-like, red to blue areas of discoloration
  • Size range: 3 - 12 cm
Diagnosis
  • Diagnosis is usually made on morphology alone
  • Immunohistochemical markers help confirm the vascular nature of the lesion
Laboratory
  • Not significant
  • Single case report of increased CA19-9 levels in a patient with splenic retiform hemangioendothelioma and concomitant hepatic amyloidosis (World J Clin Cases 2020;8:1108)
Radiology description
  • CT scan and MRI: solid enhancing mass lesion
  • Digital subtraction angiography (DSA): helps to identify feeding vessels for embolization
Radiology images

Images hosted on other servers:
Mediastinal masses

Mediastinal masses

External auditory canal lesion

External auditory canal lesion

Sphenoid bone origin

Sphenoid bone origin

Splenic lesion

Splenic lesion


Temporal bone lesion

Temporal bone lesion

Pre and postembolization DSA

Pre and postembolization DSA

Lymphography

Lymphography

Prognostic factors
  • Recurrence rates up to 60%
  • Margin involvement predisposes to recurrence
Case reports
Treatment
Clinical images

Images hosted on other servers:
Medial canthus lesion

Medial canthus lesion

Postauricular swelling

Postauricular swelling

Reddish plaques Reddish plaques

Reddish plaques


External auditory canal lesion

External auditory canal lesion

Scalp and postauricular lesions

Scalp and postauricular lesions

Raised nodular lesion on distal aspect of finger

Raised nodular lesion on distal aspect of finger

Gross description
Gross images

Images hosted on other servers:
Spinal lesion

Spinal lesion

External auditory canal lesion

External auditory canal lesion

Microscopic (histologic) description
Microscopic (histologic) images

Contributed by Nasir Ud Din, M.B.B.S. and Mowafak Hamodat, M.B.Ch.B., M.Sc. (Case #107)
Skin lesion

Skin lesion

Arborizing network Arborizing network

Arborizing network

Solid areas

Solid areas

Hobnailing

Hobnailing


Hobnailing

Hobnailing

Ectatic vessels

Ectatic vessels

Lymphoid aggregates

Lymphoid aggregates

Dabska tumor-like areas Dabska tumor-like areas

Dabska tumor-like areas


ERG

ERG

CD31 CD31

CD31

Cytology description
Positive stains
Negative stains
Molecular / cytogenetics description
  • 38% of cases in a single study were shown to harbor YAP1 gene rearrangements, with a subset showing YAP1::MAML2 fusions (Am J Surg Pathol 2020;44:1677)
Molecular / cytogenetics images

Images hosted on other servers:
YAP1 gene rearrangements

YAP1 gene rearrangements

Videos

Hemangioendotheliomas

Pathology of hemangioendotheliomas

Sample pathology report
  • Thigh lesion, shave excisional biopsy:
    • Vascular neoplasm with locally aggressive course, consistent with retiform hemangioendothelioma (see comment)
    • Comment: These tumors are known to recur (recurrence rate up to 60% in the literature), particularly when resection margins are not clear.
Differential diagnosis
  • Hobnail hemangioma:
    • Well circumscribed cutaneous lesion
    • Superficial vessels are more hobnail as compared to deep ones
    • No local recurrence reported
  • Papillary intralymphatic angioendothelioma (PILA / Dabska tumor):
    • Intraluminal papillary formations are diffusely present with perivascular and papillary core hyalinization
    • Similar areas may be seen in some examples of retiform hemangioendothelioma but this entity lacks typical retiform architecture
    • D2-40 positivity in endothelial cells
    • Adjacent lymphangioma-like areas and hemosiderin deposition may be seen
  • Composite hemangioendothelioma:
    • Shows mixture of more than hemangioendothelioma types, including Dabska tumor, retiform hemangioendothelioma and hobnail hemangioendothelioma, etc. in various combinations
    • Whenever possible, thorough sampling should be done to exclude a possibility of composite hemangioendothelioma when any hemangioendothelioma pattern is encountered
  • Kaposi sarcoma:
    • Mostly arises in HIV patients with typical clinical presentation
    • Composed of slit-like vessels with spindle cells, hyaline globules and appreciable mitotic activity
    • Immunohistochemistry for HHV8 is positive
  • Angiosarcoma:
    • Ill defined lesion with infiltrative appearance, mostly in older patients
    • Shows various architectural patterns with well formed vascular spaces, spindle cells with extravasated red blood cells, all showing prominent nuclear atypia and brisk mitotic activity
  • Atypical vascular lesion:
    • Develops in skin a few years after radiation
    • Mostly confined to dermis
    • Interconnected vascular channels lined by flattened to hobnail cells
    • May resemble retiform hemangioendothelioma in small biopsies; clinical history is important in distinguishing the two
Board review style question #1

A 30 year old man presents with a red to purple plaque involving the right thigh. A punch biopsy through the lesion shows light microscopic features as depicted in the photomicrograph above. Which of the following is the most likely diagnosis in this case?

  1. Atypical vascular lesion
  2. Intramuscular angioma
  3. Kaposi sarcoma
  4. Papillary intralymphatic angioendothelioma
  5. Retiform hemangioendothelioma
Board review style answer #1
E. Retiform hemangioendothelioma. The photomicrograph shows a vascular lesion with retiform vessels lined by a single layer of endothelial cells with hobnailing. A lymphoid aggregate is also present. Answers B - D are incorrect because other characteristic essential features of any of the given options, except atypical vascular lesion, are not seen here. Answer A is incorrect because the clinical scenario is not appropriate for a diagnosis of atypical vascular lesion. Therefore, the correct diagnosis is retiform hemangioendothelioma.

Comment Here

Reference: Retiform hemangioendothelioma
Board review style question #2
A 45 year old HIV positive woman presents with a violaceous papule on her neck. Punch biopsy of the lesion shows crushing artifacts. Preserved areas show interconnected rete testis-like vascular spaces lined by a single layer of endothelial cells with hobnailing. Which of the following is the most important immunohistochemical stain to differentiate the diagnosis from other possibilities in the scenario?

  1. CD34
  2. Claudin5
  3. D2-40
  4. HHV8
  5. Ki67
Board review style answer #2
D. HHV8. In the given clinical scenario, the most important differential diagnostic consideration here is Kaposi sarcoma. HHV8 is known to be positive in Kaposi sarcoma but not in retiform hemangioendothelioma, which is the correct diagnosis. Answers A and B are incorrect because CD34 and claudin5 are positive in both retiform hemangioendothelioma and Kaposi sarcoma. Answer C is incorrect because D2-40 can be positive in some cases of both. Answer E is incorrect because Ki67 or MIB1 proliferative index is not a diagnostic marker for either diagnosis.

Comment Here

Reference: Retiform hemangioendothelioma

Rhabdomyoma
Adult type rhabdomyoma
Definition / general

Clinical features
  • Very rare
  • Usually head and neck, particularly oral cavity
  • Median age 60 years, 75% male (Hum Pathol 1993;24:608)
  • May be multifocal (25%)

Case reports

Treatment
  • Excision is curative but may recur if incompletely excised

Clinical images

Contributed by Mark R. Wick, M.D.

Base of tongue, CT



Gross description
  • Median 3 cm, circumscribed, soft, tan-red-brown
  • Nodular or lobulated

Gross images

Contributed by Mark R. Wick, M.D.

Base of tongue



Microscopic (histologic) description
  • Well circumscribed, not encapsulated, sheets of large, well differentiated skeletal muscle cells
  • Cells are round or polygonal with abundant eosinophilic fibrillar or granular cytoplasm with frequent cross striations and intracytoplasmic rod-like inclusions
  • Nuclei are small, round and vesicular, may have prominent nucleoli
  • May have spider cells with vacuolated cytoplasm (cells resemble spider webs)
  • Variable glycogen and lipid
  • No mitotic activity, no atypia

Microscopic (histologic) images

Contributed by Mark R. Wick, M.D. and AFIP

Abundant eosinophilic and granular cytoplasm

Distinct well demarcated lobules of polygonal cells

Large closely packed polygonal cells

Haphazardly arranged crystalline material


Crystalline material resembles rods

Various images



Cytology description
  • Fragments of tumor cells, which are large, polygonal cells
  • Cytoplasm is eosinophilic and finely granular, may resemble granular cell tumor, which is S100+, muscle markers- (Diagn Cytopathol 2009;37:483)
  • Eccentrically placed nuclei
  • Cross striations and inclusions are not conspicuous (Acta Cytol 2010;54:968)

Cytology images

Images hosted on other servers:

Loose aggregates of cells

Bland round to oval nuclei

Spindle cells



Positive stains

Negative stains

Electron microscopy description
  • Myofilaments, Z bands, glycogen granules

Differential diagnosis
Fetal type rhabdomyoma
Definition / general
  • Rare benign tumor of immature skeletal muscle differentiation, usually in the head and neck
  • Retroauricular in ages 0 - 3 years

Epidemiology

Sites
  • Usually head and neck; post auricular region is the most common site

Case reports

Treatment

Gross description
  • Median 3 - 5 cm
  • Solitary, well circumscribed mass of soft tissue or mucosa
  • Gray-white-tan-pink, soft with glistening cut surface

Microscopic (histologic) description
  • Circumscribed but not encapsulated
  • Myxoid:
    • Bundles or fascicles of immature slender skeletal muscle with delicate cytoplasmic cross striations and thin tapering eosinophilic processes, resembling myotubules at week 7 - 12 of gestation
    • Also undifferentiated round / oval or spindled mesenchymal cells
    • Stroma is myxoid or fibromyxoid
    • Skeletal muscle cells mature towards periphery, may have pseudocambium layer of plasma cells and lymphocytes under mucosal epithelium
  • Cellular:
    • Bundles or fascicles of cells in parallel or plexiform patterns
    • Sparse collagenous or myxoid stroma
    • Cells have variable skeletal muscle differentiation ranging from immature cells of myxoid pattern (but in larger numbers) to ganglion cell-like rhabdomyoblasts with prominent nucleoli or strap cells with abundant basophilic or eosinophilic cytoplasm and prominent cross striations
    • Infiltration of skeletal muscle may make margins difficult to determine
    • Variable glycogen containing vacuoles
    • No / rare mitotic figures

Microscopic (histologic) images

Contributed by Mark R. Wick, M.D. and AFIP
Missing Image Missing Image Missing Image

Immature muscle cells and mesenchymal cells

Missing Image Missing Image

Undifferentiated round mesenchymal cells


Missing Image

Bipolar immature skeletal muscle cells

Missing Image

Mucosal tumors

Missing Image

Tumor cells are arranged in fascicles with less stroma

Missing Image

Immature skeletal muscle cells

Missing Image

Ganglion cell-like rhabdomyoblasts


Missing Image

Strap cells

Missing Image Missing Image

Ganglion cell-like rhabdomyoblasts

Missing Image Missing Image

Base of tongue, juvenile


Missing Image

Cellular variant

Missing Image Missing Image Missing Image Missing Image

Various images



Cytology description

Positive stains

Negative stains

Electron microscopy description
  • Hypertrophied Z band material, thick and thin filaments, numerous mitochondria, some with inclusions

Differential diagnosis
Genital type rhabdomyoma
Definition / general
  • Rare benign tumor with skeletal muscle differentiation in vagina, vulva, cervix and rarely urethra, usually seen in middle aged women

Epidemiology
  • Rarely occurs in men in the paratesticular region, epididymis or in the tunica vaginalis of the testis or prostate
  • Mean age is 42 years

Case reports

Treatment
  • Local excision is curative

Gross description
  • Well circumscribed, solitary, up to 3 cm
  • Resembles polyp
  • Covered by smooth mucosa

Microscopic (histologic) description
  • Submucosal, polypoid, well circumscribed, no capsule
  • Haphazard strap-like or round striated muscle fibers in fibrous stroma with dilated vessels
  • Cells have abundant eosinophilic cytoplasm with glycogen, cross striations, longitudinal myofibrils
  • Nucleus is round, vesicular, central and uniform
  • May have binucleated or multinucleated cells
  • No / rare mitotic figures, no cambium layer, no spider cells, no spindle cells or rhabdomyoblasts, no necrosis, no nuclear pleomorphism

Microscopic (histologic) images

AFIP images
Missing Image Missing Image Missing Image

Submucosal proliferation of haphazard skeletal muscle cells



Positive stains

Negative stains

Differential diagnosis

Schwannoma
Definition / general
  • Benign nerve sheath tumor arising from differentiated Schwann cells
Essential features
  • Encapsulated, well circumscribed
  • Most cases have a zonal pattern composed of cellular areas (Antoni A) with nuclear palisading (Verocay bodies) and a hypocellular component (Antoni B)
Terminology
  • Neurilemoma (acceptable)
ICD coding
  • ICD-10: D36.10 - benign neoplasm of peripheral nerves and autonomic nervous system, unspecified
Epidemiology
  • All ages can be affected
  • More common in 30 - 60 years of age
  • M = F
  • 90% are sporadic, 3% with neurofibromatosis type 2, 2% with schwannomatosis, 5% with meningiomatosis with or without neurofibromatosis type 2 (StatPearls: Schwannoma [Accessed 10 November 2021])
Sites
  • More frequently on the limbs with a predilection to the upper limbs, followed by the head and neck area, including the oral cavity, orbit and salivary glands
  • Deeply seated tumors are mainly in the posterior mediastinum and retroperitoneum
  • Other areas include posterior spinal roots, bone, gastrointestinal tract, pancreas, liver, thyroid, adrenal glands and lymph nodes
  • Rare sites include penis and vulva (Urol Ann 2017;9:301, World J Surg Oncol 2015;13:139)
Pathophysiology
  • May occur spontaneously
  • Can occur in familial tumor syndromes, such as neurofibromatosis type 2 (NF2), schwannomatosis or Carney complex (StatPearls: Carney Complex [Accessed 13 August 2018])
  • Loss of function of the tumor suppressor gene merlin (schwannomin)
    • Direct genetic change involving the NF2 gene on chromosome 22 or secondarily to merlin inactivation
    • Can occur in NF2 and spontaneous schwannomas
    • Can cause other neoplasms including meningioma, mesothelioma, glioma multiforme and carcinomas of breast, colon and rectum, kidney (clear cell type), liver, prostate and skin
Clinical features
  • Pain and neurological symptoms are uncommon unless the tumor is large
  • Tumor waxes and wanes in size, which may be related to cystic degeneration (J Lab Physicians 2013;5:60)
Diagnosis
  • Histologic findings in the appropriate clinical and imaging context
Radiology description
  • Well circumscribed masses that displace adjacent structures without direct invasion
  • Cystic and fatty degeneration can be seen
  • Hemorrhage and calcification are less frequent
  • Cystic degeneration or hemorrhage can be seen as areas of heterogeneity in large tumors (Radiographics 2004;24:1477, Radiographics 1999;19:1253)
Prognostic factors
Case reports
Treatment
  • Surgical excision is the treatment of choice
  • Local recurrence is uncommon
  • Delayed facial nerve palsy can occur after surgical removal of vestibular schwannoma (Neurosurgery 2016;78:251)
Clinical images

Contributed by Mark R. Wick, M.D.

Well circumscribed cutaneous mass

Gross description
  • Usually solitary
  • Most lesions are completely encapsulated
  • Nerve of origin may be present at the periphery - does not penetrate substance of tumor
  • Large tumors have an eccentric position in relation to the nerve
  • Cut section is light tan and glistening and may show yellow patches
  • Large, longstanding tumors may be cystic
  • Areas of hemorrhage may be seen (Goldblum: Enzinger and Weiss's Soft Tissue Tumors, 7th Edition, 2020)
Gross images

Contributed by Jose G. Mantilla, M.D., Mark R. Wick, M.D. and Case #334
Lesion associated with intercostal nerve

Lesion associated with intercostal nerve

Well circumscribed, homogeneous lesion

Well circumscribed, homogeneous lesion

Ancient schwannoma, cut section

Plexiform schwannoma

Microscopic (histologic) description
  • Biphasic: compact hypercellular Antoni A areas and myxoid hypocellular Antoni B areas (may be absent in small tumors)
  • Nuclear palisading around fibrillary process (Verocay bodies) is often seen in cellular areas
  • Cells are narrow, elongated and wavy with tapered ends interspersed with collagen fibers
  • Tumor cells have ill defined cytoplasm, dense chromatin
  • May have degenerative nuclear atypia, cystic degeneration and hemorrhage (ancient change)
  • Blood vessels may have thickened hyalinized walls and thrombi
  • Pseudoglandular spaces: uncommon; cystic spaces lined by Schwann cells; may represent degenerative Verocay bodies (Ann Diagn Pathol 2016;20:24, Arch Pathol Lab Med 2005;129:1106)
  • Amianthoid fibers or collagenous spherules: large nodular masses of collagen with radiating edges
  • Mitotic figures are rarely seen
  • No intratumoral axons
  • Tumors in the GI tract typically have a prominent lymphocytic rim
  • Variants:
    • Ancient schwannoma:
      • Features include hyalinization, nuclear atypia, hemosiderin deposition
      • Should not have conspicuous mitotic activity or other features suggestive of malignancy
    • Cellular schwannoma:
      • Composed entirely of Antoni A tissue and devoid of Verocay bodies
      • Most common in the paravertebral region
    • Epithelioid schwannoma:
      • Single or small groups of epithelioid schwann cells with moderate amphophilic cytoplasm and occasional nuclear pseudoinclusions (Am J Surg Pathol 2016;40:704):
      • Myxoid to hyalinized stroma, often with thick walled vessels
    • Microcystic / reticular variant (Am J Surg Pathol 2008;32:1080):
      • Has a preference for the GI tract and rarely in subcutaneous soft tissue
      • Formed of strands of bland spindle cells in a myxoid background, with various proportions of microcystic structures
      • No alternating Antoni A, Antoni B areas or Verocay bodies
    • Neuroblastoma-like variant:
    • Plexiform schwannoma:
      • Rare
      • May involve multiple nerve fascicles, making complete resection challenging
      • Typically associated with NF2 and schwannomatosis
Microscopic (histologic) images

Contributed by Dia Eldin Kamel, M.D., Ph.D. and Case #50

Schwannoma from right lateral border of tongue

S100+

Desmin-

SMA-

Left gluteus maximus schwannoma


Ancient changes

S100+

Schwannoma from the dorsum of the left hand

Plexiform schwannoma from the periosteum of the left tibia



Plexiform schwannoma:

Plexiform architecture

Nuclear palisading

Cellular areas

S100



Images hosted on other servers:

Microcystic / reticular schwannoma:

Plexiform growth pattern

Abundant eosinophilic cytoplasm

S100 positive

Virtual slides

Images hosted on other servers:

Schwannoma:

13 year old girl with cystic index finger lesion

45 year old man with thigh lump: H&E, S100

53 year old man with chest wall lump

77 year old woman with small tumor in stomach wall


Ancient schwannoma:

55 year old man with retroperitoneal tumor

Cytology description
  • Aggregates of spindled cells with indistinct cytoplasm and elongated nuclei with blunt pointed ends (World J Gastroenterol 2011;17:3459)
  • Ancient changes can show nuclear pleomorphism and occasionally nuclear inclusions
Cytology images

Case #59

Fine needle aspirate biopsy

Cell block



Images hosted on other servers:

Bland spindle cells

Microcystic structures

Positive stains
Negative stains
Electron microscopy description
  • Basal lamina consisting of electron dense material coats the surface of Schwann cells (Cancer 1981;48:1381, Acta Cytol 1983;27:441)
  • Elongated cells with continuous basal lamina, thin cytoplasmic processes, aggregates of intracytoplasmic microfibrils, peculiar intracytoplasmic lamellar bodies, extracellular long spacing collagen
  • Basal lamina is fragmented in Antoni B areas, suggesting that these areas are degenerated Antoni A areas
  • Contains lipid
  • Has characteristic Luse bodies (collagen fibers with abnormally long spacing exceeding 100 nm between electron dense bands)
Electron microscopy images

Images hosted on other servers:

Cellular schwannoma

Molecular / cytogenetics description
  • May occur spontaneously
  • Can occur in familial tumor syndromes such as neurofibromatosis type 2 (NF2), schwannomatosis or Carney complex
  • Can be caused by loss of function of the tumor suppressor gene, merlin (schwannomin)
  • Merlin acts as a tumor suppressor gene
    • Overexpression can hinder cell proliferation and the changes induced by oncogenes
    • Its downregulation leads to neoplastic transformation
  • Mutations affecting SMARCB1 have a role in the pathogenesis of a small subset of spinal schwannomas and biallelic inactivation of SMARCB1 may cooperate with deficiency of NF2 function (J Neurooncol 2018;137:33)
Sample pathology report
  • Chest wall mass, excision:
    • Schwannoma (see comment)
    • Tumor size: 3.5 cm in greatest dimension
    • Comment: Histologic sections contain an encapsulated neoplasm composed of cytologically bland spindle cells arranged in short fascicles, containing more densely cellular areas with nuclear palisading (Antoni A), alternating with paucicellular areas (Antoni B). There are focal areas of hemorrhage and myxoid degeneration. No histologic features of malignancy are identified.
Differential diagnosis
Board review style question #1
Which of the following immunohistochemical stains best differentiates schwannoma from neurofibroma?

  1. Calretinin
  2. CK5/6
  3. MNF116
  4. p63
  5. S100
Board review style answer #1
E. S100 shows strong and almost diffuse positivity with schwannoma in contrast to neurofibroma, which variably expresses the antigen due to the presence of other cell populations.

Comment Here

Reference: Schwannoma
Board review style question #2

Which of the following is true regarding the tumor represented in the picture?

  1. Its cytologic atypia is degenerative in nature
  2. The degree of cytologic atypia suggest malignant transformation
  3. Melanin pigment may be seen in some of these lesions
  4. These tumors are usually poorly circumscribed and difficult to excise
  5. Local recurrence and malignant transformation are common
Board review style answer #2
A. The picture represents schwannoma with degenerative changes (ancient schwannoma). Tumors with extensive degenerative changes may have extensive hyalinization with loss of their characteristic architectural features, as well as conspicuous degenerative cytologic atypia. Malignant transformation of schwannoma is exceedingly rare and most commonly reported in the setting of stereotactic radiation. Malignant melanotic nerve sheath tumor (formerly called melanotic schwannoma) is a rare tumor with metastatic potential, which often arises in the paraspinal region. Given its different biological behavior, it should not be labeled as schwannoma. Most schwannomas are encapsulated and easily separated from their nerve of origin. Local recurrence is rare after excision.

Comment Here

Reference: Schwannoma

Sclerosing epithelioid fibrosarcoma
Definition / general
  • Sclerosing epithelioid fibrosarcoma (SEF) is a rare, malignant mesenchymal tumor with unique architectural features consisting of cords, nests or sheets of monotonous epithelioid cells within a dense collagenous background (Virchows Arch 2020 Oct 21 [Epub ahead of print])
  • A subset is related morphologically and molecularly to low grade fibromyxoid sarcoma (LGFMS) (Am J Surg Pathol 2014;38:801)
Essential features
ICD coding
  • ICD-O:
    • 8840/3 - sclerosing epithelioid fibrosarcoma
  • ICD-11:
    • 2B5F.2 - sarcoma, not elsewhere classified of other specified sites
    • XH4BT2 - sclerosing epithelioid fibrosarcoma
Epidemiology
Sites
Pathophysiology
Etiology
  • Tumors appear to be sporadic and of unknown etiology
Clinical features
Diagnosis
Radiology description
Radiology images

Contributed by Borislav A. Alexiev, M.D.
CT

CT

Prognostic factors
Case reports
Treatment
Gross description
Gross images

Contributed by Borislav A. Alexiev, M.D.
Maxillary sinus mass

Maxillary sinus mass

Frozen section description
Microscopic (histologic) description
Microscopic (histologic) images

Contributed by Borislav A. Alexiev, M.D.
Soft tissue mass

Soft tissue mass

Sclerotic stroma

Sclerotic stroma

Epithelioid cells

Epithelioid cells

Increased cellularity

Increased cellularity

MUC4

MUC4


CD56

CD56

FLI1

FLI1

ERG

ERG

CD99

CD99

Cytology description
  • Diagnosis based solely upon cytologic features remains challenging (J Am Soc Cytopathol 2020;9:513)
  • Tumor cells range from small round, medium sized ovoid / short spindle, to epithelioid / plasmacytoid cells
  • Sclerotic, fibrous to myxoid stroma
  • Cellular variants can mimic plasmacytoma / myeloma and myoepithelioma (Int J Surg Case Rep 2020;68:228)
Positive stains
Electron microscopy description
Molecular / cytogenetics description
Sample pathology report
  • Right maxillary mass, biopsy:
    • Sclerosing epithelioid fibrosarcoma (see comment)
    • Comment: The neoplasm is composed of cords and strands of uniform epithelioid cells with scant pale cytoplasm and round to oval nuclei in a dense sclerotic stroma. Occasional mitotic figures are identified (2 mitoses/10 high power fields). Immunohistochemical stains for MUC4, CD56, FLI1, ERG and EMA (focal) are positive in tumor cells while all of the following are negative: pankeratin AE1 / AE3, S100, CD34, SMA, desmin and p63. SMARCB1 / INI1 expression is preserved. Next generation sequencing is positive for EWSR1-CREB3L1 fusion. The findings support the above diagnosis. Sclerosing epithelioid fibrosarcoma behaves more aggressively than the related low grade fibromyxoid sarcoma with a shorter survival, higher metastatic rate and greater propensity to involve deep soft tissue and bone. Recurrences and metastases, mainly to lung and bone, are common, occurring in about 50% of cases.
Differential diagnosis
Board review style question #1

The most common gene fusion in sclerosing epithelioid fibrosarcoma is:

  1. EWSR1-FLI1
  2. FUS-CREB3L2
  3. EWSR1-CREB3L1
  4. ETV6-NTRK3
  5. EP400-PHF1
Board review style answer #1
Board review style question #2

A 47 year old man presented with a mass in the soft tissue of the thigh. Hematoxylin eosin stains show a tumor composed of uniform, small, round to ovoid epithelioid cells with sparse, often clear cytoplasm and round to oval nuclei with inconspicuous nucleoli. The tumor cells are arranged in small nests and cords in a dense sclerotic stroma. Occasional mitotic figures are identified (2 mitoses/10 high power fields). The margins of the tumor are infiltrative into muscle and periosteum.

Immunohistochemical stains for MUC4, vimentin and EMA (focal) are positive in tumor cells while all of the following are negative: CD34, SOX10, S100, GFAP, desmin, p63, MyoD1, CD10 and AE1 / AE3.

Which of the following is most likely the correct diagnosis?

  1. Epithelioid sarcoma
  2. Metastatic carcinoma
  3. Ossifying fibromyxoid tumor
  4. Sclerosing epithelioid fibrosarcoma
  5. Spindle cell / sclerosing rhabdomyosarcoma
Board review style answer #2
D. Sclerosing epithelioid fibrosarcoma

Comment Here

Reference: Sclerosing epithelioid fibrosarcoma

Soft tissue and bone molecular
Table of Contents
Definition / general | Acral fibromyxoma | Alveolar soft part sarcoma | Aneurysmal bone cyst, primary | Angiofibroma of soft tissue | Angiomatoid fibrous histiocytoma | Angiosarcoma, radiation associated | Cellular angiofibroma | Chondroblastoma | Chondroid lipoma of soft tissue | Chondromyxoid fibroma | Chondrosarcoma, myxoid, extraskeletal (EMC) | Chordoma, poorly differentiated | CIC rearranged sarcoma | Clear cell sarcoma | Dermatofibrosarcoma protuberans / giant cell fibroblastoma | Desmoid type fibromatosis | Desmoplastic fibroblastoma | Desmoplastic small round cell tumor | Endometrial stromal sarcoma | Epithelioid hemangioendothelioma | Epithelioid hemangioma | Epithelioid sarcoma | Ewing sarcoma / PNET, extraosseous | EWSR1::SMAD3 positive fibroblastic tumor | Fibrous hamartoma of infancy | Giant cell fibroblastoma | Giant cell tumor of bone | Glomus tumor | Infantile fibrosarcoma / mesoblastic nephroma | Inflammatory myofibroblastic tumor | Lipoma | Lipoma, spindle cell / pleomorphic | Liposarcoma, myxoid / round cell variants | Liposarcoma, well differentiated / dedifferentiated | Low grade fibromyxoid sarcoma / sclerosing epithelioid fibrosarcoma | Low grade osteosarcoma | Malignant melanotic nerve sheath tumor | Malignant peripheral nerve sheath tumor | Mesenchymal chondrosarcoma | Myoepithelioma of soft tissue | Myopericytoma, including myofibroma | Myositis ossificans and fibro-osseous pseudotumor of digits | Myxoinflammatory fibroblastic tumor | Neuromuscular choristoma | Nodular fasciitis | NTRK rearranged spindle cell neoplasm | Ossifying fibromyxoid tumor | Osteoid osteoma / osteoblastoma | Phosphaturic mesenchymal tumor | Pseudomyogenic hemangioendothelioma | Rhabdomyosarcoma, alveolar | Round cell sarcoma with EWSR1-non-ETS fusions | Sarcoma with BCOR genetic alterations | Sclerosing epithelioid fibrosarcoma | Spindle cell / sclerosing rhabdomyosarcoma | Superficial CD34 positive fibroblastic tumor | Synovial chondromatosis | Synovial sarcoma | Tenosynovial giant cell tumor | Board review style question #1 | Board review style answer #1 | Board review style question #2 | Board review style answer #2
Definition / general
  • This topic contains characteristic molecular alterations for topics in our Soft tissue and Bone & joints chapters
  • The intent is not to catalog every alteration identified, just those that are considered characteristic
  • More information is listed in the linked topics
  • Most sarcomas are cytogenetically complex neoplasms
  • Recurrent genetic alterations are seen in a subset of bone and soft tissue neoplasms
    • Point mutations: detection may aid in diagnosis or allow targeted therapy
    • Chromosomal rearrangements: recurrent gene fusions can be useful for diagnosis
    • Copy number alterations
Acral fibromyxoma
Alveolar soft part sarcoma
Aneurysmal bone cyst, primary
Angiofibroma of soft tissue
Angiomatoid fibrous histiocytoma
Angiosarcoma, radiation associated
Cellular angiofibroma
Chondroblastoma
Chondroid lipoma of soft tissue
Chondromyxoid fibroma
Chondrosarcoma, myxoid, extraskeletal (EMC)
Chordoma, poorly differentiated
CIC rearranged sarcoma
Clear cell sarcoma
Dermatofibrosarcoma protuberans / giant cell fibroblastoma
Desmoid type fibromatosis
Desmoplastic fibroblastoma
Desmoplastic small round cell tumor
Endometrial stromal sarcoma
Epithelioid hemangioma
Epithelioid sarcoma
Ewing sarcoma / PNET, extraosseous
  • t(11;22)(q24;q12) - EWSR1::FLI1 fusion gene
  • t(2;22)(q33;q12) - EWSR1::FEV fusion gene
  • t(7,22)(p22;q12) - EWSR1::ETV1 fusion gene
  • t(17;22)(q12;q12) - EWSR1::ETV4 fusion gene
  • t(21;22)(q22;q12) - EWSR1::ERG fusion gene
  • t(16;21)(p11;q22) - FUS::ERG fusion gene
  • See Ewing sarcoma / PNET, extraosseous
  • Reference: Int J Mol Sci 2018;19:3784
Fibrous hamartoma of infancy
Giant cell fibroblastoma
Giant cell tumor of bone
Glomus tumor
Infantile fibrosarcoma / mesoblastic nephroma
Inflammatory myofibroblastic tumor
Lipoma
  • t(12;14)(q13-15;q23-24) or related changes involving HMGA2 / HMGIC at 12q13-15
  • See Lipoma
Lipoma, spindle cell / pleomorphic
Liposarcoma, myxoid / round cell variants
Liposarcoma, well differentiated / dedifferentiated
Low grade fibromyxoid sarcoma / sclerosing epithelioid fibrosarcoma
Low grade osteosarcoma
Malignant melanotic nerve sheath tumor
Malignant peripheral nerve sheath tumor
Mesenchymal chondrosarcoma
Myoepithelioma of soft tissue
Myopericytoma, including myofibroma
Myositis ossificans and fibro-osseous pseudotumor of digits
Myxoinflammatory fibroblastic tumor
Neuromuscular choristoma
Nodular fasciitis
NTRK rearranged spindle cell neoplasm
  • NTRK rearranged mesenchymal neoplasms show variable morphologic patterns, including lipofibromatosis-like neural tumor, solitary fibrous tumor-like, inflammatory myofibroblastic tumor-like, infantile fibrosarcoma, dermatofibrosarcoma protuberans-like and malignant peripheral nerve sheath tumor-like spindle cell neoplasms and various histologic grades from low to high grade, as well as benign forms (Mod Pathol 2021;34:396)
  • Pan-TRK IHC is a reliable diagnostic marker that can be used as a surrogate marker for identification of NTRK fusion in the appropriate setting (young age, morphology) to help discover these rare tumors
  • Expensive RNA based next generation sequencing (NGS) to detect / confirm specific fusions needs to be performed if patients are candidates for targeted therapy
  • Name may change in the future to kinase rearranged spindle cell neoplasm
  • May also have mutations in BRAF, RAF, RET, ALK (Am J Surg Pathol 2018;42:28, Genes Chromosomes Cancer 2018;57:611, Mod Pathol 2021;34:1710)
  • See Pan-TRK (EPR17341) [NTRK]
Ossifying fibromyxoid tumor
Osteoid osteoma / osteoblastoma
Phosphaturic mesenchymal tumor
Pseudomyogenic hemangioendothelioma
Rhabdomyosarcoma, alveolar
Round cell sarcoma with EWSR1-non-ETS fusions
Sarcoma with BCOR genetic alterations
  • BCOR::CCNB3 sarcoma (BCS) is a recently defined genetic entity among undifferentiated round cell sarcomas, initially classified as and treated similarly to the Ewing sarcoma (ES) family of tumors
  • In contrast to ES, BCS shows consistent BCOR overexpression and preliminary evidence suggests that these tumors share morphologic features with other tumors harboring BCOR genetic alterations, including BCOR internal tandem duplication (ITD) and BCOR::MAML3 (Am J Surg Pathol 2018;42:604)
  • BCOR - BCL6 transcriptional corepressor is located at Xp11.4 locus (Genes Dev 2000;14:1810, J Clin Pathol 2020;73:314)
  • See BCOR
Sclerosing epithelioid fibrosarcoma
Spindle cell / sclerosing rhabdomyosarcoma
Superficial CD34 positive fibroblastic tumor
Synovial chondromatosis
Synovial sarcoma
Tenosynovial giant cell tumor
Board review style question #1

Which of the following is true about atypical lipomatous tumor / well differentiated liposarcoma?

  1. Absence of MDM2 amplification
  2. FUS::DDIT3 or EWSR1::DDIT3 fusions are almost always present
  3. Loss of RB1 expression is observed in 50 - 70% of cases
  4. MDM2 amplification is almost always present
  5. Usually harbors USP6 rearrangement
Board review style answer #1
D. MDM2 amplification is almost always present

Comment Here

Reference: Soft tissue and bone molecular
Board review style question #2

The most common gene rearranged in ossifying fibromyxoid tumor is

  1. EWSR1
  2. FOS
  3. FUS
  4. NOTCH
  5. PHF1
Board review style answer #2

Soft tissue chondroma
Definition / general
  • Benign, cartilage forming tumor that usually arises in the vicinity of joints or tendons in the hands and feet of adults
Essential features
  • Arises from soft tissue of fingers, hands and feet
  • Not connected to the underlying bone
  • Well circumscribed, cartilaginous proliferation
Terminology
  • Soft tissue chondroma, extraskeletal chondroma, chondroma of soft parts
ICD coding
  • ICD-10: D21.9 - benign neoplasm of connective and other soft tissue, unspecified
Epidemiology
Sites
  • Mostly occurs in the fingers
  • Hands, toes, feet and trunk are less frequently affected
Clinical features
  • Occurs in soft tissue of hands and feet
  • Solitary, slowly enlarging nodule; occasionally causes pain or tenderness
  • Reference: Histopathology 1986;10:147
Diagnosis
  • Clinical presentation
    • Solitary nodule
    • Involves fingers, toes and hands
    • < 3 cm
  • Imaging
    • Computed tomography
    • Magnetic resonance imaging
  • Reference: Histopathology 1986;10:147
Radiology description
  • Well demarcated
  • Does not involve bone, although some tumors cause compression deformities or bone erosion
  • Discrete, irregular, ring-like or curvilinear calcifications
  • Reference: AJR Am J Roentgenol 1985;144:1263
Radiology images

Contributed by Andrew E. Rosenberg, M.D.
Calcified lesion

Calcified lesion

Well circumscribed, calcified soft tissue mass

Well circumscribed, calcified soft tissue mass

Prognostic factors
Case reports
Treatment
  • Simple excision
Clinical images

Contributed by Andrew E. Rosenberg, M.D.
Swelling over the toe

Swelling over the toe



Images hosted on other servers:
Foot tumor

Foot tumor

Gross description
  • Well demarcated, oval - round
  • < 3 cm in size
Gross images

Images hosted on other servers:
Well circumscribed mass

Well circumscribed mass

Foot tumor Foot tumor

Foot tumor

Frozen section description
Frozen section images

Contributed by Fireneh N. Beshah, M.D. and Jaylou M. Velez Torres, M.D.
Hypocellular cartilage

Hypocellular cartilage

Chondrocytes with small nuclei

Chondrocytes with small nuclei

Microscopic (histologic) description
  • Mature hyaline cartilage arranged in lobules with sharp borders
  • Chondrocytes in lacunae, arranged diffusely or in small clusters
  • Rarely can have moderate pleomorphism
  • 33% show focal or diffuse calcifications
  • Chondroblastoma-like soft tissue chondroma shows hypercellular areas with cells resembling chondroblasts, recently reclassified as calcified chondroid mesenchymal tumor (Mod Pathol 2021;34:1373)
Microscopic (histologic) images

Contributed by Andrew E. Rosenberg, M.D.
Well circumscribed nodule

Well circumscribed nodule

Peripheral ossification

Peripheral ossification

Enlarged nuclei

Enlarged nuclei

Fibrous pseudocapsule

Fibrous pseudocapsule

Cellular and myxoid areas

Cellular and myxoid areas

Fibrocartilage

Fibrocartilage

Cytology images

Images hosted on other servers:
Foot tumor

Foot tumor

Positive stains
Negative stains
Molecular / cytogenetics description
Sample pathology report
  • Right hip mass, excision:
    • Soft tissue chondroma
Differential diagnosis
  • Synovial chondromatosis:
    • Large joints, multiple small nodules attached to synovial membrane
    • Cloning / clustering of chondrocytes
    • FN1 gene rearrangement (Mod Pathol 2019;32:1762)
    • The mutual exclusivity of ACVR2A rearrangements observed in synovial chondromatosis and FGFR1 / 2 in soft tissue chondromas suggests these represent separate entities (Mod Pathol 2019;32:1762)
  • Calcifying aponeurotic fibroma:
    • Islands of calcification surrounded by palisaded epithelioid fibroblasts (resembling chondrocytes), poorly circumscribed and infiltrative
  • Juxtacortical chondroma:
    • Attached to surface of bone
  • Acral fibrochondromyxoid tumor:
  • Calcified chondroid mesenchymal tumor:
    • Multinodular architecture with increased cellularity towards the periphery of the nodules (Mod Pathol 2021;34:1373)
    • Matrix frequently shows coarse, grungy to lacy calcifications, which are refractive rhomboid crystals under polarized light
    • Osteoclast-like giant cells are frequently identified
Board review style question #1

A 42 year old man presented with a painless soft tissue nodule on the right index finger. Magnetic resonance imaging showed a 2 cm well circumscribed nodule (T1 hypointense) with no connection to the underlying bone. What is the diagnosis?

  1. Chondrosarcoma
  2. Osteochondroma
  3. Soft tissue chondroma
  4. Synovial chondromatosis
Board review style answer #1
C. Soft tissue chondroma. The histology shows a nodule of cartilage surrounded by fibrous tissue, which is characteristic of soft tissue chondroma. Answer D is incorrect because synovial chondromatosis is characterized by multinodular mass with superficial synovial lining. Answer A is incorrect because chondrosarcomas tend to be hypercellular and have cytologic atypia. Answer B is incorrect because the tumor is not connected to the underlying bone and does not have the characteristic trabecular bone with cartilage cap.

Comment Here

Reference: Soft tissue chondroma
Board review style question #2
Which of the following is a feature of soft tissue chondroma?

  1. Arises from big joints like hip and knee joint
  2. Size is usually > 3 cm
  3. Tends to be a solitary nodule
  4. Usually arises from the underlying bones
Board review style answer #2
C. Tends to be a solitary nodule. Soft tissue chondroma usually presents as a single nodule. Answer B is incorrect because soft tissue chondromas are < 3 cm in size in the majority of cases. Answer D is incorrect because soft tissue chondromas have no connection to the underlying bone. Answer A is incorrect because soft tissue chondromas usually involve fingers, toes and hands.

Comment Here

Reference: Soft tissue chondroma

Solitary circumscribed neuroma (pending)
Table of Contents
Definition / general
Definition / general
[Pending]

Solitary fibrous tumor
Definition / general
  • Fibroblastic tumor characterized by haphazardly arranged spindled to ovoid cells, prominent staghorn vasculature and NAB2-STAT6 gene rearrangement
Essential features
  • Haphazard arrangement of spindled to ovoid cells arranged around branching and dilated vasculature within variably collagenous stroma
  • Occurs at any anatomical site with a wide range of histological patterns
  • CD34 usually strong and diffuse but nonspecific
  • STAT6 immunohistochemistry highly sensitive and specific surrogate for NAB2-STAT6 gene fusion (Mod Pathol 2014;27:390)
  • Risk stratification preferred over anatomic staging (Mod Pathol 2017;30:1433)
Terminology
  • Hemangiopericytoma (no longer preferred) (Cancer 1975;36:2232)
  • Giant cell angiofibroma (no longer preferred)
ICD coding
  • ICD-10: D48.1 - neoplasm of uncertain behavior of connective and other soft tissue
Epidemiology
Sites
Pathophysiology
Etiology
  • Unknown
Diagrams / tables

Images hosted on other servers:

Modified 4 variable risk stratification model

Clinical features
  • Symptoms associated with anatomic location
  • Somatic soft tissue tumors present as slow growing, painless mass
  • Abdominopelvic tumors produce symptoms due to organ impingement (Eur Urol 2008;54:1188)
  • Pleural tumors often discovered incidentally but could grow exophytically into lungs
  • Paraneoplastic syndrome (Doege-Potter syndrome) extremely rare; due to IGF2 production by tumor (J Endocr Soc 2019;3:537)
Diagnosis
  • Imaging features are nonspecific and diagnosis is dependent on histologic findings
Radiology description
  • CT: well defined, sometimes lobulated mass that is isodense to skeletal muscle with heterogeneous contrast enhancement (Clin Imaging 2018;48:48)
  • MRI: T1 intermediate and T2 hypointense (cellular / fibrous areas) to hyperintense (myxoid areas) signals (Radiol Clin North Am 2016;54:565)
Radiology images

Contributed by Borislav A. Alexiev, M.D.
Pleural mass

Pleural mass



Images hosted on other servers:
Bulky thyroid tumor (CT)

Bulky thyroid tumor (CT)

Huge perineal mass (CT)

Huge perineal mass (CT)

Ischiorectal fossa mass (MRI)

Ischiorectal fossa mass (MRI)

Bladder serosa mass (CT)

Bladder serosa mass (CT)

Prognostic factors
Case reports
Treatment
Clinical images

Images hosted on other servers:
Palatal mass causing proptosis

Palatal mass causing proptosis

Huge perineal mass

Huge perineal mass

Gross description
  • Usually unencapsulated but well circumscribed mass, sometimes lobulated
  • Exophytic lesions on serosal surfaces
  • 5 - 15 cm in greatest diameter
  • Gray-white, yellow to red-brown color
  • Fibrous and firm cut surface
  • Cystic, hemorrhagic or myxoid degeneration may be present
  • Tumor necrosis and hemorrhage in malignant tumors
  • Reference: Am J Surg Pathol 1995;19:1257
Gross images

Contributed by Borislav A. Alexiev, M.D. (Case #505)
Fat forming SFT

Fat forming SFT



Images hosted on other servers:
Solid white cut surface

Solid white cut surface

Circumscribed lobulated mass

Circumscribed lobulated mass

Central necrosis

Central necrosis

Fleshy nodules

Fleshy nodules

Frozen section description
  • Circumscribed spindled cell tumor
  • Presence of staghorn vasculature helpful but not entirely specific
Microscopic (histologic) description
Microscopic (histologic) images

Contributed by Josephine Dermawan, M.D., Ph.D., Karen Fritchie, M.D.
Ovoid to fusiform cells

Ovoid to fusiform cells

Staghorn vasculature

Staghorn vasculature

Collagenous stroma

Collagenous stroma

High risk solitary fibrous tumor

High risk solitary fibrous tumor

Metastatic solitary fibrous tumor

Metastatic solitary fibrous tumor

Myxoid solitary fibrous tumor

Myxoid solitary fibrous tumor


Lipomatous solitary fibrous tumor

Lipomatous solitary fibrous tumor

Dedifferentiated solitary fibrous tumor

Dedifferentiated solitary fibrous tumor

Diffuse and strong CD34

Diffuse and strong CD34

STAT6

STAT6

STAT6 loss

STAT6 loss

Myogenin

Myogenin




Contributed by Borislav A. Alexiev, M.D. (Case #505)
Fat forming SFT Fat forming SFT Fat forming SFT Fat forming SFT Fat forming SFT Fat forming SFT

Fat forming SFT

Virtual slides

Images hosted on other servers:
Solitary fibrous tumor

Solitary fibrous tumor

Cytology description
  • Low to moderate cellularity
  • Oval, elongate, rounded or stellate cells with wispy cytoplasm and pink collagenous stroma (Cancer Cytopathol 2018;126:36)
  • High cellularity and nuclear pleomorphism in malignant cases
Cytology images

Images hosted on other servers:
Diff-Quik, fine needle aspiration

Diff-Quik, fine needle aspiration

Pap, touch prep

Pap, touch prep

Positive stains
Electron microscopy description
  • Fibroblast-like cells with well developed rough endoplasmic reticulum and surrounded by collagen fibers (Med Mol Morphol 2009;42:239)
Molecular / cytogenetics description
Molecular / cytogenetics images

Images hosted on other servers:
Capillary sequencing and RT PCR

Capillary sequencing and RT PCR

Videos

Solitary fibrous tumor

Sample pathology report
  • Soft tissue, pelvis, excision:
    • Solitary fibrous tumor (5.6 cm), completely excised (see comment)
    • Comment: Histologic sections demonstrate a moderately cellular spindled cell tumor in a collagenous stroma with prominent dilated staghorn type vasculature. Mitotic rate is approximately 1 - 2 per 10 high power fields. Overt cytologic atypia and tumor necrosis are not identified. Immunohistochemical studies show that the tumor cells are diffusely and strongly positive for CD34 and STAT6 and are negative for SOX10, S100 and AE1 / AE3, supporting the above diagnosis. According to the 2017 risk stratification criteria by Demicco et al, this should be classified as a low risk solitary fibrous tumor.
Differential diagnosis
Board review style question #1

A 54 year old woman presents with a 9.6 cm, well circumscribed, firm mass in the deep soft tissue of the thigh. Histologic sections show ovoid to spindled cells arranged haphazardly around prominent thin walled, hyalinized, dilated staghorn type vasculature (see image above). Which of the following is the most sensitive and specific immunohistochemical marker for the diagnosis of this tumor?

  1. Actin
  2. BCL2
  3. CD34
  4. Cytokeratin CAM 5.2
  5. STAT6
Board review style answer #1
Board review style question #2
Which of the following is a favorable prognostic factor for solitary fibrous tumor?

  1. Age < 55 years old
  2. Mitotic rate ≥ 4 per 10 high power fields
  3. TERT promoter mutations
  4. Tumor necrosis
  5. Tumor size ≥ 10 cm
Board review style answer #2
A. Age < 55 years old

Comment Here

Reference: Soft tissue - Solitary fibrous tumor
Board review style question #3
Which of the following is true about the majority of fat forming solitary fibrous tumor cases?

  1. All tumors show reactivity for CD34
  2. Fat forming solitary fibrous tumors usually occur in younger adults, predominantly in females
  3. Multivacuolated lipoblasts are more common in the malignant subset
  4. The imaging findings of fat forming solitary fibrous tumor are specific
  5. The mediastinum is the most commonly affected site
Board review style answer #3
C. Multivacuolated lipoblasts are more common in the malignant subset

Comment Here

Reference: Soft tissue - Solitary fibrous tumor

Spindle cell / pleomorphic lipoma
Definition / general
  • Spindle cell lipoma and pleomorphic lipoma are benign adipocytic tumors, currently regarded as morphologic variants of a single neoplasm
  • Compose approximately 1.5% of all lipomatous tumors (Histopathology 1987;11:803)
  • First described in 1975 by Enzinger and Harvey (Cancer 1975;36:1852)
Essential features
  • Spindle cell lipoma contains a varying mixture of mature adipocytes, bland spindle cells and ropy collagen fibers (Cancer 1975;36:1852)
  • Pleomorphic lipoma also contains pleomorphic and multinucleated floret-like giant cells
Terminology
  • Dendritic fibromyxolipoma (not recommended)
ICD coding
Epidemiology
  • Commonly affects middle aged to elderly men (45 - 70 years) (Cancer 1975;36:1852)
  • Very rare in patients < 20 years
  • < 10% of cases occur in women
  • Uncommon compared with conventional lipoma (ratio = 1:60)
Sites
Pathophysiology
Etiology
Clinical features
  • Asymptomatic
  • Longstanding, mobile lesion in subcutaneous tissue
  • Usually solitary
  • Shawl region of posterior neck / shoulder / upper back (Cancer 1975;36:1852)
  • Small size (most < 5 cm)
Diagnosis
  • Superficial, well circumscribed adipocytic lesion involving characteristic shawl region in a middle aged to elderly man (Cancer 1975;36:1852)
  • Demonstration of CD34 positivity and loss of RB1 helpful
Radiology description
  • MRI
    • Appearance can vary from completely nonfatty to lipoma-like (Clin Radiol 2020;75:396.e15)
    • Findings nonspecific
    • Helpful in determining extent of lesion
    • Fat free variant may be misdiagnosed as sarcoma
Radiology images

Images hosted on other servers:

Sagittal CT of posterior neck swelling

Coronal CT of posterior neck swelling

MRI characteristics

Prognostic factors
  • Benign lesion, no risk of metastasis
  • Local recurrence is rare even with incomplete excision (Cancer 1975;36:1852)
Case reports
Treatment
  • Conservative surgical excision only
Gross description
  • Well circumscribed, oval mass in subcutaneous tissue (Semin Diagn Pathol 2019;36:105)
  • Yellow-tan cut surface with grey-white and myxoid foci
  • Texture firmer than ordinary lipoma
  • Dermal and intramuscular tumors may have an infiltrative appearance
Gross images

Images hosted on other servers:

Excised tumor

Bisected surface whiter than typical lipoma

Frozen section description
  • Mature adipocytes mixed with bland spindle cells
Microscopic (histologic) description
Microscopic (histologic) images

Contributed by Laura Warmke, M.D., Jeanne Meis, M.D. and AFIP images

Spindle cell lipoma

Myxoid change

(Pseudo)
angiomatous appearance

Fat poor subtype

Myxofibrosarcoma-like appearance


Cellular spindle cell lipoma

Pleomorphic lipoma with myxoid change

Pleomorphic lipoma

CD34 positivity

RB1 loss



Contributed by @AnaPath10 on Twitter
Spindle cell / pleomorphic lipoma Spindle cell / pleomorphic lipoma

Spindle cell / pleomorphic lipoma

Spindle cell / pleomorphic lipoma Spindle cell / pleomorphic lipoma

Spindle cell / pleomorphic lipoma

Virtual slides

Images hosted on other servers:

Conventional spindle cell lipoma

Spindle cell lipoma with myxoid change

Pleomorphic lipoma

Cytology description
  • Mixture of mature adipocytes, uniform spindle cells and collagen fibers (Cancer 2001;93:381)
  • Spindle cell nuclei fusiform to ovoid with poorly defined, bipolar cytoplasmic processes
  • Nuclear grooves may be present
  • Myxoid background with mast cells common
  • Mitotic activity and nuclear pleomorphism typically absent
  • Multinucleated giant cells in pleomorphic lipoma
Cytology images

Contributed by Laura Warmke, M.D.

Bland spindle cells

Mature adipocytes



Images hosted on other servers:

Spindle / wavy nuclei

Wavy collagen fibers

Mast cells

Myxoid change

Positive stains
Negative stains
Electron microscopy description
  • Spindle cells with some features of fibroblasts
  • Nonmembrane bound lipid droplets, suggesting possible prelipoblastic nature (Cancer 2001;93:381)
Electron microscopy images

Images hosted on other servers:

Ultrastructural study

Molecular / cytogenetics description
Molecular / cytogenetics images

Images hosted on other servers:

Karyogram

Videos

Spindle cell lipoma

Pleomorphic lipoma

Sample pathology report
  • Soft tissue, back, excision:
    • Spindle cell lipoma (see comment)
    • Comment: Sections show a benign adipocytic neoplasm with bland spindle cells and eosinophilic collagen bundles in a myxoid background with scattered mast cells. No necrosis, marked cytologic atypia or significant mitotic activity is identified. Immunohistochemical stains show that the spindle cells are diffusely positive for CD34, while demonstrating loss of RB1. These results support the above diagnosis.
Differential diagnosis
Board review style question #1

What is the most typical clinical presentation of a patient with a spindle cell lipoma?

  1. 30 year old woman soft tissue mass (9 cm) involving her right lateral thigh
  2. 45 year old woman with subcutaneous mass (3 cm) involving her left arm
  3. 55 year old man with subcutaneous mass (4 cm) involving his upper back
  4. 60 year old man with soft tissue mass (15 cm) involving his retroperitoneum
Board review style answer #1
C. 55 year old man with subcutaneous mass (4 cm) involving his upper back. Spindle cell lipoma usually occurs in middle aged to elderly men. The lesion is often small (< 5 cm) and frequently involves the subcutaneous tissue of the upper back, shoulder or posterior neck.

Comment Here

Reference: Spindle cell / pleomorphic lipoma
Board review style question #2

What 2 immunohistochemical stains may be helpful in confirming the diagnosis of spindle cell lipoma?

  1. CD34 positive and RB1 loss
  2. CD34 positive and STAT6 positive
  3. Desmin positive and RB1 loss
  4. MDM2 positive and CDK4 positive
Board review style answer #2
A. Spindle cell lipoma is diffusely positive for CD34 and demonstrates loss of expression of RB1. Atypical lipomatous tumor / well differentiated liposarcoma is positive for both MDM2 and CDK4. Solitary fibrous tumor is positive for both CD34 and STAT6. Desmin positivity and loss of expression of RB1 can be seen in mammary type myofibroblastoma.

Comment Here

Reference: Spindle cell / pleomorphic lipoma

Spindle cell / sclerosing rhabdomyosarcoma
Definition / general
  • Spindle cell / sclerosing rhabdomyosarcoma (RMS) is a rare type of rhabdomyosarcoma characterized by fascicular spindle cell morphology
  • Subclassification depends on the presence of genetic alterations associated with prognosis
  • Predilection for the head and neck / extremities
  • 5 - 10% of RMS (Arch Pathol Lab Med 2015;139:1281)
Essential features
  • Rhabdomyosarcoma with prominent fascicular spindle cell or sclerosing features
  • Relatively similar morphology but a genetically and clinically heterogeneous disease, especially in the pediatric population
  • Tumors with VGLL2, NCOA2 and CITED2 gene rearrangements typically arise in infants and have a favorable prognosis (Am J Surg Pathol 2016;40:224)
  • Tumors with MYOD1 mutations usually arise in adolescents and adults and have an unfavorable prognosis (Mod Pathol 2019;32:27)
  • Tumors with TFCP2::NCOA2 gene rearrangements can be intraosseous (Mod Pathol 2020;33:404)
Terminology
  • Spindle cell / sclerosing are considered the same diagnostic variant of rhabdomyosarcoma
ICD coding
  • ICD-O: 8912/3 - spindle cell rhabdomyosarcoma
  • ICD-11: 2B55.Z & XH7NM2 - rhabdomyosarcoma, unspecified primary site & spindle cell rhabdomyosarcoma
Epidemiology
  • M > F
Sites
Pathophysiology
Clinical features
Diagnosis
Radiology description
Radiology images

Images hosted on other servers:
Missing Image

Large masticator space mass in neck

Cranial MRI

CT scan

MRI

Prognostic factors
Case reports
Treatment
Clinical images

Images hosted on other servers:
Missing Image

Cystic mass

Gross description
  • Variably circumscribed tumor with a tan, sometimes whorled cut surface
Microscopic (histologic) description
  • Fascicles of spindle cells with a herringbone growth pattern resembling leiomyosarcoma or fibrosarcoma
  • Primitive undifferentiated areas with round cells and hyperchromatic nuclei may focally be present
  • Tadpole or strap cells and rhabdomyoblasts can be seen in some cases
  • Sclerosing RMS shows prominent hyalinization / sclerosis; tumor cells in cords, nests or trabeculae in a pseudovascular growth pattern
  • In bone, there can be epithelioid cells in sheets and fascicles in addition to the typical spindle cell morphology (Histopathology 2018;73:514)
Microscopic (histologic) images

Contributed by Erica Kao, M.D. and Borislav A. Alexiev, M.D.

Sclerotic and collagenous stroma

Fascicular growth

MyoD1

Positive stains
Molecular / cytogenetics description
Sample pathology report
  • Soft tissue, neck, biopsy:
    • Rhabdomyosarcoma with spindle cell features (see comment)
    • Comment: The biopsy contains cellular fascicles of tumor cells that are strongly positive for desmin, myogenin and MyoD1. Molecular testing will be performed for further subtyping and reported in an addendum.
Differential diagnosis
Board review style question #1
Which soft tissue tumor is more commonly found in the head and neck region?

  1. Angiolipoma
  2. Dermatofibrosarcoma protuberans
  3. Liposarcoma
  4. Spindle cell / sclerosing rhabdomyosarcoma
Board review style answer #1
D. Spindle cell / sclerosing rhabdomyosarcoma

Comment Here

Reference: Spindle cell / sclerosing rhabdomyosarcoma
Board review style question #2

Which soft tissue tumor is shown in the image above and is associated with fascicular spindle cell morphology?

  1. Alveolar rhabdomyosarcoma
  2. Embryonal rhabdomyosarcoma
  3. Pleomorphic rhabdomyosarcoma
  4. Spindle cell / sclerosing rhabdomyosarcoma
Board review style answer #2
D. Spindle cell / sclerosing rhabdomyosarcoma. Fascicular spindle cell morphology is one of the essential features of spindle cell / sclerosing rhabdomyosarcoma.

Comment Here

Reference: Spindle cell / sclerosing rhabdomyosarcoma

Staging
Definition / general
  • Includes soft tissue tumors of intermediate (locally aggressive and rarely metastasizing) potential and malignant soft tissue tumors
  • Does not include carcinosarcoma, pediatric Ewing sarcoma, pediatric rhabdomyosarcoma, Kaposi sarcoma, gastroinstestinal stromal tumor and uterine sarcoma
Essential features
  • AJCC 7th edition staging was sunset on December 31, 2017; as of January 1, 2018; use of the 8th edition is mandatory
  • TNM staging system for soft tissue tumors applies to all soft tissue sarcomas of the extremities and trunk, abdomen, thoracic visceral organs and retroperitoneum except Kaposi sarcoma, gastrointestinal stromal tumors, fibromatosis (desmoid tumor) and infantile fibrosarcoma
  • Staging system applies to all soft tissue sarcomas of the head and neck except angiosarcoma, rhabdomyosarcoma of the embryonal and alveolar subtypes, Kaposi sarcoma and dermatofibrosarcoma protuberans
  • Sarcomas arising within the confines of the dura mater, including the brain, are not optimally staged by this system
  • Regardless of the anatomic site, locally aggressive soft tissue neoplasms, which may recur locally but have either no risk of metastatic disease or an extremely low risk of metastasis, are excluded from the AJCC soft tissue sarcoma staging system
  • Note: see CAP for additional information
Terminology
  • Reporting of pT (tumor), pN (lymph node) and (when applicable) pM (metastasis) categories is based on information available to the pathologist at the time the report is issued; as per the AJCC, it is the managing physician's responsibility to establish the final pathologic stage based upon all pertinent information (Amin: AJCC Cancer Staging Manual, 8th Edition, 2018)
  • Tumor (T):
    • Based on the tumor size, as measured grossly or with imaging
    • Largest dimension in any plane
    • Best to provide 3 dimensional size if possible
  • Nodes (N):
    • Lymph node status needs to be determined both clinically and microscopically
  • Metastasis (M):
    • Either not applicable (pM cannot be determined from the submitted specimen[s]) or distant metastasis (pM1)
  • Prefix:
    • If the lesion was previously treated, use the prefix y (ypTNM)
    • If the lesion is recurrent, use the prefix r (rpTNM)
Primary tumor (pT)
  • Head and neck
    • pT: not assigned (cannot be determined based on available pathological information)
    • pT1: tumor ≤ 2 cm
    • pT2: tumor > 2 cm to ≤ 4 cm
    • pT3: tumor > 4 cm
    • pT4: tumor with invasion of adjoining structures
      • pT4a: tumor with orbital invasion, skull base / dural invasion, invasion of central compartment viscera, involvement of facial skeleton or invasion of pterygoid muscles
      • pT4b: tumor with brain parenchymal invasion, carotid artery encasement, prevertebral muscle invasion or central nervous system involvement via perineural spread
      • pT4 (subcategory cannot be determined)
  • Trunk and extremities
    • pT: not assigned (cannot be determined based on available pathological information)
    • pT0: no evidence of primary tumor
    • pT1: tumor ≤ 5 cm in greatest dimension
    • pT2: tumor > 5 cm and ≤ 10 cm in greatest dimension
    • pT3: tumor > 10 cm and ≤ 15 cm in greatest dimension
    • pT4: tumor > 15 cm in greatest dimension
  • Abdomen and thoracic visceral organs
    • pT: not assigned (cannot be determined based on available pathological information)
    • pT1: organ confined
    • pT2: tumor extension into tissue beyond organ
      • pT2a: invades serosa or visceral peritoneum
      • pT2b: extension beyond serosa (mesentery)
      • pT2 (subcategory cannot be determined)
      • pT3: invades another organ
    • pT4: multifocal involvement
      • pT4a: multifocal (2 sites)
      • pT4b: multifocal (3 - 5 sites)
      • pT4c: multifocal (> 5 sites)
      • pT4 (subcategory cannot be determined)
  • Retroperitoneum
    • pT: not assigned (cannot be determined based on available pathological information)
    • pT0: no evidence of primary tumor
    • pT1: tumor ≤ 5 cm in greatest dimension
    • pT2: tumor > 5 cm and ≤ 10 cm in greatest dimension
    • pT3: tumor > 10 cm and ≤ 15 cm in greatest dimension
    • pT4: tumor > 15 cm in greatest dimension
  • Orbit
    • pN: not assigned (no nodes submitted or found)
    • pT: not assigned (cannot be determined based on available pathological information)
    • pT0: no evidence of primary tumor
    • pT1: tumor ≤ 2 cm in greatest dimension
    • pT2: tumor > 2 cm in greatest dimension without invasion of bony walls or globe
    • pT3: tumor of any size with invasion of bony walls
    • pT4: tumor of any size with invasion of globe or periorbital structures, including eyelid, conjunctiva, temporal fossa, nasal cavity, paranasal sinuses or central nervous system
Regional lymph nodes (pN)
  • pN: not assigned (no nodes submitted or found)
  • pN: not assigned (cannot be determined based on available pathological information)
  • pN0: no regional lymph node metastasis
  • pN1: regional lymph node metastasis
Distant metastasis (pM)
  • Not applicable; pM cannot be determined from the submitted specimen(s)
  • pM1: distant metastasis
Prefixes
  • Not applicable
  • m: multiple
  • y: preoperative radiotherapy or chemotherapy
  • r: recurrent tumor stage
Histologic grade (G)
  • 8th edition of the AJCC Cancer Staging Manual adopted the French Federation of Cancer Centers Sarcoma Group (FNCLCC) grading system (Amin: AJCC Cancer Staging Manual, 8th Edition, 2018)
  • FNCLCC grading
    • FNCLCC grade is based on 3 parameters: differentiation, mitotic activity and necrosis
    • Each of these parameters receives a score: differentiation (1 to 3), mitotic activity (1 to 3) and necrosis (0 to 2); the scores are summed to produce a grade (Arch Pathol Lab Med 2006;130:1448)
      • GX: cannot be assessed
      • G1: 2 or 3
      • G2: 4 or 5
      • G3: 6 to 8
  • Differentiation: tumor differentiation is scored as follows
    • Score 1: sarcomas closely resembling normal, adult mesenchymal tissue and potentially difficult to distinguish from the counterpart benign tumor (e.g., well differentiated liposarcoma, well differentiated leiomyosarcoma)
    • Score 2: sarcomas for which histologic typing is certain (e.g., myxoid liposarcoma, myxofibrosarcoma)
    • Score 3: embryonal sarcomas and undifferentiated sarcomas, synovial sarcomas and sarcomas of doubtful tumor type

    Table 1: Tumor differentiation
    Histologic type
    Score
    Atypical lipomatous tumor / well differentiated liposarcoma 1
    Well differentiated leiomyosarcoma 1
    Malignant neurofibroma 1
    Well differentiated fibrosarcoma 1
    Myxoid liposarcoma 2
    Conventional leiomyosarcoma 2
    Conventional fibrosarcoma 2
    Myxofibrosarcoma 2
    High grade myxoid (round cell) liposarcoma 3
    Pleomorphic liposarcoma 3
    Dedifferentiated liposarcoma 3
    Pleomorphic rhabdomyosarcoma 3
    Poorly differentiated / pleomorphic leiomyosarcoma 3
    Biphasic / monophasic / poorly differentiated synovial sarcoma 3
    Mesenchymal chondrosarcoma 3
    Extraskeletal osteosarcoma 3
    Extraskeletal Ewing sarcoma 3
    Malignant rhabdoid tumor 3
    Undifferentiated pleomorphic sarcoma 3
    Undifferentiated sarcoma, not otherwise specified 3

Notes:
  • Grading of malignant peripheral nerve sheath tumor, embryonal and alveolar rhabdomyosarcoma, angiosarcoma, extraskeletal myxoid chondrosarcoma, alveolar soft part sarcoma, clear cell sarcoma and epithelioid sarcoma is not recommended (Cancer 1984;53:530)
  • Case for grading malignant peripheral nerve sheath tumor is currently being debated

  • Mitosis count: the count is made in the most mitotically active area, away from areas of necrosis, in either 10 consecutive high power fields (HPF) (use the 40x objective) (1 HPF x 400 = 0.1734 mm2) or in the appropriate number of HPF to encompass 1 mm2 based on each individual microscope; if the mitotic rate is close to the cutoff between mitotic scores, the count should be repeated

    Table 2: Mitotic count score equivalent
    Mitotic score
    # mitosis/10 HPF
    # mitosis/1 mm2
    Score 1 0 - 9 mitosis/10 HPF 0 - 5 mitosis/1 mm2
    Score 2 10 - 19 mitosis/10 HPF 6 - 11 mitosis/1 mm2
    Score 3 > 19 mitosis/10 HPF > 11 mitosis/1 mm2

  • Tumor necrosis: evaluated on gross examination and validated with histologic sections

    Table 3: Necrosis
    Score
    Necrosis
    Score 0 No tumor necrosis
    Score 1 < 50% tumor necrosis
    Score 2 ≥ 50% tumor necrosis
Histopathologic type (2020 WHO classification of soft tissue tumors)
  • Adipocytic tumors
    • Intermediate (locally aggressive)
      • Atypical lipomatous tumor
    • Malignant
      • Well differentiated liposarcoma
      • Dedifferentiated liposarcoma
      • Myxoid liposarcoma
      • Pleomorphic liposarcoma
      • Epithelioid liposarcoma
      • Myxoid pleomorphic liposarcoma
  • Fibroblastic / myofibroblastic tumors
    • Intermediate (rarely metastasizing)
      • Fibrosarcomatous dermatofibrosarcoma protuberans
    • Malignant
      • Solitary fibrous tumor, malignant
      • Adult fibrosarcoma
      • Myxofibrosarcoma
      • Epithelioid myxofibrosarcoma
      • Low grade fibromyxoid sarcoma
      • Sclerosing epithelioid fibrosarcoma
  • So called fibrohistiocytic tumors
    • Malignant
      • Malignant tenosynovial giant cell tumor
  • Smooth muscle tumors
    • Malignant
      • Leiomyosarcoma
  • Pericytic (perivascular) tumors
    • Malignant
      • Malignant glomus tumor
  • Skeletal muscle tumors
    • Malignant
      • Embryonal rhabdomyosarcoma (including botryoid, anaplastic)
      • Alveolar rhabdomyosarcoma (including solid, anaplastic)
      • Pleomorphic rhabdomyosarcoma
      • Spindle cell / sclerosing rhabdomyosarcoma
        • Spindle cell / sclerosing rhabdomyosarcoma, NOS
        • Congenital spindle cell rhabdomyosarcoma with VGLL2 / NCOA2 / CITED2 rearrangements
        • MYOD1 mutant spindle cell / sclerosing rhabdomyosarcoma
        • Intraosseous spindle cell rhabdomyosarcoma (with TFCP2 / NCOA2 rearrangements)
      • Ectomesenchymoma
  • Vascular tumors
    • Malignant
      • Epithelioid hemangioendothelioma with WWTR1::CAMTA1 fusion
      • Epithelioid hemangioendothelioma with YAP1::TFE3 fusion
      • Epithelioid hemangioendothelioma, NOS
  • Peripheral nerve tumors
    • Malignant
      • Malignant peripheral nerve sheath tumor
      • Epithelioid malignant peripheral nerve sheath tumor
      • Malignant granular cell tumor
      • Malignant perineurioma
  • Chondro-osseous tumors
    • Malignant
      • Extraskeletal osteosarcoma
  • Tumors of uncertain differentiation
    • Intermediate (rarely metastasizing)
      • Ossifying fibromyxoid tumor
      • Mixed tumor
      • Mixed tumor, NOS, malignant
      • Myoepithelioma
    • Malignant
      • Phosphaturic mesenchymal tumor, malignant
      • NTRK rearranged spindle cell neoplasm
      • Synovial sarcoma, biphasic
      • Synovial sarcoma, spindle cell
      • Synovial sarcoma, poorly differentiated
      • Synovial sarcoma, NOS
      • Epithelioid sarcoma, classic type
      • Epithelioid sarcoma, proximal or large cell type
      • Alveolar soft part sarcoma
      • Clear cell sarcoma of soft tissue
      • Extraskeletal myxoid chondrosarcoma
      • Ossifying fibromyxoid tumor, malignant
      • Myoepithelial carcinoma
      • Extraskeletal Ewing sarcoma
      • Round cell sarcoma with EWSR1 non-ETS fusions
      • CIC rearranged sarcoma
  • Undifferentiated / unclassified sarcomas
    • Malignant
      • Undifferentiated spindle cell sarcoma
      • Undifferentiated pleomorphic sarcoma
      • Undifferentiated round cell sarcoma
      • Undifferentiated sarcoma, NOS
Gross images

Contributed by Borislav A. Alexiev, M.D.

Leiomyosarcoma of soft tissue

Dedifferentiated liposarcoma

Microscopic (histologic) images

Contributed by Borislav A. Alexiev, M.D.

Leiomyosarcoma of soft tissue

Atypical
lipomatous tumor /
well differentiated
liposarcoma

Board review style question #1
Which of the following tumor types should be reported / staged using the Protocol for Examination of Resection Specimens from Patients with Soft Tissue Tumors (pTNM, AJCC 8th edition)?

  1. Carcinosarcoma
  2. Extraskeletal myxoid chondrosarcoma
  3. Gastrointestinal stromal tumor
  4. Kaposi sarcoma
  5. Uterine sarcoma
Board review style answer #1
B. Extraskeletal myxoid chondrosarcoma. Answers A, C, D and E are incorrect because the following tumor types should not be reported using this protocol: carcinosarcoma (consider the appropriate site specific carcinoma protocol), gastrointestinal stromal tumor (consider the gastrointestinal stromal tumor protocol), Kaposi sarcoma and uterine sarcoma (consider the uterine sarcoma protocol).

Comment Here

Reference: Soft tissue - Staging
Board review style question #2

Which is the correct pT for a retroperitoneal dedifferentiated liposarcoma that is 12 cm?

  1. pT0
  2. pT1
  3. pT2
  4. pT3
  5. pT4
Board review style answer #2
D. pT3. pT3 is defined as a retroperitoneal tumor that is > 10 cm and ≤ 15 cm in greatest dimension. Answer A is incorrect because pT0 means no evidence of primary tumor. Answer B is incorrect because pT1 is a tumor ≤ 5 cm in greatest dimension. Answer C is incorrect because pT2 is a tumor > 5 cm and ≤ 10 cm in greatest dimension. Answer E is incorrect because pT4 is a tumor > 15 cm in greatest dimension.

Comment Here

Reference: Soft tissue - Staging

Staging-GIST
Definition / general
  • This staging protocol should be used for all resection specimens from patients with gastrointestinal stromal tumor (GIST)
  • This staging protocol is not required for the following procedures: biopsy, local excision, primary resection specimen with no residual tumor (e.g., following neoadjuvant therapy), cytologic specimens
Essential features
  • AJCC 7th edition staging was sunset on December 31, 2017; as of January 1, 2018, use of the 8th edition is mandatory
  • The TNM staging system applies to all GISTs of the esophagus, gastroesophageal junction, stomach, small intestine (duodenum, jejunum, ileum, Meckel diverticulum, small intestine, NOS), appendix, Ileocecal valve, large intestine (cecum, ascending colon, hepatic flexure of colon, transverse colon, descending colon, sigmoid colon, rectosigmoid junction, rectum, large intestine, NOS), retroperitoneum and peritoneum / abdomen
  • See CAP for additional information
Terminology
  • Reporting of pT (tumor), pN (lymph node) and when applicable, pM (metastasis) categories is based on information available to the pathologist at the time the report is issued; as per the AJCC, it is the managing physician's responsibility to establish the final pathologic stage based upon all pertinent information (Amin: AJCC Cancer Staging Manual, 8th Edition, 2018)
  • Tumor (T):
    • Based on the tumor size, as measured grossly or with imaging
    • Largest dimension in any plane
    • Best to provide 3 dimensional size if possible
  • Nodes (N):
    • Lymph node status needs to be determined both clinically and microscopically
  • Metastasis (M):
    • Metastatic (M1)
  • Prefix:
    • If the lesion was previously treated, use the prefix y (ypTNM)
    • If the lesion is recurrent, use the prefix r (rpTNM)
ICD coding
  • ICD-O: 8936/3 - gastrointestinal stromal tumor
  • ICD-11: 2B5B & XH9HQ1 - gastrointestinal stromal tumor, primary site & gastrointestinal stromal sarcoma
Diagrams / tables

Table 1: Guidelines for risk assessment of primary gastrointestinal stromal tumor (GIST)
Tumor parameters
Risk of progressive disease (%)
Mitotic rate Size Stomach Duodenum Jejunum / ileum Rectum


≤ 5 per 5 mm2
≤ 2 cm None None None None
> 2 - ≤ 5 cm Very low Low Low Low
> 5 - ≤ 10 cm Low Insufficient data Moderate Insufficient data
> 10 cm Moderate High High High


> 5 per 5 mm2
≤ 2 cm None Insufficient data High High
> 2 - ≤ 5 cm Moderate High High High
> 5 - ≤ 10 cm High Insufficient data High Insufficient data
> 10 cm High High High High

Primary tumor (pT)
  • pT not assigned (cannot be determined based on available pathological information)
  • pT0: no evidence of primary tumor
  • pT1: tumor ≤ 2 cm
  • pT2: tumor > 2 cm but not > 5 cm
  • pT3: tumor > 5 cm but not > 10 cm
  • pT4: tumor > 10 cm in greatest dimension
Regional lymph nodes (pN)
  • pN not assigned (no nodes submitted or found)
  • pN not assigned (cannot be determined based on available pathological information)
  • pN0: no regional lymph node metastasis
  • pN1: regional lymph node metastasis

Notes:
  • When no lymph nodes are present (as is often the case with resection for GIST), the pathologic N category is not assigned (pNX is not used for GIST) and should not be reported
Distant metastasis (pM)
  • Not applicable: pM cannot be determined from the submitted specimen(s)
  • pM1: distant metastasis
Prefixes
  • Not applicable
  • m: multiple primary synchronous tumors in a single organ
  • y: preoperative radiotherapy or chemotherapy
  • r: recurrent tumor stage
Risk assessment
  • None
  • Very low risk
  • Low risk
  • Moderate risk
  • High risk
  • Overtly malignant / metastatic
  • Cannot be determined: ___________

Notes:
Histologic grade (G)
  • GX: cannot be assessed
  • G1: low grade (mitotic rate ≤ 5 per 5 mm2)
  • G2: high grade (mitotic rate > 5 per 5 mm2)
  • Other (specify): ___________

Notes:
  • Mitotic count should be initiated in an area that reveals the highest level of mitotic activity on screening magnification and then should be performed as consecutive high power fields (HPF)
  • Mitoses should be counted in 5 mm2 of tumor
  • With the use of older model microscopes, 50 HPF is equivalent to 5 mm2
  • Most modern microscopes with wider fields require ~20 - 25 HPF to encompass 5 mm2
  • See CAP for additional information
Histopathologic type
  • Gastrointestinal stromal tumor, spindle cell type
  • Gastrointestinal stromal tumor, epithelioid type
  • Gastrointestinal stromal tumor, mixed
  • Gastrointestinal stromal tumor, other (specify): ___________
Gross images

Contributed by Borislav A. Alexiev, M.D.

Small bowel mass

Macroscopic appearance

Microscopic (histologic) images

Contributed by Borislav A. Alexiev, M.D.

Spindle cell tumor

Regressive vascular changes

Board review style question #1
The gastrointestinal stromal tumor (GIST) staging protocol is required for which of the following procedures?

  1. Biopsy
  2. Cytologic specimen
  3. Local excision
  4. Primary resection specimen with no residual tumor
  5. Resection
Board review style answer #1
E. Resection. The staging protocol should be used for all resection specimens from patients with GIST, except primary resection specimen with no residual tumor.

Comment Here

Reference: Staging-GIST
Board review style question #2

Which is the correct pT for a gastric gastrointestinal stromal tumor (GIST) that is 12 cm?

  1. pT0
  2. pT1
  3. pT2
  4. pT3
  5. pT4
Board review style answer #2
E. pT4. pT4 is defined as a GIST that is > 10 cm in its greatest dimension.

Comment Here

Reference: Staging-GIST

Superficial acral fibromyxoma
Definition / general
  • Benign neoplasm of fibroblastic origin, with tendency for affecting periungual regions of the digits on acral sites
  • Lesions are prone to local recurrence
Essential features
  • Acral region especially around the digits are favored sites
  • Lobulated dermal and subcutaneous spindle cell lesion with myxoid and collagenous stroma
  • Stroma and prominent vessels
  • RB1 gene deletions are observed
  • Surgical excision with adequate margins is treatment of choice
Terminology
  • Acceptable: digital fibromyxoma, superficial acral fibromyxoma
  • Not recommended: cellular digital fibroma
ICD coding
  • ICD-O: 8811/0 - fibromyxoma
  • ICD-11: EE6Y & XH8173 - other specified fibromatous disorders of skin and soft tissue & acral fibromyxoma
Epidemiology
Sites
Pathophysiology
Etiology
  • Unknown
Clinical features
Diagnosis
  • Acral soft tissue lesion affecting digits with or without cortical erosion
  • Typical location around digits should raise clinical suspicion
  • Radiological features are not characteristic
  • Histological examination of the lesion is diagnostic
  • Reference: J Cutan Pathol 2008;35:1020
Radiology description
  • Plain radiograph (Skeletal Radiol 2008;37:499)
    • Well circumscribed soft tissue mass with secondary saucer-like erosion of bone cortex
    • Calcification is absent
    • There may be sclerosis and buttressed margin of cortex in longstanding cases
  • Magnetic resonance imaging (MRI): soft tissue lesion, T2 hyperintense with focal thinning of the cortex (Skeletal Radiol 2008;37:499)
Radiology images

Contributed by Nasir Ud Din, M.B.B.S.
AP Xray distal phalanx

AP Xray of distal phalanx



Images hosted on other servers:

Plain Xray

MRI

MRI

CT and MRI

Computed tomography (CT) and MRI

Prognostic factors
  • Local recurrences developed in 3 of 14 patients (22%) in a study (J Cutan Pathol 2008;35:1020)
  • In a study of 18 patients (mean follow up interval of 10.1 years), there was 1 recurrence after local excision and 2 cases persisted / progressed after partial excision (Hum Pathol 2001;32:704)
  • Local recurrence was seen in 24% of cases in another study (all near the nail unit of the fingers or toes) after a mean interval of 27 months (Am J Surg Pathol 2012;36:789)
    • 1 tumor recurred twice
    • Margins on initial biopsy or subsequent excision had been involved
    • No other clinical or pathologic features were predictive of recurrence / persistence
  • Metastasis has not been reported
Case reports
Treatment
  • Surgical excision with adequate margin is recommended
  • Locally recurrent lesions can undergo re-excision
Clinical images

Contributed by Jose G. Mantilla, M.D.
Well circumscribed periungual mass

Well circumscribed periungual mass



Images hosted on other servers:
Retronychia of toe

Retronychia of toe

Subungual round tumor, green nail plate

Subungual round tumor, green nail plate

Subungual round pinkish tumor

Subungual round pinkish tumor

Subungual tumor distorting nail plate

Subungual tumor distorting nail plate

Dermoscopy

Dermoscopy

Gross description
Gross images

Images hosted on other servers:
Nodules with firm consistency

Nodules with firm consistency

Soft lobulated lesion

Soft lobulated lesion

Nodular pink mass

Nodular pink mass

Microscopic (histologic) description
  • Poorly circumscribed lesion in dermis composed of spindle to stellate shaped fibroblastic cells arranged in a loose storiform / fascicular pattern with alternating myxoid and collagenous stroma (Am J Surg Pathol 2012;36:789)
  • Microvasculature is prominent (Hum Pathol 2001;32:704)
  • Multinucleated stromal cells may be present and mast cells are frequent (Hum Pathol 2001;32:704)
  • Bland nuclei lacking significant mitoses or necrosis; occasionally degenerative atypia may be encountered
  • Infiltration in adipose tissue can occur
  • Cellular variants are also encountered
  • Cartilaginous and osseous metaplasia may be observed
Microscopic (histologic) images

Contributed by Nasir Ud Din, M.B.B.S.
Polypoid acral skin lesion

Polypoid acral skin lesion

Acral skin lesion

Acral skin lesion

Spindle cell lesion

Spindle cell lesion

Giant cells

Giant cells

Storiform pattern

Storiform pattern

Cellular region

Cellular region


CD34 stain

CD34 stain

Cytology description
  • Oval to spindle cells with bland nuclear chromatin with background mucoid / myxoid material
  • Groups and cohesive clusters of spindle cells with scattered thin walled capillaries (J Cytol 2015;32:39)
Cytology images

Images hosted on other servers:
Myxoid matrix with bland spindle cells

Myxoid matrix with bland spindle cells

Positive stains
Negative stains
Electron microscopy description
Molecular / cytogenetics description
  • Deletion of the RB1 gene is detected but not required to establish the diagnosis
Videos

Superficial acral fibromyxoma

Sample pathology report
  • Left big toe, excision biopsy:
    • Acral fibromyxoma (see comment)
    • Comment: There is a well circumscribed dermal based spindle cell neoplasm comprised of collagenous and myxoid areas. The stellate cells are bland, lacking atypia. Mitoses are rare. There is prominent thin vasculature. Scattered mast cells are noticed. Immunohistochemical stains for CD34 and CD99 are positive in spindle cells; features are consistent with superficial acral fibromyxoma, a benign fibroblastic tumor. The tumor is completely excised. Local recurrence is uncommon following complete excision.
Differential diagnosis
  • Onychomatricoma:
    • Rarely affects nail matrix
    • Spindle shaped stromal cells with epithelial component
    • CD34 is positive
  • Myxoid neurofibroma:
    • Spindle cells with wavy nuclei in abundant myxoid stroma
    • Prominent vasculature is lacking
    • S100 and CD34 positive
  • Superficial angiomyxoma:
    • Occurs anywhere in the body, mostly affecting the head and neck region
    • Multilobulated tumor in dermis and subcutis with infiltrative growth
    • Myxoid matrix is abundant
    • Arborizing vessels are prominent at edges
    • Intralesional neutrophils
    • SMA is negative
    • CD34 is negative
  • Acral fibrokeratoma:
    • Polypoid dome shaped
    • Dense collagenous stroma, mature spindle cells and small vessels
    • Vertical orientation of collagen fibers
  • Dermatofibrosarcoma protuberans (DFSP):
    • Affecting trunk and extremities
    • Myxoid DFSP is rare in digits
    • Storiform pattern and deep infiltration in the subcutaneous fat
    • CD34 is positive
  • Fibroma of tendon sheath:
    • Well circumscribed, attached to tendon
    • Sparsely cellular
    • Spindle cells in fibrous stroma
    • Vessels are slit-like
    • SMA is positive
    • CD34 is negative
  • Giant cell tumor of tendon sheath:
    • Lobulated, gray-brown
    • Attached to tendon sheath
    • Spindle, epithelioid and giant cells in fibrous stroma
    • Hemosiderin and foamy cells along with inflammatory cells in varying proportion
    • CD34 is negative
    • CD68 is positive
  • Glomus tumor:
    • Solitary painful mass of digits involving subcutaneous
    • Uniform round to polygonal cells arranged around vessels
    • SMA is positive
    • CD34 is negative
Board review style question #1

A 30 year old woman presented with a slow growing painless subungual mass. Plain Xray revealed uninvolved bone. Magnetic resonance imaging (MRI) confirmed a soft tissue confined lesion without bone erosion. Excision specimen revealed histological features as shown in the given image. Which positive IHC stain helps confirm the diagnosis?

  1. CD34
  2. Desmin
  3. MUC4
  4. S100
  5. SMA
Board review style answer #1
A. CD34. In superficial acral fibromyxoma (SAF), only CD34 will be diffuse and strongly positive. Answer E is incorrect because SMA is only focally positive and is not diagnostic, as it can be positive in many other spindle cell lesions. Answer D is incorrect because S100 is negative in SAF and will exclude nerve sheath tumor. Answer B is incorrect because desmin negativity excludes rhabdoid differentiation. Answer C is incorrect because MUC4 will be negative in SAF, which is a positive marker in low grade fibromyxoid sarcoma.

Comment Here

Reference: Superficial acral fibromyxoma
Board review style question #2
Superficial acral fibromyxoma Superficial acral fibromyxoma


A 30 year old woman presented with a slow growing painless subungual mass. Plain Xray revealed erosion of underlying bone. Magnetic resonance imaging (MRI) confirmed a soft tissue lesion and bone erosion. Excision specimen revealed histological features as shown in the given H&E image. The provided IHC stain is CD34. What is the treatment of choice?

  1. Local excision
  2. Moh's microsurgery
  3. Neoadjuvant chemotherapy
  4. Radiation
  5. Wide excision
Board review style answer #2
A. Local excision. The current diagnosis of superficial acral fibromyxoma warrants only local excision with adequate margins. Answer B is incorrect because Moh's microsurgery is a procedure done for skin cancers, including basal cell carcinoma, squamous cell carcinoma and melanoma at critical sites like eyes, ear, nose and lips where excision margins are sent for frozen section to secure clear margins with minimal tissue defect. Answers C, D and E are incorrect because a benign diagnosis does not merit chemotherapy, radiation or wide excision.

Comment Here

Reference: Superficial acral fibromyxoma

Superficial CD34+ fibroblastic tumor
Definition / general
  • Superficial CD34+ fibroblastic tumor is a distinctive low grade neoplasm of the skin and subcutis, characterized by a fascicular proliferation of spindled cells with abundant, eosinophilic, granular to glassy cytoplasm, marked nuclear pleomorphism and low mitotic count (Mod Pathol 2014;27:294)
  • Invariably express CD34 and are focally immunoreactive for keratin in close to 70% of cases (Mod Pathol 2014;27:294, Histopathology 2017;70:394)
Essential features
Terminology
  • PRDM10 rearranged soft tissue tumor
ICD coding
  • ICD-0: 8810/1 - superficial CD34+ fibroblastic tumor
  • ICD-11: 2B53.Y - other specified fibroblastic and myofibroblastic tumor, primary site
Epidemiology
Sites
Etiology
  • Unknown at this time
Clinical features
Diagnosis
  • Tissue sampling with microscopic examination is the gold standard for a definitive diagnosis
Radiology description
  • Well marginated tumor without calcification in the subcutaneous adipose tissue on CT / MRI scans (Oncol Lett 2017;14:3395)
  • Tumor demonstrates abnormal uptake on 2-(18F) fluoro-2-deoxy-D-glucose (18F-FDG) (PET) (Oncol Lett 2017;14:3395)
Radiology images

Contributed by Borislav Alexiev, M.D.
CT scan

CT scan

Prognostic factors
Case reports
  • 18 year old man with a painless mass in the medial aspect of his right distal thigh (Oncol Lett 2017;14:3395)
  • 48 year old man presented with a gradually increasing soft tissue mass in his right forearm of 2 years' duration (Indian J Pathol Microbiol 2018;61:421)
  • 48 year old woman presented with a painless, slow growing, brown nodule on the left thigh (J Dermatol 2016;43:934)
  • 50 year old woman presented with a four month history of a slowly enlarging, firm, painless mass at the medial aspect of her right knee below unremarkable overlying skin (Diagn Cytopathol 2016;44:926)
  • 55 year old woman with a slowly enlarging mass on the ventral antebrachium (Dermatol Surg 2017;43:1489)
  • 75 year old woman with no significant medical history presented with a firm erythematous asymptomatic nodule on the left posterior thigh (Dermatol Surg 2018;44:313)
Clinical images

Images hosted on other servers:

Thigh mass

Gross description
Gross images

Contributed by Borislav Alexiev, M.D.
Soft tissue mass

Soft tissue mass

Frozen section description
Microscopic (histologic) description
Microscopic (histologic) images

Contributed by Borislav Alexiev, M.D.
Spindle cells

Spindle cells

Chronic inflammation

Chronic inflammation

Fascicular growth pattern

Fascicular growth pattern

Pleomorphic nuclei

Pleomorphic nuclei

CD34

CD34

AE1 / AE3

AE1 / AE3

Cytology description
  • Cellular specimen composed of mostly spindle cells, arranged singly and in clusters in the background of fragments of collagenized stroma (Diagn Cytopathol 2016;44:926)
  • Numerous spindle cells with tapered nuclei and elongated cytoplasmic processes admixed with scattered large pleomorphic cells with enlarged plump nuclei, irregular nuclear contours and bizarre shapes (Diagn Cytopathol 2016;44:926)
Electron microscopy description
Electron microscopy images

Images hosted on other servers:

Lipid globules

Molecular / cytogenetics description
Molecular / cytogenetics images

Images hosted on other servers:

t(2;5)

Sample pathology report
  • Thigh, mass, excision:
    • Superficial CD34+ fibroblastic tumor (see comment)
    • Comment: Microscopic sections show a circumscribed tumor in the dermis and subcutaneous fat composed of spindle cells containing moderate to abundant glassy eosinophilic cytoplasm, vesicular nuclei with distinct nucleoli and intranuclear pseudoinclusions, along with interspersed pleomorphic giant cells. Tumor cells are arranged in fascicular or sheet-like patterns. Neither necrosis nor mitotic activity is identified. Also present is a chronic inflammatory infiltrate of lymphocytes, plasma cells, mast cells and eosinophils.
    • Immunohistochemically, neoplastic cells show diffuse and strong expression of CD34 and focal staining of cytokeratin while are negative for myogenin, S100 and STAT6. INI1 is preserved. Fluorescence in situ hybridization studies for TGFBR3 and MGEA5 are negative.
    • This constellation of morphological, immunohistochemical and molecular features strongly supports the diagnosis of superficial CD34+ fibroblastic tumor. It is a distinctive low grade neoplasm of the skin and subcutis. The majority of patients have no disease recurrence.
Differential diagnosis
Board review style question #1

A 29 year old man presented with a left thigh mass. Hematoxylin eosin stains demonstrate a circumscribed tumor in the dermis and subcutaneous fat composed of spindle cells containing moderate to abundant glassy eosinophilic cytoplasm, vesicular nuclei with distinct nucleoli and intranuclear pseudoinclusions, along with interspersed pleomorphic giant cells. Tumor cells are arranged in fascicular or sheet-like patterns. Neither necrosis nor mitotic activity is identified. Immunohistochemical stains for CD34 and keratin AE1 / AE3 are positive in tumor cells while all of the following are negative: MUC4, SMA, Desmin, S100 and STAT6. INI1 is retained. Which of the following is most likely the correct diagnosis?

  1. Epithelioid sarcoma
  2. Low grade fibromyxoid sarcoma
  3. Myxofibrosarcoma
  4. Solitary fibrous tumor
  5. Superficial CD34+ fibroblastic tumor
Board review style answer #1
E. Superficial CD34+ fibroblastic tumor

Comment Here

Reference: Superficial CD34+ fibroblastic tumor
Board review style question #2
Which of the following is true about superficial CD34+ fibroblastic tumor?

  1. High recurrence rate
  2. INI1 / SMARCB1 expression is lost
  3. Tumor is negative for keratin
  4. Tumor is positive for TGFBR3 / MGEA5 translocation
  5. Tumor is positive for PRDM10
Board review style answer #2
E. Tumor is positive for PRDM10

Comment Here

Reference: Superficial CD34+ fibroblastic tumor

Syndromes
Table of Contents
Definition / general
Definition / general
Bannayan Zonana syndrome:

Beckwith-Wiedemann syndrome:

Blue rubber bleb nevus syndrome:
  • Autosomal dominant or sporadic syndrome of skin or visceral hemangiomas
  • Elevated expression of c-kit has been noted in small venous malformations
  • 14 year old girl with multiple recurrent hemangiomas of skin and GI tract (Korean J Intern Med 2008;23:208)
  • 19 year old man with severe syndrome (Rare Tumors 2010;2:e36)


Carney syndrome / complex:
  • Autosomal dominant multiple neoplasia syndrome with myxomas and pigmented lesions of skin and mucosa, due to mutation in PRKAR1A gene (OMIM #160980) or 2p16 abnormalities (OMIM #605244)


Carney triad:

Gardner syndrome:
  • Fibromatosis, familial adenomatous polyposis and osteomas
  • OMIM #175100


Kasabach-Merritt syndrome:
  • Microangiopathic hemolytic anemia associated with childhood hemangioma or kaposiform hemangioendothelioma
  • OMIM #141000


Klippel-Trenaunay-Weber syndrome:
  • Hemangiomas with hypertrophy of associated bone and soft tissue
  • OMIM #149000


Launois-Bensaude syndrome:
  • Painless symmetrical diffuse deposits of fat beneath the skin of the neck, upper trunk, arms and legs
  • Also known as multiple symmetrical lipomatosis, cephalothoracic lipodystrophy and Madelung disease


Li Fraumeni syndrome:
  • Various soft tissue sarcomas, osteosarcoma, breast cancer, brain tumors, leukemia and adrenocortical carcinoma
  • Usually due to p53 mutation
  • OMIM #151623


Lipomatosis, multiple syndrome:

Maffucci syndrome:

Mazabraud's syndrome:

McCune-Albright syndrome:
  • Major features are polyostotic fibrous dysplasia, cafe-au-lait skin pigmentation and precocious puberty
  • Myxomas are minor feature (see Mazabrauds syndrome)
  • OMIM #174800


Multiple endocrine neoplasia 1:

Neurofibromatosis type 1:

Neurofibromatosis type 2:

Osler-Weber-Rendu syndrome:

Proteus syndrome:
  • Various cutaneous and subcutaneous lesions, including vascular malformations, lipomas, hyperpigmentation and nevi (eMedicine)


Stewart-Treves syndrome:
  • Cutaneous angiosarcoma that develops in long-standing chronic lymphedema (eMedicine)


Sturge-Weber syndrome:

Turner syndrome:
  • 45 X0, associated with cystic hygroma (eMedicine), rarely familial desmoids or GIST


Sarcomas associated with hereditary nonpolyposis colorectal cancer:

Synovial sarcoma
Definition / general
  • Malignant soft tissue tumor of uncertain histogenesis with variable epithelial differentiation
Essential features
  • Can occur in or around any tissue in the body
  • Multiple morphologies, including monophasic spindle cell, biphasic, poorly differentiated, myxoid, ossifying and monophasic epithelial
  • Monotonous spindle cells with vesicular, plump and overlapping nuclei with hemangiopericytic vessels
  • TLE1+, cytokeratin+, EMA+, BCL2+, CD99+
  • Has a characteristic chromosomal translocation t(X;18)(p11;q11) involving genes SS18 and either SSX1, SSX2 or SSX4
Terminology
  • Obsolete terms: malignant synovioma
ICD coding
  • ICD-O: 9040/3 - synovial sarcoma, NOS
  • ICD-11: 2B5A.Y & XH9B22 - synovial sarcoma, other specified primary site & synovial sarcoma, NOS
Epidemiology
  • Accounts for 5 - 10% of all soft tissue sarcomas
  • Median age: 35
  • Age range: 0 - 85+
  • Slight male predominance (M:F = 1.2:1)
  • 17.6% of cases occur in children and young adults ages 0 - 19 (Pediatr Blood Cancer 2011;57:943)
Sites
  • Can occur anywhere in the body
  • Primary site distribution (Pediatr Blood Cancer 2011;57:943):
    • Extremities: 68.7%
    • Trunk: 15.7%
    • Head and neck: 6.3%
    • Intrathoracic: 5.3%
    • Intra-abdominal: 1.8%
    • Other: 2.2%
Pathophysiology
  • Driven by the chromosomal translocation t(X;18)(p11;q11) involving genes SS18 and either SSX1, SSX2 or SSX4
  • Translocation has multiple effects on oncogenetic pathways, including the SWI / SNF chromatin remodeling complex, polycomb repressor complex and canonical Wnt pathway (Cancer Discov 2015;5:124)
  • Translocation partner affects epithelial differentiation (Int J Clin Exp Pathol 2013;6:2272):
    • SS18-SSX1: 60 - 70% monophasic, 30 - 40% biphasic
    • SS18-SSX2: 97% monophasic, 3% biphasic
    • SS18-SSX1 inhibits Snail gene while SS18-SSX2 inhibits Slug gene (OMIM: Snail Family Transcriptional Repressor 1; SNAI1 [Accessed 9 July 2021])
    • Interfering with Snail leads to stronger derepression of E-cadherin than does interfering with Slug
    • Stronger E-cadherin and increased extracellular matrix protein MMP2 expression are needed for biphasic morphology to develop
Etiology
  • Despite the name, the cells of origin are not synovial cells
  • Histogenesis is still debated
  • 1 study has shown stem cell marker expression, suggesting that origin may be a multipotent mesenchymal stem cell (Stem Cells 2010;28:1119)
  • 1 study using mice identified satellite cells (immature myoblasts) as a potential source (Cancer Cell 2007;11:375)
Clinical features
  • Very rarely associated with prior radiation therapy (Mod Pathol 2002;15:998)
  • Not classically known to be associated with any syndromes
Diagnosis
Radiology description
  • Most present as round to oval, lobulated masses
  • Bone involvement is rare
  • Ossifying synovial sarcomas have characteristic spotty radiopacities caused by focal calcifications (Pathol Res Pract 2009;205:195)
Radiology images

Contributed by Farres Obeidin, M.D.

Elbow mass

Shoulder mass

Arm mass

Prognostic factors
Case reports
Treatment
  • Primarily surgical
  • Radiotherapy and adjuvant chemotherapy have been evaluated in high risk situations (tumors > 5 cm or difficult to resect)
  • Radiation therapy: adjuvant use has shown improvement in oncologic outcome and overall survival (J Surg Oncol 2015;111:158)
  • Adjuvant chemotherapy:
    • Data is conflicting on its use but can be considered in high risk patients
    • Ifosfamide was shown to increase disease specific survival (Ann Surg 2007;246:105)
    • 2 large European Organization for Research and Treatment of Cancer (EORTC) database clinical trials showed no benefit of doxorubicin on overall survival (Ann Oncol 2014;25:2425)
  • Novel therapies:
    • Initial PDL1 and CTLA4 studies have shown no benefit (Sarcoma 2013;2013:168145)
    • Receptor tyrosine kinase inhibitor pazopanib has a promising trend toward better overall survival
    • Tazemetostat, an EZH2 inhibitor, has also been studied in INI1 deficient tumors, including synovial sarcoma (Curr Treat Options Oncol 2018;19:13)
    • Early studies on T cell receptor based immunotherapy directed towards NY-ESO-1 in HLA-A*0201+ patients have demonstrated radiological and clinical benefit (J Clin Oncol 2011;29:917)
Gross description
  • Tumor is well circumscribed and frequently multinodular
  • Most tumors are 3 - 10 cm in greatest dimension at time of diagnosis
  • Minute lesions (< 1 cm) occur, especially in hands and feet (Am J Surg Pathol 2006;30:721)
  • Cut surface may be tan or grey, yellow or pink and soft or firm (Pathol Annu 1980;15:309)
  • Myxoid change, necrosis, calcifications and metaplastic ossification may be present
Gross images

Contributed by Farres Obeidin, M.D.

Arm mass

Shoulder mass

Lung mass

Elbow mass



Images hosted on other servers:
Missing Image

Tumor within the vastus intermedius muscle

Microscopic (histologic) description
  • General (Ann Diagn Pathol 2014;18:369):
    • 2 main subtypes: biphasic and monophasic spindle cell
    • Rarer subtypes: poorly differentiated (round cell), monophasic epithelial, calcifying / ossifying and myxoid
  • Biphasic:
    • Architecture:
      • 2 components: spindle cells and gland-like epithelial structures
      • Glandular lumina contain mucin
    • Cytologic features:
      • Epithelial component has moderate, distinct amphophilic cytoplasm with round to ovoid nuclei
      • Rarely, squamous metaplasia can occur
  • Monophasic:
    • Architecture:
      • Infiltrative borders
      • Hypercellular fascicular architecture with little intervening stroma
      • Can rarely show hyalinization or myxoid change
      • Ill defined nuclear palisading may be seen
    • Cytologic features:
      • Monotonous cells with scant amphophilic cytoplasm, ovoid to spindled vesicular nuclei with evenly dispersed chromatin and inconspicuous nucleoli
      • Nuclei often close enough to overlap with adjacent nuclei
  • Poorly differentiated: highly cellular round cells with hyperchromatic nuclei and frequent mitotic activity and necrosis
  • Additional features:
    • Characteristically features focal staghorn (or hemangiopericytic), branching vascular pattern, reminiscent of solitary fibrous tumor
    • Mast cells are common
    • Focal calcification can be seen in 33% of cases
Microscopic (histologic) images

Contributed by Farres Obeidin, M.D.

Monophasic synovial sarcoma

Hemangiopericytic vessel

Mitotic activity


Focal palisading

Posttreatment

Biphasic synovial sarcoma


Patchy
AE1 / AE3+

Weak, patchy EMA+

BCL2+

CD99+

S100+

TLE1+



Contributed by Debra Zynger, M.D.

Monophasic synovial sarcoma

Monophasic synovial sarcoma, TLE1+

Positive stains
Negative stains
Electron microscopy description
  • Glandular formation of epithelioid tumor cells with sparse luminal microvilli
Electron microscopy images

Contributed by Mark R. Wick, M.D.

Biphasic, glandular cells

Biphasic, spindle cells

Molecular / cytogenetics description
Molecular / cytogenetics images

Contributed by Mark R. Wick, M.D.

X18, in situ hybridization

Translocation in synovial sarcoma

Sample pathology report
[insert representative image here]

  • Right lower extremity, biopsy:
    • Synovial sarcoma, monophasic, FNCLCC grade 2 (see comment)
    • Comment: The tumor is composed of fascicles of bland spindle cells with sparse cytoplasm and relatively uniform, ovoid hyperchromatic nuclei and inconspicuous nucleoli. The stroma contains strands of wiry collagen. Mitoses are sparse (2 mitoses/10 high power fields). No tumor necrosis is identified. Immunohistochemically, the tumor cells are immunoreactive for CD99, EMA (subset) and TLE1, while all of the following are negative: h-caldesmon, myogenin, MyoD1, STAT6, SOX10 and WT1 (C terminus). The findings support the above diagnosis. Synovial sarcoma has a variable prognosis. Major prognostic determinants are tumor stage at presentation, tumor size and FNCLCC tumor grade.
Differential diagnosis
Board review style question #1

    Which of the following is true about the lesion pictured above?

  1. Commonly has PAX3 rearrangements
  2. It is associated with HHV8 infection
  3. It is negative for cytokeratin
  4. It occurs primarily in elderly patients
  5. SS18-SSX fusion is the characteristic chromosomal abnormality
Board review style answer #1
E. Synovial sarcoma has fusion of SS18 with SSX1, SSX2 or SSX4. It typically occurs in younger patients, with the peak incidence in the third to fourth decade and in most cases shows at least focal cytokeratin expression. It is not associated with PAX3 rearrangements, a typical finding in alveolar rhabdomyosarcoma and biphenotypic sinonasal sarcoma. HHV8 is seen in Kaposi sarcoma.

Comment Here

Reference: Synovial sarcoma
Board review style question #2
    Which morphologic feature is commonly seen in synovial sarcoma?

  1. Chicken wire calcifications
  2. Extravasated red blood cells
  3. Hemangiopericytic vessels
  4. Nuclear pleomorphism
  5. Prominent storiform pattern
Board review style answer #2
C. Hemangiopericytic vessels and monotonous nuclei are commonly seen in synovial sarcoma. Chicken wire calcifications are typically seen with chondroblastoma. Extravasated red blood cells are common in nodular fasciitis, fibromatosis and Kaposi sarcoma. A storiform pattern is characteristic of dermatofibrosarcoma protuberans.

Comment Here

Reference: Synovial sarcoma

Tenosynovial giant cell tumor
Definition / general
  • Tenosynovial giant cell tumor encompasses a group of lesions that most often arise from the synovium of joints, bursae and tendon sheaths and show synovial differentiation
  • Malignant tenosynovial giant cell tumor is very uncommon and is defined by the coexistence of a benign tenosynovial giant cell tumor with overtly malignant areas or by recurrence of a typical giant cell tumor as a sarcoma
Essential features
  • Localized type
    • Second most common tumor of the hand (after ganglion cyst)
    • Well circumscribed and lobulated, usually in close association with a tendon
    • Composed of variable proportions of large epithelioid mononuclear cells, macrophages and osteoclast-like giant cells
  • Diffuse type
    • Intra-articular tumors within large joints
    • Extra-articular invasive tumors of tendon sheath, bursa or soft tissue origin
  • Malignant
    • Only ~50 cases were reported
    • Contains definite sarcomatous area
    • Arises de novo or occurs after multiple recurrences of a conventional tenosynovial giant cell tumor
Terminology
  • Acceptable: giant cell tumor of tendon sheath
  • Not recommended: pigmented villonodular synovitis
ICD coding
  • ICD-O: 9252/0 - tenosynovial giant cell tumor, NOS
  • ICD-10: D48.1 - neoplasm of uncertain behavior of connective and other soft tissue
  • ICD-11:
    • 2F7Z & XH6911 - neoplasms of uncertain behavior of unspecified site & tenosynovial giant cell tumor, localized
    • 2F7Z & XH52J9 - neoplasms of uncertain behavior of unspecified site & tenosynovial giant cell tumor, diffuse
    • 2D4Y & XH5AQ9 - other specified malignant neoplasms of ill defined or unspecified primary sites & malignant tenosynovial giant cell tumor
Epidemiology
  • Localized type
    • 40 patients/1 million
    • Fourth or fifth decade of life (most frequently)
    • F:M = 2:1
  • Diffuse type (NIH: SEER Cancer Statistics Review, 1975-2010 [Accessed 24 November 2021])
    • Annual incidence rate: 1.8 patients/1 million in United States
    • Peaks in the third and fourth decades of life with an average of 35 years
    • Female predominance
  • Malignant
    • Uncommon
    • Middle aged to older adults (most are 50 - 60 years)
Sites
  • Localized type
    • Predominantly occurs in the digits (85%), especially fingers (75%)
    • Uncommon around large joints (10%)
      • Intra-articular localized tenosynovial giant cell tumors are most frequent in the knee
  • Diffuse type (Lancet Oncol 2019;20:877)
    • Intra-articular
      • Knee (65%)
      • Hip and ankle (25%)
      • Elbow, shoulder, temporomandibular joint, spine
    • Extra-articular extension
      • Knee
      • Foot, wrist, inguinal, elbow region, digits
  • Malignant
    • Often occurs in the lower extremities
Pathophysiology
  • Translocation of the CSF gene that encodes colony stimulating factor 1 (CSF1) was frequently involved in this tumor
  • It is reported that tumor cells with CSF1 translocation synthesize large amounts of the CSF1 protein
  • Actually, only a small subset of cells harbor the translocation
Etiology
  • Unknown
Clinical features
  • Localized type (Orthop Traumatol Surg Res 2017;103:S91)
    • Skin colored nodule without specific features
    • Most are slow growing and painless
  • Diffuse type (J Rheumatol 2017;44:1476)
    • Painful mass, long duration
    • Hemorrhagic joint effusion
    • Decreased range of motion
  • Malignant (Mod Pathol 2019;32:242)
    • Similar to their benign counterparts in tumor location
    • Recurrence or metastasis to regional lymph nodes and distant locations
Diagnosis
Laboratory
  • Nonspecific
Radiology description
Radiology images

Contributed by Jigang Wang, M.D., Ph.D. and Jiufa Cui, M.D., Ph.D.

Diffuse type
Bone erosion on CT Bone erosion on CT

Bone erosion on CT

Low signal on MR Low signal on MR

Low signal on MR


Low signal on MR

Low signal on MR

Low signal on MR Low signal on MR

Low signal on MR



Images hosted on other servers:

Localized type
Mass on the tendon

Mass on the tendon

Missing Image

Ultrasound of finger lesion

Missing Image

Hand MRI


Diffuse type

MRI of the ankle


Malignant
Progressive increasing mass

Progressive increasing mass

Prognostic factors
  • Localized type
    • Benign but recurs locally (10 - 20%)
    • Risk factors for recurrence include degenerative joint disease and osseous erosion
  • Diffuse type (Eur J Cancer 2015;51:210)
    • High recurrence rate
    • Risk factors for recurrence are still unknown
    • It is very rare that tumors progress to malignancy or metastasize
  • Malignant
    • Risk factors for recurrence are still unknown
Case reports
Treatment
  • Localized type
    • Complete local excision
  • Diffuse type (Eur J Cancer 2016;63:34)
    • Surgical excision
      • Wide local excision
      • Total synovectomy
      • Prosthetic joint replacement
      • Amputation in advanced cases
    • External beam radiotherapy
    • Radiosynovectomy
    • New therapeutic strategies
      • Immunotherapy: anti-TNF-α drugs
      • Targeted therapy: tyrosine kinase inhibitors, CSF inhibitors
  • Malignant
    • Local treatment
      • Radical resection
    • Systemic therapy
      • Tyrosine kinase inhibitors (TKIs) had very limited activity in malignant tenosynovial giant cell tumor (BMC Cancer 2018;18:1296)
      • CSF1R inhibitors
      • Cytotoxic chemotherapy
Clinical images

Contributed by Mark R. Wick, M.D.

Localized type

Digit lesion



Images hosted on other servers:

Localized type
Missing Image

Mass over lateral right foot

Missing Image

Tumor at surgery

Neurovascular bundle involvement

Neurovascular bundle involvement

Swelling appearance

Swelling appearance

Lateral route

Lateral route


Diffuse type
Brownish mass

Brownish mass

Gross description
  • Localized type
    • Well circumscribed and partially encapsulated with a lobulated appearance
    • Variegated cut surface (yellow, tan, red-brown)
  • Diffuse type
    • Brown-yellow spongy tissue, firm and nodular, often 5 cm or larger
    • Diffusely covers most of synovial surface
    • Villous, nodular or villonodular
    • Poorly demarcated from adjacent tissues
  • Malignant
    • Similar to benign but may be larger
    • Sarcomatous area showed a solid growth pattern with whitish color, massive hemorrhage and necrosis (Pathol Int 2012;62:559)
Gross images

Contributed by Mark R. Wick, M.D.

Localized type
Missing Image

Cut surface



Images hosted on other servers:

Diffuse type
Tumor mass

Tumor mass

Microscopic (histologic) description
  • Composed of mononuclear cells, multinucleated giant cells, foamy macrophages, inflammatory cells and hemosiderin
  • 2 principal cell types of the mononuclear components:
    • Small histiocyte-like cells: pale cytoplasm and round or reniform nuclei
    • Large epithelioid cells: amphophilic cytoplasm and rounded vesicular nuclei, often containing a peripheral rim of hemosiderin granules
  • Mitotic activity may be brisk
  • Necrosis can be present
  • Localized type (J Surg Oncol 1998;68:100)
    • Lobulated and well circumscribed
    • Osteoclast-like giant cells are usually readily apparent
    • Xanthoma cells are frequent, tend to aggregate locally near the periphery of nodules, and may be associated with cholesterol clefts
    • Hemosiderin deposits
    • Stroma showed variable degrees of hyalinization
  • Diffuse type (J Surg Oncol 1998;68:100, J Oral Maxillofac Surg 2019;77:1022.e1)
    • Infiltrative, and grow as diffuse, expansile sheets
    • Osteoclast-like giant cells are less common in the diffuse form than the localized form and may be absent or extremely rare in as many as 20% of cases
    • Cleft-like spaces are common and appear either as artifactual tears or as synovial lined spaces
    • Stromal hyalinization is common and may mimic osteoid
    • Blood filled pseudoalveolar spaces are seen in approximately 10% of cases
  • Malignant (Hum Pathol 2017;63:144, Am J Surg Pathol 1997;21:153)
    • Composed of sheets and nodules of enlarged mononuclear cells
    • Significantly increased mitotic count, including atypical mitoses, necrosis, enlarged nuclei with nucleoli, spindling of mononucleated cells and myxoid changes
    • May contain areas that resemble undifferentiated pleomorphic sarcoma or myxofibrosarcoma
Microscopic (histologic) images

Contributed by Jigang Wang, M.D., Ph.D., Jiufa Cui, M.D., Ph.D. and Michella Whisman, M.D.

Localized type
Missing Image

Fibrous bands

Missing Image

Lobular appearance

Large epithelioid cells Large epithelioid cells

Large epithelioid cells

Missing Image

Multinucleated giant cells

Missing Image

Pigment laden histiocytes


Missing Image

Foamy histiocytes

Foamy histiocytes Missing Image

Foamy histiocytes

Missing Image Missing Image

Mononuclear component



Diffuse type
Foamy histiocytes (xanthoma cells)

Foamy histiocytes (xanthoma cells)

Mononuclear cells

Mononuclear cells

Large mononuclear cells

Large mononuclear cells

Chondroid metaplasia

Chondroid metaplasia

Papillary growth pattern

Papillary growth pattern

Numerous mononuclear cells

Numerous mononuclear cells

Cytology description
  • Localized type (Rare Tumors 2015;7:5814)
    • Moderately cellular smears with a variable admixture of cell populations:
      • Polygonal to spindled mononuclear cells in loose clusters or individually dispersed with moderate cytoplasm and round to oval nuclei
      • Scattered osteoclast-like multinucleated giant cells
      • Foamy histiocytes
      • Hemosiderin laden histiocytes
    • Relatively scant background inflammation in most cases
  • Diffuse type (Acta Cytol 2017;61:160)
    • Heterogeneous cell populations:
      • Large epithelioid mononuclear cells
        • Abundant eosinophilic cytoplasm, usually containing ring-like hemosiderin pigment
        • Vesicular, round to oval, eccentric nuclei with prominent nucleoli
      • Macrophages with smaller, oval or reniform nuclei
      • Osteoclast-like giant cells
    • Frequent mitotic figures
    • Chondroid metaplasia is common in the temporomandibular joint
  • Malignant (J Cytol 2017;34:174)
Cytology images

Images hosted on other servers:

Localized type

Mononuclear stromal cells, osteoclast giant cells


Malignant
Mitosis

Mitosis

Immunofluorescence description
  • Unknown at this time
Positive stains
Negative stains
Electron microscopy description
Molecular / cytogenetics description
Molecular / cytogenetics images

Images hosted on other servers:

Diffuse type
Structural chromosomal aberration

Structural chromosomal aberration

Sample pathology report
  • Right finger, index, excision:
    • Tenosynovial giant cell tumor, localized type (giant cell tumor of tendon sheath), 1.6 cm (see comment)
    • Comment: Surgery is the main treatment. Localized type rarely recurs after complete removal while diffuse type often recurs.

  • Right hip, index, excision:
    • Tenosynovial giant cell tumor, diffuse type (pigmented villonodular synovitis)

  • Right hip, excision:
    • Malignant diffuse tenosynovial giant cell tumor
Differential diagnosis
  • Localized type
    • Diffuse type tenosynovial giant cell tumor:
      • Essentially identical appearance to localized form at high magnification
      • Distinguished from localized form by large size, infiltrative growth or anatomic site (often intra-articular and in larger joints)
      • Villonodular architecture when intra-articular
    • Giant cell tumor of soft tissue:
      • More uniform background of mononuclear cells
      • Typically have sheets of osteoclastic giant cells similar to giant cell tumor of bone
      • Less association with a tendon
    • Fibroma of tendon sheath:
      • Slit-like vascular spaces at the periphery
      • Cracking artifact in collagenous background
      • Spindled to stellate fibroblasts / myofibroblasts
      • Lacks giant cells, large histiocytoid cells with eccentric nucleus, hemosiderin laden histiocytes and foamy histiocytes
  • Diffuse type
    • Localized type tenosynovial giant cell tumor:
      • Well circumscribed and encapsulated
      • Lacks villi
      • Localized to digits, uncommon in large joint
    • Hemarthrosis:
      • Villiform synovial hyperplasia with hemosiderosis
      • Solid cellular areas are rare
      • History is important: repetitive hemarthrosis (e.g., trauma or hemophilia)
    • Malignant tenosynovial giant cell tumor:
      • Often with past history of radiation therapy
      • Pleomorphic spindle cells and high mitotic rate and necrosis
      • Epithelioid cells with prominent nucleoli
  • Malignant
    • Benign diffuse tenosynovial giant cell tumor:
      • Lacks sarcomatous area
    • Other soft tissue sarcoma:
      • Lacks typical area of diffuse tenosynovial giant cell tumor
      • Lacks medical history of diffuse tenosynovial giant cell tumor
  • Dermatofibroma:
    • Dermal lesions
    • Rare in digits
    • Lacks large epithelioid cells and osteoclast-like giant cells
    • Lacks CSF1 rearrangement
  • Plexiform fibrohistiocytic tumor:
    • Usually involves dermis and subcutis
    • Nodules or clusters are interconnected in characteristic plexiform arrangement
    • Plump fibroblastic cells and histiocyte-like cells within a finely granular myxoid background
    • Large epithelioid mononulear cells are absent
Board review style question #1
Which of the following features must be identified to the make the diagnosis of tenosynovial giant cell tumor?

  1. Epithelioid mononuclear cells
  2. Foamy macrophages
  3. Hemosiderin laden macrophages
  4. Inflammatory cells
  5. Multinucleated giant cells
Board review style answer #1
A. Epithelioid mononuclear cells. The lesion is a tenosynovial giant cell tumor, localized type (also called giant cell tumor of tendon sheath). Although all the listed cell types can be seen in varying proportions within the tumor, the histiocytoid mononuclear cells are the neoplastic component and should always be present.

Comment Here

Reference: Tenosynovial giant cell tumor
Board review style question #2
Which of the following is true regarding tenosynovial giant cell tumor, localized type?

  1. Both genders are affected equally
  2. Diagnosis can be made even in the absence of osteoclast-like giant cells
  3. Highly infiltrative lesion, which leads to recurrence in the majority of cases
  4. It is the most common tumor of the hand
  5. The patient always reports a history of trauma
Board review style answer #2
B. The diagnosis can be made even in the absence of osteoclast-like giant cells. Some cases show a paucity of giant cells, which is why it is best to know the constellation of histologic features aside from the presence of giant cells in order to make the diagnosis in giant cell poor cases. Tenosynovial giant cell tumor, localized type, is the second most common tumor of the hand (ganglion cyst is most common). It shows a predilection for females (F:M = 2:1). The tumors are usually well circumscribed. They may recur but simple excision is curative most of the time. Some patients report a history of trauma but not all.

Comment Here

Reference: Tenosynovial giant cell tumor
Board review style question #3
Which of the following is true regarding the location of diffuse tenosynovial giant cell tumor?

  1. Knee is the most frequent, followed by hip and ankle
  2. Often in foot and hand
  3. Often occurs in small joint
  4. Only intra-articular
Board review style answer #3
A. Knee is the most frequent, followed by hip and ankle. Diffuse tenosynovial giant cell tumor often involves large joint and can be extra-articular. Localized tenosynovial giant cell tumor often occurs in small joint, such as foot and hand.

Comment Here

Reference: Tenosynovial giant cell tumor
Board review style question #4
BRQ image BRQ image BRQ image


A 15 year old girl presented with finger mass for 1 year. The tumor (shown above) was well defined, attached to the tendon and was approximately 1 cm in diameter. What is your diagnosis?

  1. Chondroblastoma
  2. Dermatofibroma
  3. Fibroma of tendon sheath
  4. Giant cell tumor
  5. Tenosynovial giant cell tumor
Board review style answer #4
E. Tenosynovial giant cell tumor. The pictured lesion is a tenosynovial giant cell tumor, localized type (also called giant cell tumor of tendon sheath). The tumor usually occurs in small joint and is attached to tendon sheath. Microscopically, the tumor was a giant cell rich tumor with prominent epithelioid mononuclear cells that can be observed.

Comment Here

Reference: Tenosynovial giant cell tumor
Board review style question #5
Which of the following is true regarding malignant tenosynovial giant cell tumor?

  1. Malignant tumor from the location around large joint
  2. May have a previous history or typical morphological area of tenosynovial giant cell tumor
  3. Often occurs in upper extremities
  4. Usually affects young people
Board review style answer #5
B. May have a previous history or typical morphological area of tenosynovial giant cell tumor

Comment Here

Reference: Tenosynovial giant cell tumor

Undifferentiated / unclassified sarcoma
Definition / general
  • Undifferentiated sarcoma shows no identifiable line of differentiation when analyzed by presently available technology (WHO 5th edition) (Surg Oncol Clin N Am 2022;31:321)
  • Not included are dedifferentiated subtypes of specific soft tissue sarcomas, such as dedifferentiated liposarcoma, in which the high grade component is commonly undifferentiated
Essential features
  • Spindle cell, pleomorphic, epithelioid and round cells (WHO 5th edition)
  • Most often high grade morphology
  • Absence of any morphological and immunohistochemical features of specific differentiation
  • Absence of distinctive molecular aberration
ICD coding
  • ICD-O: 8805/3 - undifferentiated sarcoma
  • ICD-11
    • 2B5F.2 & XH73J4 - sarcoma, not elsewhere classified of other specific sites & giant cell sarcoma
    • 2B5F.2 & XH0947 - sarcoma, not elsewhere classified of other specific sites & malignant fibrous histiocytoma
    • 2B5F.2 & XH6HY6 - sarcoma, not elsewhere classified of other specific sites & undifferentiated sarcoma
    • 2B5F.2 & XH85G7 - sarcoma, not elsewhere classified of other specific sites & small cell sarcoma
Epidemiology
  • Accounts for as many as 20% of all soft tissue sarcomas
  • Tumor occurs most frequently in the soft tissues of extremities (50% in lower limbs, 20% in upper limbs) and occasionally in bone and viscera (WHO 5th edition) (Cancer Control 2021;28:10732748211036775)
  • Peaks around the age of 60 years and is more common in men
  • Tumors with pleomorphic morphology (undifferentiated pleomorphic sarcoma) occur mostly in older adults
Sites
  • May be found in any location (WHO 5th edition)
  • Most common in the somatic soft tissue
Pathophysiology
  • Genetic alterations, such as mutations, deletions, epigenetic modifications, may be important for undifferentiated sarcoma development and progression; although they are nonspecific
  • Recent investigation had classified TP53, ATRX, H3F3A, ZFHX3, CSMD3, PRPRT, TRIO, CLTC, PDGFRB, ALK, PTCH1, RET, ERBB4, JAK3, GATA1, PIK3CG, RARA and MYH9 as cancer driver genes (Front Genet 2023;14:1109491, J Pathol 2019;247:166)
  • Numerical and structural variants underlying chromosomal aberrations are frequently observed in undifferentiated sarcoma, indicating the critical role of high chromosomal instability in the sarcomagenesis and progression (Front Genet 2023;14:1109491)
  • Losses of 13q12-q14 or 13q21 were observed in a large proportion of tumors, suggesting that a gene localized in this region could act as a tumor suppressor gene (Cancer Genet Cytogenet 1999;111:134)
Etiology
  • Exact pathogenic mechanisms of undifferentiated sarcoma remain obscure
  • Hedgehog signaling pathway plays a role in the proliferation and malignancy (Cancer Res 2012;72:1013)
  • Hippo pathway may also be implicated in undifferentiated sarcoma tumor biology, as vestigial-like family member 3 (VGLL3) and YES1 associated transcriptional regulator (YAP1) cofactors were found to be highly amplified on a genome sequencing study (Genes Chromosomes Cancer 2010;49:1161)
  • Radiation therapy associated undifferentiated sarcomas (Ann Surg Oncol 2015;22:3913)
  • PRDM10 fusions are present in ~5% of undifferentiated sarcoma; all tumors were morphologically low grade and none of the patients developed metastases
    • PRDM10 fusion positive sarcomas may constitute a clinically important subset of undifferentiated sarcoma (Clin Cancer Res 2015;21:864)
Diagrams / tables

Images hosted on other servers:
Potential pathways for sarcomagenesis

Potential pathways for sarcomagenesis

Role of Skp2 and p16

Role of Skp2 and p16

Confluent network of pathways

Confluent network of pathways

Potential involvement of pathways

Potential involvement of pathways

Clinical features
Diagnosis
  • Diagnosis of exclusion (WHO 5th edition)
  • Atypical spindle cell, pleomorphic, epithelioid and round cells
  • High grade morphology in the majority of cases
  • Absence of any morphological and immunohistochemical features of specific differentiation
  • Complex and not specific cytogenetic abnormalities
  • Undifferentiated round cell sarcoma can be diagnosed only if distinctive gene fusions have been ruled out; they are most frequent in young patients
Radiology description
Radiology images

Contributed by Borislav A. Alexiev, M.D.
T1 isointense mass

T1 isointense mass

T2 hyperintense mass

T2 hyperintense mass

Prognostic factors
  • Deep tumor location and AJCC stage are the most important predictive prognostic factors (Pathol Int 2002;52:595)
  • Other factors that indicate a poorer prognosis include inadequate surgical margin around the tumor, metastatic spread, large tumor size and older age
  • Vast majority of undifferentiated sarcomas are high grade lesions, with a local recurrence rate of 19 - 31%, a metastatic rate of 31 - 35% and a 5 year survival of 65 - 70% (Mod Pathol 2014;27:S39)
  • Only a minority of patients develop metastases after 5 years, with the common metastatic sites being lung (90%), bone (8%) and liver (1%); regional lymph node metastases are decidedly uncommon (Mod Pathol 2014;27:S39)
  • Undifferentiated sarcoma arising in the limbs or trunk has a reported 5 year metastasis free survival rate of 83% (J Clin Oncol 2001;19:3045)
  • Undifferentiated sarcoma with epithelioid morphology seems to be more aggressive; in a recent study, the 5 and 10 year disease specific survival rates were 43% and 42%, respectively (Oncologist 2011;16:512)
Case reports
Treatment
  • Surgery in combination with radiotherapy (sporadically combined with chemotherapy in case of high risk of development of metastasis) is the common therapy of choice for undifferentiated sarcoma, which is similar to the treatment of other soft tissue sarcomas (Eur J Surg Oncol 2022;48:985)
  • Standard treatment option for patients who present with unresectable soft tissue sarcomas is use of chemotherapeutic agents (Cancer 2023;129:3417)
Clinical images

Images hosted on other servers
Lower back mass

Lower back mass

Gross description
Gross images

Contributed by Borislav A. Alexiev, M.D.
Ulcerated mass

Ulcerated mass

Intramuscular mass

Intramuscular mass

Necrotic mass

Necrotic mass

Frozen section description
Microscopic (histologic) description
Microscopic (histologic) images

Contributed by Borislav A. Alexiev, M.D., Nikhil Sangle, M.D. (Case #387), AFIP and @JMGardnerMD on Twitter
Fascicular growth pattern

Fascicular growth pattern

Spindle cells

Spindle cells

Epithelioid cells

Epithelioid cells

Pleomorphic cells

Pleomorphic cells

Bizarre multinucleated tumor cells

Bizarre multinucleated tumor cells


High mitotic activity

High mitotic activity

Bizarre nuclei

Abundant eosinophilic cytoplasm

Characteristic giant cells

Mixture of fibroblasts and histiocyte-like cells


Vague storiform pattern

Strap-like cells

Numerous mitotic figures

Frequent tumor cell necrosis

Acute inflammatory and histiocyte-like cells


Osteoclast-like giant cells with uniform nuclei in undifferentiated pleomorphic sarcoma

Contributed by @JMGardnerMD on Twitter (see original post here)"> Undifferentiated / unclassified sarcoma

Undifferentiated / unclassified sarcoma

CD10+

CD10+

CD68+

CD68+


Vimentin+

Vimentin+

CK-

CK-

CD34-

CD34-

Desmin-

Desmin-

S100-

S100-

Cytology description
  • Main advantages of soft tissue fine needle aspiration cytology (FNAC) are good sensitivity and specificity, low morbidity, speed of diagnosis and low cost / benefit ratio
  • The most important disadvantages stem from limited experience in cytological diagnosis of soft tissue tumors and a lack of standardized and uniform reporting system for soft tissue FNAC (Cytopathology 2020;31:271)
  • FNAC is of utility not only in primary lesions but also for metastatic tumors and for the documentation of locally recurrent soft tissue neoplasms (Diagn Cytopathol 2022;50:463, Cancer 1997;81:228)
  • Although making an initial diagnosis of sarcoma by fine needle aspiration biopsy is reliable, specific subtyping of sarcomas as undifferentiated sarcoma is more problematic
  • Cellular smears
  • Patterns range from single cells to large storiform fragments
  • Spindled, plasmacytoid, round and pleomorphic cell shapes are found; pleomorphic cells are often multinucleated
  • Anisonucleosis, nuclear hyperchromasia, frequent mitoses
  • Nuclear membrane irregularities, nucleoli are often prominent
Cytology images

Contributed by Borislav A. Alexiev, M.D.
Cellular smear

Cellular smear

Nuclear atypia

Nuclear atypia

Positive stains
Negative stains
Electron microscopy description
  • Immunohistochemistry has progressively supplanted electron microscopy as the method of choice to characterize the phenotype of neoplastic cells in soft tissue tumors and in solving most differential diagnoses (Ultrastruct Pathol 2008;32:51)
  • Occasionally, in tumors with smooth muscle, skeletal muscle, adipocytic, vascular endothelial and Schwann cell differentiation, electron microscopy may help in the differential diagnosis
Molecular / cytogenetics description
  • Numerical and structural variants underlying chromosomal aberrations are frequently observed in undifferentiated sarcoma (Front Genet 2023;14:1109491)
  • Losses of 13q12-q14 or 13q21 were observed in a large proportion of tumors, suggesting that a gene localized in this region could act as a tumor suppressor gene (Cancer Genet Cytogenet 1999;111:134)
  • Comprehensive, integrated genomics shows that undifferentiated sarcoma and myxofibrosarcoma are largely indistinguishable across the multiplatform molecular landscape (Front Genet 2023;14:1109491)
  • PRDM10 fusions are present in ~5% of undifferentiated sarcoma (Clin Cancer Res 2015;21:864)
Sample pathology report
  • Soft tissue, left thigh, excision:
    • Undifferentiated pleomorphic sarcoma, FNCLCC grade 3 (see comment)
    • Comment: H&E stained tissue sections show a patternless arrangement of highly atypical spindled and polygonal cells with eosinophilic cytoplasm. There is a marked nuclear pleomorphism, including bizarre nuclei and multinucleation. Abundant mitoses with atypical forms (> 20 mitoses/10 high power fields) and focal coagulative necrosis (~20%) are seen.
    • Immunohistochemical stains for vimentin and CD68 are positive in tumor cells, while all of the following are negative: keratin AE1 / AE3, MelanA, SOX10, desmin, h-caldesmon, ALK1, CD30, MyoD1 and myogenin. H3K27me3 expression is retained. FISH is negative for MDM2 amplification. The findings support the diagnosis of undifferentiated sarcoma.
Differential diagnosis
Board review style question #1

Which of the following is true for undifferentiated soft tissue sarcomas?

  1. Areas with prominent myxoid matrix and numerous curvilinear capillaries are common
  2. Complex and nonspecific cytogenetic abnormalities
  3. Most arise in the retroperitoneum
  4. Tumor cells often display cross striations
  5. Usually are tumors of children
Board review style answer #1
B. Complex and nonspecific cytogenetic abnormalities aberrations are frequently observed in undifferentiated soft tissue sarcomas. Answer C is incorrect because undifferentiated soft tissue sarcomas rarely arise in the retroperitoneum. Answer A is incorrect because prominent myxoid matrix with numerous curvilinear capillaries are characteristic findings in myxofibrosarcoma. Answer D is incorrect because tumor cells in undifferentiated soft tissue sarcomas do not display cross striation. Answer E is incorrect because undifferentiated soft tissue sarcomas usually affect older and elderly adults (50 - 70 years).

Comment Here

Reference: Undifferentiated / unclassified sarcoma
Board review style question #2
Which morphologic feature is commonly seen in undifferentiated soft tissue sarcomas?

  1. Chicken wire calcifications
  2. Lipoblasts
  3. Neoplastic bone formation
  4. Nuclear pleomorphism
  5. Tumor cells often display cross striations
Board review style answer #2
D. Nuclear pleomorphism is frequently observed in undifferentiated soft tissue sarcomas. Answer C is incorrect because neoplastic bone formation is typically seen with osteosarcoma. Answer A is incorrect because chicken wire calcifications are typically seen with chondroblastoma. Answer E is incorrect because cross striations are typically seen with rhabdomyosarcomas. Answer B is incorrect because lipoblasts are typically seen with liposarcomas.

Comment Here

Reference: Undifferentiated / unclassified sarcoma

Venous hemangioma
Definition / general
  • Also called venous malformation (VM)
  • Rare slow growing benign tumor of aberrant and ectatic venous connections
  • Vascular anomalies, including venous hemangiomas, are congenital lesions of abnormal vascular development
Essential features
  • Mainly present in adults
  • Tumor present in the subcutaneous or deep soft tissues with predilection for the limbs
  • Also present as a long standing slow growing tumor
  • Rapid growth may occur during puberty, pregnancy or traumatic injury
  • Calcifications can be seen in these tumors due to phleboliths (calcified thrombi)
  • Deep seated tumor is difficult to excise and can recur locally but subcutaneous tumors do not show a tendency to recur
Terminology
  • Venous hemangioma / venous malformation
Epidemiology
  • Incidence ~ 1 in 10,000
  • Most occur sporadically
  • Both men and women are affected
Sites
  • Mostly in adult limbs
Pathophysiology
  • Aberrant venous connections lead to venous congestion, thrombosis, phlebolith formation and gradual expansion of these lesions
Etiology
  • Vascular / venous malformation
  • Inherited forms of venous hemangiomas are autosomal dominant disorders and have been localized to chromosome 9p
  • Mutation of angiopoetin receptor gene TIE2 / TEK was recently discovered in multiple sporadic venous hemangiomas
Clinical features
  • Visible at birth but presents as a deep mass
  • Overlying skin may appear normal or possess a bluish discoloration
  • With more cutaneous involvement, the lesions appear darker blue or purple
Diagnosis
  • Calcifications can be seen in the radiological examinations due to phleboliths (calcified thrombi) within venous hemangiomas
  • MRI is the imaging modality of choice for diagnosing venous hemangioma; it offers superior delineation of the disease, which is necessary for treatment planning
Laboratory
  • D dimer may be elevated and a marker of disease
Radiology description
  • Radiographic studies, angiogram and MRI
Prognostic factors
  • Deep seated tumor is difficult to excise and can recur locally
  • Subcutaneous tumors do not show a tendency to recur
Treatment
  • Different treatment modalities based on size, location and patient age, including periodic observation, embolization, sclerotherapy, radiofrequency ablation and wide surgical excision
Gross description
  • Hemorrhagic cutaneous or deep soft tissue mass with ill defined and dilated vessels
Microscopic (histologic) description
  • Typically consists of closely packed thick walled vessels, which are variably dilated and commonly display thrombosis with occasionally formation of phleboliths (calcified thrombi)
  • Vein valves are present
  • Occasional hemosiderin interposition can be seen in vessel walls
Microscopic (histologic) images

Contributed by Jian-Hua Qiao, M.D.

Low power

High power

Mural thrombus formation

Large laminated thrombus

Thrombus with proliferation of fibroblasts

Focal hemosiderin deposition

Board review style question #1
Which statements are true about venous hemangioma?

  1. It can grow rapidly during pregnancy
  2. The deep soft tissue tumor can be entirely excised without recurrence
  3. The tumor can be found in subcutaneous tissue or deep soft tissue
  4. Venous hemangioma is a slow growing tumor


  1. (1,3,4)
  2. (1,4)
  3. (2,3)
  4. (1,2,3,4)
Board review style answer #1
A. (All but 2 are correct; 2 is wrong, since deep seated tumors are difficult to excise and can recur locally)

Comment Here

Reference: Venous hemangioma

WHO classification
Definition / general
  • WHO classification of soft tissue tumors serves as a guide to clarify diagnoses among a multidisciplinary team composed of pathologists, radiologists and clinicians (Adv Anat Pathol 2021;28:44)
Major updates
  • Adipocytic tumors
    • 2 new entities are considered in the new classification: atypical spindle cell / pleomorphic lipomatous tumor and myxoid pleomorphic liposarcoma
    • Benign
    • Malignant
    • Extrarenal angiomyolipoma was deleted in this category and reclassified in the category of tumors of uncertain differentiation and is now referred to as angiomyolipoma (Adv Anat Pathol 2021;28:44)
    • Extra-adrenal myelolipoma was removed in this edition
  • Fibroblastic and myofibroblastic tumors
  • So called fibrohistiocytic tumors
    • Most relevant entity that has disappeared since 2013 is represented by the family of so called malignant fibrous histiocytoma (MFH) (Pathologica 2021;113:70)
      • Undifferentiated pleomorphic sarcoma currently represents the correct designation for the storiform and pleomorphic variant of MFH
      • Giant cell MFH is currently replaced by 3 distinct tumor types: giant cell tumor of soft tissues, extraskeletal osteosarcoma and giant cell rich osteosarcoma (Pathologica 2021;113:70)
      • Myxoid MFH is currently recognized as a purely fibroblastic tumor, identified with the original name myxofibrosarcoma (Pathologica 2021;113:70)
      • Inflammatory MFH overlaps entirely with the inflammatory variant of dedifferentiated liposarcomas (Pathologica 2021;113:70)
      • Angiomatoid MFH (an indolent lesion most often harboring a EWSR1-CREB1 fusion gene and more rarely a EWSR1-ATF1 or FUS-ATF1 fusion gene) is currently listed within the group of soft tissue lesion of unknown differentiation (Pathologica 2021;113:70, Clin Cancer Res 2007;13:7322)
  • Vascular tumors
  • Smooth muscle tumors
    • 2 new entities are considered in the new classification: EBV associated smooth muscle tumors and inflammatory leiomyosarcoma (Pathologica 2021;113:70)
    • Intermediate
    • Malignant
  • Peripheral nerve sheath tumors
    • Single major change introduced by the 2020 WHO classification is the recognition that so called melanotic schwannoma actually represents a clinically aggressive neoplasm (no longer belonging to the intermediate category) and is consequently relabeled as malignant melanotic nerve sheath tumor (Pathologica 2021;113:70)
    • Malignant
  • Tumors of uncertain differentiation
    • NTRK rearranged spindle cell neoplasms (excluding infantile fibrosarcoma that represent a distinct clinicopathologic entity defined molecularly by the presence of NTRK3-ETV6 fusion gene) represent an emerging group of molecularly defined rare soft tissue tumors (Pathologica 2021;113:70)
    • Prognosis of NTRK rearranged adult tumors appear to be related to histological grade
    • NTRK rearranged spindle cell neoplasm
  • Undifferentiated small round cell sarcoma of bone and soft tissue
WHO (2020)











    Undifferentiated small round cell sarcoma of bone and soft tissue ICD-O
  • Ewing sarcoma9364/3
  • Round cell sarcoma with EWSR1 non-ETS fusions9366/3
  • CIC rearranged sarcoma9367/3
  • Sarcoma with BCOR genetic alterations9368/3
Microscopic (histologic) images

Contributed by Borislav A. Alexiev, M.D.
Myxoid liposarcoma

Myxoid liposarcoma

Pleomorphic liposarcoma

Pleomorphic liposarcoma

Inflammatory leiomyosarcoma

Inflammatory leiomyosarcoma

Sclerosing epithelioid fibrosarcoma

Sclerosing epithelioid fibrosarcoma

Biphasic synovial sarcoma

Biphasic synovial sarcoma

Proximal type epithelioid sarcoma

Proximal type epithelioid sarcoma


Extraskeletal myxoid chondrosarcoma

Extraskeletal myxoid chondrosarcoma

Alveolar soft part sarcoma

Alveolar soft part sarcoma

Clear cell sarcoma

Clear cell sarcoma

Desmoplastic small round cell tumor

Desmoplastic small round cell tumor

CIC rearranged sarcoma

CIC rearranged sarcoma

Sarcoma with BCOR genetic alterations

Sarcoma with BCOR genetic alterations

Board review style question #1
The single new entity appearing among vascular lesions in the WHO 2020 classification is named anastomosing hemangioma. What is the most common site of anastomosing hemangioma?

  1. Head and neck
  2. Intestinal tract
  3. Kidney and retroperitoneal adipose tissue
  4. Limbs
  5. Lungs and uterus
Board review style answer #1
C. Kidney and retroperitoneal adipose tissue

Comment Here

Reference: WHO classification of soft tissue tumors
Board review style question #2

The histologic pattern shown above would be most commonly expected in which of the following soft tissue tumors listed in WHO 2020?

  1. Cellular angiofibroma
  2. Low grade fibromyxoid sarcoma
  3. Myofibroblastoma
  4. Perineurioma
  5. Solitary fibrous tumor
Board review style answer #2
E. Solitary fibrous tumor. Solitary fibrous tumor is characterized by haphazardly arranged spindled to ovoid cells with indistinct, pale eosinophilic cytoplasm within a variably collagenous stroma and branching staghorn vascular pattern (hemangiopericytomatous vessels).

Comment Here

Reference: WHO classification of soft tissue tumors

WHO classification-GI mesenchymal tumors
Table of Contents
Definition / general | WHO (2019)
Definition / general
WHO (2019)






Back to top
Recent Soft tissue Pathology books

Bocklage: 2014

Boto: 2016

Dodd: 2014

Fisher: 2015

Folpe: 2022

Goldblum: 2019

Hornick: 2018

Horvai: 2012

IARC: 2020

Jiang: 2023

Lindberg: 2019

Miettinen: 2016

Mody: 2022

Montgomery: 2018

Montgomery: 2020

Paolo Dei Tos : 2019

Perry: 2017



Find related Pathology books: cytopathology, soft tissue & bone
Image 01 Image 02